You are on page 1of 196

PERSONS AND FAMILY RELATIONS

CIVIL PERSONALITY
ARTICLE 41
Republic of the Philippines
SUPREME COURT
Manila

EN BANC

G.R. No. 26795 July 31, 1970

CARMEN QUIMIGUING, Suing through her parents, ANTONIO QUIMIGUING and JACOBA
CABILIN, plaintiffs-appellants,
vs.
FELIX ICAO, defendant-appellee.

Torcuato L. Galon for plaintiffs-appellants.

Godardo Jacinto for defendant-appellee.

REYES, J.B.L., J.:

Appeal on points of law from an order of the Court of First Instance of Zamboanga del Norte (Judge Onofre Sison
Abalos, presiding), in its Civil Case No. 1590, dismissing a complaint for support and damages, and another
order denying amendment of the same pleading.

The events in the court of origin can be summarized as follows:

Appellant, Carmen Quimiguing, assisted by her parents, sued Felix Icao in the court below. In her complaint it
was averred that the parties were neighbors in Dapitan City, and had close and confidential relations; that
defendant Icao, although married, succeeded in having carnal intercourse with plaintiff several times by force and
intimidation, and without her consent; that as a result she became pregnant, despite efforts and drugs supplied
by defendant, and plaintiff had to stop studying. Hence, she claimed support at P120.00 per month, damages
and attorney's fees.

Duly summoned, defendant Icao moved to dismiss for lack of cause of action since the complaint did not allege
that the child had been born; and after hearing arguments, the trial judge sustained defendant's motion and
dismissed the complaint.

Thereafter, plaintiff moved to amend the complaint to allege that as a result of the intercourse, plaintiff had later
given birth to a baby girl; but the court, sustaining defendant's objection, ruled that no amendment was allowable,
since the original complaint averred no cause of action. Wherefore, the plaintiff appealed directly to this Court.

We find the appealed orders of the court below to be untenable. A conceived child, although as yet unborn, is
given by law a provisional personality of its own for all purposes favorable to it, as explicitly provided in Article 40
of the Civil Code of the Philippines. The unborn child, therefore, has a right to support from its progenitors,
particularly of the defendant-appellee (whose paternity is deemed admitted for the purpose of the motion to
dismiss), even if the said child is only "en ventre de sa mere;" just as a conceived child, even if as yet unborn,
may receive donations as prescribed by Article 742 of the same Code, and its being ignored by the parent in his
testament may result in preterition of a forced heir that annuls the institution of the testamentary heir, even if such
child should be born after the death of the testator Article 854, Civil Code).

ART. 742. Donations made to conceived and unborn children may be accepted by those persons
who would legally represent them if they were already born.

ART. 854. The preterition or omission of one, some, or all of the compulsory heirs in the direct
line, whether living at the time of the execution of the will or born after the death of the testator,
shall annul the institution of heir; but the devises and legacies shall be valid insofar as they are
not inofficious.
If the omitted compulsory heirs should die before the testator, the institution shall be effectual,
without prejudice to the right of 'representation.

It is thus clear that the lower court's theory that Article 291 of the Civil Code declaring that support is an obligation
of parents and illegitimate children "does not contemplate support to children as yet unborn," violates Article 40
aforesaid, besides imposing a condition that nowhere appears in the text of Article 291. It is true that Article 40
prescribing that "the conceived child shall be considered born for all purposes that are favorable to it" adds
further "provided it be born later with the conditions specified in the following article" (i.e., that the foetus be alive
at the time it is completely delivered from the mother's womb). This proviso, however, is not a condition
precedent to the right of the conceived child; for if it were, the first part of Article 40 would become entirely
useless and ineffective. Manresa, in his Commentaries (5th Ed.) to the corresponding Article 29 of the Spanish
Civil Code, clearly points this out:

Los derechos atribuidos al nasciturus no son simples expectativas, ni aun en el sentido tecnico
que la moderna doctrina da a esta figura juridica sino que constituyen un caso de los
propiamente Ilamados 'derechos en estado de pendenci'; el nacimiento del sujeto en las
condiciones previstas por el art. 30, no determina el nacimiento de aquellos derechos (que ya
existian de antemano), sino que se trata de un hecho que tiene efectos declarativos. (1 Manresa,
Op. cit., page 271)

A second reason for reversing the orders appealed from is that for a married man to force a woman not his wife
to yield to his lust (as averred in the original complaint in this case) constitutes a clear violation of the rights of his
victim that entitles her to claim compensation for the damage caused. Says Article 21 of the Civil Code of the
Philippines:

ART. 21. Any person who wilfully causes loss or injury to another in a manner that is contrary to
morals, good customs or public policy shall compensate the latter for the damage.

The rule of Article 21 is supported by Article 2219 of the same Code:

ART 2219. Moral damages may be recovered in the following and analogous cases:

(3) Seduction, abduction, rape or other lascivious acts:

xxx xxx xxx

(10) Acts and actions referred to in Articles 21, 26, 27, 28 ....

Thus, independently of the right to Support of the child she was carrying, plaintiff herself had a cause of action for
damages under the terms of the complaint; and the order dismissing it for failure to state a cause of action was
doubly in error.

WHEREFORE, the orders under appeal are reversed and set aside. Let the case be remanded to the court of
origin for further proceedings conformable to this decision. Costs against appellee Felix Icao. So ordered.

Concepcion, C.J., Dizon, Makalintal, Zaldivar, Castro, Fernando, Teehankee, Barredo and Villamor, JJ., concur.
Republic of the Philippines
SUPREME COURT
Manila

EN BANC

G.R. No. L-16439 July 20, 1961

ANTONIO GELUZ, petitioner,


vs.
THE HON. COURT OF APPEALS and OSCAR LAZO, respondents.

Mariano H. de Joya for petitioner.


A.P. Salvador for respondents.

REYES, J.B.L., J.:

This petition for certiorari brings up for review question whether the husband of a woman, who voluntarily
procured her abortion, could recover damages from physician who caused the same.

The litigation was commenced in the Court of First Instance of Manila by respondent Oscar Lazo, the of Nita
Villanueva, against petitioner Antonio Geluz, a physician. Convinced of the merits of the complaint upon the
evidence adduced, the trial court rendered judgment favor of plaintiff Lazo and against defendant Geluz, ordering
the latter to pay P3,000.00 as damages, P700.00 attorney's fees and the costs of the suit. On appeal, Court of
Appeals, in a special division of five, sustained the award by a majority vote of three justices as against two, who
rendered a separate dissenting opinion.

The facts are set forth in the majority opinion as follows:

Nita Villanueva came to know the defendant (Antonio Geluz) for the first time in 1948 — through her aunt
Paula Yambot. In 1950 she became pregnant by her present husband before they were legally married.
Desiring to conceal her pregnancy from her parent, and acting on the advice of her aunt, she had herself
aborted by the defendant. After her marriage with the plaintiff, she again became pregnant. As she was
then employed in the Commission on Elections and her pregnancy proved to be inconvenient, she had
herself aborted again by the defendant in October 1953. Less than two years later, she again became
pregnant. On February 21, 1955, accompanied by her sister Purificacion and the latter's daughter Lucida,
she again repaired to the defendant's clinic on Carriedo and P. Gomez streets in Manila, where the three
met the defendant and his wife. Nita was again aborted, of a two-month old foetus, in consideration of the
sum of fifty pesos, Philippine currency. The plaintiff was at this time in the province of Cagayan,
campaigning for his election to the provincial board; he did not know of, nor gave his consent, to the
abortion.

It is the third and last abortion that constitutes plaintiff's basis in filing this action and award of damages. Upon
application of the defendant Geluz we granted certiorari.

The Court of Appeals and the trial court predicated the award of damages in the sum of P3,000.06 upon the
provisions of the initial paragraph of Article 2206 of the Civil Code of the Philippines. This we believe to be error,
for the said article, in fixing a minimum award of P3,000.00 for the death of a person, does not cover the case of
an unborn foetus that is not endowed with personality. Under the system of our Civil Code, "la criatura abortiva
no alcanza la categoria de persona natural y en consscuencia es un ser no nacido a la vida del Derecho"
(Casso-Cervera, "Diccionario de Derecho Privado", Vol. 1, p. 49), being incapable of having rights and
obligations.

Since an action for pecuniary damages on account of personal injury or death pertains primarily to the one
injured, it is easy to see that if no action for such damages could be instituted on behalf of the unborn child on
account of the injuries it received, no such right of action could derivatively accrue to its parents or heirs. In fact,
even if a cause of action did accrue on behalf of the unborn child, the same was extinguished by its pre-natal
death, since no transmission to anyone can take place from on that lacked juridical personality (or juridical
capacity as distinguished from capacity to act). It is no answer to invoke the provisional personality of a
conceived child (conceptus pro nato habetur) under Article 40 of the Civil Code, because that same article
expressly limits such provisional personality by imposing the condition that the child should be subsequently born
alive: "provided it be born later with the condition specified in the following article". In the present case, there is
no dispute that the child was dead when separated from its mother's womb.

The prevailing American jurisprudence is to the same effect; and it is generally held that recovery can not had for
the death of an unborn child (Stafford vs. Roadway Transit Co., 70 F. Supp. 555; Dietrich vs. Northampton, 52
Am. Rep. 242; and numerous cases collated in the editorial note, 10 ALR, (2d) 639).

This is not to say that the parents are not entitled to collect any damages at all. But such damages must be those
inflicted directly upon them, as distinguished from the injury or violation of the rights of the deceased, his right to
life and physical integrity. Because the parents can not expect either help, support or services from an unborn
child, they would normally be limited to moral damages for the illegal arrest of the normal development of
the spes hominis that was the foetus, i.e., on account of distress and anguish attendant to its loss, and the
disappointment of their parental expectations (Civ. Code Art. 2217), as well as to exemplary damages, if the
circumstances should warrant them (Art. 2230). But in the case before us, both the trial court and the Court of
Appeals have not found any basis for an award of moral damages, evidently because the appellee's indifference
to the previous abortions of his wife, also caused by the appellant herein, clearly indicates that he was
unconcerned with the frustration of his parental hopes and affections. The lower court expressly found, and the
majority opinion of the Court of Appeals did not contradict it, that the appellee was aware of the second abortion;
and the probabilities are that he was likewise aware of the first. Yet despite the suspicious repetition of the event,
he appeared to have taken no steps to investigate or pinpoint the causes thereof, and secure the punishment of
the responsible practitioner. Even after learning of the third abortion, the appellee does not seem to have taken
interest in the administrative and criminal cases against the appellant. His only concern appears to have been
directed at obtaining from the doctor a large money payment, since he sued for P50,000.00 damages and
P3,000.00 attorney's fees, an "indemnity" claim that, under the circumstances of record, was clearly exaggerated.

The dissenting Justices of the Court of Appeals have aptly remarked that:

It seems to us that the normal reaction of a husband who righteously feels outraged by the abortion which
his wife has deliberately sought at the hands of a physician would be highminded rather than mercenary;
and that his primary concern would be to see to it that the medical profession was purged of an unworthy
member rather than turn his wife's indiscretion to personal profit, and with that idea in mind to press either
the administrative or the criminal cases he had filed, or both, instead of abandoning them in favor of a
civil action for damages of which not only he, but also his wife, would be the beneficiaries.

It is unquestionable that the appellant's act in provoking the abortion of appellee's wife, without medical necessity
to warrant it, was a criminal and morally reprehensible act, that can not be too severely condemned; and the
consent of the woman or that of her husband does not excuse it. But the immorality or illegality of the act does
not justify an award of damage that, under the circumstances on record, have no factual or legal basis.

The decision appealed from is reversed, and the complaint ordered dismissed. Without costs.

Let a copy of this decision be furnished to the Department of Justice and the Board of Medical Examiners for their
information and such investigation and action against the appellee Antonio Geluz as the facts may warrant.

Bengzon, C.J., Padilla, Labrador, Barrera, Paredes, Dizon and Natividad, JJ., concur.
Concepcion, J., took no part.
De Leon, J., took no part.
ARTICLE 42
Republic of the Philippines
SUPREME COURT
Manila

EN BANC

G.R. No. L-15499 February 28, 1962

ANGELA M. BUTTE, plaintiff-appellant,


vs.
MANUEL UY and SONS, INC., defendant-appellee.

Delgado, Flores and Macapagal for plaintiff-appellant.


Pelaez and Jalandoni for defendant-appellee.

REYES, J.B.L., J.:

Appeal from a decision of the Court of First instance of Manila dismissing the action for legal redemption filed by
plaintiff-appellant.

It appears that Jose V. Ramirez, during his lifetime, was a co-owner of a house and lot located at Sta. Cruz,
Manila, as shown by Transfer Certificate of Title No. 52789, issued in the name of the following co-owners: Marie
Garnier Vda. de Ramirez, 1/6; Jose V. Ramirez, 1/6; Jose E. Ramirez, 1/6; Rita de Ramirez, 1/6; and Jose Ma.
Ramirez, 1/6.

On October 20, 1951, Jose V. Ramirez died. Subsequently, Special Proceeding No. 15026 was instituted to
settle his estate, that included the one-sixth (1/6) undivided share in the aforementioned property. And although
his last will and testament, wherein he bequeathed his estate to his children and grandchildren and one-third
(1/3) of the free portion to Mrs. Angela M. Butte, hereinafter referred to as plaintiff-appellant, has been admitted
to probate, the estate proceedings are still pending up to the present on account of the claims of creditors which
exceed the assets of the deceased. The Bank of the Philippine Islands was appointed judicial administrator.

Meanwhile, on December 9, 1958, Mrs. Marie Garnier Vda. de Ramirez, one of the co-owners of the late Jose V.
Ramirez in the Sta. Cruz property, sold her undivided 1/6 share to Manuel Uy & Sons, Inc. defendant-appellant
herein, for the sum of P500,000.00. After the execution by her attorney-in-fact, Mrs. Elsa R. Chambers, of an
affidavit to the effect that formal notices of the sale had been sent to all possible redemptioners, the deed of sale
was duly registered and Transfer Certificate of Title No. 52789 was cancelled in lieu of which a new one was
issued in the name of the vendee and the other-co-owners.

On the same day (December 9, 1958), Manuel Uy & Sons, Inc. sent a letter to the Bank of the Philippine Islands
as judicial administrator of the estate of the late Jose V. Ramirez informing it of the above-mentioned sale. This
letter, together with that of the bank, was forwarded by the latter to Mrs. Butte c/o her counsel Delgado, Flores &
Macapagal, Escolta, Manila, and having received the same on December 10, 1958, said law office delivered
them to plaintiff-appellant's son, Mr. Miguel Papa, who in turn personally handed the letters to his mother, Mrs.
Butte, on December 11 and 12, 1958. Aside from this letter of defendant-appellant, the vendor, thru her attorney-
in-fact Mrs. Chambers, wrote said bank on December 11, 1958 confirming vendee's letter regarding the sale of
her 1/6 share in the Sta. Cruz property for the sum of P500,000.00. Said letter was received by the bank on
December 15, 1958 and having endorsed it to Mrs. Butte's counsel, the latter received the same on December
16, 1958. Appellant received the letter on December 19, 1958.

On January 15, 1959, Mrs. Angela M. Butte, thru Atty. Resplandor Sobretodo, sent a letter and a Philippine
National Bank cashier's check in the amount of P500,000.00 to Manuel Uy & Sons, Inc. offering to redeem the
1/6 share sold by Mrs. Marie Garnier Vda. de Ramirez. This tender having been refused, plaintiff on the same
day consigned the amount in court and filed the corresponding action for legal redemption. Without prejudice to
the determination by the court of the reasonable and fair market value of the property sold which she alleged to
be grossly excessive, plaintiff prayed for conveyance of the property, and for actual, moral and exemplary
damages.

After the filing by defendant of its answer containing a counterclaim, and plaintiff's reply thereto, trial was held,
after which the court rendered decision on May 13, 1959, dismissing plaintiff's complaint on the grounds that she
has no right to redeem the property and that, if ever she had any, she exercised the same beyond the statutory
30-day period for legal redemptions provided by the Civil Code. The counterclaim of defendant for damages was
likewise dismissed for not being sufficiently established. Both parties appealed directly to this Court.

Based on the foregoing facts, the main issues posed in this appeal are: (1) whether or not plaintiff-appellant,
having been bequeathed 1/3 of the free portion of the estate of Jose V. Ramirez, can exercise the right of legal
redemption over the 1/6 share sold by Mrs. Marie Garnier Vda. de Ramirez despite the presence of the judicial
administrator and pending the final distribution of her share in the testate proceedings; and (2) whether or not she
exercised the right of legal redemption within the period prescribed by law.

The applicable law involved in the present case is contained in Articles 1620, p. 1, and 1623 of the Civil Code of
the Philippines, which read as follows:

ART. 1620. A co-owner of a thing may exercise the right of redemption in case the shares of all the other-
co-owners or of any of them, are sold to a third person. If the price of the alienation is grossly excessive,
the redemptioner shall pay only a reasonable one.

Should two or more co-owners desire to exercise the right of redemption, they may only do so in
proportion to the share they may respectively have in the thing owned in common. (1522a)

ART. 1623. The right of legal predemption or redemption shall not be exercised except within thirty days
from the notice in writing by the respective vendor, or by the vendor, as the case may be. The deed of
sale shall not be accorded in the Registry of Property, unless accompanied by an affidavit of the vendor
that he has given written notice thereof at all possible redemptioners.

The right of redemption of co-owners excludes that of adjoining owners. (1524a)

That the appellant Angela M. Butte is entitled to exercise the right of legal redemption is clear. As testamentary
heir of the estate of J.V. Ramirez, she and her co-heirs acquired an interest in the undivided one-sixth (1/6) share
owned by her predecessor (causante) in the Santa Cruz property, from the moment of the death of the aforesaid
co-owner, J.V. Ramirez. By law, the rights to the succession of a deceased persons are transmitted to his heirs
from the moment of his death, and the right of succession includes all property rights and obligations that survive
the decedent.

ART. 776. The inheritance includes all the property, rights and obligations of a person which are not
extinguished by his death. (659)

ART. 777. The rights to the succession are transmitted from the moment of the death of the decedent.
(657a)

ART. 947. The legatee or devisee acquires a right to the pure and simple legacies or devisees from the
death of the testator, and transmits it to his heirs. (881a)

The principle of transmission as of the time of the predecessor's death is basic in our Civil Code, and is
supported by other related articles. Thus, the capacity of the heir is determined as of the time the decedent died
(Art. 1034); the legitime is to be computed as of the same moment(Art. 908), and so is the in officiousness of the
donation inter vivos (Art. 771). Similarly, the legacies of credit and remission are valid only in the amount due and
outstanding at the death of the testator (Art. 935),and the fruits accruing after that instant are deemed to pertain
to the legatee (Art. 948).

As a consequence of this fundamental rule of succession, the heirs of Jose V. Ramirez acquired his undivided
share in the Sta. Cruz property from the moment of his death, and from that instant, they became co-owners in
the aforesaid property, together with the original surviving co-owners of their decedent (causante). A co-owner of
an undivided share is necessarily a co-owner of the whole. Wherefore, any one of the Ramirez heirs, as such co-
owner, became entitled to exercise the right of legal redemption (retracto de comuneros) as soon as another co-
owner (Maria Garnier Vda. de Ramirez) had sold her undivided share to a stranger, Manuel Uy & Sons, Inc. This
right of redemption vested exclusively in consideration of the redemptioner's share which the law nowhere takes
into account.

The situation is in no wise altered by the existence of a judicial administrator of the estate of Jose V. Ramirez
while under the Rules of Court the administrator has the right to the possession of the real and personal estate of
the deceased, so far as needed for the payment of the decedent's debts and the expenses of administration (sec.
3, Rule 85), and the administrator may bring or defend actions for the recovery or protection of the property or
rights of the deceased (sec. 2, Rule 88), such rights of possession and administration do not include the right of
legal redemption of the undivided share sold to Uy & Company by Mrs. Garnier Ramirez. The reason is obvious:
this right of legal redemption only came into existence when the sale to Uy & Sons, Inc. was perfected, eight (8)
years after the death of Jose V. Ramirez, and formed no part of his estate. The redemption right vested in the
heirs originally, in their individual capacity, they did not derivatively acquire it from their decedent, for when Jose
V. Ramirez died, none of the other co-owners of the Sta. Cruz property had as yet sold his undivided share to a
stranger. Hence, there was nothing to redeem and no right of redemption; and if the late Ramirez had no such
right at his death, he could not transmit it to his own heirs. Much less could Ramirez acquire such right of
redemption eight years after his death, when the sale to Uy & Sons, Inc. was made; because death extinguishes
civil personality, and, therefore, all further juridical capacity to acquire or transmit rights and obligations of any
kind (Civil Code of the Phil., Art. 42).

It is argued that the actual share of appellant Mrs. Butte in the estate of Jose V. Ramirez has not been
specifically determined as yet, that it is still contingent; and that the liquidation of estate of Jose V. Ramirez may
require the alienation of the decedent's undivided portion in the Sta. Cruz property, in which event Mrs. Butte
would have no interest in said undivided portion. Even if it were true, the fact would remain that so long as that
undivided share remains in the estate, the heirs of Jose V. Ramirez own it, as the deceased did own it before his
demise, so that his heirs are now as much co-owners of the Sta. Cruz property as Jose V. Ramirez was himself a
co-owner thereof during his lifetime. As co-owners of the property, the heirs of Jose V. Ramirez, or any one of
them, became personally vested with right of legal redemption as soon as Mrs. Garnier sold her own pro-indiviso
interest to Uy & Sons. Even if subsequently, the undivided share of Ramirez (and of his heirs) should eventually
be sold to satisfy the creditors of the estate, it would not destroy their ownership of it before the sale, but would
only convey or transfer it as in turn sold (of it actually is sold) to pay his creditors. Hence, the right of any of the
Ramirez heirs to redeem the Garnier share will not be retroactively affected. All that the law requires is that the
legal redemptioner should be a co-owner at the time the undivided share of another co-owner is sold to a
stranger. Whether or not the redemptioner will continue being a co-owner after exercising the legal redemptioner
is irrelevant for the purposes of law.

Nor it can be argued that if the original share of Ramirez is sold by the administrator, his heirs would stand in law
as never having acquired that share. This would only be true if the inheritance is repudiated or the heir's quality
as such is voided. But where the heirship is undisputed, the purchaser of hereditary property is not deemed to
have acquired the title directly from the deceased Ramirez, because a dead man can not convey title, nor from
the administrator who owns no part of the estate; the purchaser can only derive his title from the Ramirez heirs,
represented by the administrator, as their trustee or legal representative.

The right of appellant Angela M. Butte to make the redemption being established, the next point of inquiry is
whether she had made or tendered the redemption price within the 30 days from notices as prescribed by law.
This period, be it noted, is peremptory, because the policy of the law is not to leave the purchaser's title in
uncertainty beyond the established 30-day period. In considering whether or not the offer to redeem was timely,
we think that the notice given by the vendee (buyer) should not be taken into account. The text of Article 1623
clearly and expressly prescribes that the thirty days for making the redemption are to be counted from notice in
writing by the vendor. Under the old law (Civ. Code of 1889, Art. 1524), it was immaterial who gave the notice; so
long as the redeeming co-owner learned of the alienation in favor of the stranger, the redemption period began to
run. It is thus apparent that the Philippine legislature in Article 1623 deliberately selected a particular method of
giving notice, and that method must be deemed exclusive (39 Am. Jur., 237; Payne vs. State, 12 S.W. [2d] 528).
As ruled in Wampler vs. Lecompte, 150 Atl. 458 (affd. in 75 Law Ed. [U.S.] 275) —

Why these provisions were inserted in the statute we are not informed, but we may assume until the
contrary is shown, that a state of facts in respect thereto existed, which warranted the legislature in so
legislating.

The reasons for requiring that the notice should be given by the seller, and not by the buyer, are easily divined.
The seller of an undivided interest is in the best position to know who are his co-owners that under the law must
be notified of the sale. Also, the notice by the seller removes all doubts as to the fact of the sale, its perfection;
and its validity, the notice being a reaffirmation thereof, so that the party need not entertain doubt that the seller
may still contest the alienation. This assurance would not exist if the notice should be given by the buyer.

The notice which became operative is that given by Mrs. Chambers, in her capacity as attorney-in-fact of the
vendor Marie Garnier Vda. de Ramirez. Under date of December 11, 1958, she wrote the Administrator Bank of
the Philippine Islands that her principal's one-sixth (1/6) share in the Sta. Cruz property had been sold to Manuel
Uy & Sons, Inc. for P500,000.00. The Bank received this notice on December 15, 1958, and on the same day
endorsed it to Mrs. Butte, care of Delgado, Flores and Macapagal (her attorneys), who received the same on
December 16, 1958. Mrs. Butte tendered redemption and upon the vendee's refusal, judicially consigned the
price of P500,000.00 on January 15, 1959. The latter date was the last one of the thirty days allowed by the Code
for the redemption, counted by excluding December 16, 1958 and including January 15, 1959, pursuant to Article
13 of the Civil Code. Therefore, the redemption was made in due time.

The date of receipt of the vendor's notice by the Administrator Bank (December 15) can not be counted as
determining the start of thirty days; for the Administrator of the estate was not a proper redemptioner, since, as
previously shown, the right to redeem the share of Marie Garnier did not form part of the estate of Jose V.
Ramirez.
We find no jurisdiction for appellant's claim that the P500,000,00. paid by Uy & Sons, Inc. for the Garnier share is
grossly excessive. Gross excess cannot be predicated on mere individual estimates of market price by a single
realtor.

The redemption and consignation having been properly made, the Uy counterclaim for damages and attorney's
fees predicated on the assumption that plaintiff's action was clearly unfounded, becomes untenable.

PREMISES CONSIDERED, the judgment appealed from is hereby reversed and set aside, and another one
entered:

(a) Declaring the consignation of P500,000,00 made by appellant Angela M. Butte duly and properly
made;

(b) Declaring that said appellant properly exercised in due time the legal redemption of the one-sixth (1/6)
undivided portion of the land covered by Certificate of Title No. 59363 of the Office of the Register of
Deeds of the City of Manila, sold on December 9, 1958 by Marie Garnier Vda. de Ramirez to appellant
Manuel Uy & Sons, Inc.

(c) Ordering appellant Manuel Uy & Sons, Inc. to accept the consigned price and to convey to Angela M.
Butte the undivided portion above referred to, within 30 days from the time our decision becomes final,
and subsequently to account for the rentals and fruits of the redeemed share from and after January 15,
1958, until its conveyance; and.

(d) Ordering the return of the records to the court of origin for further proceedings conformable to this
opinion.

Without finding as to costs.

Bengzon, C.J., Padilla, Bautista Angelo, Labrador, Concepcion, Barrera and Dizon, JJ., concur.
Paredes and De Leon, JJ., took no part.
ARTICLE 43
Republic of the Philippines
SUPREME COURT
Manila

EN BANC

G.R. No. L-5426 May 29, 1953

RAMON JOAQUIN, petitioner,


vs.
ANTONIO C. NAVARRO, respondent.

Agrava, Peralta & Agrava for petitioner.


Leonardo Abola for respondent.

TUASON, J.:

This three proceedings was instituted in the Court of First Instance of Manila in the summary settlement of states
of Joaquin Navarro, Sr., his wife Angela Joaquin de Navarro, Joaquin Navarro, Jr., and Pilar Navarro, deceased.
All of them having been heard jointly, Judge Rafael Amparo handed down a single decision which was appealed
to the Court of Appeals, whose decision, modifying that the Court of First Instance, in turn was elevated to the
Supreme Court for review.

The main question represented in the first two courts related to the sequence of the deaths of Joaquin Navarro,
Sr., his wife, and their children, all of whom were killed in the massacre of civilians by Japanese troops in Manila
in February 1945. The trial court found the deaths of this persons to have accurred in this order: 1st. The Navarro
girls, named Pilar, Concepcion and Natividad; 2nd. Joaquin Navarro, Jr.; 3rd. Angela Joaquin de Navarro, and
4th, Joaquin Navarro, Sr. The Court of Appeals concurred with the trial court except that, with regard to Angela
Joaquin de Navarro and Joaquin Navarro, Jr., the latter was declared to have survived his mother.

It is this modification of the lower court's finding which is now being contested by the petitioner. The importance
of the question whether Angela Joaquin de Navarro died before Joaquin Navarro, Jr., or vice versa, lies in the
fact that it radically affects the rights of succession of Ramon Joaquin, the present petitioner who was an
acknowledged natural child of Angela Joaquin and adopted child of the deceased spouses, and Antonio C.
Navarro, respondent, son of Joaquin Navarro, Sr. by first marriage.

The facts, which is not disputed, are outlined in the statement in the decision of the Court of Appeals as follows:

"On February 6, 1945, while the battle for the liberation of Manila was raging, the spouses Joaquin Navarro, Sr.
and Angela Joaquin, together with their three daughters, Pilar, Concepcion, and Natividad, and their son Joaquin
Navarro, Jr., and the latter's wife, Adela Conde, sought refuge in the ground floor of the building known as the
German Club, at the corner of San Marcelino and San Luis Streets of this City. During their stay, the building was
packed with refugees, shells were exploding around, and the Club was set on fire. Simultaneously, the Japanese
started shooting at the people inside the building, especially those who were trying to escape. The three
daughters were hit and fell of the ground near the entrance; and Joaquin Navarro, Sr., and his son decided to
abandon the premises to seek a safer heaven. They could not convince Angela Joaquin who refused to join
them; and son Joaquin Navarro, Sr., his son, Joaquin Navarro, Jr., and the latter's wife, Angela Conde, and a
friend and former neighbor, Francisco Lopez, dashed out of the burning edifice. As they came out, Joaquin
Navarro, Jr. was shot in the head by a Japanese soldier and immediately dropped. The others lay flat on the
ground in front of the Club premises to avoid the bullets. Minutes later, the German Club, already on fire,
collapsed, trapping many people inside, presumably including Angela Joaquin.

"Joaquin Navarro, Sr., Mrs. Joaquin Navarro, Jr., and Francisco Lopez managed to reach an air raid shelter
nearby, the stayed there about three days, until February 10, 1915, when they were forced to leave the shelter
be- cause the shelling tore it open. They flied toward the St. Theresa Academy in San Marcelino Street, but
unfortunately met Japanese Patrols, who fired at the refugees, killing Joaquin Navarro, Sr., and his daughter-in-
law.

"At the time of the masaccre, Joaquin Navarro, Sr. was aged 70; his wife Angela Joaquin was about 67 years old;
Joaquin Navarro, Jr., about 30; Pilar Navarro was two or three years older than her brother; while the other
sisters, Concepcion and Natividad Navarro y Joaquin, were between 23 and 25."
The Court of Appeals' finding were all taken from the testimony of Francisco Lopez, who miraculously survived
the holocaust, and upon them the Court of Appeals opined that, "as between the mother Angela Joaquin and the
son Joaquin Navarro, Jr., the evidence of the survivorship is uncertain and insufficient" and the statutory
presumption must be applied. The appellate Court's reasoning for its conclusion is thus stated:

"It does not require argument to show that survivorship cannot be established by proof of the death of only one of
the parties; but that there must be adequate proof that one was alive when the other had already died. Now in
this case before us, the testimony of the sole witness Lopez is to the effect that Joaquin Navarro, Jr. was shot
and died shortly after the living the German Club in the company of his father and the witness, and that the
burning edified entirely collapsed minutes after the shooting of the son; but there is not a scintilla of evidence,
direct or circumstantial, from which we may infer the condition of the mother, Angela Joaquin, during the
appreciable interval from the instant his son turned his back to her, to dash out to the Club, until he died. All we
can glean from the evidence is that Angela Joaquin was unhurt when her son left her to escape from the German
Club; but she could have died almost immediately after, from a variety of causes. She might have been shot by
the Japanese, like her daughters, killed by falling beams from the burning edifice, overcome by the fumes, or
fatally struck by splinters from the exploding shells. We cannot say for certain. No evidence is available on the
point. All we can decide is that no one saw her alive after her son left her aside, and that there is no proof when
she died. Clearly, this circumstance alone cannot support a finding that she died latter than her son, and we are
thus compelled to fall back upon the statutory presumption. In deed, it could be said that the purpose of the
presumption of survivorship would be precisely to afford a solution to uncertainties like these. Hence the son
Joaquin Navarro, Jr. aged 30, must be deemed to have survived his mother, Angela Joaquin, who was admittedly
above 60 years of age (Rule 123, sec. 69, subsec. (ii), Rules of Court).

"The total lack of evidence on how Angela Joaquin died likewise disposes of the question whether she and her
deceased children perished in the same calamity. There being no evidence to the contrary, the only guide is the
occasion of the deaths, which is identical for all of them; that battle for the liberation of Manila. A second reason
is that the law, in declaring that those fallen in the same battle are to be regarded as perishing in the same
calamity, could not overlooked that a variety of cause of death can ( and usually do) operate in the source of
combats. During the same battle, some may die from wounds, other from gages, fire, or drowning. It is clear that
the law disregards episodic details, and treats the battle as an overall cause of death in applying the presumption
of survivorship.

"We are thus led the conclusion that the order in which the members of the Navarro-Joaquin family met their end
is as follows: first, the three daughters Pilar, Concepcion, and Natividad; then the mother Angela Joaquin; then
the son Joaquin Navarro, Jr., and days later (of which there is no doubt), the father Joaquin Navarro, Sr."

Much space in the briefs is taken in a discussion of whether section 334(37) of Act No. 129, now section 69 (ii) of
Rule 123 of the Rules of Court, has repealed article 33 of the civil code of 1889, now article 43 of the New Civil
Code. It is the contention of the petitioner that it did not, and that on the assumption that there is total lack of
evidence, as the Court of Appeals said, then Angela Joaquin and Joaquin Navarro, Jr. should, under article 33,
be held to have died at the same time.

The point is not of much if any relevancy and will be left open for the consideration when obsolute necessity there
for arises. We say irrelevant because our opinion is that neither of the two provisions is applicable for the reasons
to be presently set forth.

Rule 123, section 69 (ii) of the Revised Rules of Court, reads:

When two person perish in the same calamity, such as wreck, battle or conflagration, and it is not (1)
shown who died first, and there are no (2) particular circumstances from when it can be inferred, the
survivorship is presumed from the probabilities resulting from the strength and ages of the sexes,
according to the following rules:

xxx xxx xxx

Article 33 of the Civil Code of 1889 of the following tenor:

Whenever a doubt arises as to which was the first to die to the two or more persons who would inherent
one from the other, the persons who alleges the prior death of either must prove the allegation; in the
absence of proof the presumption shall be that they died at the same time, and no transmission of rights
from one to the other shall take place.

Most provisions, as their language plainly implies, are intended as a substitute for lacks and so are not to be
available when there are facts. With particular reference to section 69 (ii) of Rule 123, "the situation which it
present is one in which the facts are not only unknown but unknowable. By hypothesis, there is no specific
evidence as to the time of death . . . ." . . . it is assumed that no evidence can be produced. . . . Since the facts
are unknown and unknowable, the law may apply the law of fairness appropriate to the different legal situation
that arises." (IX Wigmore on Evidence, 1940 ed., 483.)

In In re Wallace's Estate, 220 Pac. 683, which the Court of Appeals cited the applied with the respect to the
deaths of the Navarro girls, pointing out that "our rule is taken from the Fourth Division of sec. 1936 of the
California Code of Civil Procedure," the Supreme Court of California said:

When the statue speaks of "particular circumstances from which it can be inferred" that one died before
the other it means that there are circumstances from which the fact of death by one before the other may
be inferred as a relation conclusion from the facts proven. The statue does not mean circumstances
which would shown, or which would tend to show, probably that one died before the other. Grand Lodge
A.O.W.W. vs. Miller, 8 Cal. App. 28, 96 Pac. 22. When by circumstantial evidence alone, a party seeks to
prove a survivorship contrary to the statutory presumption, the circumstances by which it is sought to
prove the survivorship must be such as are competent and sufficient when tested by the general rules of
evidence in civil cases. The inference of survivorship cannot rest upon mere surmise, speculation, or
conjecture. As was said in Grand Lodge vs. Miller, supra, "if the matter is left to probably, then the statue
of the presumption."

It is manifest from the language of section 69 (ii) of Rule 123 and of that of the foregoing decision that the
evidence of the survivorship need not be direct; it may be indirect, circumstantial, or inferential. Where there are
facts, known or knowable, from which a rational conclusion can be made, the presumption does not step in, and
the rule of preponderance of evidence controls.

Are there particular circumstances on record from which reasonable inference of survivorship between Angela
Joaquin and her son can be drawn? Is Francisco Lopez' testimony competent and sufficient for this purpose? For
a better appreciation of this issue, it is convenient and necessary to detail the testimony, which was described by
the trial court as "disinterested and trustworthy" and by the Court of Appeals as "entitled to credence."

Lopez testified:

Q. You said you were also heat at that time as you leave the German Club with Joaquin Navarro, Sr.,
Joaquin Navarro, Jr. and the latter's wife?- A. Yes, sir.

Q. Did you fall? — A. I fell down.

Q. And you said you fell down close to Joaquin Navarro, Jr.? A. Yes, sir.

Q. When the German Club collapsed where were you? — A. We were out 15 meters away from the
building but I could see what was going on.

xxx xxx xxx

Q. Could there have been an interval of fifteen minutes between the two events, that is the shooting of
Joaquin Navarro, Jr. and the collapse of the German Club? — A. Yes, sir, I could not say exactly,
Occasions like that, you know, you are confused.

Q. Could there (have) been an interval of an hour instead of fifteen minutes? — A. Possible, but not
probable.

Q. Could it have been 40 minutes? — A. Yes, sir, about 40 minutes.

xxx xxx xxx

Q. You also know that Angela Joaquin is already dead? — A. Yes, sir.

Q. Can you tell the Honorable Court when did Angela Joaquin die? — A. Well, a few minutes after we
have dashed out, the German Club, which was burning, collapsed over them, including Mrs. Joaquin
Navarro, Sr.

xxx xxx xxx

Q. From your testimony it would appear that while you can give positive evidence to the fact that Pilar,
Concepcion and Natividad Navarro, and Joaquin Navarro, Jr. died, you can not give the same positive
evidence to the fact that Angela Joaquin also died? — A. Yes, sir, in the sense that I did not see her
actually die, but when the building collapsed over her I saw and I am positive and I did not see her come
out of that building so I presumed she died there.
xxx xxx xxx

Q. Why did you have to dash out of the German Club, you, Mr. Joaquin Navarro, Sr. and Mr. Joaquin
Navarro Jr. and the latter's wife? — A. Because the Japanese had set fire to the Club and they were
shooting people outside, so we thought of running away rather than be roasted.

xxx xxx xxx

Q. You mean to say that before you jumped out of the German Club all the Navarro girls, Pilar,
Concepcion, and Natividad, were already wounded? — A. to my knowledge, yes.

Q. They were wounded? — A. Yes, sir.

Q. Were they lying on the ground or not? — A. On the ground near the entrance, because most of the
people who were shot by the Japanese were those who were trying to escape, and as far as I can
remember they were among those killed.

xxx xxx xxx

Q. So you noticed that they were killed or shot by the Japanese a few minutes before you left the place?
— A. That is what I think, because those Japanese soldiers were shooting the people inside especially
those trying to escape.

xxx xxx xxx

Q. And none of them was not except the girls, is that what you mean? A — . There were many people
shot because they were trying to escape.

xxx xxx xxx

Q. How come that these girls were shot when they were inside the building, can you explain that? — A.
They were trying to escape probably.

It is our opinion that the preceding testimony contains facts quite adequate to solve the problem of survivorship
between Angela Joaquin and Joaquin Navarro, Jr. and keep the statutory presumption out of the case. It is
believed that in the light of the conditions painted by Lopez, a fair and reasonable inference can be arrived at,
namely: that Joaquin Navarro, Jr. died before his mother.

While the possibility that the mother died before the son can not be ruled out, it must be noted that this possibility
is entirely speculative and must yield to the more rational deduction from proven facts that it was the other way
around. Joaquin Navarro, Jr., it will be recalled, was killed, while running, in front of, and 15 meters from, the
German Club. Still in the prime of life, 30, he must have negotiated that distance in five seconds or less, and so
died within that interval from the time he dashed out of the building. Now, when Joaquin Navarro, Jr. with his
father and wife started to flee from the clubhouse, the old lady was alive and unhurt, so much so that the Navarro
father and son tried hard to have her come along. She could have perished within those five or fewer seconds, as
stated, but the probabilities that she did seem very remote. True, people in the building were also killed but these,
according to Lopez, were mostly refugees who had tried to slip away from it and were shot by Japanese troops. It
was not very likely that Mrs. Joaquin Navarro, Sr. made an attempt to escape. She even made frantic efforts to
dissuade her husband and son from leaving the place and exposing themselves to gun fire.

This determination of Mrs. Angela Joaquin to stay where she was may well give an idea, at the same time, of a
condition of relative safety in the clubhouse at the moment her husband, son, and daughter-in-law left her. It
strongly tends to prove that, as the situation looked to her, the perils of death from staying were not so imminent.
And it lends credence to Mr. Lopez' statement that the collapse of the clubhouse occurred about 40 minutes after
Joaquin Navarro the son was shot in the head and dropped dead, and that it was the collapse that killed Mrs.
Angela Navarro. The Court of Appeals said the interval between Joaquin Navarro's death and the breaking down
of the edifice was "minutes". Even so, it was much longer than five seconds, long enough to warrant the
inference that Mrs. Angela Joaquin was sill alive when her son expired

The Court of Appeals mentioned several causes, besides the collapse of the building, by which Mrs. Navarro
could have been killed. All these are speculative , and the probabilities, in the light of the known facts, are against
them. Dreading Japanese sharpshooters outside as evidenced by her refusal to follow the only remaining living
members of her family, she could not have kept away form protective walls. Besides, the building had been set
on fire trap the refugees inside, and there was no necessity for the Japanese to was their ammunition except
upon those who tried to leave the premises. Nor was Angela Joaquin likely to have been killed by falling beams
because the building was made of concrete and its collapse, more likely than not, was sudden. As to fumes,
these do not cause instantaneous death; certainly not within the brief space of five seconds between her son's
departure and his death.

It will be said that all this is indulging in inferences that are not conclusive. Section 69(ii) of Rule 123 does not
require that the inference necessary to exclude the presumption therein provided be certain. It is the "particular
circumstances from which it (survivorship) can be inferred" that are required to be certain as tested by the rules
of evidence. In speaking of inference the rule can not mean beyond doubt, for "inference is never certainty, but if
may be plain enough to justify a finding of fact." (In re Bohenko's Estate, 4 N.Y.S. 2nd. 427, citing Tortora vs.
State of New York, 269 N.Y. 199 N.E. 44; Hart vs. Hudson River Bridge Co., 80 N.Y.). 622.) As the California
courts have said, it is enough that "the circumstances by which it is sought to prove the survivorship must be
such as are competent and sufficient when tested by the general rules of evidence in civil cases." (In re Wallace's
Estate, supra.) "Juries must often reason," says one author, "according to probabilities, drawing an inference that
the main fact in issue existed from collateral facts not directly proving, but strongly tending to prove, its existence.
The vital question in such cases is the cogency of the proof afforded by the secondary facts. How likely,
according to experience, is the existence of the primary fact if certain secondary facts exist?" (1 Moore on Facts,
Sec. 596.) The same author tells us of a case where "a jury was justified in drawing the inference that the person
who was caught firing a shot at an animal trespassing on his land was the person who fired a shot about an hour
before at the same animal also trespassing." That conclusion was not airtight, but rational. In fact, the
circumstances in the illustration leave greater room for another possibility than do the facts of the case at hand.

In conclusion the presumption that Angela Joaquin de Navarro died before her son is based purely on surmises,
speculations, or conjectures without any sure foundation in the evidence. the opposite theory — that the mother
outlived her son — is deduced from established facts which, weighed by common experience, engender the
inference as a very strong probability. Gauged by the doctrine of preponderance of evidence by, which civil cases
are decided, this inference ought to prevail. It can not be defeated as in an instance, cited by Lord Chief Justice
Kenyon, "bordering on the ridiculous, where in an action on the game laws it was suggested that the gun with
which the defendant fired was not charged with shot, but that the bird might have died in consequence of the
fright." (1 Moore on Facts, 63, citing Wilkinson vs. Payne, 4 T. R. 468.)

It is said that part of the decision of the Court of Appeals which the appellant impugns, and which has been
discussed, involves findings of fact which can not be disturbed. The point is not, in our judgment, well considered.
The particular circumstances from which the parties and the Court of Appeals drew conclusions are, as above
seen, undisputed, and this being the case, the correctness or incorrectness of those conclusions raises a
question of law, not of fact, which the Supreme Court has jurisdiction to look into. As was said in 1 Moran
Commentaries on the Rules of ?Court, 3rd Ed. 856, 857, "Undisputed evidence is one thing, and contradicted
evidence is another. An incredible witness does not cease to be such because he is not impeached or
contradicted. But when the evidence is purely documentary, the authenticity of which is not questioned and the
only issue is the construction to be placed thereon, or where a case is submitted upon an agreement of facts, or
where all the facts are stated in the judgment and the issue is the correctness of the conclusions drawn
therefrom, the question is one of law which may be reviewed by the Supreme Court."

The question of whether upon given facts the operation of the statutory presumption is to be invoked is a
question of law.

The prohibition against intermeddling with decisions on questions of evidence refers to decisions supported by
substantial evidence. By substantial evidence is meant real evidence or at least evidence about which
reasonable men may disagree. Findings grounded entirely on speculations, surmises, or conjectures come within
the exception to the general rule.

We are constrained to reverse the decision under review, and hold that the distribution of the decedents' estates
should be made in accordance with the decision of the trial court. This result precludes the necessity of passing
upon the question of "reserva troncal" which was put forward on the hypothetical theory that Mrs. Joaquin
Navarro's death preceded that of her son. Without costs.

Feria, Pablo, Bengzon, Montemayor, Reyes, Jugo, Bautista Angelo and Labrador, JJ., concur.
ARTICLE 44
105 Phil. 1081

[ G.R. No. L-11973, June 30, 1959 ]

FELIPE M. ROLDAN, PLAINTIFF AND APPELLANT, VS. PHILIPPINE VETERANS BOARD, ET AL.,
DEFENDANTS AND APPELLEES.

DECISION

MONTEMAYOR, J.:
Plaintiff Roldan is appealing the decision of the Court of First Instance of Manila, dismissing his complaint on the
ground that the action brought against the members of the Philippine Veterans Board, which was a mere agency
of the government, was in effect a suit against the state and that it was done without its consent.

The facts in this case are not controverted. Roldan was a first grade Civil Service eligible. On March 26, 1953,
he was appointed clerk in the Philippine Veterans Board with compensation at the rate of P2,160 a year, and he
entered upon the performance of his duties. Defendant Antonio F. Garcia, acting Administrative Officer of
the Philippine Veterans Board of which he was a member and signing for the Chairman, in a letter dated
March 10, 1954 addressed to Roldan, among other things, said:

"In this connection, attention may be invited to the provision of section 2 of Act 2589 and the Cabinet
Resolution dated December 23, 1946, reiterating its former policy against the reinstatement in the service of
officers and employees of the Government who have retired under existing retirement Acts and also to the
provision of Sec. 6 of Republic Act 728, which states that '.no person shall be appointed or reinstated in the
service when he is already fifty seven years of age, etc.'

"In view of the foregoing, and as you were already fifty-seven (57) years of age on March 11, 1953, you are
hereby advised that your services in the Board will terminate effective at the close of business on March 25,
1954."
So, Roldan was separated from the service on March 25, 1954 and in his place Juan Domingo was appointed.
Roldan initiated Quo Warranto proceedings against Domingo in Civil Case No. 25603 of the
CFI of Manila. The trial court in said case decided in favor of Roldan, declaring his ouster to have
been illegal. The dispositive part of the decision reads thus:

"IN VIEW WHEREOF, granted; judgment is rendered declaring Juan Domingo not entitled to said office, and
declaring plaintiff, Felipe M. Roldan, as the person legally entitled and with authority to exercise the same; and
the Court orders that plaintiff be restored to said position. Costs against defendant."
Said decision became final and was executed resulting in the reinstatement of Roldan to his former position on
September 24, 1955.

For the period of about 18 months that he was out of the service due to his separation therefrom on March 25,
1954, Roldan filed the present action against the Philippine Veterans Board and its five members to recover his
back wages during said period plus moral damages in the amount of P5,000.00
including P600.00 for attorneys fees. The trial court, through Judge Luis B. Reyes, dismissed the complaint
on the ground that Republic Act No. 65 creating the Philippine Veterans Board made said Board a mere agency
of the Government to carry out the purposes of said Act No. 65; that the salaries of the employees of said
Board, like that of the plaintiff, were appropriated every year by law and that the salary corresponding to the
position of Roldan for the period from March 26, 1954, when separated from the service, until September
24,1955, when he was reinstated, had already been paid to Juan N. Domingo, the defendant over whom
he won in the Quo Warranto Proceeding; and that neither the Philippine Veterans Board nor its members can
provide for the payment of Roldan's back wages, having no power to do so under the law, Congress being the
only body that can make the appropriation. In support of its ruling the trial court cited the case of Metropolitan
Transportation Service (Metran) vs. Paredes, 79 Phil., 819.

After a careful study of the case we agree with the trial court that the ruling laid down in the case of Metropolitan
Transportation Service (Metran) vs. Paredes, supra, is directly applicable. In that case the Metran was created
by an Executive Order shortly after liberation in. order to provide transportation service for the government and
its employees. It would appear that as a result of a collision resulting in damages^ action was brought against it
to recover damages. This Court held that the Metran was a mere office or agency of the government,
unincorporated and possessing no juridical personality under the law, incapable of suing or being sued and that
a claim against it would in effect be a suit against the government, which suit may not prosper without the
government's consent. In the case of Metran, the latter was a mere agency of the government operating under
the Bureau of Public Works. In the present case, the Philippine Veterans Board was created and functioned
under the Department of National Defense. It is also a mere agency of the government. It is not a
body corporate and politic in deed and in law, incapable of suing or being sued.

Appellant contends that the Philippine Veterans Board is a juridical entity within the meaning of Article 44 of the
Civil Code, which reads as follows:

"Art. 44. The following are juridical persons:

(1) The state and its political subdivisions;

(2) Other corporation, institutions and entities for public interest or purpose, created by law; their personality
begins as soon as they have been constituted according to law;

(3) Corporations, partnerships and associations for private interest or purpose to which the law grants a juridical
personality, separate and distinct from that of each shareholder, partner or member."
Counsel for the appellant merely quotes the above-reproduced article without giving reasons why the
Philippine Veterans Board is included in its provisions. A juridical person is a "being of legal existence,
susceptible of rights and obligations, or of being subject of juridical relations" (2 Sanchez Roman, p. 119,
quoted in Padilla's Civil Code Annotated, Vol. 1, 94, 1956 Ed.).

It is clear that the Philippine Veterans Board which was created under Section 7 of Republic Act No. 65 under
the Department of National Defense to carry into effect the purpose of said act and to take charge of
effectuating the duties assigned to it by law, which Board is composed of a chairman and four other members
to be appointed by the President with the consent of the Commission on Appointment from among veterans of
the Philippine Army and of recognized or deserving guerrilla organizations, which members are entitled to per
diems of P15 each for every meeting actually attended, may not be considered a juridical person within the
meaning of the law, capable of being sued, especially for the recovery of back salaries, which salaries are
appropriated only by Congress. So, a suit like the present one against the Board is in reality an action against
the government itself.

In the case of Syquia vs. Almeda Lopez, et al., (84 Phil., 312; 47 Off. Gaz., 665), we held that a suit against
an officer of a government by a private citizen which would result in a charge against or financial liability to
the government must be regarded as a suit against the government itself, and it cannot prosper or be entertained
by the Court except with the consent of said government. In the present case, a judgment in favor of Roldan for
the payment of his back salaries for the period of 18 months when he was out of the service cannot be a
charge against the Philippine Veterans Board or against its members for the reason that the board member
acting as chairman in effecting the separation of Roldan from the service, assuming the same to be illegal, acted
officially and in the name of the government. Naturally, any judgment in favor of Roldan would mean a charge to
or a liability against the Philippine Government.

In view of the foregoing, the decision appealed from is hereby affirmed, with costs against appellant.

Paras, C. J., Bengzon, Padilla, Bautista Angelo, Concepcion, Endencia, and Barrera, JJ., concur.
CITIZENSHIP AND DOMICILE
Republic of the Philippines
SUPREME COURT
Manila

EN BANC

G.R. No. 119976 September 18, 1995

IMELDA ROMUALDEZ-MARCOS, petitioner,


vs.
COMMISSION ON ELECTIONS and CIRILO ROY MONTEJO, respondents.

KAPUNAN, J.:

A constitutional provision should be construed as to give it effective operation and suppress the mischief at which
it is aimed.1 The 1987 Constitution mandates that an aspirant for election to the House of Representatives be "a
registered voter in the district in which he shall be elected, and a resident thereof for a period of not less than one
year immediately preceding the election."2 The mischief which this provision — reproduced verbatim from the
1973 Constitution — seeks to prevent is the possibility of a "stranger or newcomer unacquainted with the
conditions and needs of a community and not identified with the latter, from an elective office to serve that
community."3

Petitioner Imelda Romualdez-Marcos filed her Certificate of Candidacy for the position of Representative of the
First District of Leyte with the Provincial Election Supervisor on March 8, 1995, providing the following information
in item no. 8:4

RESIDENCE IN THE CONSTITUENCY WHERE I SEEK TO BE ELECTED IMMEDIATELY


PRECEDING THE ELECTION: __________ Years and seven Months.

On March 23, 1995, private respondent Cirilo Roy Montejo, the incumbent Representative of the First District of
Leyte and a candidate for the same position, filed a "Petition for Cancellation and Disqualification"5 with the
Commission on Elections alleging that petitioner did not meet the constitutional requirement for residency. In his
petition, private respondent contended that Mrs. Marcos lacked the Constitution's one year residency
requirement for candidates for the House of Representatives on the evidence of declarations made by her in
Voter Registration Record 94-No. 33497726 and in her Certificate of Candidacy. He prayed that "an order be
issued declaring (petitioner) disqualified and canceling the certificate of candidacy." 7

On March 29, 1995, petitioner filed an Amended/Corrected Certificate of Candidacy, changing the entry "seven"
months to "since childhood" in item no. 8 of the amended certificate. 8 On the same day, the Provincial Election
Supervisor of Leyte informed petitioner that:

[T]his office cannot receive or accept the aforementioned Certificate of Candidacy on the ground
that it is filed out of time, the deadline for the filing of the same having already lapsed on March
20, 1995. The Corrected/Amended Certificate of Candidacy should have been filed on or before
the March 20, 1995 deadline.9

Consequently, petitioner filed the Amended/Corrected Certificate of Candidacy with the COMELEC's Head Office
in Intramuros, Manila on
March 31, 1995. Her Answer to private respondent's petition in SPA No. 95-009 was likewise filed with the head
office on the same day. In said Answer, petitioner averred that the entry of the word "seven" in her original
Certificate of Candidacy was the result of an "honest misinterpretation" 10 which she sought to rectify by adding
the words "since childhood" in her Amended/Corrected Certificate of Candidacy and that "she has always
maintained Tacloban City as her domicile or residence. 11 Impugning respondent's motive in filing the petition
seeking her disqualification, she noted that:

When respondent (petitioner herein) announced that she was intending to register as a voter in
Tacloban City and run for Congress in the First District of Leyte, petitioner immediately opposed
her intended registration by writing a letter stating that "she is not a resident of said city but of
Barangay Olot, Tolosa, Leyte. After respondent had registered as a voter in Tolosa following
completion of her six month actual residence therein, petitioner filed a petition with the COMELEC
to transfer the town of Tolosa from the First District to the Second District and pursued such a
move up to the Supreme Court, his purpose being to remove respondent as petitioner's opponent
in the congressional election in the First District. He also filed a bill, along with other Leyte
Congressmen, seeking the creation of another legislative district to remove the town of Tolosa out
of the First District, to achieve his purpose. However, such bill did not pass the Senate. Having
failed on such moves, petitioner now filed the instant petition for the same objective, as it is
obvious that he is afraid to submit along with respondent for the judgment and verdict of the
electorate of the First District of Leyte in an honest, orderly, peaceful, free and clean elections on
May 8, 1995. 12

On April 24, 1995, the Second Division of the Commission on Elections (COMELEC), by a vote of 2 to 1, 13 came
up with a Resolution 1) finding private respondent's Petition for Disqualification in SPA 95-009 meritorious; 2)
striking off petitioner's Corrected/Amended Certificate of Candidacy of March 31, 1995; and 3) canceling her
original Certificate of Candidacy. 14 Dealing with two primary issues, namely, the validity of amending the original
Certificate of Candidacy after the lapse of the deadline for filing certificates of candidacy, and petitioner's
compliance with the one year residency requirement, the Second Division held:

Respondent raised the affirmative defense in her Answer that the printed word "Seven" (months)
was a result of an "honest misinterpretation or honest mistake" on her part and, therefore, an
amendment should subsequently be allowed. She averred that she thought that what was asked
was her "actual and physical" presence in Tolosa and not residence of origin or domicile in the
First Legislative District, to which she could have responded "since childhood." In an
accompanying affidavit, she stated that her domicile is Tacloban City, a component of the First
District, to which she always intended to return whenever absent and which she has never
abandoned. Furthermore, in her memorandum, she tried to discredit petitioner's theory of
disqualification by alleging that she has been a resident of the First Legislative District of Leyte
since childhood, although she only became a resident of the Municipality of Tolosa for seven
months. She asserts that she has always been a resident of Tacloban City, a component of the
First District, before coming to the Municipality of Tolosa.

Along this point, it is interesting to note that prior to her registration in Tolosa, respondent
announced that she would be registering in Tacloban City so that she can be a candidate for the
District. However, this intention was rebuffed when petitioner wrote the Election Officer of
Tacloban not to allow respondent since she is a resident of Tolosa and not Tacloban. She never
disputed this claim and instead implicitly acceded to it by registering in Tolosa.

This incident belies respondent's claim of "honest misinterpretation or honest mistake." Besides,
the Certificate of Candidacy only asks for RESIDENCE. Since on the basis of her Answer, she
was quite aware of "residence of origin" which she interprets to be Tacloban City, it is curious why
she did not cite Tacloban City in her Certificate of Candidacy. Her explanation that she thought
what was asked was her actual and physical presence in Tolosa is not easy to believe because
there is none in the question that insinuates about Tolosa. In fact, item no. 8 in the Certificate of
Candidacy speaks clearly of "Residency in the CONSTITUENCY where I seek to be elected
immediately preceding the election." Thus, the explanation of respondent fails to be persuasive.

From the foregoing, respondent's defense of an honest mistake or misinterpretation, therefore, is


devoid of merit.

To further buttress respondent's contention that an amendment may be made, she cited the case
of Alialy v. COMELEC (2 SCRA 957). The reliance of respondent on the case of Alialy is
misplaced. The case only applies to the "inconsequential deviations which cannot affect the result
of the election, or deviations from provisions intended primarily to secure timely and orderly
conduct of elections." The Supreme Court in that case considered the amendment only as a
matter of form. But in the instant case, the amendment cannot be considered as a matter of form
or an inconsequential deviation. The change in the number of years of residence in the place
where respondent seeks to be elected is a substantial matter which determines her qualification
as a candidacy, specially those intended to suppress, accurate material representation in the
original certificate which adversely affects the filer. To admit the amended certificate is to
condone the evils brought by the shifting minds of manipulating candidate, of the detriment of the
integrity of the election.

Moreover, to allow respondent to change the seven (7) month period of her residency in order to
prolong it by claiming it was "since childhood" is to allow an untruthfulness to be committed before
this Commission. The arithmetical accuracy of the 7 months residency the respondent indicated
in her certificate of candidacy can be gleaned from her entry in her Voter's Registration Record
accomplished on January 28, 1995 which reflects that she is a resident of Brgy. Olot, Tolosa,
Leyte for 6 months at the time of the said registration (Annex A, Petition). Said accuracy is further
buttressed by her letter to the election officer of San Juan, Metro Manila, dated August 24, 1994,
requesting for the cancellation of her registration in the Permanent List of Voters thereat so that
she can be re-registered or transferred to Brgy. Olot, Tolosa, Leyte. The dates of these three (3)
different documents show the respondent's consistent conviction that she has transferred her
residence to Olot, Tolosa, Leyte from Metro Manila only for such limited period of time, starting in
the last week of August 1994 which on March 8, 1995 will only sum up to 7 months. The
Commission, therefore, cannot be persuaded to believe in the respondent's contention that it was
an error.

xxx xxx xxx

Based on these reasons the Amended/Corrected Certificate of Candidacy cannot be admitted by


this Commission.

xxx xxx xxx

Anent the second issue, and based on the foregoing discussion, it is clear that respondent has
not complied with the one year residency requirement of the Constitution.

In election cases, the term "residence" has always been considered as synonymous with
"domicile" which imports not only the intention to reside in a fixed place but also personal
presence in-that place, coupled with conduct indicative of such intention. Domicile denotes a fixed
permanent residence to which when absent for business or pleasure, or for like reasons, one
intends to return. (Perfecto Faypon vs. Eliseo Quirino, 96 Phil 294; Romualdez vs. RTC-
Tacloban, 226 SCRA 408). In respondent's case, when she returned to the Philippines in 1991,
the residence she chose was not Tacloban but San Juan, Metro Manila. Thus, her animus
revertendi is pointed to Metro Manila and not Tacloban.

This Division is aware that her claim that she has been a resident of the First District since
childhood is nothing more than to give her a color of qualification where she is otherwise
constitutionally disqualified. It cannot hold ground in the face of the facts admitted by the
respondent in her affidavit. Except for the time that she studied and worked for some years after
graduation in Tacloban City, she continuously lived in Manila. In 1959, after her husband was
elected Senator, she lived and resided in San Juan, Metro Manila where she was a registered
voter. In 1965, she lived in San Miguel, Manila where she was again a registered voter. In 1978,
she served as member of the Batasang Pambansa as the representative of the City of Manila and
later on served as the Governor of Metro Manila. She could not have served these positions if she
had not been a resident of the City of Manila. Furthermore, when she filed her certificate of
candidacy for the office of the President in 1992, she claimed to be a resident of San Juan, Metro
Manila. As a matter of fact on August 24, 1994, respondent wrote a letter with the election officer
of San Juan, Metro Manila requesting for the cancellation of her registration in the permanent list
of voters that she may be re-registered or transferred to Barangay Olot, Tolosa, Leyte. These
facts manifest that she could not have been a resident of Tacloban City since childhood up to the
time she filed her certificate of candidacy because she became a resident of many places,
including Metro Manila. This debunks her claim that prior to her residence in Tolosa, Leyte, she
was a resident of the First Legislative District of Leyte since childhood.

In this case, respondent's conduct reveals her lack of intention to make Tacloban her domicile.
She registered as a voter in different places and on several occasions declared that she was a
resident of Manila. Although she spent her school days in Tacloban, she is considered to have
abandoned such place when she chose to stay and reside in other different places. In the case
of Romualdez vs. RTC (226 SCRA 408) the Court explained how one acquires a new domicile by
choice. There must concur: (1) residence or bodily presence in the new locality; (2) intention to
remain there; and (3) intention to abandon the old domicile. In other words there must basically
be animus manendi with animus non revertendi. When respondent chose to stay in Ilocos and
later on in Manila, coupled with her intention to stay there by registering as a voter there and
expressly declaring that she is a resident of that place, she is deemed to have abandoned
Tacloban City, where she spent her childhood and school days, as her place of domicile.

Pure intention to reside in that place is not sufficient, there must likewise be conduct indicative of
such intention. Respondent's statements to the effect that she has always intended to return to
Tacloban, without the accompanying conduct to prove that intention, is not conclusive of her
choice of residence. Respondent has not presented any evidence to show that her conduct, one
year prior the election, showed intention to reside in Tacloban. Worse, what was evident was that
prior to her residence in Tolosa, she had been a resident of Manila.
It is evident from these circumstances that she was not a resident of the First District of Leyte
"since childhood."

To further support the assertion that she could have not been a resident of the First District of
Leyte for more than one year, petitioner correctly pointed out that on January 28, 1995
respondent registered as a voter at precinct No. 18-A of Olot, Tolosa, Leyte. In doing so, she
placed in her Voter Registration Record that she resided in the municipality of Tolosa for a period
of six months. This may be inconsequential as argued by the respondent since it refers only to
her residence in Tolosa, Leyte. But her failure to prove that she was a resident of the First District
of Leyte prior to her residence in Tolosa leaves nothing but a convincing proof that she had been
a resident of the district for six months only. 15

In a Resolution promulgated a day before the May 8, 1995 elections, the COMELEC en banc denied petitioner's
Motion for Reconsideration 16 of the April 24, 1995 Resolution declaring her not qualified to run for the position of
Member of the House of Representatives for the First Legislative District of Leyte. 17 The Resolution tersely
stated:

After deliberating on the Motion for Reconsideration, the Commission RESOLVED to DENY it, no
new substantial matters having been raised therein to warrant re-examination of the resolution
granting the petition for disqualification. 18

On May 11, 1995, the COMELEC issued a Resolution allowing petitioner's proclamation should the results of the
canvass show that she obtained the highest number of votes in the congressional elections in the First District of
Leyte. On the same day, however, the COMELEC reversed itself and issued a second Resolution directing that
the proclamation of petitioner be suspended in the event that she obtains the highest number of votes. 19

In a Supplemental Petition dated 25 May 1995, petitioner averred that she was the overwhelming winner of the
elections for the congressional seat in the First District of Leyte held May 8, 1995 based on the canvass
completed by the Provincial Board of Canvassers on May 14, 1995. Petitioner alleged that the canvass showed
that she obtained a total of 70,471 votes compared to the 36,833 votes received by Respondent Montejo. A copy
of said Certificate of Canvass was annexed to the Supplemental Petition.

On account of the Resolutions disqualifying petitioner from running for the congressional seat of the First District
of Leyte and the public respondent's Resolution suspending her proclamation, petitioner comes to this court for
relief.

Petitioner raises several issues in her Original and Supplemental Petitions. The principal issues may be classified
into two general areas:

I. The issue of Petitioner's qualifications

Whether or not petitioner was a resident, for election purposes, of the First District of Leyte for a
period of one year at the time of the May 9, 1995 elections.

II. The Jurisdictional Issue

a) Prior to the elections

Whether or not the COMELEC properly exercised its jurisdiction in disqualifying petitioner outside
the period mandated by the Omnibus Election Code for disqualification cases under Article 78 of
the said Code.

b) After the Elections

Whether or not the House of Representatives Electoral Tribunal assumed exclusive jurisdiction
over the question of petitioner's qualifications after the May 8, 1995 elections.

I. Petitioner's qualification

A perusal of the Resolution of the COMELEC's Second Division reveals a startling confusion in the application of
settled concepts of "Domicile" and "Residence" in election law. While the COMELEC seems to be in agreement
with the general proposition that for the purposes of election law, residence is synonymous with domicile, the
Resolution reveals a tendency to substitute or mistake the concept of domicile for actual residence, a conception
not intended for the purpose of determining a candidate's qualifications for election to the House of
Representatives as required by the 1987 Constitution. As it were, residence, for the purpose of meeting the
qualification for an elective position, has a settled meaning in our jurisdiction.
Article 50 of the Civil Code decrees that "[f]or the exercise of civil rights and the fulfillment of civil obligations, the
domicile of natural persons is their place of habitual residence." In Ong vs. Republic 20 this court took the concept
of domicile to mean an individual's "permanent home", "a place to which, whenever absent for business or for
pleasure, one intends to return, and depends on facts and circumstances in the sense that they disclose
intent." 21 Based on the foregoing, domicile includes the twin elements of "the fact of residing or physical presence
in a fixed place" and animus manendi, or the intention of returning there permanently.

Residence, in its ordinary conception, implies the factual relationship of an individual to a certain place. It is the
physical presence of a person in a given area, community or country. The essential distinction between residence
and domicile in law is that residence involves the intent to leave when the purpose for which the resident has
taken up his abode ends. One may seek a place for purposes such as pleasure, business, or health. If a person's
intent be to remain, it becomes his domicile; if his intent is to leave as soon as his purpose is established it is
residence. 22 It is thus, quite perfectly normal for an individual to have different residences in various places.
However, a person can only have a single domicile, unless, for various reasons, he successfully abandons his
domicile in favor of another domicile of choice. In Uytengsu vs. Republic, 23 we laid this distinction quite clearly:

There is a difference between domicile and residence. "Residence" is used to indicate a place of
abode, whether permanent or temporary; "domicile" denotes a fixed permanent residence to
which, when absent, one has the intention of returning. A man may have a residence in one place
and a domicile in another. Residence is not domicile, but domicile is residence coupled with the
intention to remain for an unlimited time. A man can have but one domicile for the same purpose
at any time, but he may have numerous places of residence. His place of residence is generally
his place of domicile, but it is not by any means necessarily so since no length of residence
without intention of remaining will constitute domicile.

For political purposes the concepts of residence and domicile are dictated by the peculiar criteria of political laws.
As these concepts have evolved in our election law, what has clearly and unequivocally emerged is the fact that
residence for election purposes is used synonymously with domicile.

In Nuval vs. Guray, 24 the Court held that "the term residence. . . is synonymous with domicile which imports not
only intention to reside in a fixed place, but also personal presence in that place, coupled with conduct indicative
of such intention." 25 Larena vs. Teves 26 reiterated the same doctrine in a case involving the qualifications of the
respondent therein to the post of Municipal President of Dumaguete, Negros Oriental. Faypon vs. Quirino, 27 held
that the absence from residence to pursue studies or practice a profession or registration as a voter other than in
the place where one is elected does not constitute loss of residence. 28 So settled is the concept (of domicile) in
our election law that in these and other election law cases, this Court has stated that the mere absence of an
individual from his permanent residence without the intention to abandon it does not result in a loss or change of
domicile.

The deliberations of the 1987 Constitution on the residence qualification for certain elective positions have placed
beyond doubt the principle that when the Constitution speaks of "residence" in election law, it actually means only
"domicile" to wit:

Mr. Nolledo: With respect to Section 5, I remember that in the 1971 Constitutional Convention,
there was an attempt to require residence in the place not less than one year immediately
preceding the day of the elections. So my question is: What is the Committee's concept of
residence of a candidate for the legislature? Is it actual residence or is it the concept of domicile
or constructive residence?

Mr. Davide: Madame President, insofar as the regular members of the National Assembly are
concerned, the proposed section merely provides, among others, "and a resident thereof", that is,
in the district for a period of not less than one year preceding the day of the election. This was in
effect lifted from the 1973 Constitution, the interpretation given to it was domicile. 29

xxx xxx xxx

Mrs. Rosario Braid: The next question is on Section 7, page 2. I think Commissioner Nolledo has
raised the same point that "resident" has been interpreted at times as a matter of intention rather
than actual residence.

Mr. De los Reyes: Domicile.

Ms. Rosario Braid: Yes, So, would the gentleman consider at the proper time to go back to actual
residence rather than mere intention to reside?

Mr. De los Reyes: But we might encounter some difficulty especially considering that a provision
in the Constitution in the Article on Suffrage says that Filipinos living abroad may vote as enacted
by law. So, we have to stick to the original concept that it should be by domicile and not physical
residence. 30

In Co vs. Electoral Tribunal of the House of Representatives, 31 this Court concluded that the framers of the 1987
Constitution obviously adhered to the definition given to the term residence in election law, regarding it as having
the same meaning as domicile. 32

In the light of the principles just discussed, has petitioner Imelda Romualdez Marcos satisfied the residency
requirement mandated by Article VI, Sec. 6 of the 1987 Constitution? Of what significance is the questioned entry
in petitioner's Certificate of Candidacy stating her residence in the First Legislative District of Leyte as seven (7)
months?

It is the fact of residence, not a statement in a certificate of candidacy which ought to be decisive in determining
whether or not and individual has satisfied the constitution's residency qualification requirement. The said
statement becomes material only when there is or appears to be a deliberate attempt to mislead, misinform, or
hide a fact which would otherwise render a candidate ineligible. It would be plainly ridiculous for a candidate to
deliberately and knowingly make a statement in a certificate of candidacy which would lead to his or her
disqualification.

It stands to reason therefore, that petitioner merely committed an honest mistake in jotting the word "seven" in
the space provided for the residency qualification requirement. The circumstances leading to her filing the
questioned entry obviously resulted in the subsequent confusion which prompted petitioner to write down the
period of her actual stay in Tolosa, Leyte instead of her period of residence in the First district, which was "since
childhood" in the space provided. These circumstances and events are amply detailed in the COMELEC's
Second Division's questioned resolution, albeit with a different interpretation. For instance, when herein petitioner
announced that she would be registering in Tacloban City to make her eligible to run in the First District, private
respondent Montejo opposed the same, claiming that petitioner was a resident of Tolosa, not Tacloban City.
Petitioner then registered in her place of actual residence in the First District, which is Tolosa, Leyte, a fact which
she subsequently noted down in her Certificate of Candidacy. A close look at said certificate would reveal the
possible source of the confusion: the entry for residence (Item No. 7) is followed immediately by the entry for
residence in the constituency where a candidate seeks election thus:

7. RESIDENCE (complete Address): Brgy. Olot, Tolosa, Leyte

POST OFFICE ADDRESS FOR ELECTION PURPOSES: Brgy. Olot, Tolosa, Leyte

8. RESIDENCE IN THE CONSTITUENCY WHERE I SEEK TO


BE ELECTED IMMEDIATELY PRECEDING THE ELECTION:_________ Years
and Seven Months.

Having been forced by private respondent to register in her place of actual residence in Leyte instead of
petitioner's claimed domicile, it appears that petitioner had jotted down her period of stay in her legal residence or
domicile. The juxtaposition of entries in Item 7 and Item 8 — the first requiring actual residence and the second
requiring domicile — coupled with the circumstances surrounding petitioner's registration as a voter in Tolosa
obviously led to her writing down an unintended entry for which she could be disqualified. This honest mistake
should not, however, be allowed to negate the fact of residence in the First District if such fact were established
by means more convincing than a mere entry on a piece of paper.

We now proceed to the matter of petitioner's domicile.

In support of its asseveration that petitioner's domicile could not possibly be in the First District of Leyte, the
Second Division of the COMELEC, in its assailed Resolution of April 24,1995 maintains that "except for the time
when (petitioner) studied and worked for some years after graduation in Tacloban City, she continuously lived in
Manila." The Resolution additionally cites certain facts as indicative of the fact that petitioner's domicile ought to
be any place where she lived in the last few decades except Tacloban, Leyte. First, according to the Resolution,
petitioner, in 1959, resided in San Juan, Metro Manila where she was also registered voter. Then, in 1965,
following the election of her husband to the Philippine presidency, she lived in San Miguel, Manila where she as a
voter. In 1978 and thereafter, she served as a member of the Batasang Pambansa and Governor of Metro
Manila. "She could not, have served these positions if she had not been a resident of Metro Manila," the
COMELEC stressed. Here is where the confusion lies.

We have stated, many times in the past, that an individual does not lose his domicile even if he has lived and
maintained residences in different places. Residence, it bears repeating, implies a factual relationship to a given
place for various purposes. The absence from legal residence or domicile to pursue a profession, to study or to
do other things of a temporary or semi-permanent nature does not constitute loss of residence. Thus, the
assertion by the COMELEC that "she could not have been a resident of Tacloban City since childhood up to the
time she filed her certificate of candidacy because she became a resident of many places" flies in the face of
settled jurisprudence in which this Court carefully made distinctions between (actual) residence and domicile for
election law purposes. In Larena vs. Teves, 33 supra, we stressed:

[T]his court is of the opinion and so holds that a person who has his own house wherein he lives
with his family in a municipality without having ever had the intention of abandoning it, and without
having lived either alone or with his family in another municipality, has his residence in the former
municipality, notwithstanding his having registered as an elector in the other municipality in
question and having been a candidate for various insular and provincial positions, stating every
time that he is a resident of the latter municipality.

More significantly, in Faypon vs. Quirino, 34 We explained that:

A citizen may leave the place of his birth to look for "greener pastures," as the saying goes, to
improve his lot, and that, of course includes study in other places, practice of his avocation, or
engaging in business. When an election is to be held, the citizen who left his birthplace to improve
his lot may desire to return to his native town to cast his ballot but for professional or business
reasons, or for any other reason, he may not absent himself from his professional or business
activities; so there he registers himself as voter as he has the qualifications to be one and is not
willing to give up or lose the opportunity to choose the officials who are to run the government
especially in national elections. Despite such registration, the animus revertendi to his home, to
his domicile or residence of origin has not forsaken him. This may be the explanation why the
registration of a voter in a place other than his residence of origin has not been deemed sufficient
to constitute abandonment or loss of such residence. It finds justification in the natural desire and
longing of every person to return to his place of birth. This strong feeling of attachment to the
place of one's birth must be overcome by positive proof of abandonment for another.

From the foregoing, it can be concluded that in its above-cited statements supporting its proposition that
petitioner was ineligible to run for the position of Representative of the First District of Leyte, the COMELEC was
obviously referring to petitioner's various places of (actual) residence, not her domicile. In doing so, it not only
ignored settled jurisprudence on residence in election law and the deliberations of the constitutional commission
but also the provisions of the Omnibus Election Code (B.P. 881). 35

What is undeniable, however, are the following set of facts which establish the fact of petitioner's domicile, which
we lift verbatim from the COMELEC's Second Division's assailed Resolution: 36

In or about 1938 when respondent was a little over 8 years old, she established her domicile in
Tacloban, Leyte (Tacloban City). She studied in the Holy Infant Academy in Tacloban from 1938
to 1949 when she graduated from high school. She pursued her college studies in St. Paul's
College, now Divine Word University in Tacloban, where she earned her degree in Education.
Thereafter, she taught in the Leyte Chinese School, still in Tacloban City. In 1952 she went to
Manila to work with her cousin, the late speaker Daniel Z. Romualdez in his office in the House of
Representatives. In 1954, she married ex-President Ferdinand E. Marcos when he was still a
congressman of Ilocos Norte and registered there as a voter. When her husband was elected
Senator of the Republic in 1959, she and her husband lived together in San Juan, Rizal where
she registered as a voter. In 1965, when her husband was elected President of the Republic of
the Philippines, she lived with him in Malacanang Palace and registered as a voter in San Miguel,
Manila.

[I]n February 1986 (she claimed that) she and her family were abducted and kidnapped to
Honolulu, Hawaii. In November 1991, she came home to Manila. In 1992, respondent ran for
election as President of the Philippines and filed her Certificate of Candidacy wherein she
indicated that she is a resident and registered voter of San Juan, Metro Manila.

Applying the principles discussed to the facts found by COMELEC, what is inescapable is that petitioner held
various residences for different purposes during the last four decades. None of these purposes unequivocally
point to an intention to abandon her domicile of origin in Tacloban, Leyte. Moreover, while petitioner was born in
Manila, as a minor she naturally followed the domicile of her parents. She grew up in Tacloban, reached her
adulthood there and eventually established residence in different parts of the country for various reasons. Even
during her husband's presidency, at the height of the Marcos Regime's powers, petitioner kept her close ties to
her domicile of origin by establishing residences in Tacloban, celebrating her birthdays and other important
personal milestones in her home province, instituting well-publicized projects for the benefit of her province and
hometown, and establishing a political power base where her siblings and close relatives held positions of power
either through the ballot or by appointment, always with either her influence or consent. These well-publicized ties
to her domicile of origin are part of the history and lore of the quarter century of Marcos power in our country.
Either they were entirely ignored in the COMELEC'S Resolutions, or the majority of the COMELEC did not know
what the rest of the country always knew: the fact of petitioner's domicile in Tacloban, Leyte.
Private respondent in his Comment, contends that Tacloban was not petitioner's domicile of origin because she
did not live there until she was eight years old. He avers that after leaving the place in 1952, she "abandoned her
residency (sic) therein for many years and . . . (could not) re-establish her domicile in said place by merely
expressing her intention to live there again." We do not agree.

First, minor follows the domicile of his parents. As domicile, once acquired is retained until a new one is gained, it
follows that in spite of the fact of petitioner's being born in Manila, Tacloban, Leyte was her domicile of origin by
operation of law. This domicile was not established only when her father brought his family back to Leyte contrary
to private respondent's averments.

Second, domicile of origin is not easily lost. To successfully effect a change of domicile, one must demonstrate: 37

1. An actual removal or an actual change of domicile;

2. A bona fide intention of abandoning the former place of residence and establishing a new one;
and

3. Acts which correspond with the purpose.

In the absence of clear and positive proof based on these criteria, the residence of origin should be deemed to
continue. Only with evidence showing concurrence of all three requirements can the presumption of continuity or
residence be rebutted, for a change of residence requires an actual and deliberate abandonment, and one
cannot have two legal residences at the same time. 38 In the case at bench, the evidence adduced by private
respondent plainly lacks the degree of persuasiveness required to convince this court that an abandonment of
domicile of origin in favor of a domicile of choice indeed occurred. To effect an abandonment requires the
voluntary act of relinquishing petitioner's former domicile with an intent to supplant the former domicile with one of
her own choosing (domicilium voluntarium).

In this connection, it cannot be correctly argued that petitioner lost her domicile of origin by operation of law as a
result of her marriage to the late President Ferdinand E. Marcos in 1952. For there is a clearly established
distinction between the Civil Code concepts of "domicile" and "residence." 39 The presumption that the wife
automatically gains the husband's domicile by operation of law upon marriage cannot be inferred from the use of
the term "residence" in Article 110 of the Civil Code because the Civil Code is one area where the two concepts
are well delineated. Dr. Arturo Tolentino, writing on this specific area explains:

In the Civil Code, there is an obvious difference between domicile and residence. Both terms
imply relations between a person and a place; but in residence, the relation is one of fact while in
domicile it is legal or juridical, independent of the necessity of physical presence. 40

Article 110 of the Civil Code provides:

Art. 110. — The husband shall fix the residence of the family. But the court may exempt the wife
from living with the husband if he should live abroad unless in the service of the Republic.

A survey of jurisprudence relating to Article 110 or to the concepts of domicile or residence as they affect the
female spouse upon marriage yields nothing which would suggest that the female spouse automatically loses her
domicile of origin in favor of the husband's choice of residence upon marriage.

Article 110 is a virtual restatement of Article 58 of the Spanish Civil Code of 1889 which states:

La mujer esta obligada a seguir a su marido donde quiera que fije su residencia. Los Tribunales,
sin embargo, podran con justa causa eximirla de esta obligacion cuando el marido transende su
residencia a ultramar o' a pais extranjero.

Note the use of the phrase "donde quiera su fije de residencia" in the aforequoted article, which means wherever
(the husband) wishes to establish residence. This part of the article clearly contemplates only actual residence
because it refers to a positive act of fixing a family home or residence. Moreover, this interpretation is further
strengthened by the phrase "cuando el marido translade su residencia" in the same provision which means,
"when the husband shall transfer his residence," referring to another positive act of relocating the family to
another home or place of actual residence. The article obviously cannot be understood to refer to domicile which
is a fixed,
fairly-permanent concept when it plainly connotes the possibility of transferring from one place to another not only
once, but as often as the husband may deem fit to move his family, a circumstance more consistent with the
concept of actual residence.
The right of the husband to fix the actual residence is in harmony with the intention of the law to strengthen and
unify the family, recognizing the fact that the husband and the wife bring into the marriage different domiciles (of
origin). This difference could, for the sake of family unity, be reconciled only by allowing the husband to fix a
single place of actual residence.

Very significantly, Article 110 of the Civil Code is found under Title V under the heading: RIGHTS AND
OBLIGATIONS BETWEEN HUSBAND AND WIFE. Immediately preceding Article 110 is Article 109 which
obliges the husband and wife to live together, thus:

Art. 109. — The husband and wife are obligated to live together, observe mutual respect and
fidelity and render mutual help and support.

The duty to live together can only be fulfilled if the husband and wife are physically together. This takes into
account the situations where the couple has many residences (as in the case of the petitioner). If the husband
has to stay in or transfer to any one of their residences, the wife should necessarily be with him in order that they
may "live together." Hence, it is illogical to conclude that Art. 110 refers to "domicile" and not to "residence."
Otherwise, we shall be faced with a situation where the wife is left in the domicile while the husband, for
professional or other reasons, stays in one of their (various) residences. As Dr. Tolentino further explains:

Residence and Domicile — Whether the word "residence" as used with reference to particular
matters is synonymous with "domicile" is a question of some difficulty, and the ultimate decision
must be made from a consideration of the purpose and intent with which the word is used.
Sometimes they are used synonymously, at other times they are distinguished from one another.

xxx xxx xxx

Residence in the civil law is a material fact, referring to the physical presence of a person in a
place. A person can have two or more residences, such as a country residence and a city
residence. Residence is acquired by living in place; on the other hand, domicile can exist without
actually living in the place. The important thing for domicile is that, once residence has been
established in one place, there be an intention to stay there permanently, even if residence is also
established in some other
place. 41

In fact, even the matter of a common residence between the husband and the wife during the marriage is not an
iron-clad principle; In cases applying the Civil Code on the question of a common matrimonial residence, our
jurisprudence has recognized certain situations 42 where the spouses could not be compelled to live with each
other such that the wife is either allowed to maintain a residence different from that of her husband or, for
obviously practical reasons, revert to her original domicile (apart from being allowed to opt for a new one). In De
la Vina vs. Villareal 43 this Court held that "[a] married woman may acquire a residence or domicile separate from
that of her husband during the existence of the marriage where the husband has given cause for divorce." 44 Note
that the Court allowed the wife either to obtain new residence or to choose a new domicile in such an event. In
instances where the wife actually opts, .under the Civil Code, to live separately from her husband either by taking
new residence or reverting to her domicile of origin, the Court has held that the wife could not be compelled to
live with her husband on pain of contempt. In Arroyo vs. Vasques de Arroyo 45 the Court held that:

Upon examination of the authorities, we are convinced that it is not within the province of the
courts of this country to attempt to compel one of the spouses to cohabit with, and render
conjugal rights to, the other. Of course where the property rights of one of the pair are invaded, an
action for restitution of such rights can be maintained. But we are disinclined to sanction the
doctrine that an order, enforcible (sic) by process of contempt, may be entered to compel the
restitution of the purely personal right of consortium. At best such an order can be effective for no
other purpose than to compel the spouses to live under the same roof; and he experience of
those countries where the courts of justice have assumed to compel the cohabitation of married
people shows that the policy of the practice is extremely questionable. Thus in England, formerly
the Ecclesiastical Court entertained suits for the restitution of conjugal rights at the instance of
either husband or wife; and if the facts were found to warrant it, that court would make a
mandatory decree, enforceable by process of contempt in case of disobedience, requiring the
delinquent party to live with the other and render conjugal rights. Yet this practice was sometimes
criticized even by the judges who felt bound to enforce such orders, and in Weldon v. Weldon (9
P.D. 52), decided in 1883, Sir James Hannen, President in the Probate, Divorce and Admiralty
Division of the High Court of Justice, expressed his regret that the English law on the subject was
not the same as that which prevailed in Scotland, where a decree of adherence, equivalent to the
decree for the restitution of conjugal rights in England, could be obtained by the injured spouse,
but could not be enforced by imprisonment. Accordingly, in obedience to the growing sentiment
against the practice, the Matrimonial Causes Act (1884) abolished the remedy of imprisonment;
though a decree for the restitution of conjugal rights can still be procured, and in case of
disobedience may serve in appropriate cases as the basis of an order for the periodical payment
of a stipend in the character of alimony.

In the voluminous jurisprudence of the United States, only one court, so far as we can discover,
has ever attempted to make a preemptory order requiring one of the spouses to live with the
other; and that was in a case where a wife was ordered to follow and live with her husband, who
had changed his domicile to the City of New Orleans. The decision referred to (Bahn v. Darby, 36
La. Ann., 70) was based on a provision of the Civil Code of Louisiana similar to article 56 of the
Spanish Civil Code. It was decided many years ago, and the doctrine evidently has not been
fruitful even in the State of Louisiana. In other states of the American Union the idea of enforcing
cohabitation by process of contempt is rejected. (21 Cyc., 1148).

In a decision of January 2, 1909, the Supreme Court of Spain appears to have affirmed an order
of the Audiencia Territorial de Valladolid requiring a wife to return to the marital domicile, and in
the alternative, upon her failure to do so, to make a particular disposition of certain money and
effects then in her possession and to deliver to her husband, as administrator of the ganancial
property, all income, rents, and interest which might accrue to her from the property which she
had brought to the marriage. (113 Jur. Civ., pp. 1, 11) But it does not appear that this order for the
return of the wife to the marital domicile was sanctioned by any other penalty than the
consequences that would be visited upon her in respect to the use and control of her property;
and it does not appear that her disobedience to that order would necessarily have been followed
by imprisonment for contempt.

Parenthetically when Petitioner was married to then Congressman Marcos, in 1954, petitioner was obliged — by
virtue of Article 110 of the Civil Code — to follow her husband's actual place of residence fixed by him. The
problem here is that at that time, Mr. Marcos had several places of residence, among which were San Juan, Rizal
and Batac, Ilocos Norte. There is no showing which of these places Mr. Marcos did fix as his family's residence.
But assuming that Mr. Marcos had fixed any of these places as the conjugal residence, what petitioner gained
upon marriage was actual residence. She did not lose her domicile of origin.

On the other hand, the common law concept of "matrimonial domicile" appears to have been incorporated, as a
result of our jurisprudential experiences after the drafting of the Civil Code of 1950, into the New Family Code. To
underscore the difference between the intentions of the Civil Code and the Family Code drafters, the term
residence has been supplanted by the term domicile in an entirely new provision (Art. 69) distinctly different in
meaning and spirit from that found in Article 110. The provision recognizes revolutionary changes in the concept
of women's rights in the intervening years by making the choice of domicile a product of mutual agreement
between the spouses. 46

Without as much belaboring the point, the term residence may mean one thing in civil law (or under the Civil
Code) and quite another thing in political law. What stands clear is that insofar as the Civil Code is concerned-
affecting the rights and obligations of husband and wife — the term residence should only be interpreted to mean
"actual residence." The inescapable conclusion derived from this unambiguous civil law delineation therefore, is
that when petitioner married the former President in 1954, she kept her domicile of origin and merely gained a
new home, not a domicilium necessarium.

Even assuming for the sake of argument that petitioner gained a new "domicile" after her marriage and only
acquired a right to choose a new one after her husband died, petitioner's acts following her return to the country
clearly indicate that she not only impliedly but expressly chose her domicile of origin (assuming this was lost by
operation of law) as her domicile. This "choice" was unequivocally expressed in her letters to the Chairman of the
PCGG when petitioner sought the PCGG's permission to "rehabilitate (our) ancestral house in Tacloban and
Farm in Olot, Leyte. . . to make them livable for the Marcos family to have a home in our
homeland." 47 Furthermore, petitioner obtained her residence certificate in 1992 in Tacloban, Leyte, while living in
her brother's house, an act which supports the domiciliary intention clearly manifested in her letters to the PCGG
Chairman. She could not have gone straight to her home in San Juan, as it was in a state of disrepair, having
been previously looted by vandals. Her "homes" and "residences" following her arrival in various parts of Metro
Manila merely qualified as temporary or "actual residences," not domicile. Moreover, and proceeding from our
discussion pointing out specific situations where the female spouse either reverts to her domicile of origin or
chooses a new one during the subsistence of the marriage, it would be highly illogical for us to assume that she
cannot regain her original domicile upon the death of her husband absent a positive act of selecting a new one
where situations exist within the subsistence of the marriage itself where the wife gains a domicile different from
her husband.

In the light of all the principles relating to residence and domicile enunciated by this court up to this point, we are
persuaded that the facts established by the parties weigh heavily in favor of a conclusion supporting petitioner's
claim of legal residence or domicile in the First District of Leyte.

II. The jurisdictional issue


Petitioner alleges that the jurisdiction of the COMELEC had already lapsed considering that the assailed
resolutions were rendered on April 24, 1995, fourteen (14) days before the election in violation of Section 78 of
the Omnibus Election Code. 48 Moreover, petitioner contends that it is the House of Representatives Electoral
Tribunal and not the COMELEC which has jurisdiction over the election of members of the House of
Representatives in accordance with Article VI Sec. 17 of the Constitution. This is untenable.

It is a settled doctrine that a statute requiring rendition of judgment within a specified time is generally construed
to be merely directory, 49 "so that non-compliance with them does not invalidate the judgment on the theory that if
the statute had intended such result it would have clearly indicated it." 50 The difference between a mandatory and
a directory provision is often made on grounds of necessity. Adopting the same view held by several American
authorities, this court in Marcelino vs. Cruz held that: 51

The difference between a mandatory and directory provision is often determined on grounds of
expediency, the reason being that less injury results to the general public by disregarding than
enforcing the letter of the law.

In Trapp v. Mc Cormick, a case calling for the interpretation of a statute containing a limitation of
thirty (30) days within which a decree may be entered without the consent of counsel, it was held
that "the statutory provisions which may be thus departed from with impunity, without affecting the
validity of statutory proceedings, are usually those which relate to the mode or time of doing that
which is essential to effect the aim and purpose of the Legislature or some incident of the
essential act." Thus, in said case, the statute under examination was construed merely to be
directory.

The mischief in petitioner's contending that the COMELEC should have abstained from rendering a decision after
the period stated in the Omnibus Election Code because it lacked jurisdiction, lies in the fact that our courts and
other quasi-judicial bodies would then refuse to render judgments merely on the ground of having failed to reach
a decision within a given or prescribed period.

In any event, with the enactment of Sections 6 and 7 of R.A. 6646 in relation to Section 78 of B.P. 881, 52 it is
evident that the respondent Commission does not lose jurisdiction to hear and decide a pending disqualification
case under Section 78 of B.P. 881 even after the elections.

As to the House of Representatives Electoral Tribunal's supposed assumption of jurisdiction over the issue of
petitioner's qualifications after the May 8, 1995 elections, suffice it to say that HRET's jurisdiction as the sole
judge of all contests relating to the elections, returns and qualifications of members of Congress begins only after
a candidate has become a member of the House of Representatives. 53 Petitioner not being a member of the
House of Representatives, it is obvious that the HRET at this point has no jurisdiction over the question.

It would be an abdication of many of the ideals enshrined in the 1987 Constitution for us to either to ignore or
deliberately make distinctions in law solely on the basis of the personality of a petitioner in a case. Obviously a
distinction was made on such a ground here. Surely, many established principles of law, even of election laws
were flouted for the sake perpetuating power during the pre-EDSA regime. We renege on these sacred ideals,
including the meaning and spirit of EDSA ourselves bending established principles of principles of law to deny an
individual what he or she justly deserves in law. Moreover, in doing so, we condemn ourselves to repeat the
mistakes of the past.

WHEREFORE, having determined that petitioner possesses the necessary residence qualifications to run for a
seat in the House of Representatives in the First District of Leyte, the COMELEC's questioned Resolutions dated
April 24, May 7, May 11, and May 25, 1995 are hereby SET ASIDE. Respondent COMELEC is hereby directed to
order the Provincial Board of Canvassers to proclaim petitioner as the duly elected Representative of the First
District of Leyte.

SO ORDERED.

Feliciano, J., is on leave.

Separate Opinions

PUNO, J., concurring:


It was Aristotle who taught mankind that things that are alike should be treated alike, while things that are unalike
should be treated unalike in proportion to their unalikeness.1 Like other candidates, petitioner has clearly met the
residence requirement provided by Section 6, Article VI of the Constitution. 2 We cannot disqualify her and treat
her unalike, for the Constitution guarantees equal protection of the law. I proceed from the following factual and
legal propositions:

First. There is no question that petitioner's original domicile is in Tacloban, Leyte. Her parents were domiciled in
Tacloban. Their ancestral house is in Tacloban. They have vast real estate in the place. Petitioner went to school
and thereafter worked there. I consider Tacloban as her initial domicile, both her domicile of origin and her
domicile of choice. Her domicile of origin as it was the domicile of her parents when she was a minor; and her
domicile of choice, as she continued living there even after reaching the age of majority.

Second. There is also no question that in May, 1954, petitioner married the late President Ferdinand E. Marcos.
By contracting marriage, her domicile became subject to change by law, and the right to change it was given by
Article 110 of the Civil Code provides:

Art. 110. The husband shall fix the residence of the family. But the court may exempt the wife
from living with the husband if he should live abroad unless in the service of the
Republic.3 (Emphasis supplied)

In De la Viña v. Villareal and Geopano,4 this Court explained why the domicile of the wife ought to follow
that of the husband. We held: "The reason is founded upon the theoretic identity of person and interest
between the husband and the wife, and the presumption that, from the nature of the relation, the home of
one is the home of the other. It is intended to promote, strengthen, and secure their interests in this
relation, as it ordinarily exists, where union and harmony prevail." 5 In accord with this objective, Article
109 of the Civil Code also obligated the husband and wife "to live together."

Third. The difficult issues start as we determine whether petitioner's marriage to former President Marcos ipso
facto resulted in the loss of her Tacloban domicile. I respectfully submit that her marriage by itself alone did not
cause her to lose her Tacloban domicile. Article 110 of the Civil Code merely gave the husband the right to fix the
domicile of the family. In the exercise of the right, the husband may explicitly choose the prior domicile of his wife,
in which case, the wife's domicile remains unchanged. The husband can also implicitly acquiesce to his wife's
prior domicile even if it is different. So we held in de la Viña,6

. . . . When married women as well as children subject to parental authority live, with the
acquiescence of their husbands or fathers, in a place distinct from where the latter live, they have
their own independent domicile. . . .

It is not, therefore, the mere fact of marriage but the deliberate choice of a different domicile by the
husband that will change the domicile of a wife from what it was prior to their marriage. The domiciliary
decision made by the husband in the exercise of the right conferred by Article 110 of the Civil Code binds
the wife. Any and all acts of a wife during her coverture contrary to the domiciliary choice of the husband
cannot change in any way the domicile legally fixed by the husband. These acts are void not only
because the wife lacks the capacity to choose her domicile but also because they are contrary to law and
public policy.

In the case at bench, it is not disputed that former President Marcos exercised his right to fix the family domicile
and established it in Batac, Ilocos Norte, where he was then the congressman. At that particular point of time and
throughout their married life, petitioner lost her domicile in Tacloban, Leyte. Since petitioner's Batac domicile has
been fixed by operation of law, it was not affected in 1959 when her husband was elected as Senator, when they
lived in San Juan, Rizal and where she registered as a voter. It was not also affected in 1965 when her husband
was elected President, when they lived in Malacañang Palace, and when she registered as a voter in San Miguel,
Manila. Nor was it affected when she served as a member of the Batasang Pambansa, Minister of Human
Settlements and Governor of Metro Manila during the incumbency of her husband as President of the nation.
Under Article 110 of the Civil Code, it was only her husband who could change the family domicile in Batac and
the evidence shows he did not effect any such change. To a large degree, this follows the common law that "a
woman on her marriage loses her own domicile and by operation of law, acquires that of her husband, no matter
where the wife actually lives or what she believes or intends."7

Fourth. The more difficult task is how to interpret the effect of the death on September 28, 1989 of former
President Marcos on petitioner's Batac domicile. The issue is of first impression in our jurisdiction and two (2)
schools of thought contend for acceptance. One is espoused by our distinguished colleague, Mr. Justice Davide,
Jr., heavily relying on American authorities. 8 He echoes the theory that after the husband's death, the wife retains
the last domicile of her husband until she makes an actual change.

I do not subscribe to this submission. The American case law that the wife still retains her dead husband's
domicile is based on ancient common law which we can no longer apply in the Philippine setting today. The
common law identified the domicile of a wife as that of the husband and denied to her the power of acquiring a
domicile of her own separate and apart from him. 9 Legal scholars agree that two (2) reasons support this
common law doctrine. The first reason as pinpointed by the legendary Blackstone is derived from the view that
"the very being or legal existence of the woman is suspended during
the marriage, or at least is incorporated and consolidated into that of the husband." 10 The second reason lies in
"the desirability of having the interests of each member of the family unit governed by the same
law."11 The presumption that the wife retains the domicile of her deceased husband is an extension of this
common law concept. The concept and its extension have provided some of the most iniquitous jurisprudence
against women. It was under common law that the 1873 American case of Bradwell v. Illinois 12 was decided
where women were denied the right to practice law. It was unblushingly ruled that "the natural and proper timidity
and delicacy which belongs to the female sex evidently unfits it for many of the occupations of civil life . . . This is
the law of the Creator." Indeed, the rulings relied upon by Mr. Justice Davide in CJS 13 and AM JUR 2d14 are
American state court decisions handed down between the years 1917 15 and 1938,16 or before the time when
women were accorded equality of rights with men. Undeniably, the women's liberation movement resulted in far-
ranging state legislations in the United States to eliminate gender inequality. 17 Starting in the decade of the
seventies, the courts likewise liberalized their rulings as they started invalidating laws infected with gender-bias. It
was in 1971 when the US Supreme Court in Reed v. Reed,18 struck a big blow for women equality when it
declared as unconstitutional an Idaho law that required probate courts to choose male family members over
females as estate administrators. It held that mere administrative inconvenience cannot justify a sex-based
distinction. These significant changes both in law and in case law on the status of women virtually obliterated the
iniquitous common law surrendering the rights of married women to their husbands based on the dubious theory
of the parties' theoretic oneness. The Corpus Juris Secundum editors did not miss the relevance of this revolution
on women's right as they observed: "However, it has been declared that under modern statutes changing the
status of married women and departing from the common law theory of marriage, there is no reason why a wife
may not acquire a separate domicile for every purpose known to the law."19 In publishing in 1969 the Restatement
of the Law, Second (Conflict of Laws 2d), the reputable American Law Institute also categorically stated that the
view of Blackstone ". . . is no longer held. As the result of statutes and court decisions, a wife now possesses
practically the same rights and powers as her unmarried sister."20

In the case at bench, we have to decide whether we should continue clinging to the anachronistic common
law that demeans women, especially married women. I submit that the Court has no choice except to break away
from this common law rule, the root of the many degradations of Filipino women. Before 1988, our laws
particularly the Civil Code, were full of gender discriminations against women. Our esteemed colleague, Madam
Justice Flerida Ruth Romero, cited a few of them as follows: 21

xxx xxx xxx

Legal Disabilities Suffered by Wives

Not generally known is the fact that under the Civil Code, wives suffer under certain restrictions or
disabilities. For instance, the wife cannot accept gifts from others, regardless of the sex of the
giver or the value of the gift, other than from her very close relatives, without her husband's
consent. She may accept only from, say, her parents, parents-in-law, brothers, sisters and the
relatives within the so-called fourth civil degree. She may not exercise her profession or
occupation or engage in business if her husband objects on serious grounds or if his income is
sufficient to support their family in accordance with their social standing. As to what constitutes
"serious grounds" for objecting, this is within the discretion of the husband.

xxx xxx xxx

Because of the present inequitable situation, the amendments to the Civil Law being proposed by
the University of the Philippines Law Center would allow absolute divorce which severes the
matrimonial ties, such that the divorced spouses are free to get married a year after the divorce is
decreed by the courts. However, in order to place the husband and wife on an equal footing
insofar as the bases for divorce are concerned, the following are specified as the grounds for
absolute divorce: (1) adultery or having a paramour committed by the respondent in any of the
ways specified in the Revised Penal Code or (2) an attempt by the respondent against the life of
the petitioner which amounts to attempted parricide under the Revised Penal Code; (3)
abandonment of the petitioner by the respondent without just cause for a period of three
consecutive years; or (4) habitual maltreatment.

With respect to property relations, the husband is automatically the administrator of the conjugal
property owned in common by the married couple even if the wife may be the more astute or
enterprising partner. The law does not leave it to the spouses to decide who shall act as such
administrator. Consequently, the husband is authorized to engage in acts and enter into
transactions beneficial to the conjugal partnership. The wife, however, cannot similarly bind the
partnership without the husband's consent.
And while both exercise joint parental authority over their children, it is the father whom the law
designates as the legal administrator of the property pertaining to the unemancipated child.

Taking the lead in Asia, our government exerted efforts, principally through legislations, to eliminate
inequality between men and women in our land. The watershed came on August 3, 1988 when our
Family Code took effect which, among others, terminated the unequal treatment of husband and wife as
to their rights and responsibilities.22

The Family Code attained this elusive objective by giving new rights to married women and by abolishing sex-
based privileges of husbands. Among others, married women are now given the joint right to administer the
family property, whether in the absolute community system or in the system of conjugal partnership; 23 joint
parental authority over their minor children, both over their persons as well as their properties; 24 joint responsibility
for the support of the family;25 the right to jointly manage the household; 26 and, the right to object to their
husband's exercise of profession, occupation, business or activity. 27 Of particular relevance to the case at bench
is Article 69 of the Family Code which took away the exclusive right of the husband to fix the family domicile and
gave it jointly to the husband and the wife, thus:

Art. 69. The husband and wife shall fix the family domicile. In case of disagreement, the court
shall decide.

The court may exempt one spouse from living with the other if the latter should live abroad or
there are other valid and compelling reasons for the exemption. However, such exemption shall
not apply if the same is not compatible with the solidarity of the family. (Emphasis supplied)

Article 69 repealed Article 110 of the Civil Code. Commenting on the duty of the husband and wife to live
together, former Madam Justice Alice Sempio-Diy of the Court of Appeals specified the instances when a
wife may now refuse to live with her husband, thus:28

(2) The wife has the duty to live with her husband, but she may refuse to do so in certain cases
like:

(a) If the place chosen by the husband as family residence is dangerous to her
Life;

(b) If the husband subjects her to maltreatment or abusive conduct or insults,


making common life impossible;

(c) If the husband compels her to live with his parents, but she cannot get along
with her mother-in-law and they have constant quarrels (Del Rosario v. Del
Rosario, CA, 46 OG 6122);

(d) Where the husband has continuously carried illicit relations for 10 years with
different women and treated his wife roughly and without consideration. (Dadivas
v. Villanueva, 54 Phil. 92);

(e) Where the husband spent his time in gambling, giving no money to his family
for food and necessities, and at the same time insulting his wife and laying hands
on her. (Panuncio v. Sula, CA, 34 OG 129);

(f) If the husband has no fixed residence and lives a vagabond life as a tramp (1
Manresa 329);

(g) If the husband is carrying on a shameful business at home (Gahn v. Darby, 38


La. Ann. 70).

The inescapable conclusion is that our Family Code has completely emancipated the wife from the
control of the husband, thus abandoning the parties' theoretic identity of interest. No less than the late
revered Mr. Justice J.B.L. Reyes who chaired the Civil Code Revision Committee of the UP Law Center
gave this insightful view in one of his rare lectures after retirement: 29

xxx xxx xxx

The Family Code is primarily intended to reform the family law so as to emancipate the wife from
the exclusive control of the husband and to place her at parity with him insofar as the family is
concerned. The wife and the husband are now placed on equal standing by the Code. They are
now joint administrators of the family properties and exercise joint authority over the persons and
properties of their children. This means a dual authority in the family. The husband will no longer
prevail over the wife but she has to agree on all matters concerning the family. (Emphasis
supplied)

In light of the Family Code which abrogated the inequality between husband and wife as started and
perpetuated by the common law, there is no reason in espousing the anomalous rule that the wife still
retains the domicile of her dead husband. Article 110 of the Civil Code which provides the statutory
support for this stance has been repealed by Article 69 of the Family Code. By its repeal, it becomes a
dead-letter law, and we are not free to resurrect it by giving it further effect in any way or manner such as
by ruling that the petitioner is still bound by the domiciliary determination of her dead husband.

Aside from reckoning with the Family Code, we have to consider our Constitution and its firm guarantees of due
process and equal protection of
law.30 It can hardly be doubted that the common law imposition on a married woman of her dead husband's
domicile even beyond his grave is patently discriminatory to women. It is a gender-based discrimination and is
not rationally related to the objective of promoting family solidarity. It cannot survive a constitutional challenge.
Indeed, compared with our previous fundamental laws, the 1987 Constitution is more concerned with equality
between sexes as it explicitly commands that the State ". . . shall ensure fundamental equality before the law of
women and men." To be exact, section 14, Article II provides: "The State recognizes the role of women in nation
building, and shall ensure fundamental equality before the law of women and men. We shall be transgressing the
sense and essence of this constitutional mandate if we insist on giving our women the caveman's treatment.

Prescinding from these premises, I respectfully submit that the better stance is to rule that petitioner reacquired
her Tacloban domicile upon the death of her husband in 1989. This is the necessary consequence of the view
that petitioner's Batac dictated domicile did not continue after her husband's death; otherwise, she would have no
domicile and that will violate the universal rule that no person can be without a domicile at any point of time. This
stance also restores the right of petitioner to choose her domicile before it was taken away by Article 110 of the
Civil Code, a right now recognized by the Family Code and protected by the Constitution. Likewise, I cannot see
the fairness of the common law requiring petitioner to choose again her Tacloban domicile before she could be
released from her Batac domicile. She lost her Tacloban domicile not through her act but through the act of her
deceased husband when he fixed their domicile in Batac. Her husband is dead and he cannot rule her beyond
the grave. The law disabling her to choose her own domicile has been repealed. Considering all these, common
law should not put the burden on petitioner to prove she has abandoned her dead husband's domicile. There is
neither rhyme nor reason for this gender-based burden.

But even assuming arguendo that there is need for convincing proof that petitioner chose to reacquire her
Tacloban domicile, still, the records reveal ample evidence to this effect. In her affidavit submitted to the
respondent COMELEC, petitioner averred:

xxx xxx xxx

36. In November, 1991, I came home to our beloved country, after several requests for my return
were denied by President Corazon C. Aquino, and after I filed suits for our Government to issue
me my passport.

37. But I came home without the mortal remains of my beloved husband, President Ferdinand E.
Marcos, which the Government considered a threat to the national security and welfare.

38. Upon my return to the country, I wanted to immediately live and reside in Tacloban City or in
Olot, Tolosa, Leyte, even if my residences there were not livable as they had been destroyed and
cannibalized. The PCGG, however, did not permit and allow me.

39. As a consequence, I had to live at various times in the Westin Philippine Plaza in Pasay City,
a friend's apartment on Ayala Avenue, a house in South Forbes Park which my daughter rented,
and Pacific Plaza, all in Makati.

40. After the 1992 Presidential Elections, I lived and resided in the residence of my brother in San
Jose, Tacloban City, and pursued my negotiations with PCGG to recover my sequestered
residences in Tacloban City and Barangay Olot, Tolosa, Leyte.

40.1 In preparation for my observance of All Saints' Day and All Souls' Day that
year, I renovated my parents' burial grounds and entombed their bones which had
been excalvated, unearthed and scattered.

41. On November 29, 1993, I formally wrote PCGG Chairman Magtanggol Gunigundo for
permissions to —
. . . rehabilitate . . . (o)ur ancestral house in Tacloban and farmhouse in Olot,
Leyte . . . to make them livable for us the Marcos family to have a home in our
own motherland.

xxx xxx xxx

42. It was only on 06 June 1994, however, when PCGG Chairman Gunigundo, in his letter to Col.
Simeon Kempis, Jr., PCGG Region 8 Representative, allowed me to repair and renovate my
Leyte residences. I quote part of his letter:

Dear Col. Kempis,

Upon representation by Mrs. Imelda R. Marcos to this Commission, that she


intends to visit our sequestered properties in Leyte, please allow her access
thereto. She may also cause repairs and renovation of the sequestered
properties, in which event, it shall be understood that her undertaking said repairs
is not authorization for her to take over said properties, and that all expenses shall
be for her account and not reimbursable. Please extend the necessary courtesy to
her.

xxx xxx xxx

43. I was not permitted, however, to live and stay in the Sto. Niño Shrine residence in Tacloban
City where I wanted to stay and reside, after repairs and renovations were completed. In August
1994, I transferred from San Jose, Tacloban City, to my residence in Barangay Olot, Tolosa,
Leyte, when PCGG permitted me to stay and live there.

It is then clear that in 1992 petitioner reestablished her domicile in the First District of Leyte. It is not
disputed that in 1992, she first lived at the house of her brother in San Jose, Tacloban City and later, in
August 1994, she transferred her residence in Barangay Olot, Tolosa, Leyte. Both Tacloban City and the
municipality of Olot are within the First District of Leyte. Since petitioner reestablished her old domicile in
1992 in the First District of Leyte, she more than complied with the constitutional requirement of
residence
". . . for a period of not less than one year immediately preceding the day of the election," i.e., the May 8,
1995 elections.

The evidence presented by the private respondent to negate the Tacloban domicile of petitioner is nil. He
presented petitioner's Voter's Registration Record filed with the Board of Election Inspectors of Precinct 10-A of
Barangay Olot, Tolosa, Leyte wherein she stated that her period of residence in said barangay was six (6)
months as of the date of her filing of said Voter's Registration Record on January 28, 1995. 31 This statement in
petitioner's Voter's Registration Record is a non-prejudicial admission. The Constitution requires at least one (1)
year residence in the district in which the candidate shall be elected. In the case at bench, the reference is the
First District of Leyte. Petitioner's statement proved that she resided in Olot six (6) months before January 28,
1995 but did not disprove that she has also resided in Tacloban City starting 1992. As aforestated, Olot and
Tacloban City are both within the First District of Leyte, hence, her six (6) months residence in Olot should be
counted not against, but in her favor. Private respondent also presented petitioner's Certificate of Candidacy filed
on March 8, 199532 where she placed seven (7) months after Item No. 8 which called for information regarding
"residence in the constituency where I seek to be elected immediately preceding the election." Again, this original
certificate of candidacy has no evidentiary value because an March 1, 1995 it was corrected by petitioner. In her
Amended/Corrected Certificate of Candidacy,33 petitioner wrote "since childhood" after Item No. 8. The
amendment of a certificate of candidacy to correct a bona fide mistake has been allowed by this Court as a
matter of course and as a matter of right. As we held in Alialy v. COMELEC,34 viz.:

xxx xxx xxx

The absence of the signature of the Secretary of the local chapter N.P in the original certificate of
candidacy presented before the deadline September 11, 1959, did not render the certificate
invalid. The amendment of the certificate, although at a date after the deadline, but before the
election, was substantial compliance with the law, and the defect was cured.

It goes without saying that petitioner's erroneous Certificate of Candidacy filed on March 8, 1995 cannot
be used as evidence against her. Private respondent's petition for the disqualification of petitioner rested
alone on these two (2) brittle pieces of documentary evidence — petitioner's Voter's Registration Record
and her original Certificate of Candidacy. Ranged against the evidence of the petitioner showing her
ceaseless contacts with Tacloban, private respondent's two (2) pieces of evidence are too insufficient to
disqualify petitioner, more so, to deny her the right to represent the people of the First District of Leyte
who have overwhelmingly voted for her.
Fifth. Section 10, Article IX-C of the Constitution mandates that "bona fide candidates for any public office shall
be free from any form of harassment and discrimination."35 A detached reading of the records of the case at
bench will show that all forms of legal and extra-legal obstacles have been thrown against petitioner to prevent
her from running as the people's representative in the First District of Leyte. In petitioner's Answer to the petition
to disqualify her, she averred:36

xxx xxx xxx

10. Petitioner's (herein private respondent Montejo) motive in filing the instant petition is devious.
When respondent (petitioner herein) announced that she was intending to register as a voter in
Tacloban City and run for Congress in the First District of Leyte, petitioner (Montejo) immediately
opposed her intended registration by writing a letter stating that "she is not a resident of said city
but of Barangay Olot, Tolosa, Leyte." (Annex "2" of respondent's affidavit, Annex "2"). After
respondent (petitioner herein) had registered as a voter in Tolosa following completion of her six-
month actual residence therein, petitioner (Montejo) filed a petition with the COMELEC to transfer
the town of Tolosa from the First District to the Second District and pursued such move up to the
Supreme Court in G.R. No. 118702, his purpose being to remove respondent (petitioner herein)
as petitioner's (Montejo's) opponent in the congressional election in the First District. He also filed
a bill, along with other Leyte Congressmen, seeking to create another legislative district, to
remove the town of Tolosa out of the First District and to make it a part of the new district, to
achieve his purpose. However, such bill did not pass the Senate. Having, failed on such moves,
petitioner now filed the instant petition, for the same objective, as it is obvious that he is afraid to
submit himself along with respondent (petitioner herein) for the judgment and verdict of the
electorate of the First District of Leyte in an honest, orderly, peaceful, free and clean elections on
May 8, 1995.

These allegations which private respondent did not challenge were not lost
to the perceptive eye of Commissioner Maambong who in his Dissenting Opinion, 37 held:

xxx xxx xxx

Prior to the registration date — January 28, 1995 the petitioner (herein private respondent
Montejo) wrote the Election Officer of Tacloban City not to allow respondent (petitioner herein) to
register thereat since she is a resident of Tolosa and not Tacloban City. The purpose of this move
of the petitioner (Montejo) is not lost to (sic) the Commission. In UND No. 95-001 (In the matter of
the Legislative Districts of the Provinces of Leyte, Iloilo, and South Cotabato, Out of Which the
New Provinces of Biliran, Guimaras and Saranggani Were Respectively Created), . . . Hon. Cirilo
Roy G. Montejo, Representative, First District of Leyte, wanted the Municipality of Tolosa, in the
First District of Leyte, transferred to the Second District of Leyte. The Hon. Sergio A.F. Apostol,
Representative of the Second District of Leyte, opposed the move of the petitioner (Montejo).
Under Comelec Resolution No. 2736 (December 29, 1994), the Commission on Elections refused
to make the proposed transfer. Petitioner (Montejo) filed "Motion for Reconsideration of
Resolution
No. 2736" which the Commission denied in a Resolution promulgated on February 1, 1995.
Petitioner (Montejo) filed a petition for certiorari before the Honorable Supreme Court (Cirilo Roy
G. Montejo vs. Commission on Elections, G.R. No. 118702) questioning the resolution of the
Commission. Believing that he could get a favorable ruling from the Supreme Court, petitioner
(Montejo) tried to make sure that the respondent (petitioner herein) will register as a voter in
Tolosa so that she will be forced to run as Representative not in the First but in the Second
District.

It did not happen. On March 16, 1995, the Honorable Supreme Court unanimously promulgated a
"Decision," penned by Associate Justice Reynato S. Puno, the dispositive portion of which reads:

IN VIEW WHEREOF, Section 1 of Resolution No. 2736 insofar as it transferred


the municipality of Capoocan of the Second District and the municipality of
Palompon of the Fourth District to the Third District of the province of Leyte, is
annulled and set aside. We also deny the Petition praying for the transfer of the
municipality of Tolosa from the First District to the Second District of the province
of Leyte. No costs.

Petitioner's (Montejo's) plan did not work. But the respondent (petitioner herein) was constrained
to register in the Municipality of Tolosa where her house is instead of Tacloban City, her domicile.
In any case, both Tacloban City and Tolosa are in the First Legislative District.

All these attempts to misuse our laws and legal processes are forms of rank harassments and invidious
discriminations against petitioner to deny her equal access to a public office. We cannot commit any
hermeneutic violence to the Constitution by torturing the meaning of equality, the end result of which will
allow the harassment and discrimination of petitioner who has lived a controversial life, a past of
alternating light and shadow. There is but one Constitution for all Filipinos. Petitioner cannot be adjudged
by a "different" Constitution, and the worst way to interpret the Constitution is to inject in its interpretation,
bile and bitterness.

Sixth. In Gallego v. Vera,38 we explained that the reason for this residence requirement is "to exclude a stranger
or newcomer, unacquainted, with the conditions and needs of a community and not identified with the latter, from
an elective office to serve that community . . . ." Petitioner's lifetime contacts with the First District of Leyte cannot
be contested. Nobody can claim that she is not acquainted with its problems because she is a stranger to the
place. None can argue she cannot satisfy the intent of the Constitution.

Seventh. In resolving election cases, a dominant consideration is the need to effectuate the will of the electorate.
The election results show that petitioner received Seventy Thousand Four Hundred Seventy-one (70,471) votes,
while private respondent got only Thirty-Six Thousand Eight Hundred Thirty-Three (36,833) votes. Petitioner is
clearly the overwhelming choice of the electorate of the First District of Leyte and this is not a sleight of statistics.
We cannot frustrate this sovereign will on highly arguable technical considerations. In case of doubt, we should
lean towards a rule that will give life to the people's political judgment.

A final point. The case at bench provides the Court with the rare opportunity to rectify the inequality of status
between women and men by rejecting the iniquitous common law precedents on the domicile of married women
and by redefining domicile in accord with our own culture, law, and Constitution. To rule that a married woman is
eternally tethered to the domicile dictated by her dead husband is to preserve the anachronistic and anomalous
balance of advantage of a husband over his wife. We should not allow the dead to govern the living even if the
glories of yesteryears seduce us to shout long live the dead! The Family Code buried this gender-based
discrimination against married women and we should not excavate what has been entombed. More importantly,
the Constitution forbids it.

I vote to grant the petition.

Bellosillo and Melo, JJ., concur.

FRANCISCO, J., concurring:

I concur with Mr. Justice Kapunan's ponencia finding petitioner qualified for the position of Representative of the
First Congressional District of Leyte. I wish, however, to express a few comments on the issue of petitioner's
domicile.

Domicile has been defined as that place in which a person's habitation is fixed, without any present intention of
removing therefrom, and that place is properly the domicile of a person in which he has voluntarily fixed his
abode, or habitation, not for a mere special or temporary purpose, but with a present intention of making it his
permanent home (28 C.J.S. §1). It denotes a fixed permanent residence to which when absent for business, or
pleasure, or for like reasons one intends to return, and depends on facts and circumstances, in the sense that
they disclose intent. (Ong Huan Tin v. Republic, 19 SCRA 966, 969)

Domicile is classified into domicile of origin and domicile of choice. The law attributes to every individual a
domicile of origin, which is the domicile of his parents, or of the head of his family, or of the person on whom he is
legally dependent at the time of his birth. While the domicile of origin is generally the place where one is born or
reared, it maybe elsewhere (28 C.J.S. §5). Domicile of choice, on the other hand, is the place which the person
has elected and chosen for himself to displace his previous domicile; it has for its true basis or foundation the
intention of the person (28 C.J.S. §6). In order to hold that a person has abandoned his domicile and acquired a
new one called domicile of choice, the following requisites must concur, namely, (a) residence or bodily presence
in the new locality, (b) intention to remain there or animus manendi, and (c) an intention to abandon the old
domicile or animus non revertendi (Romualdez v. RTC, Br. 7, Tacloban City, 226 SCRA 408, 415). A third
classification is domicile by operation of law which attributes to a person a domicile independent of his own
intention or actual residence, ordinarily resulting from legal domestic relations, as that of the wife arising from
marriage, or the relation of a parent and a child (28 C.J.S. §7).

In election law, when our Constitution speaks of residence for election purposes it means domicile (Co v.
Electoral Tribunal of the House of Representatives, 199 SCRA 692, 713; Nuval v. Guray, 52 Phil. 645, 651). To
my mind, public respondent Commission on Elections misapplied this concept, of domicile which led to
petitioner's disqualification by ruling that petitioner failed to comply with the constitutionally mandated one-year
residence requirement. Apparently, public respondent Commission deemed as conclusive petitioner's stay and
registration as voter in many places as conduct disclosing her intent to abandon her established domicile of origin
in Tacloban, Leyte. In several decisions, though, the Court has laid down the rule that registration of a voter in a
place other than his place of origin is not sufficient to constitute abandonment or loss of such residence (Faypon
v. Quirino, 96 Phil. 294, 300). Respondent Commission offered no cogent reason to depart from this rule except
to surmise petitioner's intent of abandoning her domicile of origin.

It has been suggested that petitioner's domicile of origin was supplanted by a new domicile due to her marriage,
a domicile by operation of law. The proposition is that upon the death of her husband in 1989 she retains her
husband's domicile, i.e., Batac, Ilocos Norte, until she makes an actual change thereof. I find this proposition
quite untenable.

Tacloban, Leyte, is petitioner's domicile of origin which was involuntarily supplanted with another, i.e., Batac,
Ilocos Norte, upon her marriage in 1954 with then Congressman Marcos. By legal fiction she followed the
domicile of her husband. In my view, the reason for the law is for the spouses to fully and effectively perform their
marital duties and obligations to one another. 1 The question of domicile, however, is not affected by the fact that it
was the legal or moral duty of the individual to reside in a given place (28 C.J.S. §11). Thus, while the wife retains
her marital domicile so long as the marriage subsists, she automatically loses it upon the latter's termination, for
the reason behind the law then ceases. Otherwise, petitioner, after her marriage was ended by the death of her
husband, would be placed in a quite absurd and unfair situation of having been freed from all wifely obligations
yet made to hold on to one which no longer serves any meaningful purpose.

It is my view therefore that petitioner reverted to her original domicile of Tacloban, Leyte upon her husband's
death without even signifying her intention to that effect. It is for the private respondent to prove, not for petitioner
to disprove, that petitioner has effectively abandoned Tacloban, Leyte for Batac, Ilocos Norte or for some other
place/s. The clear rule is that it is the party (herein private respondent) claiming that a person has abandoned or
lost his residence of origin who must show and prove preponderantly such abandonment or loss (Faypon v.
Quirino, supra at 298; 28 C.J.S. §16), because the presumption is strongly in favor of an original or former
domicile, as against an acquired one (28 C.J.S. §16). Private respondent unfortunately failed to discharge this
burden as the record is devoid of convincing proof that petitioner has acquired whether voluntarily or involuntarily,
a new domicile to replace her domicile of origin.

The records, on the contrary, clearly show that petitioner has complied with the constitutional one-year residence
requirement. After her exile abroad, she returned to the Philippines in 1991 to reside in Olot, Tolosa, Leyte, but
the Presidential Commission on Good Government which sequestered her residential house and other properties
forbade her necessitating her transient stay in various places in Manila (Affidavit p.6, attached as Annex I of the
Petition). In 1992, she ran for the position of president writing in her certificate of candidacy her residence as San
Juan, Metro Manila. After her loss therein, she went back to Tacloban City, acquired her residence
certificate2 and resided with her brother in San Jose. She resided in San Jose, Tacloban City until August of 1994
when she was allowed by the PCGG to move and reside in her sequestered residential house in Olot, Tolosa,
Leyte (Annex I, p. 6).3 It was in the same month of August when she applied for the cancellation of her previous
registration in San Juan, Metro Manila in order to register anew as voter of Olot, Tolosa, Leyte, which she did on
January 28, 1995. From this sequence of events, I find it quite improper to use as the reckoning period of the
one-year residence requirement the date when she applied for the cancellation of her previous registration in San
Juan, Metro Manila. The fact which private respondent never bothered to disprove is that petitioner transferred
her residence after the 1992 presidential election from San Juan, Metro Manila to San Jose, Tacloban City, and
resided therein until August of 1994. She later transferred to Olot, Tolosa, Leyte (Annex I, p. 7). It appearing that
both Tacloban City and Tolosa, Leyte are within the First Congressional District of Leyte, it indubitably stands that
she had more than a year of residence in the constituency she sought to be elected. Petitioner, therefore, has
satisfactorily complied with the one-year qualification required by the 1987 Constitution.

I vote to grant the petition.

ROMERO, J., separate opinion:

Petitioner has appealed to this Court for relief after the COMELEC ruled that she was disqualified from running
for Representative of her District and that, in the event that she should, nevertheless, muster a majority vote, her
proclamation should be suspended. Not by a straightforward ruling did the COMELEC pronounce its decision as
has been its unvarying practice in the past, but by a startling succession of "reverse somersaults." Indicative of its
shifting stance vis-a-vis petitioner's certificate of candidacy were first, the action of its Second Division
disqualifying her and canceling her original Certificate of Candidacy by a vote of 2-1 on April 24, 1995; then the
denial by the COMELEC en banc of her Motion for Reconsideration on May 7, 1995, a day before the election;
then because she persisted in running, its decision on
May 11, 1995 or three days after the election, allowing her proclamation in the event that the results of the
canvass should show that she obtained the highest number of votes (obviously noting that petitioner had won
overwhelmingly over her opponent), but almost simultaneously reversing itself by directing that even if she wins,
her proclamation should nonetheless be suspended.

Crucial to the resolution of the disqualification issue presented by the case at bench is the interpretation to be
given to the one-year residency requirement imposed by the Constitution on aspirants for a Congressional seat. 1
Bearing in mind that the term "resident" has been held to be synonymous with "domicile" for election purposes, it
is important to determine whether petitioner's domicile was in the First District of Leyte and if so, whether she had
resided there for at least a period of one year. Undisputed is her domicile of origin, Tacloban, where her parents
lived at the time of her birth. Depending on what theory one adopts, the same may have been changed when she
married Ferdinand E. Marcos, then domiciled in Batac, by operation of law. Assuming it did, his death certainly
released her from the obligation to live with him at the residence fixed by him during his lifetime. What may
confuse the layman at this point is the fact that the term "domicile" may refer to "domicile of origin," "domicile of
choice," or "domicile by operation of law," which subject we shall not belabor since it has been amply discussed
by the ponente and in the other separate opinions.

In any case, what assumes relevance is the divergence of legal opinion as to the effect of the husband's death on
the domicile of the widow. Some scholars opine that the widow's domicile remains unchanged; that the deceased
husband's wishes perforce still bind the wife he has left behind. Given this interpretation, the widow cannot
possibly go far enough to sever the domiciliary tie imposed by her husband.

It is bad enough to interpret the law as empowering the husband unilaterally to fix the residence or domicile of the
family, as laid down in the Civil Code,2 but to continue giving obeisance to his wishes even after the rationale
underlying the mutual duty of the spouses to live together has ceased, is to close one's eyes to the stark realities
of the present.

At the other extreme is the position that the widow automatically reverts to her domicile of origin upon the demise
of her husband. Does the law so abhor a vacuum that the widow has to be endowed somehow with a domicile?
To answer this question which is far from rhetorical, one will have to keep in mind the basic principles of domicile.
Everyone must have a domicile. Then one must have only a single domicile for the same purpose at any given
time. Once established, a domicile remains until a new one is acquired, for no person lives who has no domicile,
as defined by the law be is subject to.

At this juncture, we are confronted with an unexplored legal terrain in this jurisdiction, rendered more murky by
the conflicting opinions of foreign legal authorities. This being the state of things, it is imperative as it is opportune
to illumine the darkness with the beacon light of truth, as dictated by experience and the necessity of according
petitioner her right to choose her domicile in keeping with the enlightened global trend to recognize and protect
the human rights of women, no less than men.

Admittedly, the notion of placing women at par with men, insofar as civil, political and social rights are concerned,
is a relatively recent phenomenon that took seed only in the middle of this century. It is a historical fact that for
over three centuries, the Philippines had been colonized by Spain, a conservative, Catholic country which
transplanted to our shores the Old World cultures, mores and attitudes and values. Through the imposition on our
government of the Spanish Civil Code in 1889, the people, both men and women, had no choice but to accept
such concepts as the husband's being the head of the family and the wife's subordination to his authority. In such
role, his was the right to make vital decisions for the family. Many instances come to mind, foremost being what is
related to the issue before us, namely, that "the husband shall fix the residence of the family." 3 Because he is
made responsible for the support of the wife and the rest of the family, 4 he is also empowered to be the
administrator of the conjugal property, with a few exceptions 5 and may, therefore, dispose of the conjugal
partnership property for the purposes specified under the law;6 whereas, as a general rule, the wife
cannot bind the conjugal partnership without the husband's consent.7 As regards the property pertaining
to the children under parental authority, the father is the legal administrator and only in his absence may
the mother assume his powers.8 Demeaning to the wife's dignity are certain strictures on her personal
freedoms, practically relegating her to the position of minors and disabled persons. To illustrate a few:
The wife cannot, without the husband's consent, acquire any gratuitous title, except from her
ascendants, descendants, parents-in-law, and collateral relatives within the fourth degree.9 With respect
to her employment, the husband wields a veto power in the case the wife exercises her profession or
occupation or engages in business, provided his income is sufficient for the family, according to its
social standing and his opposition is founded on serious and valid grounds. 10 Most offensive, if not
repulsive, to the liberal-minded is the effective prohibition upon a widow to get married till after three
hundred days following the death of her husband, unless in the meantime, she has given birth to a
child. 11 The mother who contracts a subsequent marriage loses the parental authority over her children,
unless the deceased husband, father of the latter, has expressly provided in his will that his widow might
marry again, and has ordered that in such case she should keep and exercise parental authority over
their children. 12 Again, an instance of a husband's overarching influence from beyond the grave.

All these indignities and disabilities suffered by Filipino wives for hundreds of years evoked no protest
from them until the concept of human rights and equality between and among nations and individuals
found hospitable lodgment in the United Nations Charter of which the Philippines was one of the original
signatories. By then, the Spanish "conquistadores" had been overthrown by the American forces at the
turn of the century. The bedrock of the U.N. Charter was firmly anchored on this credo: "to reaffirm faith
in the fundamental human rights, in the dignity and worth of the human person, in the equal rights of
men and women." (Emphasis supplied)
It took over thirty years before these egalitarian doctrines bore fruit, owing largely to the
burgeoning of the feminist movement. What may be regarded as the international bill of rights for
women was implanted in the Convention on the Elimination of All Forms of Discrimination
Against Women (CEDAW) adopted by the U.N. General Assembly which entered into force as an
international treaty on September 3, 1981. In ratifying the instrument, the Philippines bound itself
to implement its liberating spirit and letter, for its Constitution, no less, declared that "The
Philippines. . . adopts the generally accepted principles of international law as part of the law of
the land and adheres to the policy of peace, equality, justice, freedom, cooperation, and amity
with all nations." 13 One such principle embodied in the CEDAW is granting to men and women
"the same rights with regard to the law relating to the movement of persons and the freedom to
choose their residence and domicile." 14 (Emphasis supplied).

CEDAW's pro-women orientation which was not lost on Filipino women was reflected in the 1987
Constitution of the Philippines and later, in the Family Code, 15 both of which were speedily approved by
the first lady President of the country, Corazon C. Aquino. Notable for its emphasis on the human rights
of all individuals and its bias for equality between the sexes are the following provisions: "The State
values the dignity of every human person and guarantees full respect for human rights"16 and "The State
recognizes the role of women in nation-building, and shall ensure the fundamental equality before the
law of women and men."17

A major accomplishment of women in their quest for equality with men and the elimination of discriminatory
provisions of law was the deletion in the Family Code of almost all of the unreasonable strictures on wives and
the grant to them of personal rights equal to that of their husbands. Specifically, the husband and wife are now
given the right jointly to fix the family domicile;18 concomitant to the spouses' being jointly responsible for the
support of the family is the right and duty of both spouses to manage the household; 19 the administration and the
enjoyment of the community property shall belong to both spouses jointly; 20 the father and mother shall now jointly
exercise legal guardianship over the property of their unemancipated common child 21 and several others.

Aware of the hiatus and continuing gaps in the law, insofar as women's rights are concerned, Congress passed a
law popularly known as "Women in Development and Nation Building Act" 22 Among the rights given to married
women evidencing their capacity to act in contracts equal to that of men are:

(1) Women shall have the capacity to borrow and obtain loans and execute security and credit arrangements
under the same conditions as men;

(2) Women shall have equal access to all government and private sector programs granting agricultural credit,
loans and non material resources and shall enjoy equal treatment in agrarian reform and land resettlement
programs;

(3) Women shall have equal rights to act as incorporators and enter into insurance contracts; and

(4) Married women shall have rights equal to those of married men in applying for passports, secure visas and
other travel documents, without need to secure the consent of their spouses.

As the world draws the curtain on the Fourth World Conference of Women in Beijing, let this Court now be the
first to respond to its clarion call that "Women's Rights are Human Rights" and that "All obstacles to women's full
participation in decision-making at all levels, including the family" should be removed. Having been herself a
Member of the Philippine Delegation to the International Women's Year Conference in Mexico in 1975, this writer
is only too keenly aware of the unremitting struggle being waged by women the world over, Filipino women not
excluded, to be accepted as equals of men and to tear down the walls of discrimination that hold them back from
their proper places under the sun.

In light of the inexorable sweep of events, local and global, legislative, executive and judicial, according more
rights to women hitherto denied them and eliminating whatever pockets of discrimination still exist in their civil,
political and social life, can it still be insisted that widows are not at liberty to choose their domicile upon the death
of their husbands but must retain the same, regardless?

I submit that a widow, like the petitioner and others similarly situated, can no longer be bound by the domicile of
the departed husband, if at all she was before. Neither does she automatically revert to her domicile of origin, but
exercising free will, she may opt to reestablish her domicile of origin. In returning to Tacloban and subsequently,
to Barangay Olot, Tolosa, both of which are located in the First District of Leyte, petitioner amply demonstrated
by overt acts, her election of a domicile of choice, in this case, a reversion to her domicile of origin. Added
together, the time when she set up her domicile in the two places sufficed to meet the one-year requirement to
run as Representative of the First District of Leyte.

In view of the foregoing expatiation, I vote to GRANT the petition.


VITUG, J., separate opinion:

The case at bench deals with explicit Constitutional mandates.

The Constitution is not a pliable instrument. It is a bedrock in our legal system that sets up ideals and directions
and render steady our strides hence. It only looks back so as to ensure that mistakes in the past are not
repeated. A compliant transience of a constitution belittles its basic function and weakens its goals. A constitution
may well become outdated by the realities of time. When it does, it must be changed but while it remains, we owe
it respect and allegiance. Anarchy, open or subtle, has never been, nor must it ever be, the answer to perceived
transitory needs, let alone societal attitudes, or the Constitution might lose its very essence.

Constitutional provisions must be taken to be mandatory in character unless, either by express statement or by
necessary implication, a different intention is manifest (see Marcelino vs. Cruz, 121 SCRA 51).

The two provisions initially brought to focus are Section 6 and Section 17 of Article VI of the fundamental law.
These provisions read:

Sec. 6. No person shall be a Member of the House of Representatives unless he is a natural-born


citizen of the Philippines and, on the day of the election, is at least twenty-five years of age, able
to read and write, and, except the party-list representatives, a registered voter in the district in
which he shall be elected, and a resident thereof for a period of not less than one year
immediately preceding the day of the election.

Sec. 17. The Senate and the House of Representatives shall each have an Electoral Tribunal
which shall be the sole judge of all contests relating to the election, returns, and qualifications of
their respective Members. Each Electoral Tribunal shall be composed of nine Members, three of
whom shall be Justices of the Supreme Court to be designated by the Chief Justice, and the
remaining six shall be Members of the Senate or the House of Representatives, as the case may
be, who shall be chosen on the basis of proportional representation from the political parties and
the parties or organizations registered under the party-list system represented therein. The senior
Justice in the Electoral Tribunal shall be its Chairman.

The Commission on Election (the "COMELEC") is constitutionally bound to enforce and administer "all laws and
regulations relative to the conduct of election . . ." (Art. IX, C, Sec. 2, Constitution) that, there being nothing said
to the contrary, should include its authority to pass upon the qualification and disqualification prescribed by law
of candidates to an elective office. Indeed, pre-proclamation controversies are expressly placed under the
COMELEC's jurisdiction to hear and resolve (Art. IX, C, Sec. 3, Constitution).

The matter before us specifically calls for the observance of the constitutional one-year residency requirement.
The issue (whether or not there is here such compliance), to my mind, is basically a question of fact or at least
inextricably linked to such determination. The findings and judgment of the COMELEC, in accordance with the
long established rule and subject only to a number of exceptions under the basic heading of "grave abuse of
discretion," are not reviewable by this Court.

I do not find much need to do a complex exercise on what seems to me to be a plain matter. Generally, the term
"residence" has a broader connotation that may mean permanent (domicile), official (place where one's official
duties may require him to stay) or temporary (the place where he sojourns during a considerable length of time).
For civil law purposes, i.e., as regards the exercise of civil rights and the fulfillment of civil obligations, the
domicile of a natural person is the place of his habitual residence (see Article 50, Civil Code). In election cases,
the controlling rule is that heretofore announced by this Court in Romualdez vs. Regional Trial Court, Branch 7,
Tacloban City (226 SCRA 408, 409); thus:

In election cases, the Court treats domicile and residence as synonymous terms, thus: "(t)he term
"residence" as used in the election law is synonymous with "domicile," which imports not only an
intention to reside in a fixed place but also personal presence in that place, coupled with conduct
indicative of such intention." "Domicile" denotes a fixed permanent residence to which when
absent for business or pleasure, or for like reasons, one intends to return. . . . . Residence thus
acquired, however, may be lost by adopting another choice of domicile. In order, in turn, to
acquire a new domicile by choice, there must concur (1) residence or bodily presence in the new
locality, (2) an intention to remain there, and (3) an intention to abandon the old domicile. In other
words, there must basically be animus manendi coupled with animus non revertendi. The purpose
to remain in or at the domicile of choice must be for an indefinite period of time; the change of
residence must be voluntary; and the residence at the place chosen for the new domicile must be
actual.

Using the above tests, I am not convinced that we can charge the COMELEC with having committed
grave abuse of discretion in its assailed resolution.
The COMELEC's jurisdiction, in the case of congressional elections, ends when the jurisdiction of the Electoral
Tribunal concerned begins. It signifies that the protestee must have theretofore been duly proclaimed and has
since become a "member" of the Senate or the House of Representatives. The question can be asked on
whether or not the proclamation of a candidate is just a ministerial function of the Commission on Elections
dictated solely on the number of votes cast in an election exercise. I believe, it is not. A ministerial duty is an
obligation the performance of which, being adequately defined, does not allow the use of further judgment or
discretion. The COMELEC, in its particular case, is tasked with the full responsibility of ascertaining all the facts
and conditions such as may be required by law before a proclamation is properly done.

The Court, on its part, should, in my view at least, refrain from any undue encroachment on the ultimate exercise
of authority by the Electoral Tribunals on matters which, by no less than a constitutional fiat, are explicitly within
their exclusive domain. The nagging question, if it were otherwise, would be the effect of the Court's peremptory
pronouncement on the ability of the Electoral Tribunal to later come up with its own judgment in a contest
"relating to the election, returns and qualification" of its members.

Prescinding from all the foregoing, I should like to next touch base on the applicability to this case of Section 6 of
Republic Act No. 6646, in relation to Section 72 of Batas Pambansa Blg. 881, each providing thusly:

REPUBLIC ACT NO. 6646

xxx xxx xxx

Sec. 6. Effect of Disqualification Case. — Any candidate who has been declared by final
judgment to be disqualified shall not be voted for, and the votes cast for him shall not be counted.
If for any reason a candidate is not declared by final judgment before an election to be
disqualified and he is voted for and receives the winning number of votes in such election, the
Court or Commission shall continue with the trial and hearing of the action, inquiry or protest and,
upon motion of the complainant or any intervenor, may during the pendency thereof order the
suspension of the proclamation of such candidate whenever the evidence of his guilt is strong.

BATAS PAMBANSA BLG. 881

xxx xxx xxx

Sec. 72. Effects of disqualification cases and priority. — The Commission and the courts shall
give priority to cases of disqualification by reason of violation of this Act to the end that a final
decision shall be rendered not later than seven days before the election in which the
disqualification is sought.

Any candidate who has been declared by final judgment to be disqualified shall not be voted for,
and the votes cast for him shall not be counted. Nevertheless, if for any reason, a candidate is not
declared by final, judgment before an election to be disqualified, and he is voted for and receives
the winning number of votes in such election, his violation of the provisions of the preceding
sections shall not prevent his proclamation and assumption to office.

I realize that in considering the significance of the law, it may be preferable to look for not so much the specific
instances they ostensibly would cover as the principle they clearly convey. Thus, I will not scoff at the argument
that it should be sound to say that votes cast in favor of the disqualified candidate, whenever ultimately declared
as such, should not be counted in his or her favor and must accordingly be considered to be stray votes. The
argument, nevertheless, is far outweighed by the rationale of the now prevailing doctrine first enunciated in the
case of Topacio vs. Paredes (23 Phil. 238 [1912]) which, although later abandoned in Ticzon vs. Comelec (103
SCRA 687 [1981]), and Santos vs. COMELEC (137 SCRA 740 [1985]), was restored, along with the interim case
of Geronimo vs. Ramos (136 SCRA 435 [1985]), by the Labo (176 SCRA 1 (1989]), Abella (201 SCRA 253
[1991]), Labo (211 SCRA 297 [1992]) and, most recently, Benito (235 SCRA 436 [1994]) rulings. Benito
vs. Comelec was a unanimous decision penned by Justice Kapunan and concurred in by Chief Justice Narvasa,
Justices Feliciano, Padilla, Bidin, Regalado, Davide, Romero, Melo, Quiason, Puno, Vitug and Mendoza (Justices
Cruz and Bellosillo were on official leave). For easy reference, let me quote from the first Labo decision:

Finally, there is the question of whether or not the private respondent, who filed the quo
warranto petition, can replace the petitioner as mayor. He cannot. The simple reason is that as he
obtained only the second highest number of votes in the election, he was obviously not the choice
of the people of Baguio City.

The latest ruling of the Court on this issue is Santos v. Commission on Elections, (137 SCRA
740) decided in 1985. In that case, the candidate who placed second was proclaimed elected
after the votes for his winning rival, who was disqualified as a turncoat and considered a non-
candidate, were all disregard as stray. In effect, the second placer won by default. That decision
was supported by eight members of the Court then, (Cuevas, J., ponente, with Makasiar,
Concepcion, Jr., Escolin, Relova, De la Fuente, Alampay and Aquino, JJ., concurring.) with three
dissenting (Teehankee, Acting C.J., Abad Santos and Melencio-Herrera, JJ.) and another two
reserving their vote. (Plana and Gutierrez, Jr., JJ.) One was on official leave. (Fernando, C.J.)

Re-examining that decision, the Court finds, and so holds, that it should be reversed in favor of
the earlier case of Geronimo v. Ramos, (136 SCRA 435) which represents the more logical and
democratic rule. That case, which reiterated the doctrine first announced in 1912 in Topacio
v. Paredes, (23 Phil. 238) was supported by ten members of the Court, (Gutierrez, Jr., ponente,
with Teehankee, Abad Santos, Melencio-Herrera, Plana, Escolin, Relova, De la Fuente, Cuevas
and Alampay, JJ., concurring) without any dissent, although one reserved his vote, (Makasiar, J.)
another took no part, (Aquino, J.) and two others were on leave. (Fernando, C.J. and Concepcion,
Jr., J.) There the Court held:

. . . it would be extremely repugnant to the basic concept of the constitutionally


guaranteed right to suffrage if a candidate who has not acquired the majority or
plurality of votes is proclaimed a winner and imposed as the representative of a
constituency, the majority of which have positively declared through their ballots
that they do not choose him.

Sound policy dictates that public elective offices are filled by those who have
received the highest number of votes cast in the election for that office, and it is a
fundamental idea in all republican forms of government that no one can be
declared elected and no measure can be declared carried unless he or it receives
a majority or plurality of the legal votes cast in the election. (20 Corpus Juris 2nd,
S 243, p. 676.)

The fact that the candidate who obtained the highest number of votes is later declared to be
disqualified or not eligible for the office to which he was elected does not necessarily entitle the
candidate who obtained the second highest number of votes to be declared the winner of the
elective office. The votes cast for a dead, disqualified, or non-eligible person may not be valid to
vote the winner into office or maintain him there. However, in the absence of a statute which
clearly asserts a contrary political and legislative policy on the matter, if the votes were cast in the
sincere belief that the candidate was alive, qualified, or eligible, they should not be treated as
stray, void or meaningless. (at pp. 20-21)

Considering all the foregoing, I am constrained to vote for the dismissal of the petition.

MENDOZA, J., separate opinion:

In my view the issue in this case is whether the Commission on Elections has the power to disqualify candidates
on the ground that they lack eligibility for the office to which they seek to be elected. I think that it has none and
that the qualifications of candidates may be questioned only in the event they are elected, by filing a petition
for quo warranto or an election protest in the appropriate forum, not necessarily in the COMELEC but, as in this
case, in the House of Representatives Electoral Tribunal. That the parties in this case took part in the
proceedings in the COMELEC is of no moment. Such proceedings were unauthorized and were not rendered
valid by their agreement to submit their dispute to that body.

The various election laws will be searched in vain for authorized proceedings for determining a candidate's
qualifications for an office before his election. There are none in the Omnibus Election Code (B.P. Blg. 881), in
the Electoral Reforms Law of 1987 (R.A. No. 6646), or in the law providing for synchronized elections (R.A. No.
7166). There are, in other words, no provisions for pre-proclamation contests but only election protests or quo
warranto proceedings against winning candidates.

To be sure, there are provisions denominated for "disqualification," but they are not concerned with a declaration
of the ineligibility of a candidate. These provisions are concerned with the incapacity (due to insanity,
incompetence or conviction of an offense) of a person either to be a candidate or to continue as a candidate for
public office. There is also a provision for the denial or cancellation of certificates of candidacy, but it applies only
to cases involving false representations as to certain matters required by law to be stated in the certificates.

These provisions are found in the following parts of the Omnibus Election Code:

§ 12. Disqualifications. — Any person who has been declared by competent authority insane or
incompetent, or has been sentenced by final judgment for subversion, insurrection, rebellion or for
any offense for which he has been sentenced to a penalty of more than eighteen months or for a
crime involving moral turpitude, shall be disqualified to be a candidate and to hold any office,
unless he has been given plenary pardon or granted amnesty.
The disqualifications to be a candidate herein provided shall be deemed removed upon the
declaration by competent authority that said insanity or incompetence had been removed or after
the expiration of a period of five years from his service of sentence, unless within the same period
he again becomes disqualified. (Emphasis added)

§ 68. Disqualifications. — Any candidate who, in an action or protest in which he is a party is


declared by final decision of a competent court guilty of, or found by the Commission of having (a)
given money or other material consideration to influence, induce or corrupt the voters or public
officials performing electoral functions; (b) committed acts of terrorism to enhance his candidacy;
(c) spent in his election campaign an amount in excess of that allowed by this Code; (d) solicited,
received or made any contribution prohibited under Sections 89, 95, 96, 97 and 104; or (e)
violated any of Sections 80, 83, 85, 86 and 261, paragraphs d, e, k, v, and cc, sub-paragraph 6,
shall be disqualified from continuing as a candidate, or if he has been elected, from holding the
office. Any person who is a permanent resident of or an immigrant to a foreign country shall not
be qualified to run for any elective office under this Code, unless said person has waived his
status as permanent resident or immigrant of a foreign country in accordance with the residence
requirement provided for in the election laws. (Emphasis added)

§ 78. Petition to deny due course to or cancel a certificate of


candidacy. — A verified petition seeking to deny due course or to cancel a certificate of candidacy
may be filed by any person exclusively on the ground that any material representation contained
therein as required under Section 74 hereof is false. The petition may be filed at any time not later
than twenty-five days from the time of the filing of the certificate of candidacy and shall be
decided, after due notice and hearing, not later than fifteen days before the election. (Emphasis
added)

the Electoral Reforms Law of 1987 (R.A. No. 6646):

§ 6. Effect of Disqualification Case. — Any candidate who has been declared by final judgment to
be disqualified shall not be voted for, and the votes cast for him shall not be counted. If for
any reason a candidate is not declared by final judgment before an election to be disqualified and
he is voted for and receives the winning number of votes in such election, the Court or
Commission shall continue with the trial and hearing of the action, inquiry or protest and; upon
motion for the complainant or any intervenor, may during the pendency thereof order the
suspension of the proclamation of such candidate whenever the evidence of his guilt is strong.
(Emphasis added).

§ 7. Petition to Deny Due Course to or Cancel a Certificate of Candidacy. — The procedure


hereinabove provided shall apply to petitions to deny due course to or cancel a certificate of
candidacy as provided in Section 78 of Batas Pambansa Blg. 881.

and the Local Government Code of 1991 (R.A. No. 7160):

§ 40. Disqualifications. — The following persons are disqualified from running for any elective
local position:

(a) Those sentenced by final judgment for an offense involving moral turpitude or for an offense
punishable by one (1) year or more of imprisonment, within two (2) years after serving sentence;

(b) Those removed from office as a result of on administrative case;

(c) Those convicted by final judgment for violating the oath of allegiance to the Republic;

(d) Those with dual citizenship;

(e) Fugitive from justice in criminal or nonpolitical cases here or abroad;

(f) Permanent residents in a foreign country or those who have acquired the right to reside abroad
and continue to avail of the same right after the effectivity of this Code; and

(g) The insane or feeble-minded.

The petition filed by private respondent Cirilo Roy Montejo in the COMELEC, while entitled "For Cancellation and
Disqualification," contained no allegation that private respondent Imelda Romualdez-Marcos made material
representations in her certificate of candidacy which were false, it sought her disqualification on the ground that
"on the basis of her Voter Registration Record and Certificate of Candidacy, [she] is disqualified from running for
the position of Representative, considering that on election day, May 8, 1995, [she] would have resided less than
ten (10) months in the district where she is seeking to be elected." For its part, the COMELEC's Second Division,
in its resolution of April 24, 1995, cancelled her certificate of candidacy and corrected certificate of candidacy on
the basis of its finding that petitioner is "not qualified to run for the position of Member of the House of
Representatives for the First Legislative District of Leyte" and not because of any finding that she had made false
representations as to material matters in her certificate of candidacy.

Montejo's petition before the COMELEC was therefore not a petition for cancellation of certificate of candidacy
under § 78 of the Omnibus Election Code, but essentially a petition to declare private respondent ineligible. It is
important to note this, because, as will presently be explained, proceedings under § 78 have for their purpose to
disqualify a person from being a candidate, whereas quo warranto proceedings have for their purpose to
disqualify a person from holding public office. Jurisdiction over quo warranto proceedings involving members of
the House of Representatives is vested in the Electoral Tribunal of that body.

Indeed, in the only cases in which this Court dealt with petitions for the cancellation of certificates of candidacy,
the allegations were that the respondent candidates had made false representations in their certificates of
candidacy with regard to their citizenship,1 age,2 or residence.3 But in the generality of cases in which this Court
passed upon the qualifications of respondents for office, this Court did so in the context of election
protests4 or quo warranto proceedings5 filed after the proclamation of the respondents or protestees as winners.

Three reasons may be cited to explain the absence of an authorized proceeding for determining before
election the qualifications of a candidate.

First is the fact that unless a candidate wins and is proclaimed elected, there is no necessity for determining his
eligibility for the office. In contrast, whether an individual should be disqualified as a candidate for acts
constituting election offenses (e.g., vote buying, over spending, commission of prohibited acts) is a prejudicial
question which should be determined lest he wins because of the very acts for which his disqualification is being
sought. That is why it is provided that if the grounds for disqualification are established, a candidate will not be
voted for; if he has been voted for, the votes in his favor will not be counted; and if for some reason he has been
voted for and he has won, either he will not be proclaimed or his proclamation will be set aside. 6

Second is the fact that the determination of a candidate's eligibility, e.g., his citizenship or, as in this case, his
domicile, may take a long time to make, extending beyond the beginning of the term of the office. This is amply
demonstrated in the companion case (G.R. No. 120265, Agapito A. Aquino v. COMELEC) where the
determination of Aquino's residence was still pending in the COMELEC even after the elections of May 8, 1995.
This is contrary to the summary character of proceedings relating to certificates of candidacy. That is why the law
makes the receipt of certificates of candidacy a ministerial duty of the COMELEC and its officers.7 The law is
satisfied if candidates state in their certificates of candidacy that they are eligible for the position which they seek
to fill, leaving the determination of their qualifications to be made after the election and only in the event they are
elected. Only in cases involving charges of false representations made in certificates of candidacy is the
COMELEC given jurisdiction.

Third is the policy underlying the prohibition against pre-proclamation cases in elections for President, Vice
President, Senators and members of the House of Representatives. (R.A. No. 7166, § 15) The purpose is to
preserve the prerogatives of the House of Representatives Electoral Tribunal and the other Tribunals as "sole
judges" under the Constitution of the election, returns and qualifications of members of Congress or of the
President and Vice President, as the case may be.

By providing in § 253 for the remedy of quo warranto for determining an elected official's qualifications after the
results of elections are proclaimed, while being conspicuously silent about a pre-proclamation remedy based on
the same ground, the Omnibus Election Code, or OEC, by its silence underscores the policy of not authorizing
any inquiry into the qualifications of candidates unless they have been elected.

Apparently realizing the lack of an authorized proceeding for declaring the ineligibility of candidates, the
COMELEC amended its rules on February 15, 1993 so as to provide in Rule 25, § 1 the following:

Grounds for disqualification. — Any candidate who does not possess all the qualifications of a
candidate as provided for by the Constitution or by existing law or who commits any act declared
by law to be grounds for disqualification may be disqualified from continuing as a candidate.

The lack of provision for declaring the ineligibility of candidates, however, cannot be supplied by a mere rule.
Such an act is equivalent to the creation of a cause of action which is a substantive matter which the COMELEC,
in the exercise of its rulemaking power under Art. IX, A, § 6 of the Constitution, cannot do. It is noteworthy that
the Constitution withholds from the COMELEC even the power to decide cases involving the right to vote, which
essentially involves an inquiry into qualifications based on age, residence and citizenship of voters. (Art. IX, C, §
2(3))
The assimilation in Rule 25 of the COMELEC rules of grounds for ineligibility into grounds for disqualification is
contrary to the evident intention of the law. For not only in their grounds but also in their consequences are
proceedings for "disqualification" different from those for a declaration of "ineligibility." "Disqualification"
proceedings, as already stated, are based on grounds specified in §§ 12 and 68 of the Omnibus Election Code
and in § 40 of the Local Government Code and are for the purpose of barring an individual from becoming a
candidate or from continuing as a candidate for public office. In a word, their purpose is to eliminate a candidate
from the race either from the start or during its progress. "Ineligibility," on the other hand, refers to the lack of the
qualifications prescribed in the Constitution or the statutes for holding public office and the purpose of the
proceedings for declaration of ineligibility is to remove the incumbent from office.

Consequently, that an individual possesses the qualifications for a public office does not imply that he is not
disqualified from becoming a candidate or continuing as a candidate for a public office and vice versa. We have
this sort of dichotomy in our Naturalization Law. (C.A. No. 473) That an alien has the qualifications prescribed in
§ 2 of the law does not imply that he does not suffer from any of disqualifications provided in § 4.

Indeed, provisions for disqualifications on the ground that the candidate is guilty of prohibited election practices or
offenses, like other pre-proclamation remedies, are aimed at the detestable practice of "grabbing the
proclamation and prolonging the election protest," 8 through the use of "manufactured" election returns or resort to
other trickery for the purpose of altering the results of the election. This rationale does not apply to cases for
determining a candidate's qualifications for office before the election. To the contrary, it is the candidate against
whom a proceeding for disqualification is brought who could be prejudiced because he could be prevented from
assuming office even though in end he prevails.

To summarize, the declaration of ineligibility of a candidate may only be sought in an election protest or action
for quo warranto filed pursuant to § 253 of the Omnibus Election Code within 10 days after his proclamation. With
respect to elective local officials (e.g., Governor, Vice Governor, members of the Sangguniang Panlalawigan,
etc.) such petition must be filed either with the COMELEC, the Regional Trial Courts, or Municipal Trial Courts, as
provided in Art. IX, C, § 2(2) of the Constitution. In the case of the President and Vice President, the petition must
be filed with the Presidential Electoral Tribunal (Art. VII, § 4, last paragraph), and in the case of the Senators, with
the Senate Electoral Tribunal, and in the case of Congressmen, with the House of Representatives Electoral
Tribunal. (Art. VI, § 17) There is greater reason for not allowing before the election the filing of disqualification
proceedings based on alleged ineligibility in the case of candidates for President, Vice President, Senators and
members of the House of Representatives, because of the same policy prohibiting the filing of pre-proclamation
cases against such candidates.

For these reasons, I am of the opinion that the COMELEC had no jurisdiction over SPA No. 95-009; that its
proceedings in that case, including its questioned orders, are void; and that the eligibility of petitioner Imelda
Romualdez-Marcos for the office of Representative of the First District of Leyte may only be inquired into by the
HRET.

Accordingly, I vote to grant the petition and to annul the proceedings of the Commission on Elections in SPA No.
95-009, including its questioned orders doted April 24, 1995, May 7, 1995, May 11, 1995 and May 25, 1995,
declaring petitioner Imelda Romualdez-Marcos ineligible and ordering her proclamation as Representative of the
First District of Leyte suspended. To the extent that Rule 25 of the COMELEC Rules of Procedure authorizes
proceedings for the disqualification of candidates on the ground of ineligibility for the office, it should considered
void.

The provincial board of canvassers should now proceed with the proclamation of petitioner.

Narvasa, C.J., concurs.

PADILLA, J., dissenting:

I regret that I cannot join the majority opinion as expressed in the well-written ponencia of Mr. Justice Kapunan.

As in any controversy arising out of a Constitutional provision, the inquiry must begin and end with the provision
itself. The controversy should not be blurred by what, to me, are academic disquisitions. In this particular
controversy, the Constitutional provision on point states that — "no person shall be a member of the House of
Representatives unless he is a natural-born citizen of the Philippines, and on the day of the election, is at least
twenty-five (25) years of age, able to read and write, and except the party list representatives, a registered voter
in the district in which he shall be elected, and a resident thereof for a period of not less than one year
immediately preceding the day of the election." (Article VI, section 6)

It has been argued that for purposes of our election laws, the term residence has been understood as
synonymous with domicile. This argument has been validated by no less than the Court in numerous
cases1 where significantly the factual circumstances clearly and convincingly proved that a person does not
effectively lose his domicile of origin if the intention to reside therein is manifest with his personal presence in the
place, coupled with conduct indicative of such intention.

With this basic thesis in mind, it would not be difficult to conceive of different modalities within which the phrase
"a resident thereof (meaning, the legislative district) for a period of not less than one year" would fit.

The first instance is where a person's residence and domicile coincide in which case a person only has to prove
that he has been domiciled in a permanent location for not less than a year before the election.

A second situation is where a person maintains a residence apart from his domicile in which case he would have
the luxury of district shopping, provided of course, he satisfies the one-year residence period in the district as the
minimum period for eligibility to the position of congressional representative for the district.

In either case, one would not be constitutionally disqualified for abandoning his residence in order to return to his
domicile of origin, or better still, domicile of choice; neither would one be disqualified for abandoning altogether
his domicile in favor of his residence in the district where he desires to be a candidate.

The most extreme circumstance would be a situation wherein a person maintains several residences in different
districts. Since his domicile of origin continues as an option as long as there is no effective abandonment (animus
non revertendi), he can practically choose the district most advantageous for him.

All these theoretical scenarios, however, are tempered by the unambiguous limitation that "for a period of not less
than one year immediately preceding the day of the election", he must be a resident in the district where he
desires to be elected.

To my mind, the one year residence period is crucial regardless of whether or not the term "residence" is to be
synonymous with "domicile." In other words, the candidate's intent and actual presence in one district must
in all situations satisfy the length of time prescribed by the fundamental law. And this, because of a definite
Constitutional purpose. He must be familiar with the environment and problems of a district he intends to
represent in Congress and the one-year residence in said district would be the minimum period to acquire such
familiarity, if not versatility.

In the case of petitioner Imelda R. Marcos, the operative facts are distinctly set out in the now assailed decision
of the Comelec 2nd Division dated 24 April 1995 (as affirmed by the Comelec en banc) —

In or about 1938 when respondent was a little over 8 years old, she established her domicile in
Tacloban, Leyte (Tacloban City). She studied in the Holy Infant Academy in Tacloban from 1938
to 1948 when she graduated from high school. She pursued her college studies in St. Paul's
College, now Divine Word University of Tacloban, where she earned her degree in Education.
Thereafter, she taught in the Leyte Chinese High School, still in Tacloban City. In 1952 she went
to Manila to work with her cousin, the late Speaker Daniel Z. Romualdez in his office in the House
of Representatives. In 1954, she married ex-president Ferdinand Marcos when he was still a
congressman of Ilocos Norte. She lived with him in Batac, Ilocos Norte and registered there as a
voter. When her husband was elected Senator of the Republic in 1959, she and her husband
lived together in San Juan, Rizal where she registered as a voter. In 1965 when her husband was
elected President of the Republic of the Philippines, she lived with him in Malacanang Palace and
registered as a voter in San Miguel, Manila.

During the Marcos presidency, respondent served as a Member of the Batasang Pambansa,
Minister of Human Settlements and Governor of Metro Manila. She claimed that in February
1986, she and her family were abducted and kidnapped to Honolulu, Hawaii. In November 1991,
she came home to Manila. In 1992 respondent ran for election as President of the Philippines and
filed her Certificate of Candidacy wherein she indicated that she is a resident and registered voter
of San Juan, Metro Manila. On August 24, 1994, respondent filed a letter with the election officer
of San Juan, Metro Manila, requesting for cancellation of her registration in the Permanent List of
Voters in Precinct No. 157 of San Juan, Metro Manila, in order that she may be re-registered or
transferred to Brgy. Olot, Tolosa, Leyte. (Annex 2-B, Answer). On August 31, 1994, respondent
filed her Sworn Application for Cancellation of Voter's Previous Registration (Annex 2-C, Answer)
stating that she is a duly registered voter in 157-A, Brgy. Maytunas, San Juan, Metro that she
intends to register at Brgy. Olot, Tolosa, Leyte.

On January 28, 1995 respondent registered as a voter at Precinct No. 18-A of Olot, Tolosa,
Leyte. She filed with the Board of Election Inspectors CE Form No. 1, Voter Registration Record
No. 94-3349772, wherein she alleged that she has resided in the municipality of Tolosa for a
period of 6 months (Annex A, Petition).
On March 8, 1995, respondent filed with the Office of the Provincial Election Supervisor, Leyte, a
Certificate of Candidacy for the position of Representative of the First District of Leyte wherein
she also alleged that she has been a resident in the constituency where she seeks to be elected
for a period of 7 months. The pertinent entries therein are as follows:

7. PROFESSION OR OCCUPATION: House-wife/ Teacher/ Social


Worker

8. RESIDENCE (complete address): Brgy. Olot, Tolosa, Leyte

Post Office Address for election purposes: Brgy. Olot, Tolosa,


Leyte

9. RESIDENCE IN THE CONSTITUENCY WHEREIN I SEEK TO


BE ELECTED IMMEDIATELY PRECEDING ELECTION:
________ Years Seven Months

10. I AM NOT A PERMANENT RESIDENT OF, OR IMMIGRANT


TO, A FOREIGN COUNTRY.

THAT I AM ELIGIBLE for said office; That I will support and defend the Constitution of the
Republic of the Philippines and will maintain true faith and allegiance thereto; That I will obey the
laws, legal orders and decrees promulgated by the duly-constituted authorities; That the
obligation imposed by my oath is assumed voluntarily, without mental reservation or purpose of
evasion; and That the facts stated herein are true to the best of my knowledge.

(Sgd.) Imelda Romualdez-Marcos


(Signature of Candidate)2

Petitioner's aforestated certificate of candidacy filed on 8 March 1995 contains the decisive component or seed of
her disqualification. It is contained in her answer under oath of "seven months" to the query of "residence in the
constituency wherein I seek to be elected immediately preceding the election."

It follows from all the above that the Comelec committed no grave abuse of discretion in holding that petitioner is
disqualified from the position of representative for the 1st congressional district of Leyte in the elections of
8 May 1995, for failure to meet the "not less than one-year residence in the constituency (1st district, Leyte)
immediately preceding the day of election
(8 May 1995)."

Having arrived at petitioner's disqualification to be a representative of the first district of Leyte, the next important
issue to resolve is whether or not the Comelec can order the Board of Canvassers to determine and proclaim the
winner out of the remaining qualified candidates for representative in said district.

I am not unaware of the pronouncement made by this Court in the case of Labo vs. Comelec, G.R. 86564,
August 1, 1989, 176 SCRA 1 which gave the rationale as laid down in the early 1912 case of Topacio
vs. Paredes, 23 Phil. 238 that:

. . . . Sound policy dictates that public elective offices are filled by those who have received the
highest number of votes cast in the election for that office, and it is a fundamental idea in all
republican forms of government that no one can be declared elected and no measure can be
declared carried unless he or it receives a majority or plurality of the legal votes cast in the
election. (20 Corpus Juris 2nd, S 243, p. 676)

The fact that the candidate who obtained the highest number of votes is later declared to be
disqualified or not eligible for the office to which he was elected does not necessarily entitle the
candidate who obtained the second highest number of votes to be declared the winner of the
elective office. The votes cast for a dead, disqualified, or non-eligible person may not be valid to
vote the winner into office or maintain him there. However, in the absence of a statute which
clearly asserts a contrary political and legislative policy on the matter, if the votes were cast in the
sincere belief that the candidate was alive, qualified, or eligible, they should not be treated as
stray, void or meaningless.

Under Sec. 6 RA 6646, (An Act Introducing Additional Reforms in the Electoral System and for other purposes)
(84 O.G. 905, 22 February 1988) it is provided that:
. . . — Any candidate who has been declared by final judgment to be disqualified shall not be
voted for, and the votes cast for him shall not be counted. If for any reason a candidate is not
declared by final judgment before an election to be disqualified and he is voted for and receives
the winning number of votes in such election, the Court or Commission shall continue with the
trial and hearing of the action, inquiry or protest and, upon motion of the complainant or any
intervenor, may, during the pendency thereof order the suspension of the proclamation of such
candidate whenever the evidence of his guilt is strong.

There is no need to indulge in legal hermeneutics to sense the plain and unambiguous meaning of the provision
quoted above. As the law now stands, the legislative policy does not limit its concern with the effect of a final
judgement of disqualification only before the election, but even during or after the election. The law is clear that in
all situations, the votes cast for a disqualified candidate SHALL NOT BE COUNTED. The law has also validated
the jurisdiction of the Court or Commission on Election to continue hearing the petition for disqualification in case
a candidate is voted for and receives the highest number of votes, if for any reason, he is not declared by final
judgment before an election to be disqualified.

Since the present case is an after election scenario, the power to suspend proclamation (when evidence of his
guilt is strong) is also explicit under the law. What happens then when after the elections are over, one is
declared disqualified? Then, votes cast for him "shall not be counted" and in legal contemplation, he no longer
received the highest number of votes.

It stands to reason that Section 6 of RA 6646 does not make the second placer the winner simply because a
"winning candidate is disqualified," but that the law considers him as the candidate who had obtained the highest
number of votes as a result of the votes cast for the disqualified candidate not being counted or considered.

As this law clearly reflects the legislative policy on the matter, then there is no reason why this Court should not
re-examine and consequently abandon the doctrine in the Jun Labo case. It has been stated that "the
qualifications prescribed for elective office cannot be erased by the electorate alone. The will of the people as
expressed through the ballot cannot cure the vice of ineligibility" most especially when it is mandated by no less
than the Constitution.

ACCORDINGLY, I vote to DISMISS the petition and to order the Provincial Board of Canvassers of Leyte to
proclaim the candidate receiving the highest number of votes, from among the qualified candidates, as the duly
elected representative of the 1st district of Leyte.

Hermosisima, Jr. J., dissent.

REGALADO, J., dissenting:

While I agree with same of the factual bases of the majority opinion, I cannot arrive conjointly at the same
conclusion drawn therefrom Hence, this dissent which assuredly is not formulated "on the basis of the personality
of a petitioner in a case."

I go along with the majority in their narration of antecedent facts, insofar as the same are pertinent to this case,
and which I have simplified as follows:

1. Petitioner, although born in Manila, resided during her childhood in the present Tacloban City,
she being a legitimate daughter of parents who appear to have taken up permanent residence
therein. She also went to school there and, for a time, taught in one of the schools in that city.

2. When she married then Rep. Ferdinand E. Marcos who was then domiciled in Batac, Ilocos
Norte, by operation of law she acquired a new domicile in that place in 1954.

3. In the successive years and during the events that happened thereafter, her husband having
been elected as a Senator and then as President, she lived with him and their family in San Juan,
Rizal and then in Malacanang Palace in San Miguel, Manila.

4. Over those years, she registered as a voter and actually voted in Batac, Ilocos Norte, then in
San Juan, Rizal, and also in San Miguel, Manila, all these merely in the exercise of the right of
suffrage.

5. It does not appear that her husband, even after he had assumed those lofty positions
successively, ever abandoned his domicile of origin in Batac, Ilocos Norte where he maintained
his residence and invariably voted in all elections.
6. After the ouster of her husband from the presidency in 1986 and the sojourn of the Marcos
family in Honolulu, Hawaii, U.S.A., she eventually returned to the Philippines in 1991 and resided
in different places which she claimed to have been merely temporary residences.

7. In 1992, petitioner ran for election as President of the Philippines and in her certificate of
candidacy she indicated that she was then a registered voter and resident of San Juan, Metro
Manila.

8. On August 24, 1994, she filed a letter for the cancellation of her registration in the Permanent
List of Voters in Precinct No. 157 of San Juan, Metro Manila in order that she may "be re-
registered or transferred to Brgy. Olot, Tolosa, Leyte." On August 31, 1994, she followed this up
with her Sworn Application for Cancellation of Voter's Previous Registration wherein she stated
that she was a registered voter in Precinct No. 157-A, Brgy. Maytunas, San Juan, Metro Manila
and that she intended to register in Brgy. Olot, Tolosa, Leyte.

9. On January 28, 1995, petitioner registered as a voter at Precinct No. 18-A of Olot, Tolosa,
Leyte, for which purpose she filed with the therein Board of Election Inspectors a voter's
registration record form alleging that she had resided in that municipality for six months.

10. On March 8, 1995, petitioner filed her certificate of candidacy for the position of
Representative of the First District of Leyte wherein she alleged that she had been a resident for
"Seven Months" of the constituency where she sought to be elected.

11. On March 29, 1995, she filed an "Amended/Corrected Certificate of Candidacy" wherein her
answer in the original certificate of candidacy to item "8. RESIDENCE IN THE CONSTITUENCY
WHERE I SEEK, TO BE ELECTED IMMEDIATELY PRECEDING THE ELECTION:" was
changed or replaced with a new entry reading "SINCE CHILDHOOD."

The sole issue for resolution is whether, for purposes of her candidacy, petitioner had complied with the
residency requirement of one year as mandated by no less than Section 6, Article VI of the 1987 Constitution.

I do not intend to impose upon the time of my colleagues with a dissertation on the difference between residence
and domicile. We have had enough of that and I understand that for purposes of political law and, for that matter
of international law, residence is understood to be synonymous with domicile. That is so understood in our
jurisprudence and in American Law, in contradistinction to the concept of residence for purposes of civil,
commercial and procedural laws whenever an issue thereon is relevant or controlling.

Consequently, since in the present case the question of petitioner's residence is integrated in and inseparable
from her domicile, I am addressing the issue from the standpoint of the concept of the latter term, specifically its
permutations into the domicile of origin, domicile of choice and domicile by operation of law, as understood in
American law from which for this case we have taken our jurisprudential bearings.

My readings inform me that the domicile of the parents at the time of birth, or what is termed the "domicile of
origin," constitutes the domicile of an infant until abandoned, or until the acquisition of a new domicile in a
different place.1 In the instant case, we may grant that petitioner's domicile of origin, 2 at least as of 1938, was
what is now Tacloban City.

Now, as I have observed earlier, domicile is said to be of three kinds, that is, domicile by birth, domicile by
choice, and domicile by operation of law. The first is the common case of the place of birth or domicilium originis,
the second is that which is voluntarily acquired by a party or domicilium propio motu; the last which is
consequential, as that of a wife arising from marriage,3 is sometimes called domicilium necesarium. There is no
debate that the domicile of origin can be lost or replaced by a domicile of choice or a domicile by operation of law
subsequently acquired by the party.

When petitioner contracted marriage in 1954 with then Rep. Marcos, by operation of law, not only international or
American but of our own enactment, 4 she acquired her husband's domicile of origin in Batac, Ilocos Norte and
correspondingly lost her own domicile of origin in Tacloban City.

Her subsequent changes of residence — to San Juan, Rizal, then to San Miguel, Manila, thereafter to Honolulu,
Hawaii, and back to now San Juan, Metro Manila — do not appear to have resulted in her thereby acquiring new
domiciles of choice. In fact, it appears that her having resided in those places was by reason of the fortunes or
misfortunes of her husband and his peregrinations in the assumption of new official positions or the loss of them.
Her residence in Honolulu and, of course, those after her return to the Philippines were, as she claimed, against
her will or only for transient purposes which could not have invested them with the status of domiciles of choice. 5
After petitioner's return to the Philippines in 1991 and up to the present imbroglio over her requisite residency in
Tacloban City or Olot, Tolosa, Leyte, there is no showing that she ever attempted to acquire any other domicile of
choice which could have resulted in the abandonment of her legal domicile in Batac, Ilocos Norte. On that score,
we note the majority's own submission 6 that, to successfully effect a change of domicile, one must demonstrate
(a) an actual removal or an actual change of domicile, (b) a bona fide intention of abandoning the former place of
residence and establishing a new one, and (c) acts which correspond with the purpose.

We consequently have to also note that these requirements for the acquisition of a domicile of choice apply
whether what is sought to be changed or substituted is a domicile of origin (domicilium originis) or a domicile by
operation of law (domicilium necesarium). Since petitioner had lost her domicilium originis which had been
replaced by her domicilium necesarium, it is therefore her continuing domicile in Batac, Ilocos Norte which, if at
all, can be the object of legal change under the contingencies of the case at bar.

To get out of this quandary, the majority decision echoes the dissenting opinion of Commissioner Regalado E.
Maambong in SPA 95-009 of the Commission on Elections,7 and advances this novel proposition.

It may be said that petitioner lost her domicile of origin by operation of law as a result of her
marriage to the late President Ferdinand E. Marcos in 1952 (sic, 1954). By operation of law
(domicilium necesarium), her legal domicile at the time of her marriage became Batac, Ilocos
Norte although there were no indications of an intention on her part to abandon her domicile of
origin. Because of her husband's subsequent death and through the operation of the provisions of
the New Family Code already in force at the time, however, her legal domicile automatically
reverted to her domicile of origin. . . . (Emphasis supplied).

Firstly, I am puzzled why although it is conceded that petitioner had acquired a domicilium necesarium in Batac,
Ilocos Norte, the majority insists on making a qualification that she did not intend to abandon her domicile of
origin. I find this bewildering since, in this situation, it is the law that declares where petitioner's domicile is at any
given time, and not her self-serving or putative intent to hold on to her former domicile. Otherwise, contrary to
their own admission that one cannot have more than one domicile at a time, 8 the majority would be suggesting
that petitioner retained Tacloban City as (for lack of a term in law since it does not exist therein) the equivalent of
what is fancied as a reserved, dormant, potential, or residual domicile.

Secondly, domicile once lost in accordance with law can only be recovered likewise in accordance with law.
However, we are here being titillated with the possibility of an automatic reversion to or reacquisition of a domicile
of origin after the termination of the cause for its loss by operation of law. The majority agrees that since
petitioner lost her domicile of origin by her marriage, the termination of the marriage also terminates that effect
thereof. I am impressed by the ingeniousness of this theory which proves that, indeed, necessity is the mother of
inventions. Regretfully, I find some difficulty in accepting either the logic or the validity of this argument.

If a party loses his domicile of origin by obtaining a new domicile of choice, he thereby voluntarily abandons the
former in favor of the latter. If, thereafter, he abandons that chosen domicile, he does not per se recover his
original domicile unless, by subsequent acts legally indicative thereof, he evinces his intent and desire to
establish the same as his new domicile, which is precisely what petitioner belatedly and, evidently just for
purposes of her candidacy, unsuccessfully tried to do.

One's subsequent abandonment of his domicile of choice cannot automatically restore his domicile of origin, not
only because there is no legal authority therefor but because it would be absurd Pursued to its logical
consequence, that theory of ipso jure reversion would rule out the fact that said party could already very well
have obtained another domicile, either of choice or by operation of law, other than his domicile of origin.
Significantly and obviously for this reason, the Family Code, which the majority inexplicably invokes, advisedly
does not regulate this contingency since it would impinge on one's freedom of choice.

Now, in the instant case, petitioner not only voluntarily abandoned her domicile of choice (unless we assume that
she entered into the marital state against her will) but, on top of that, such abandonment was further affirmed
through her acquisition of a new domicile by operation of law. In fact, this is even a case of
both voluntary and legal abandonment of a domicile of origin. With much more reason, therefore, should we
reject the proposition that with the termination of her marriage in 1989, petitioner had supposedly per se and ipso
facto reacquired her domicile of origin which she lost in 1954. Otherwise, this would be tantamount to saying that
during the period of marital coverture, she was simultaneously in possession and enjoyment of a domicile of
origin which was only in a state of suspended animation.

Thus, the American rule is likewise to the effect that while after the husband's death the wife has the right to elect
her own domicile,9 she nevertheless retains the last domicile of her deceased husband until she makes an actual
change. 10 In the absence of affirmative evidence, to the contrary, the presumption is that a wife's domicile or legal
residence follows that of her husband and will continue after his death. 11
I cannot appreciate the premises advanced in support of the majority's theory based on Articles 68 and 69 of the
Family Code. All that is of any relevance therein is that under this new code, the right and power to fix the family
domicile is now shared by the spouses. I cannot perceive how that joint right, which in the first place was never
exercised by the spouses, could affect the domicile fixed by the law for petitioner in 1954 and, for her husband,
long prior thereto. It is true that a wife now has the coordinate power to determine the conjugal or family domicile,
but that has no bearing on this case. With the death of her husband, and each of her children having gotten
married and established their own respective domiciles, the exercise of that joint power was and is no longer
called for or material in the present factual setting of this controversy. Instead, what is of concern in petitioner's
case was the matter of her having acquired or not her own domicile of choice.

I agree with the majority's discourse on the virtues of the growing and expanded participation of women in the
affairs of the nation, with equal rights and recognition by Constitution and statutory conferment. However, I have
searched in vain for a specific law or judicial pronouncement which either expressly or by necessary implication
supports the majority's desired theory of automatic reacquisition of or reversion to the domicilium originis of
petitioner. Definitely, as between the settled and desirable legal norms that should govern this issue, there is a
world of difference; and, unquestionably, this should be resolved by legislative articulation but not by the
eloquence of the well-turned phrase.

In sum, petitioner having lost Tacloban City as her domicile of origin since 1954 and not having automatically
reacquired any domicile therein, she cannot legally claim that her residency in the political constituency of which
it is a part continued since her birth up to the present. Respondent commission was, therefore, correct in rejecting
her pretension to that effect in her amended/corrected certificate of candidacy, and in holding her to her
admission in the original certificate that she had actually resided in that constituency for only seven months prior
to the election. These considerations render it unnecessary to further pass upon the procedural issues raised by
petitioner.

ON THE FOREGOING PREMISES, I vote to DISMISS the petition for lack of merit.

DAVIDE, JR., J., dissenting:

I respectfully dissent from the opinion of the majority written by Mr. Justice Santiago M. Kapunan, more
particularly on the issue of the petitioner's qualification.

Under Section 7, Subdivision A, Article IX of the Constitution, decisions, orders, or rulings of the COMELEC may
be brought to this Court only by the special civil action for certiorari under Rule 65 of the Rules of Court (Aratuc
vs. COMELEC, 88 SCRA 251 [1979]; Dario vs. Mison, 176 SCRA 84 [1989]).

Accordingly, a writ of certiorari may be granted only if the COMELEC has acted without or in excess of
jurisdiction or with grave abuse of discretion (Section 1, Rule 65, Rules of Court). Since the COMELEC has,
undoubtedly, jurisdiction over the private respondent's petition, the only issue left is whether it acted with grave
abuse of discretion in disqualifying the petitioner.

My careful and meticulous perusal of the challenged resolution of 24 April 1995 of the COMELEC Second
Division and the En Banc resolution of 7 May 1995 discloses total absence of abuse of discretion, much less
grave abuse thereof. The resolution of the Second Division dispassionately and objectively discussed in minute
details the facts which established beyond cavil that herein petitioner was disqualified as a candidate on the
ground of lack of residence in the First Congressional District of Leyte. It has not misapplied, miscomprehended,
or misunderstood facts or circumstances of substance pertinent to the issue of her residence.

The majority opinion, however, overturned the COMELEC's findings of fact for lack of proof that the petitioner has
abandoned Tolosa as her domicile of origin, which is allegedly within the First Congressional District of Leyte.

I respectfully submit that the petitioner herself has provided the COMELEC, either by admission or by
documentary evidence, overwhelming proof of the loss or abandonment of her domicile of origin, which is
Tacloban City and not Tolosa, Leyte. Assuming that she decided to live again in her domicile of origin, that
became her second domicile of choice, where her stay, unfortunately, was for only seven months before the day
of the election. She was then disqualified to be a candidate for the position of Representative of the First
Congressional District of Leyte. A holding to the contrary would be arbitrary.

It may indeed be conceded that the petitioner's domicile of choice was either Tacloban City or Tolosa, Leyte.
Nevertheless, she lost it by operation of law sometime in May 1954 upon her marriage to the then Congressman
(later, President) Ferdinand E. Marcos. A domicile by operation of law is that domicile which the law attributes to
a person, independently of his own intention or actual residence, as results from legal domestic relations as that
of the wife arising from marriage (28 C.J.S. Domicile § 7, 11). Under the governing law then, Article 110 of the
Civil Code, her new domicile or her domicile of choice was the domicile of her husband, which was Batac, Ilocos
Norte. Said Article reads as follows:
Art. 110. The husband shall fix the residence of the family. But the court may exempt the wife
from living with the husband if he should live abroad unless in the service of the Republic.

Commenting thereon, civilist Arturo M. Tolentino states:

Although the duty of the spouses to live together is mutual, the husband has a predominant right
because he is empowered by law to fix the family residence. This right even predominates over
some rights recognized by law in the wife. For instance, under article 117 the wife may engage in
business or practice a profession or occupation. But because of the power of the husband to fix
the family domicile he may fix it at such a place as would make it impossible for the wife to
continue in business or in her profession. For justifiable reasons, however, the wife may be
exempted from living in the residence chosen by the husband. The husband cannot validly allege
desertion by the wife who refuses to follow him to a new place of residence, when it appears that
they have lived for years in a suitable home belonging to the wife, and that his choice of a
different home is not made in good faith. (Commentaries and Jurisprudence on the Civil Code of
the Philippines, vol. 1, 1985 ed., 339).

Under common law, a woman upon her marriage loses her own domicile and, by operation of law, acquires that
of her husband, no matter where the wife actually lives or what she believes or intends. Her domicile is fixed in
the sense that it is declared to be the same as his, and subject to certain limitations, he can change her domicile
by changing his own (25 Am Jur 2d Domicile § 48, 37).

It must, however, be pointed out that under Article 69 of the Family Code, the fixing of the family domicile is no
longer the sole prerogative of the husband, but is now a joint decision of the spouses, and in case of
disagreement the court shall decide. The said article uses the term "family domicile," and not family residence, as
"the spouses may have multiple residences, and the wife may elect to remain in one of such residences, which
may destroy the duty of the spouses to live together and its corresponding benefits" (ALICIA V. SEMPIO-DIY,
Handbook on the Family Code of the Philippines, [1988], 102).

The theory of automatic restoration of a woman's domicile of origin upon the death of her husband, which the
majority opinion adopts to overcome the legal effect of the petitioner's marriage on her domicile, is unsupported
by law and by jurisprudence. The settled doctrine is that after the husband's death the wife has a right to elect her
own domicile, but she retains the last domicile of her husband until she makes an actual change (28 C.J.S.
Domicile § 12, 27). Or, on the death of the husband, the power of the wife to acquire her own domicile is revived,
but until she exercises the power her domicile remains that of the husband at the time of his death (25 Am Jur 2d
Domicile § 62, 45). Note that what is revived is not her domicile of origin but her power to acquire her own
domicile.

Clearly, even after the death of her husband, the petitioner's domicile was that of her husband at the time of his
death — which was Batac, Ilocos Norte, since their residences in San Juan, Metro Manila, and San Miguel,
Manila, were their residences for convenience to enable her husband to effectively perform his official duties.
Their residence in San Juan was a conjugal home, and it was there to which she returned in 1991 when she was
already a widow. In her sworn certificate of candidacy for the Office of the President in the synchronized elections
of May 1992, she indicated therein that she was a resident of San Juan, Metro Manila. She also voted in the said
elections in that place.

On the basis of her evidence, it was only on 24 August 1994 when she exercised her right as a widow to acquire
her own domicile in Tolosa, Leyte, through her sworn statement requesting the Election Officer of San Juan,
Metro Manila, to cancel her registration in the permanent list of voters in Precinct 157 thereat and praying that
she be "re-registered or transferred to Brgy. Olot, Tolosa, Leyte, the place of [her] birth and permanent
residence" (photocopy of Exhibit "B," attached as Annex "2" of private respondent Montejo's Comment). Notably,
she contradicted this sworn statement regarding her place of birth when, in her Voter's Affidavit sworn to on 15
March 1992 (photocopy of Exhibit "C," attached as Annex "3," Id.), her Voter Registration Record sworn to on 28
January 1995 (photocopy of Exhibit "E," attached as Annex "5," Id.), and her Certificate of Candidacy sworn to on
8 March 1995 (photocopy of Exhibit "A," attached as Annex "1," Id.), she solemnly declared that she was born in
Manila.

The petitioner is even uncertain as to her domicile of origin. Is it Tacloban City or Tolosa, Leyte? In the affidavit
attached to her Answer to the petition for disqualification (Annex "I" of Petition), she declared under oath that her
"domicile or residence is Tacloban City." If she did intend to return to such domicile or residence of origin why did
she inform the Election Officer of San Juan that she would transfer to Olot, Tolosa, Leyte, and indicate in her
Voter's Registration Record and in her certificate of candidacy that her residence is Olot, Tolosa, Leyte? While
this uncertainty is not important insofar as residence in the congressional district is concerned, it nevertheless
proves that forty-one years had already lapsed since she had lost or abandoned her domicile of origin by virtue of
marriage and that such length of time diminished her power of recollection or blurred her memory.
I find to be misplaced the reliance by the majority opinion on Faypon vs. Quirino (96 Phil. 294 [1954]), and the
subsequent cases which established the principle that absence from original residence or domicile of origin to
pursue studies, practice one's profession, or engage in business in other states does not constitute loss of such
residence or domicile. So is the reliance on Section 117 of the Omnibus Election Code which provides that
transfer of residence to any other place by reason of one's "occupation; profession; employment in private and
public service; educational activities; work in military or naval reservations; service in the army, navy or air force,
the constabulary or national police force; or confinement or detention in government institutions in accordance
with law" is not deemed as loss of original residence. Those cases and legal provision do not include marriage of
a woman. The reason for the exclusion is, of course, Article 110 of the Civil Code. If it were the intention of this
Court or of the legislature to consider the marriage of a woman as a circumstance which would not operate as an
abandonment of domicile (of origin or of choice), then such cases and legal provision should have expressly
mentioned the same.

This Court should not accept as gospel truth the self-serving claim of the petitioner in her affidavit (Annex "A" of
her Answer in COMELEC SPA No. 95-009; Annex "I" of Petition) that her "domicile or residence of origin is
Tacloban City," and that she "never intended to abandon this domicile or residence of origin to which [she]
always intended to return whenever absent." Such a claim of intention cannot prevail over the effect of Article 110
of the Civil Code. Besides, the facts and circumstances or the vicissitudes of the petitioner's life after her
marriage in 1954 conclusively establish that she had indeed abandoned her domicile of origin and had acquired a
new one animo et facto (KOSSUTH KENT KENNAN, A Treatise on Residence and Domicile, [1934], 214, 326).

Neither should this Court place complete trust on the petitioner's claim that she "merely committed an honest
mistake" in writing down the word "seven" in the space provided for the residency qualification requirement in the
certificate of candidacy. Such a claim is self-serving and, in the light of the foregoing disquisitions, would be all
sound and fury signifying nothing. To me, she did not commit any mistake, honest or otherwise; what she stated
was the truth.

The majority opinion also disregards a basic rule in evidence that he who asserts a fact or the affirmative of an
issue has the burden of proving it (Imperial Victory Shipping Agency vs. NLRC, 200 SCRA 178 [1991]; P.T.
Cerna Corp. vs. Court of Appeals, 221 SCRA 19 [1993]). Having admitted marriage to the then Congressman
Marcos, the petitioner could not deny the legal consequence thereof on the change of her domicile to that of her
husband. The majority opinion rules or at least concludes that "[b]y operation of law (domicilium necesarium), her
legal domicile at the time of her marriage automatically became Batac, Ilocos Norte." That conclusion is
consistent with Article 110 of the Civil Code. Since she is presumed to retain her deceased husband's domicile
until she exercises her revived power to acquire her own domicile, the burden is upon her to prove that she has
exercised her right to acquire her own domicile. She miserably failed to discharge that burden.

I vote to deny the petition.

Separate Opinions

PUNO, J., concurring:

It was Aristotle who taught mankind that things that are alike should be treated alike, while things that are unalike
should be treated unalike in proportion to their unalikeness.1 Like other candidates, petitioner has clearly met the
residence requirement provided by Section 6, Article VI of the Constitution. 2 We cannot disqualify her and treat
her unalike, for the Constitution guarantees equal protection of the law. I proceed from the following factual and
legal propositions:

First. There is no question that petitioner's original domicile is in Tacloban, Leyte. Her parents were domiciled in
Tacloban. Their ancestral house is in Tacloban. They have vast real estate in the place. Petitioner went to school
and thereafter worked there. I consider Tacloban as her initial domicile, both her domicile of origin and her
domicile of choice. Her domicile of origin as it was the domicile of her parents when she was a minor; and her
domicile of choice, as she continued living there even after reaching the age of majority.

Second. There is also no question that in May, 1954, petitioner married the late President Ferdinand E. Marcos.
By contracting marriage, her domicile became subject to change by law, and the right to change it was given by
Article 110 of the Civil Code provides:

Art. 110. The husband shall fix the residence of the family. But the court may exempt the wife
from living with the husband if he should live abroad unless in the service of the
Republic.3 (Emphasis supplied)

In De la Viña v. Villareal and Geopano,4 this Court explained why the domicile of the wife ought to follow
that of the husband. We held: "The reason is founded upon the theoretic identity of person and interest
between the husband and the wife, and the presumption that, from the nature of the relation, the home of
one is the home of the other. It is intended to promote, strengthen, and secure their interests in this
relation, as it ordinarily exists, where union and harmony prevail." 5 In accord with this objective, Article
109 of the Civil Code also obligated the husband and wife "to live together."

Third. The difficult issues start as we determine whether petitioner's marriage to former President Marcos ipso
facto resulted in the loss of her Tacloban domicile. I respectfully submit that her marriage by itself alone did not
cause her to lose her Tacloban domicile. Article 110 of the Civil Code merely gave the husband the right to fix the
domicile of the family. In the exercise of the right, the husband may explicitly choose the prior domicile of his wife,
in which case, the wife's domicile remains unchanged. The husband can also implicitly acquiesce to his wife's
prior domicile even if it is different. So we held in de la Viña,6

. . . . When married women as well as children subject to parental authority live, with the
acquiescence of their husbands or fathers, in a place distinct from where the latter live, they have
their own independent domicile. . . .

It is not, therefore, the mere fact of marriage but the deliberate choice of a different domicile by the
husband that will change the domicile of a wife from what it was prior to their marriage. The domiciliary
decision made by the husband in the exercise of the right conferred by Article 110 of the Civil Code binds
the wife. Any and all acts of a wife during her coverture contrary to the domiciliary choice of the husband
cannot change in any way the domicile legally fixed by the husband. These acts are void not only
because the wife lacks the capacity to choose her domicile but also because they are contrary to law and
public policy.

In the case at bench, it is not disputed that former President Marcos exercised his right to fix the family domicile
and established it in Batac, Ilocos Norte, where he was then the congressman. At that particular point of time and
throughout their married life, petitioner lost her domicile in Tacloban, Leyte. Since petitioner's Batac domicile has
been fixed by operation of law, it was not affected in 1959 when her husband was elected as Senator, when they
lived in San Juan, Rizal and where she registered as a voter. It was not also affected in 1965 when her husband
was elected President, when they lived in Malacañang Palace, and when she registered as a voter in San Miguel,
Manila. Nor was it affected when she served as a member of the Batasang Pambansa, Minister of Human
Settlements and Governor of Metro Manila during the incumbency of her husband as President of the nation.
Under Article 110 of the Civil Code, it was only her husband who could change the family domicile in Batac and
the evidence shows he did not effect any such change. To a large degree, this follows the common law that "a
woman on her marriage loses her own domicile and by operation of law, acquires that of her husband, no matter
where the wife actually lives or what she believes or intends."7

Fourth. The more difficult task is how to interpret the effect of the death on September 28, 1989 of former
President Marcos on petitioner's Batac domicile. The issue is of first impression in our jurisdiction and two (2)
schools of thought contend for acceptance. One is espoused by our distinguished colleague, Mr. Justice Davide,
Jr., heavily relying on American authorities. 8 He echoes the theory that after the husband's death, the wife retains
the last domicile of her husband until she makes an actual change.

I do not subscribe to this submission. The American case law that the wife still retains her dead husband's
domicile is based on ancient common law which we can no longer apply in the Philippine setting today. The
common law identified the domicile of a wife as that of the husband and denied to her the power of acquiring a
domicile of her own separate and apart from him. 9 Legal scholars agree that two (2) reasons support this
common law doctrine. The first reason as pinpointed by the legendary Blackstone is derived from the view that
"the very being or legal existence of the woman is suspended during
the marriage, or at least is incorporated and consolidated into that of the husband." 10 The second reason lies in
"the desirability of having the interests of each member of the family unit governed by the same
law."11 The presumption that the wife retains the domicile of her deceased husband is an extension of this
common law concept. The concept and its extension have provided some of the most iniquitous jurisprudence
against women. It was under common law that the 1873 American case of Bradwell v. Illinois 12 was decided
where women were denied the right to practice law. It was unblushingly ruled that "the natural and proper timidity
and delicacy which belongs to the female sex evidently unfits it for many of the occupations of civil life . . . This is
the law of the Creator." Indeed, the rulings relied upon by Mr. Justice Davide in CJS 13 and AM JUR 2d14 are
American state court decisions handed down between the years 1917 15 and 1938,16 or before the time when
women were accorded equality of rights with men. Undeniably, the women's liberation movement resulted in far-
ranging state legislations in the United States to eliminate gender inequality. 17 Starting in the decade of the
seventies, the courts likewise liberalized their rulings as they started invalidating laws infected with gender-bias. It
was in 1971 when the US Supreme Court in Reed v. Reed,18 struck a big blow for women equality when it
declared as unconstitutional an Idaho law that required probate courts to choose male family members over
females as estate administrators. It held that mere administrative inconvenience cannot justify a sex-based
distinction. These significant changes both in law and in case law on the status of women virtually obliterated the
iniquitous common law surrendering the rights of married women to their husbands based on the dubious theory
of the parties' theoretic oneness. The Corpus Juris Secundum editors did not miss the relevance of this revolution
on women's right as they observed: "However, it has been declared that under modern statutes changing the
status of married women and departing from the common law theory of marriage, there is no reason why a wife
may not acquire a separate domicile for every purpose known to the law."19 In publishing in 1969 the Restatement
of the Law, Second (Conflict of Laws 2d), the reputable American Law Institute also categorically stated that the
view of Blackstone ". . . is no longer held. As the result of statutes and court decisions, a wife now possesses
practically the same rights and powers as her unmarried sister."20

In the case at bench, we have to decide whether we should continue clinging to the anachronistic common
law that demeans women, especially married women. I submit that the Court has no choice except to break away
from this common law rule, the root of the many degradations of Filipino women. Before 1988, our laws
particularly the Civil Code, were full of gender discriminations against women. Our esteemed colleague, Madam
Justice Flerida Ruth Romero, cited a few of them as follows:21

xxx xxx xxx

Legal Disabilities Suffered by Wives

Not generally known is the fact that under the Civil Code, wives suffer under certain restrictions or
disabilities. For instance, the wife cannot accept gifts from others, regardless of the sex of the
giver or the value of the gift, other than from her very close relatives, without her husband's
consent. She may accept only from, say, her parents, parents-in-law, brothers, sisters and the
relatives within the so-called fourth civil degree. She may not exercise her profession or
occupation or engage in business if her husband objects on serious grounds or if his income is
sufficient to support their family in accordance with their social standing. As to what constitutes
"serious grounds" for objecting, this is within the discretion of the husband.

xxx xxx xxx

Because of the present inequitable situation, the amendments to the Civil Law being proposed by
the University of the Philippines Law Center would allow absolute divorce which severes the
matrimonial ties, such that the divorced spouses are free to get married a year after the divorce is
decreed by the courts. However, in order to place the husband and wife on an equal footing
insofar as the bases for divorce are concerned, the following are specified as the grounds for
absolute divorce: (1) adultery or having a paramour committed by the respondent in any of the
ways specified in the Revised Penal Code or (2) an attempt by the respondent against the life of
the petitioner which amounts to attempted parricide under the Revised Penal Code; (3)
abandonment of the petitioner by the respondent without just cause for a period of three
consecutive years; or (4) habitual maltreatment.

With respect to property relations, the husband is automatically the administrator of the conjugal
property owned in common by the married couple even if the wife may be the more astute or
enterprising partner. The law does not leave it to the spouses to decide who shall act as such
administrator. Consequently, the husband is authorized to engage in acts and enter into
transactions beneficial to the conjugal partnership. The wife, however, cannot similarly bind the
partnership without the husband's consent.

And while both exercise joint parental authority over their children, it is the father whom the law
designates as the legal administrator of the property pertaining to the unemancipated child.

Taking the lead in Asia, our government exerted efforts, principally through legislations, to eliminate
inequality between men and women in our land. The watershed came on August 3, 1988 when our
Family Code took effect which, among others, terminated the unequal treatment of husband and wife as
to their rights and responsibilities.22

The Family Code attained this elusive objective by giving new rights to married women and by abolishing sex-
based privileges of husbands. Among others, married women are now given the joint right to administer the
family property, whether in the absolute community system or in the system of conjugal partnership; 23 joint
parental authority over their minor children, both over their persons as well as their properties; 24 joint responsibility
for the support of the family;25 the right to jointly manage the household; 26 and, the right to object to their
husband's exercise of profession, occupation, business or activity. 27 Of particular relevance to the case at bench
is Article 69 of the Family Code which took away the exclusive right of the husband to fix the family domicile and
gave it jointly to the husband and the wife, thus:

Art. 69. The husband and wife shall fix the family domicile. In case of disagreement, the court
shall decide.

The court may exempt one spouse from living with the other if the latter should live abroad or
there are other valid and compelling reasons for the exemption. However, such exemption shall
not apply if the same is not compatible with the solidarity of the family. (Emphasis supplied)
Article 69 repealed Article 110 of the Civil Code. Commenting on the duty of the husband and wife to live
together, former Madam Justice Alice Sempio-Diy of the Court of Appeals specified the instances when a
wife may now refuse to live with her husband, thus:28

(2) The wife has the duty to live with her husband, but she may refuse to do so in certain cases
like:

(a) If the place chosen by the husband as family residence is dangerous to her
Life;

(b) If the husband subjects her to maltreatment or abusive conduct or insults,


making common life impossible;

(c) If the husband compels her to live with his parents, but she cannot get along
with her mother-in-law and they have constant quarrels (Del Rosario v. Del
Rosario, CA, 46 OG 6122);

(d) Where the husband has continuously carried illicit relations for 10 years with
different women and treated his wife roughly and without consideration. (Dadivas
v. Villanueva, 54 Phil. 92);

(e) Where the husband spent his time in gambling, giving no money to his family
for food and necessities, and at the same time insulting his wife and laying hands
on her. (Panuncio v. Sula, CA, 34 OG 129);

(f) If the husband has no fixed residence and lives a vagabond life as a tramp (1
Manresa 329);

(g) If the husband is carrying on a shameful business at home (Gahn v. Darby, 38


La. Ann. 70).

The inescapable conclusion is that our Family Code has completely emancipated the wife from the
control of the husband, thus abandoning the parties' theoretic identity of interest. No less than the late
revered Mr. Justice J.B.L. Reyes who chaired the Civil Code Revision Committee of the UP Law Center
gave this insightful view in one of his rare lectures after retirement: 29

xxx xxx xxx

The Family Code is primarily intended to reform the family law so as to emancipate the wife from
the exclusive control of the husband and to place her at parity with him insofar as the family is
concerned. The wife and the husband are now placed on equal standing by the Code. They are
now joint administrators of the family properties and exercise joint authority over the persons and
properties of their children. This means a dual authority in the family. The husband will no longer
prevail over the wife but she has to agree on all matters concerning the family. (Emphasis
supplied)

In light of the Family Code which abrogated the inequality between husband and wife as started and
perpetuated by the common law, there is no reason in espousing the anomalous rule that the wife still
retains the domicile of her dead husband. Article 110 of the Civil Code which provides the statutory
support for this stance has been repealed by Article 69 of the Family Code. By its repeal, it becomes a
dead-letter law, and we are not free to resurrect it by giving it further effect in any way or manner such as
by ruling that the petitioner is still bound by the domiciliary determination of her dead husband.

Aside from reckoning with the Family Code, we have to consider our Constitution and its firm guarantees of due
process and equal protection of
law.30 It can hardly be doubted that the common law imposition on a married woman of her dead husband's
domicile even beyond his grave is patently discriminatory to women. It is a gender-based discrimination and is
not rationally related to the objective of promoting family solidarity. It cannot survive a constitutional challenge.
Indeed, compared with our previous fundamental laws, the 1987 Constitution is more concerned with equality
between sexes as it explicitly commands that the State ". . . shall ensure fundamental equality before the law of
women and men." To be exact, section 14, Article II provides: "The State recognizes the role of women in nation
building, and shall ensure fundamental equality before the law of women and men. We shall be transgressing the
sense and essence of this constitutional mandate if we insist on giving our women the caveman's treatment.

Prescinding from these premises, I respectfully submit that the better stance is to rule that petitioner reacquired
her Tacloban domicile upon the death of her husband in 1989. This is the necessary consequence of the view
that petitioner's Batac dictated domicile did not continue after her husband's death; otherwise, she would have no
domicile and that will violate the universal rule that no person can be without a domicile at any point of time. This
stance also restores the right of petitioner to choose her domicile before it was taken away by Article 110 of the
Civil Code, a right now recognized by the Family Code and protected by the Constitution. Likewise, I cannot see
the fairness of the common law requiring petitioner to choose again her Tacloban domicile before she could be
released from her Batac domicile. She lost her Tacloban domicile not through her act but through the act of her
deceased husband when he fixed their domicile in Batac. Her husband is dead and he cannot rule her beyond
the grave. The law disabling her to choose her own domicile has been repealed. Considering all these, common
law should not put the burden on petitioner to prove she has abandoned her dead husband's domicile. There is
neither rhyme nor reason for this gender-based burden.

But even assuming arguendo that there is need for convincing proof that petitioner chose to reacquire her
Tacloban domicile, still, the records reveal ample evidence to this effect. In her affidavit submitted to the
respondent COMELEC, petitioner averred:

xxx xxx xxx

36. In November, 1991, I came home to our beloved country, after several requests for my return
were denied by President Corazon C. Aquino, and after I filed suits for our Government to issue
me my passport.

37. But I came home without the mortal remains of my beloved husband, President Ferdinand E.
Marcos, which the Government considered a threat to the national security and welfare.

38. Upon my return to the country, I wanted to immediately live and reside in Tacloban City or in
Olot, Tolosa, Leyte, even if my residences there were not livable as they had been destroyed and
cannibalized. The PCGG, however, did not permit and allow me.

39. As a consequence, I had to live at various times in the Westin Philippine Plaza in Pasay City,
a friend's apartment on Ayala Avenue, a house in South Forbes Park which my daughter rented,
and Pacific Plaza, all in Makati.

40. After the 1992 Presidential Elections, I lived and resided in the residence of my brother in San
Jose, Tacloban City, and pursued my negotiations with PCGG to recover my sequestered
residences in Tacloban City and Barangay Olot, Tolosa, Leyte.

40.1 In preparation for my observance of All Saints' Day and All Souls' Day that
year, I renovated my parents' burial grounds and entombed their bones which had
been excalvated, unearthed and scattered.

41. On November 29, 1993, I formally wrote PCGG Chairman Magtanggol Gunigundo for
permissions to —

. . . rehabilitate . . . (o)ur ancestral house in Tacloban and farmhouse in Olot,


Leyte . . . to make them livable for us the Marcos family to have a home in our
own motherland.

xxx xxx xxx

42. It was only on 06 June 1994, however, when PCGG Chairman Gunigundo, in his letter to Col.
Simeon Kempis, Jr., PCGG Region 8 Representative, allowed me to repair and renovate my
Leyte residences. I quote part of his letter:

Dear Col. Kempis,

Upon representation by Mrs. Imelda R. Marcos to this Commission, that she


intends to visit our sequestered properties in Leyte, please allow her access
thereto. She may also cause repairs and renovation of the sequestered
properties, in which event, it shall be understood that her undertaking said repairs
is not authorization for her to take over said properties, and that all expenses shall
be for her account and not reimbursable. Please extend the necessary courtesy to
her.

xxx xxx xxx


43. I was not permitted, however, to live and stay in the Sto. Niño Shrine residence in Tacloban
City where I wanted to stay and reside, after repairs and renovations were completed. In August
1994, I transferred from San Jose, Tacloban City, to my residence in Barangay Olot, Tolosa,
Leyte, when PCGG permitted me to stay and live there.

It is then clear that in 1992 petitioner reestablished her domicile in the First District of Leyte. It is not
disputed that in 1992, she first lived at the house of her brother in San Jose, Tacloban City and later, in
August 1994, she transferred her residence in Barangay Olot, Tolosa, Leyte. Both Tacloban City and the
municipality of Olot are within the First District of Leyte. Since petitioner reestablished her old domicile in
1992 in the First District of Leyte, she more than complied with the constitutional requirement of
residence
". . . for a period of not less than one year immediately preceding the day of the election," i.e., the May 8,
1995 elections.

The evidence presented by the private respondent to negate the Tacloban domicile of petitioner is nil. He
presented petitioner's Voter's Registration Record filed with the Board of Election Inspectors of Precinct 10-A of
Barangay Olot, Tolosa, Leyte wherein she stated that her period of residence in said barangay was six (6)
months as of the date of her filing of said Voter's Registration Record on January 28, 1995. 31 This statement in
petitioner's Voter's Registration Record is a non-prejudicial admission. The Constitution requires at least one (1)
year residence in the district in which the candidate shall be elected. In the case at bench, the reference is the
First District of Leyte. Petitioner's statement proved that she resided in Olot six (6) months before January 28,
1995 but did not disprove that she has also resided in Tacloban City starting 1992. As aforestated, Olot and
Tacloban City are both within the First District of Leyte, hence, her six (6) months residence in Olot should be
counted not against, but in her favor. Private respondent also presented petitioner's Certificate of Candidacy filed
on March 8, 199532 where she placed seven (7) months after Item No. 8 which called for information regarding
"residence in the constituency where I seek to be elected immediately preceding the election." Again, this original
certificate of candidacy has no evidentiary value because an March 1, 1995 it was corrected by petitioner. In her
Amended/Corrected Certificate of Candidacy,33 petitioner wrote "since childhood" after Item No. 8. The
amendment of a certificate of candidacy to correct a bona fide mistake has been allowed by this Court as a
matter of course and as a matter of right. As we held in Alialy v. COMELEC,34 viz.:

xxx xxx xxx

The absence of the signature of the Secretary of the local chapter N.P in the original certificate of
candidacy presented before the deadline September 11, 1959, did not render the certificate
invalid. The amendment of the certificate, although at a date after the deadline, but before the
election, was substantial compliance with the law, and the defect was cured.

It goes without saying that petitioner's erroneous Certificate of Candidacy filed on March 8, 1995 cannot
be used as evidence against her. Private respondent's petition for the disqualification of petitioner rested
alone on these two (2) brittle pieces of documentary evidence — petitioner's Voter's Registration Record
and her original Certificate of Candidacy. Ranged against the evidence of the petitioner showing her
ceaseless contacts with Tacloban, private respondent's two (2) pieces of evidence are too insufficient to
disqualify petitioner, more so, to deny her the right to represent the people of the First District of Leyte
who have overwhelmingly voted for her.

Fifth. Section 10, Article IX-C of the Constitution mandates that "bona fide candidates for any public office shall
be free from any form of harassment and discrimination."35 A detached reading of the records of the case at
bench will show that all forms of legal and extra-legal obstacles have been thrown against petitioner to prevent
her from running as the people's representative in the First District of Leyte. In petitioner's Answer to the petition
to disqualify her, she averred:36

xxx xxx xxx

10. Petitioner's (herein private respondent Montejo) motive in filing the instant petition is devious.
When respondent (petitioner herein) announced that she was intending to register as a voter in
Tacloban City and run for Congress in the First District of Leyte, petitioner (Montejo) immediately
opposed her intended registration by writing a letter stating that "she is not a resident of said city
but of Barangay Olot, Tolosa, Leyte." (Annex "2" of respondent's affidavit, Annex "2"). After
respondent (petitioner herein) had registered as a voter in Tolosa following completion of her six-
month actual residence therein, petitioner (Montejo) filed a petition with the COMELEC to transfer
the town of Tolosa from the First District to the Second District and pursued such move up to the
Supreme Court in G.R. No. 118702, his purpose being to remove respondent (petitioner herein)
as petitioner's (Montejo's) opponent in the congressional election in the First District. He also filed
a bill, along with other Leyte Congressmen, seeking to create another legislative district, to
remove the town of Tolosa out of the First District and to make it a part of the new district, to
achieve his purpose. However, such bill did not pass the Senate. Having, failed on such moves,
petitioner now filed the instant petition, for the same objective, as it is obvious that he is afraid to
submit himself along with respondent (petitioner herein) for the judgment and verdict of the
electorate of the First District of Leyte in an honest, orderly, peaceful, free and clean elections on
May 8, 1995.

These allegations which private respondent did not challenge were not lost
to the perceptive eye of Commissioner Maambong who in his Dissenting Opinion, 37 held:

xxx xxx xxx

Prior to the registration date — January 28, 1995 the petitioner (herein private respondent
Montejo) wrote the Election Officer of Tacloban City not to allow respondent (petitioner herein) to
register thereat since she is a resident of Tolosa and not Tacloban City. The purpose of this move
of the petitioner (Montejo) is not lost to (sic) the Commission. In UND No. 95-001 (In the matter of
the Legislative Districts of the Provinces of Leyte, Iloilo, and South Cotabato, Out of Which the
New Provinces of Biliran, Guimaras and Saranggani Were Respectively Created), . . . Hon. Cirilo
Roy G. Montejo, Representative, First District of Leyte, wanted the Municipality of Tolosa, in the
First District of Leyte, transferred to the Second District of Leyte. The Hon. Sergio A.F. Apostol,
Representative of the Second District of Leyte, opposed the move of the petitioner (Montejo).
Under Comelec Resolution No. 2736 (December 29, 1994), the Commission on Elections refused
to make the proposed transfer. Petitioner (Montejo) filed "Motion for Reconsideration of
Resolution
No. 2736" which the Commission denied in a Resolution promulgated on February 1, 1995.
Petitioner (Montejo) filed a petition for certiorari before the Honorable Supreme Court (Cirilo Roy
G. Montejo vs. Commission on Elections, G.R. No. 118702) questioning the resolution of the
Commission. Believing that he could get a favorable ruling from the Supreme Court, petitioner
(Montejo) tried to make sure that the respondent (petitioner herein) will register as a voter in
Tolosa so that she will be forced to run as Representative not in the First but in the Second
District.

It did not happen. On March 16, 1995, the Honorable Supreme Court unanimously promulgated a
"Decision," penned by Associate Justice Reynato S. Puno, the dispositive portion of which reads:

IN VIEW WHEREOF, Section 1 of Resolution No. 2736 insofar as it transferred


the municipality of Capoocan of the Second District and the municipality of
Palompon of the Fourth District to the Third District of the province of Leyte, is
annulled and set aside. We also deny the Petition praying for the transfer of the
municipality of Tolosa from the First District to the Second District of the province
of Leyte. No costs.

Petitioner's (Montejo's) plan did not work. But the respondent (petitioner herein) was constrained
to register in the Municipality of Tolosa where her house is instead of Tacloban City, her domicile.
In any case, both Tacloban City and Tolosa are in the First Legislative District.

All these attempts to misuse our laws and legal processes are forms of rank harassments and invidious
discriminations against petitioner to deny her equal access to a public office. We cannot commit any
hermeneutic violence to the Constitution by torturing the meaning of equality, the end result of which will
allow the harassment and discrimination of petitioner who has lived a controversial life, a past of
alternating light and shadow. There is but one Constitution for all Filipinos. Petitioner cannot be adjudged
by a "different" Constitution, and the worst way to interpret the Constitution is to inject in its interpretation,
bile and bitterness.

Sixth. In Gallego v. Vera,38 we explained that the reason for this residence requirement is "to exclude a stranger
or newcomer, unacquainted, with the conditions and needs of a community and not identified with the latter, from
an elective office to serve that community . . . ." Petitioner's lifetime contacts with the First District of Leyte cannot
be contested. Nobody can claim that she is not acquainted with its problems because she is a stranger to the
place. None can argue she cannot satisfy the intent of the Constitution.

Seventh. In resolving election cases, a dominant consideration is the need to effectuate the will of the electorate.
The election results show that petitioner received Seventy Thousand Four Hundred Seventy-one (70,471) votes,
while private respondent got only Thirty-Six Thousand Eight Hundred Thirty-Three (36,833) votes. Petitioner is
clearly the overwhelming choice of the electorate of the First District of Leyte and this is not a sleight of statistics.
We cannot frustrate this sovereign will on highly arguable technical considerations. In case of doubt, we should
lean towards a rule that will give life to the people's political judgment.

A final point. The case at bench provides the Court with the rare opportunity to rectify the inequality of status
between women and men by rejecting the iniquitous common law precedents on the domicile of married women
and by redefining domicile in accord with our own culture, law, and Constitution. To rule that a married woman is
eternally tethered to the domicile dictated by her dead husband is to preserve the anachronistic and anomalous
balance of advantage of a husband over his wife. We should not allow the dead to govern the living even if the
glories of yesteryears seduce us to shout long live the dead! The Family Code buried this gender-based
discrimination against married women and we should not excavate what has been entombed. More importantly,
the Constitution forbids it.

I vote to grant the petition.

Bellosillo and Melo, JJ., concur.

FRANCISCO, J., concurring:

I concur with Mr. Justice Kapunan's ponencia finding petitioner qualified for the position of Representative of the
First Congressional District of Leyte. I wish, however, to express a few comments on the issue of petitioner's
domicile.

Domicile has been defined as that place in which a person's habitation is fixed, without any present intention of
removing therefrom, and that place is properly the domicile of a person in which he has voluntarily fixed his
abode, or habitation, not for a mere special or temporary purpose, but with a present intention of making it his
permanent home (28 C.J.S. §1). It denotes a fixed permanent residence to which when absent for business, or
pleasure, or for like reasons one intends to return, and depends on facts and circumstances, in the sense that
they disclose intent. (Ong Huan Tin v. Republic, 19 SCRA 966, 969)

Domicile is classified into domicile of origin and domicile of choice. The law attributes to every individual a
domicile of origin, which is the domicile of his parents, or of the head of his family, or of the person on whom he is
legally dependent at the time of his birth. While the domicile of origin is generally the place where one is born or
reared, it maybe elsewhere (28 C.J.S. §5). Domicile of choice, on the other hand, is the place which the person
has elected and chosen for himself to displace his previous domicile; it has for its true basis or foundation the
intention of the person (28 C.J.S. §6). In order to hold that a person has abandoned his domicile and acquired a
new one called domicile of choice, the following requisites must concur, namely, (a) residence or bodily presence
in the new locality, (b) intention to remain there or animus manendi, and (c) an intention to abandon the old
domicile or animus non revertendi (Romualdez v. RTC, Br. 7, Tacloban City, 226 SCRA 408, 415). A third
classification is domicile by operation of law which attributes to a person a domicile independent of his own
intention or actual residence, ordinarily resulting from legal domestic relations, as that of the wife arising from
marriage, or the relation of a parent and a child (28 C.J.S. §7).

In election law, when our Constitution speaks of residence for election purposes it means domicile (Co v.
Electoral Tribunal of the House of Representatives, 199 SCRA 692, 713; Nuval v. Guray, 52 Phil. 645, 651). To
my mind, public respondent Commission on Elections misapplied this concept, of domicile which led to
petitioner's disqualification by ruling that petitioner failed to comply with the constitutionally mandated one-year
residence requirement. Apparently, public respondent Commission deemed as conclusive petitioner's stay and
registration as voter in many places as conduct disclosing her intent to abandon her established domicile of origin
in Tacloban, Leyte. In several decisions, though, the Court has laid down the rule that registration of a voter in a
place other than his place of origin is not sufficient to constitute abandonment or loss of such residence (Faypon
v. Quirino, 96 Phil. 294, 300). Respondent Commission offered no cogent reason to depart from this rule except
to surmise petitioner's intent of abandoning her domicile of origin.

It has been suggested that petitioner's domicile of origin was supplanted by a new domicile due to her marriage,
a domicile by operation of law. The proposition is that upon the death of her husband in 1989 she retains her
husband's domicile, i.e., Batac, Ilocos Norte, until she makes an actual change thereof. I find this proposition
quite untenable.

Tacloban, Leyte, is petitioner's domicile of origin which was involuntarily supplanted with another, i.e., Batac,
Ilocos Norte, upon her marriage in 1954 with then Congressman Marcos. By legal fiction she followed the
domicile of her husband. In my view, the reason for the law is for the spouses to fully and effectively perform their
marital duties and obligations to one another. 1 The question of domicile, however, is not affected by the fact that it
was the legal or moral duty of the individual to reside in a given place (28 C.J.S. §11). Thus, while the wife retains
her marital domicile so long as the marriage subsists, she automatically loses it upon the latter's termination, for
the reason behind the law then ceases. Otherwise, petitioner, after her marriage was ended by the death of her
husband, would be placed in a quite absurd and unfair situation of having been freed from all wifely obligations
yet made to hold on to one which no longer serves any meaningful purpose.

It is my view therefore that petitioner reverted to her original domicile of Tacloban, Leyte upon her husband's
death without even signifying her intention to that effect. It is for the private respondent to prove, not for petitioner
to disprove, that petitioner has effectively abandoned Tacloban, Leyte for Batac, Ilocos Norte or for some other
place/s. The clear rule is that it is the party (herein private respondent) claiming that a person has abandoned or
lost his residence of origin who must show and prove preponderantly such abandonment or loss (Faypon v.
Quirino, supra at 298; 28 C.J.S. §16), because the presumption is strongly in favor of an original or former
domicile, as against an acquired one (28 C.J.S. §16). Private respondent unfortunately failed to discharge this
burden as the record is devoid of convincing proof that petitioner has acquired whether voluntarily or involuntarily,
a new domicile to replace her domicile of origin.

The records, on the contrary, clearly show that petitioner has complied with the constitutional one-year residence
requirement. After her exile abroad, she returned to the Philippines in 1991 to reside in Olot, Tolosa, Leyte, but
the Presidential Commission on Good Government which sequestered her residential house and other properties
forbade her necessitating her transient stay in various places in Manila (Affidavit p.6, attached as Annex I of the
Petition). In 1992, she ran for the position of president writing in her certificate of candidacy her residence as San
Juan, Metro Manila. After her loss therein, she went back to Tacloban City, acquired her residence
certificate2 and resided with her brother in San Jose. She resided in San Jose, Tacloban City until August of 1994
when she was allowed by the PCGG to move and reside in her sequestered residential house in Olot, Tolosa,
Leyte (Annex I, p. 6).3 It was in the same month of August when she applied for the cancellation of her previous
registration in San Juan, Metro Manila in order to register anew as voter of Olot, Tolosa, Leyte, which she did on
January 28, 1995. From this sequence of events, I find it quite improper to use as the reckoning period of the
one-year residence requirement the date when she applied for the cancellation of her previous registration in San
Juan, Metro Manila. The fact which private respondent never bothered to disprove is that petitioner transferred
her residence after the 1992 presidential election from San Juan, Metro Manila to San Jose, Tacloban City, and
resided therein until August of 1994. She later transferred to Olot, Tolosa, Leyte (Annex I, p. 7). It appearing that
both Tacloban City and Tolosa, Leyte are within the First Congressional District of Leyte, it indubitably stands that
she had more than a year of residence in the constituency she sought to be elected. Petitioner, therefore, has
satisfactorily complied with the one-year qualification required by the 1987 Constitution.

I vote to grant the petition.

ROMERO, J., separate opinion:

Petitioner has appealed to this Court for relief after the COMELEC ruled that she was disqualified from running
for Representative of her District and that, in the event that she should, nevertheless, muster a majority vote, her
proclamation should be suspended. Not by a straightforward ruling did the COMELEC pronounce its decision as
has been its unvarying practice in the past, but by a startling succession of "reverse somersaults." Indicative of its
shifting stance vis-a-vis petitioner's certificate of candidacy were first, the action of its Second Division
disqualifying her and canceling her original Certificate of Candidacy by a vote of 2-1 on April 24, 1995; then the
denial by the COMELEC en banc of her Motion for Reconsideration on May 7, 1995, a day before the election;
then because she persisted in running, its decision on
May 11, 1995 or three days after the election, allowing her proclamation in the event that the results of the
canvass should show that she obtained the highest number of votes (obviously noting that petitioner had won
overwhelmingly over her opponent), but almost simultaneously reversing itself by directing that even if she wins,
her proclamation should nonetheless be suspended.

Crucial to the resolution of the disqualification issue presented by the case at bench is the interpretation to be
given to the one-year residency requirement imposed by the Constitution on aspirants for a Congressional seat. 1

Bearing in mind that the term "resident" has been held to be synonymous with "domicile" for election purposes, it
is important to determine whether petitioner's domicile was in the First District of Leyte and if so, whether she had
resided there for at least a period of one year. Undisputed is her domicile of origin, Tacloban, where her parents
lived at the time of her birth. Depending on what theory one adopts, the same may have been changed when she
married Ferdinand E. Marcos, then domiciled in Batac, by operation of law. Assuming it did, his death certainly
released her from the obligation to live with him at the residence fixed by him during his lifetime. What may
confuse the layman at this point is the fact that the term "domicile" may refer to "domicile of origin," "domicile of
choice," or "domicile by operation of law," which subject we shall not belabor since it has been amply discussed
by the ponente and in the other separate opinions.

In any case, what assumes relevance is the divergence of legal opinion as to the effect of the husband's death on
the domicile of the widow. Some scholars opine that the widow's domicile remains unchanged; that the deceased
husband's wishes perforce still bind the wife he has left behind. Given this interpretation, the widow cannot
possibly go far enough to sever the domiciliary tie imposed by her husband.

It is bad enough to interpret the law as empowering the husband unilaterally to fix the residence or domicile of the
family, as laid down in the Civil Code,2 but to continue giving obeisance to his wishes even after the rationale
underlying the mutual duty of the spouses to live together has ceased, is to close one's eyes to the stark realities
of the present.

At the other extreme is the position that the widow automatically reverts to her domicile of origin upon the demise
of her husband. Does the law so abhor a vacuum that the widow has to be endowed somehow with a domicile?
To answer this question which is far from rhetorical, one will have to keep in mind the basic principles of domicile.
Everyone must have a domicile. Then one must have only a single domicile for the same purpose at any given
time. Once established, a domicile remains until a new one is acquired, for no person lives who has no domicile,
as defined by the law be is subject to.

At this juncture, we are confronted with an unexplored legal terrain in this jurisdiction, rendered more murky by
the conflicting opinions of foreign legal authorities. This being the state of things, it is imperative as it is opportune
to illumine the darkness with the beacon light of truth, as dictated by experience and the necessity of according
petitioner her right to choose her domicile in keeping with the enlightened global trend to recognize and protect
the human rights of women, no less than men.

Admittedly, the notion of placing women at par with men, insofar as civil, political and social rights are concerned,
is a relatively recent phenomenon that took seed only in the middle of this century. It is a historical fact that for
over three centuries, the Philippines had been colonized by Spain, a conservative, Catholic country which
transplanted to our shores the Old World cultures, mores and attitudes and values. Through the imposition on our
government of the Spanish Civil Code in 1889, the people, both men and women, had no choice but to accept
such concepts as the husband's being the head of the family and the wife's subordination to his authority. In such
role, his was the right to make vital decisions for the family. Many instances come to mind, foremost being what is
related to the issue before us, namely, that "the husband shall fix the residence of the family." 3 Because he is
made responsible for the support of the wife and the rest of the family, 4 he is also empowered to be the
administrator of the conjugal property, with a few exceptions 5 and may, therefore, dispose of the conjugal
partnership property for the purposes specified under the law;6 whereas, as a general rule, the wife
cannot bind the conjugal partnership without the husband's consent. 7 As regards the property pertaining
to the children under parental authority, the father is the legal administrator and only in his absence may
the mother assume his powers.8 Demeaning to the wife's dignity are certain strictures on her personal
freedoms, practically relegating her to the position of minors and disabled persons. To illustrate a few:
The wife cannot, without the husband's consent, acquire any gratuitous title, except from her
ascendants, descendants, parents-in-law, and collateral relatives within the fourth degree.9 With respect
to her employment, the husband wields a veto power in the case the wife exercises her profession or
occupation or engages in business, provided his income is sufficient for the family, according to its
social standing and his opposition is founded on serious and valid grounds. 10 Most offensive, if not
repulsive, to the liberal-minded is the effective prohibition upon a widow to get married till after three
hundred days following the death of her husband, unless in the meantime, she has given birth to a
child. 11 The mother who contracts a subsequent marriage loses the parental authority over her children,
unless the deceased husband, father of the latter, has expressly provided in his will that his widow might
marry again, and has ordered that in such case she should keep and exercise parental authority over
their children. 12 Again, an instance of a husband's overarching influence from beyond the grave.

All these indignities and disabilities suffered by Filipino wives for hundreds of years evoked no protest
from them until the concept of human rights and equality between and among nations and individuals
found hospitable lodgment in the United Nations Charter of which the Philippines was one of the original
signatories. By then, the Spanish "conquistadores" had been overthrown by the American forces at the
turn of the century. The bedrock of the U.N. Charter was firmly anchored on this credo: "to reaffirm faith
in the fundamental human rights, in the dignity and worth of the human person, in the equal rights of
men and women." (Emphasis supplied)

It took over thirty years before these egalitarian doctrines bore fruit, owing largely to the
burgeoning of the feminist movement. What may be regarded as the international bill of rights for
women was implanted in the Convention on the Elimination of All Forms of Discrimination
Against Women (CEDAW) adopted by the U.N. General Assembly which entered into force as an
international treaty on September 3, 1981. In ratifying the instrument, the Philippines bound itself
to implement its liberating spirit and letter, for its Constitution, no less, declared that "The
Philippines. . . adopts the generally accepted principles of international law as part of the law of
the land and adheres to the policy of peace, equality, justice, freedom, cooperation, and amity
with all nations." 13 One such principle embodied in the CEDAW is granting to men and women
"the same rights with regard to the law relating to the movement of persons and the freedom to
choose their residence and domicile." 14 (Emphasis supplied).

CEDAW's pro-women orientation which was not lost on Filipino women was reflected in the 1987
Constitution of the Philippines and later, in the Family Code, 15 both of which were speedily approved by
the first lady President of the country, Corazon C. Aquino. Notable for its emphasis on the human rights
of all individuals and its bias for equality between the sexes are the following provisions: "The State
values the dignity of every human person and guarantees full respect for human rights"16 and "The State
recognizes the role of women in nation-building, and shall ensure the fundamental equality before the
law of women and men."17
A major accomplishment of women in their quest for equality with men and the elimination of discriminatory
provisions of law was the deletion in the Family Code of almost all of the unreasonable strictures on wives and
the grant to them of personal rights equal to that of their husbands. Specifically, the husband and wife are now
given the right jointly to fix the family domicile;18 concomitant to the spouses' being jointly responsible for the
support of the family is the right and duty of both spouses to manage the household; 19 the administration and the
enjoyment of the community property shall belong to both spouses jointly; 20 the father and mother shall now jointly
exercise legal guardianship over the property of their unemancipated common child 21 and several others.

Aware of the hiatus and continuing gaps in the law, insofar as women's rights are concerned, Congress passed a
law popularly known as "Women in Development and Nation Building Act" 22 Among the rights given to married
women evidencing their capacity to act in contracts equal to that of men are:

(1) Women shall have the capacity to borrow and obtain loans and execute security and credit arrangements
under the same conditions as men;

(2) Women shall have equal access to all government and private sector programs granting agricultural credit,
loans and non material resources and shall enjoy equal treatment in agrarian reform and land resettlement
programs;

(3) Women shall have equal rights to act as incorporators and enter into insurance contracts; and

(4) Married women shall have rights equal to those of married men in applying for passports, secure visas and
other travel documents, without need to secure the consent of their spouses.

As the world draws the curtain on the Fourth World Conference of Women in Beijing, let this Court now be the
first to respond to its clarion call that "Women's Rights are Human Rights" and that "All obstacles to women's full
participation in decision-making at all levels, including the family" should be removed. Having been herself a
Member of the Philippine Delegation to the International Women's Year Conference in Mexico in 1975, this writer
is only too keenly aware of the unremitting struggle being waged by women the world over, Filipino women not
excluded, to be accepted as equals of men and to tear down the walls of discrimination that hold them back from
their proper places under the sun.

In light of the inexorable sweep of events, local and global, legislative, executive and judicial, according more
rights to women hitherto denied them and eliminating whatever pockets of discrimination still exist in their civil,
political and social life, can it still be insisted that widows are not at liberty to choose their domicile upon the death
of their husbands but must retain the same, regardless?

I submit that a widow, like the petitioner and others similarly situated, can no longer be bound by the domicile of
the departed husband, if at all she was before. Neither does she automatically revert to her domicile of origin, but
exercising free will, she may opt to reestablish her domicile of origin. In returning to Tacloban and subsequently,
to Barangay Olot, Tolosa, both of which are located in the First District of Leyte, petitioner amply demonstrated
by overt acts, her election of a domicile of choice, in this case, a reversion to her domicile of origin. Added
together, the time when she set up her domicile in the two places sufficed to meet the one-year requirement to
run as Representative of the First District of Leyte.

In view of the foregoing expatiation, I vote to GRANT the petition.

VITUG, J., separate opinion:

The case at bench deals with explicit Constitutional mandates.

The Constitution is not a pliable instrument. It is a bedrock in our legal system that sets up ideals and directions
and render steady our strides hence. It only looks back so as to ensure that mistakes in the past are not
repeated. A compliant transience of a constitution belittles its basic function and weakens its goals. A constitution
may well become outdated by the realities of time. When it does, it must be changed but while it remains, we owe
it respect and allegiance. Anarchy, open or subtle, has never been, nor must it ever be, the answer to perceived
transitory needs, let alone societal attitudes, or the Constitution might lose its very essence.

Constitutional provisions must be taken to be mandatory in character unless, either by express statement or by
necessary implication, a different intention is manifest (see Marcelino vs. Cruz, 121 SCRA 51).

The two provisions initially brought to focus are Section 6 and Section 17 of Article VI of the fundamental law.
These provisions read:

Sec. 6. No person shall be a Member of the House of Representatives unless he is a natural-born


citizen of the Philippines and, on the day of the election, is at least twenty-five years of age, able
to read and write, and, except the party-list representatives, a registered voter in the district in
which he shall be elected, and a resident thereof for a period of not less than one year
immediately preceding the day of the election.

Sec. 17. The Senate and the House of Representatives shall each have an Electoral Tribunal
which shall be the sole judge of all contests relating to the election, returns, and qualifications of
their respective Members. Each Electoral Tribunal shall be composed of nine Members, three of
whom shall be Justices of the Supreme Court to be designated by the Chief Justice, and the
remaining six shall be Members of the Senate or the House of Representatives, as the case may
be, who shall be chosen on the basis of proportional representation from the political parties and
the parties or organizations registered under the party-list system represented therein. The senior
Justice in the Electoral Tribunal shall be its Chairman.

The Commission on Election (the "COMELEC") is constitutionally bound to enforce and administer "all laws and
regulations relative to the conduct of election . . ." (Art. IX, C, Sec. 2, Constitution) that, there being nothing said
to the contrary, should include its authority to pass upon the qualification and disqualification prescribed by law
of candidates to an elective office. Indeed, pre-proclamation controversies are expressly placed under the
COMELEC's jurisdiction to hear and resolve (Art. IX, C, Sec. 3, Constitution).

The matter before us specifically calls for the observance of the constitutional one-year residency requirement.
The issue (whether or not there is here such compliance), to my mind, is basically a question of fact or at least
inextricably linked to such determination. The findings and judgment of the COMELEC, in accordance with the
long established rule and subject only to a number of exceptions under the basic heading of "grave abuse of
discretion," are not reviewable by this Court.

I do not find much need to do a complex exercise on what seems to me to be a plain matter. Generally, the term
"residence" has a broader connotation that may mean permanent (domicile), official (place where one's official
duties may require him to stay) or temporary (the place where he sojourns during a considerable length of time).
For civil law purposes, i.e., as regards the exercise of civil rights and the fulfillment of civil obligations, the
domicile of a natural person is the place of his habitual residence (see Article 50, Civil Code). In election cases,
the controlling rule is that heretofore announced by this Court in Romualdez vs. Regional Trial Court, Branch 7,
Tacloban City (226 SCRA 408, 409); thus:

In election cases, the Court treats domicile and residence as synonymous terms, thus: "(t)he term
"residence" as used in the election law is synonymous with "domicile," which imports not only an
intention to reside in a fixed place but also personal presence in that place, coupled with conduct
indicative of such intention." "Domicile" denotes a fixed permanent residence to which when
absent for business or pleasure, or for like reasons, one intends to return. . . . . Residence thus
acquired, however, may be lost by adopting another choice of domicile. In order, in turn, to
acquire a new domicile by choice, there must concur (1) residence or bodily presence in the new
locality, (2) an intention to remain there, and (3) an intention to abandon the old domicile. In other
words, there must basically be animus manendi coupled with animus non revertendi. The purpose
to remain in or at the domicile of choice must be for an indefinite period of time; the change of
residence must be voluntary; and the residence at the place chosen for the new domicile must be
actual.

Using the above tests, I am not convinced that we can charge the COMELEC with having committed
grave abuse of discretion in its assailed resolution.

The COMELEC's jurisdiction, in the case of congressional elections, ends when the jurisdiction of the Electoral
Tribunal concerned begins. It signifies that the protestee must have theretofore been duly proclaimed and has
since become a "member" of the Senate or the House of Representatives. The question can be asked on
whether or not the proclamation of a candidate is just a ministerial function of the Commission on Elections
dictated solely on the number of votes cast in an election exercise. I believe, it is not. A ministerial duty is an
obligation the performance of which, being adequately defined, does not allow the use of further judgment or
discretion. The COMELEC, in its particular case, is tasked with the full responsibility of ascertaining all the facts
and conditions such as may be required by law before a proclamation is properly done.

The Court, on its part, should, in my view at least, refrain from any undue encroachment on the ultimate exercise
of authority by the Electoral Tribunals on matters which, by no less than a constitutional fiat, are explicitly within
their exclusive domain. The nagging question, if it were otherwise, would be the effect of the Court's peremptory
pronouncement on the ability of the Electoral Tribunal to later come up with its own judgment in a contest
"relating to the election, returns and qualification" of its members.

Prescinding from all the foregoing, I should like to next touch base on the applicability to this case of Section 6 of
Republic Act No. 6646, in relation to Section 72 of Batas Pambansa Blg. 881, each providing thusly:
REPUBLIC ACT NO. 6646

xxx xxx xxx

Sec. 6. Effect of Disqualification Case. — Any candidate who has been declared by final
judgment to be disqualified shall not be voted for, and the votes cast for him shall not be counted.
If for any reason a candidate is not declared by final judgment before an election to be
disqualified and he is voted for and receives the winning number of votes in such election, the
Court or Commission shall continue with the trial and hearing of the action, inquiry or protest and,
upon motion of the complainant or any intervenor, may during the pendency thereof order the
suspension of the proclamation of such candidate whenever the evidence of his guilt is strong.

BATAS PAMBANSA BLG. 881

xxx xxx xxx

Sec. 72. Effects of disqualification cases and priority. — The Commission and the courts shall
give priority to cases of disqualification by reason of violation of this Act to the end that a final
decision shall be rendered not later than seven days before the election in which the
disqualification is sought.

Any candidate who has been declared by final judgment to be disqualified shall not be voted for,
and the votes cast for him shall not be counted. Nevertheless, if for any reason, a candidate is not
declared by final, judgment before an election to be disqualified, and he is voted for and receives
the winning number of votes in such election, his violation of the provisions of the preceding
sections shall not prevent his proclamation and assumption to office.

I realize that in considering the significance of the law, it may be preferable to look for not so much the specific
instances they ostensibly would cover as the principle they clearly convey. Thus, I will not scoff at the argument
that it should be sound to say that votes cast in favor of the disqualified candidate, whenever ultimately declared
as such, should not be counted in his or her favor and must accordingly be considered to be stray votes. The
argument, nevertheless, is far outweighed by the rationale of the now prevailing doctrine first enunciated in the
case of Topacio vs. Paredes (23 Phil. 238 [1912]) which, although later abandoned in Ticzon vs. Comelec (103
SCRA 687 [1981]), and Santos vs. COMELEC (137 SCRA 740 [1985]), was restored, along with the interim case
of Geronimo vs. Ramos (136 SCRA 435 [1985]), by the Labo (176 SCRA 1 (1989]), Abella (201 SCRA 253
[1991]), Labo (211 SCRA 297 [1992]) and, most recently, Benito (235 SCRA 436 [1994]) rulings. Benito
vs. Comelec was a unanimous decision penned by Justice Kapunan and concurred in by Chief Justice Narvasa,
Justices Feliciano, Padilla, Bidin, Regalado, Davide, Romero, Melo, Quiason, Puno, Vitug and Mendoza (Justices
Cruz and Bellosillo were on official leave). For easy reference, let me quote from the first Labo decision:

Finally, there is the question of whether or not the private respondent, who filed the quo
warranto petition, can replace the petitioner as mayor. He cannot. The simple reason is that as he
obtained only the second highest number of votes in the election, he was obviously not the choice
of the people of Baguio City.

The latest ruling of the Court on this issue is Santos v. Commission on Elections, (137 SCRA
740) decided in 1985. In that case, the candidate who placed second was proclaimed elected
after the votes for his winning rival, who was disqualified as a turncoat and considered a non-
candidate, were all disregard as stray. In effect, the second placer won by default. That decision
was supported by eight members of the Court then, (Cuevas, J., ponente, with Makasiar,
Concepcion, Jr., Escolin, Relova, De la Fuente, Alampay and Aquino, JJ., concurring.) with three
dissenting (Teehankee, Acting C.J., Abad Santos and Melencio-Herrera, JJ.) and another two
reserving their vote. (Plana and Gutierrez, Jr., JJ.) One was on official leave. (Fernando, C.J.)

Re-examining that decision, the Court finds, and so holds, that it should be reversed in favor of
the earlier case of Geronimo v. Ramos, (136 SCRA 435) which represents the more logical and
democratic rule. That case, which reiterated the doctrine first announced in 1912 in Topacio
v. Paredes, (23 Phil. 238) was supported by ten members of the Court, (Gutierrez, Jr., ponente,
with Teehankee, Abad Santos, Melencio-Herrera, Plana, Escolin, Relova, De la Fuente, Cuevas
and Alampay, JJ., concurring) without any dissent, although one reserved his vote, (Makasiar, J.)
another took no part, (Aquino, J.) and two others were on leave. (Fernando, C.J. and Concepcion,
Jr., J.) There the Court held:

. . . it would be extremely repugnant to the basic concept of the constitutionally


guaranteed right to suffrage if a candidate who has not acquired the majority or
plurality of votes is proclaimed a winner and imposed as the representative of a
constituency, the majority of which have positively declared through their ballots
that they do not choose him.

Sound policy dictates that public elective offices are filled by those who have
received the highest number of votes cast in the election for that office, and it is a
fundamental idea in all republican forms of government that no one can be
declared elected and no measure can be declared carried unless he or it receives
a majority or plurality of the legal votes cast in the election. (20 Corpus Juris 2nd,
S 243, p. 676.)

The fact that the candidate who obtained the highest number of votes is later declared to be
disqualified or not eligible for the office to which he was elected does not necessarily entitle the
candidate who obtained the second highest number of votes to be declared the winner of the
elective office. The votes cast for a dead, disqualified, or non-eligible person may not be valid to
vote the winner into office or maintain him there. However, in the absence of a statute which
clearly asserts a contrary political and legislative policy on the matter, if the votes were cast in the
sincere belief that the candidate was alive, qualified, or eligible, they should not be treated as
stray, void or meaningless. (at pp. 20-21)

Considering all the foregoing, I am constrained to vote for the dismissal of the petition.

MENDOZA, J., separate opinion:

In my view the issue in this case is whether the Commission on Elections has the power to disqualify candidates
on the ground that they lack eligibility for the office to which they seek to be elected. I think that it has none and
that the qualifications of candidates may be questioned only in the event they are elected, by filing a petition
for quo warranto or an election protest in the appropriate forum, not necessarily in the COMELEC but, as in this
case, in the House of Representatives Electoral Tribunal. That the parties in this case took part in the
proceedings in the COMELEC is of no moment. Such proceedings were unauthorized and were not rendered
valid by their agreement to submit their dispute to that body.

The various election laws will be searched in vain for authorized proceedings for determining a candidate's
qualifications for an office before his election. There are none in the Omnibus Election Code (B.P. Blg. 881), in
the Electoral Reforms Law of 1987 (R.A. No. 6646), or in the law providing for synchronized elections (R.A. No.
7166). There are, in other words, no provisions for pre-proclamation contests but only election protests or quo
warranto proceedings against winning candidates.

To be sure, there are provisions denominated for "disqualification," but they are not concerned with a declaration
of the ineligibility of a candidate. These provisions are concerned with the incapacity (due to insanity,
incompetence or conviction of an offense) of a person either to be a candidate or to continue as a candidate for
public office. There is also a provision for the denial or cancellation of certificates of candidacy, but it applies only
to cases involving false representations as to certain matters required by law to be stated in the certificates.

These provisions are found in the following parts of the Omnibus Election Code:

§ 12. Disqualifications. — Any person who has been declared by competent authority insane or
incompetent, or has been sentenced by final judgment for subversion, insurrection, rebellion or for
any offense for which he has been sentenced to a penalty of more than eighteen months or for a
crime involving moral turpitude, shall be disqualified to be a candidate and to hold any office,
unless he has been given plenary pardon or granted amnesty.

The disqualifications to be a candidate herein provided shall be deemed removed upon the
declaration by competent authority that said insanity or incompetence had been removed or after
the expiration of a period of five years from his service of sentence, unless within the same period
he again becomes disqualified. (Emphasis added)

§ 68. Disqualifications. — Any candidate who, in an action or protest in which he is a party is


declared by final decision of a competent court guilty of, or found by the Commission of having (a)
given money or other material consideration to influence, induce or corrupt the voters or public
officials performing electoral functions; (b) committed acts of terrorism to enhance his candidacy;
(c) spent in his election campaign an amount in excess of that allowed by this Code; (d) solicited,
received or made any contribution prohibited under Sections 89, 95, 96, 97 and 104; or (e)
violated any of Sections 80, 83, 85, 86 and 261, paragraphs d, e, k, v, and cc, sub-paragraph 6,
shall be disqualified from continuing as a candidate, or if he has been elected, from holding the
office. Any person who is a permanent resident of or an immigrant to a foreign country shall not
be qualified to run for any elective office under this Code, unless said person has waived his
status as permanent resident or immigrant of a foreign country in accordance with the residence
requirement provided for in the election laws. (Emphasis added)

§ 78. Petition to deny due course to or cancel a certificate of


candidacy. — A verified petition seeking to deny due course or to cancel a certificate of candidacy
may be filed by any person exclusively on the ground that any material representation contained
therein as required under Section 74 hereof is false. The petition may be filed at any time not later
than twenty-five days from the time of the filing of the certificate of candidacy and shall be
decided, after due notice and hearing, not later than fifteen days before the election. (Emphasis
added)

the Electoral Reforms Law of 1987 (R.A. No. 6646):

§ 6. Effect of Disqualification Case. — Any candidate who has been declared by final judgment to
be disqualified shall not be voted for, and the votes cast for him shall not be counted. If for
any reason a candidate is not declared by final judgment before an election to be disqualified and
he is voted for and receives the winning number of votes in such election, the Court or
Commission shall continue with the trial and hearing of the action, inquiry or protest and; upon
motion for the complainant or any intervenor, may during the pendency thereof order the
suspension of the proclamation of such candidate whenever the evidence of his guilt is strong.
(Emphasis added).

§ 7. Petition to Deny Due Course to or Cancel a Certificate of Candidacy. — The procedure


hereinabove provided shall apply to petitions to deny due course to or cancel a certificate of
candidacy as provided in Section 78 of Batas Pambansa Blg. 881.

and the Local Government Code of 1991 (R.A. No. 7160):

§ 40. Disqualifications. — The following persons are disqualified from running for any elective
local position:

(a) Those sentenced by final judgment for an offense involving moral turpitude or for an offense
punishable by one (1) year or more of imprisonment, within two (2) years after serving sentence;

(b) Those removed from office as a result of on administrative case;

(c) Those convicted by final judgment for violating the oath of allegiance to the Republic;

(d) Those with dual citizenship;

(e) Fugitive from justice in criminal or nonpolitical cases here or abroad;

(f) Permanent residents in a foreign country or those who have acquired the right to reside abroad
and continue to avail of the same right after the effectivity of this Code; and

(g) The insane or feeble-minded.

The petition filed by private respondent Cirilo Roy Montejo in the COMELEC, while entitled "For Cancellation and
Disqualification," contained no allegation that private respondent Imelda Romualdez-Marcos made material
representations in her certificate of candidacy which were false, it sought her disqualification on the ground that
"on the basis of her Voter Registration Record and Certificate of Candidacy, [she] is disqualified from running for
the position of Representative, considering that on election day, May 8, 1995, [she] would have resided less than
ten (10) months in the district where she is seeking to be elected." For its part, the COMELEC's Second Division,
in its resolution of April 24, 1995, cancelled her certificate of candidacy and corrected certificate of candidacy on
the basis of its finding that petitioner is "not qualified to run for the position of Member of the House of
Representatives for the First Legislative District of Leyte" and not because of any finding that she had made false
representations as to material matters in her certificate of candidacy.

Montejo's petition before the COMELEC was therefore not a petition for cancellation of certificate of candidacy
under § 78 of the Omnibus Election Code, but essentially a petition to declare private respondent ineligible. It is
important to note this, because, as will presently be explained, proceedings under § 78 have for their purpose to
disqualify a person from being a candidate, whereas quo warranto proceedings have for their purpose to
disqualify a person from holding public office. Jurisdiction over quo warranto proceedings involving members of
the House of Representatives is vested in the Electoral Tribunal of that body.
Indeed, in the only cases in which this Court dealt with petitions for the cancellation of certificates of candidacy,
the allegations were that the respondent candidates had made false representations in their certificates of
candidacy with regard to their citizenship,1 age,2 or residence.3 But in the generality of cases in which this Court
passed upon the qualifications of respondents for office, this Court did so in the context of election
protests4 or quo warranto proceedings5 filed after the proclamation of the respondents or protestees as winners.

Three reasons may be cited to explain the absence of an authorized proceeding for determining before
election the qualifications of a candidate.

First is the fact that unless a candidate wins and is proclaimed elected, there is no necessity for determining his
eligibility for the office. In contrast, whether an individual should be disqualified as a candidate for acts
constituting election offenses (e.g., vote buying, over spending, commission of prohibited acts) is a prejudicial
question which should be determined lest he wins because of the very acts for which his disqualification is being
sought. That is why it is provided that if the grounds for disqualification are established, a candidate will not be
voted for; if he has been voted for, the votes in his favor will not be counted; and if for some reason he has been
voted for and he has won, either he will not be proclaimed or his proclamation will be set aside. 6

Second is the fact that the determination of a candidate's eligibility, e.g., his citizenship or, as in this case, his
domicile, may take a long time to make, extending beyond the beginning of the term of the office. This is amply
demonstrated in the companion case (G.R. No. 120265, Agapito A. Aquino v. COMELEC) where the
determination of Aquino's residence was still pending in the COMELEC even after the elections of May 8, 1995.
This is contrary to the summary character of proceedings relating to certificates of candidacy. That is why the law
makes the receipt of certificates of candidacy a ministerial duty of the COMELEC and its officers.7 The law is
satisfied if candidates state in their certificates of candidacy that they are eligible for the position which they seek
to fill, leaving the determination of their qualifications to be made after the election and only in the event they are
elected. Only in cases involving charges of false representations made in certificates of candidacy is the
COMELEC given jurisdiction.

Third is the policy underlying the prohibition against pre-proclamation cases in elections for President, Vice
President, Senators and members of the House of Representatives. (R.A. No. 7166, § 15) The purpose is to
preserve the prerogatives of the House of Representatives Electoral Tribunal and the other Tribunals as "sole
judges" under the Constitution of the election, returns and qualifications of members of Congress or of the
President and Vice President, as the case may be.

By providing in § 253 for the remedy of quo warranto for determining an elected official's qualifications after the
results of elections are proclaimed, while being conspicuously silent about a pre-proclamation remedy based on
the same ground, the Omnibus Election Code, or OEC, by its silence underscores the policy of not authorizing
any inquiry into the qualifications of candidates unless they have been elected.

Apparently realizing the lack of an authorized proceeding for declaring the ineligibility of candidates, the
COMELEC amended its rules on February 15, 1993 so as to provide in Rule 25, § 1 the following:

Grounds for disqualification. — Any candidate who does not possess all the qualifications of a
candidate as provided for by the Constitution or by existing law or who commits any act declared
by law to be grounds for disqualification may be disqualified from continuing as a candidate.

The lack of provision for declaring the ineligibility of candidates, however, cannot be supplied by a mere rule.
Such an act is equivalent to the creation of a cause of action which is a substantive matter which the COMELEC,
in the exercise of its rulemaking power under Art. IX, A, § 6 of the Constitution, cannot do. It is noteworthy that
the Constitution withholds from the COMELEC even the power to decide cases involving the right to vote, which
essentially involves an inquiry into qualifications based on age, residence and citizenship of voters. (Art. IX, C, §
2(3))

The assimilation in Rule 25 of the COMELEC rules of grounds for ineligibility into grounds for disqualification is
contrary to the evident intention of the law. For not only in their grounds but also in their consequences are
proceedings for "disqualification" different from those for a declaration of "ineligibility." "Disqualification"
proceedings, as already stated, are based on grounds specified in §§ 12 and 68 of the Omnibus Election Code
and in § 40 of the Local Government Code and are for the purpose of barring an individual from becoming a
candidate or from continuing as a candidate for public office. In a word, their purpose is to eliminate a candidate
from the race either from the start or during its progress. "Ineligibility," on the other hand, refers to the lack of the
qualifications prescribed in the Constitution or the statutes for holding public office and the purpose of the
proceedings for declaration of ineligibility is to remove the incumbent from office.

Consequently, that an individual possesses the qualifications for a public office does not imply that he is not
disqualified from becoming a candidate or continuing as a candidate for a public office and vice versa. We have
this sort of dichotomy in our Naturalization Law. (C.A. No. 473) That an alien has the qualifications prescribed in
§ 2 of the law does not imply that he does not suffer from any of disqualifications provided in § 4.
Indeed, provisions for disqualifications on the ground that the candidate is guilty of prohibited election practices or
offenses, like other pre-proclamation remedies, are aimed at the detestable practice of "grabbing the
proclamation and prolonging the election protest," 8 through the use of "manufactured" election returns or resort to
other trickery for the purpose of altering the results of the election. This rationale does not apply to cases for
determining a candidate's qualifications for office before the election. To the contrary, it is the candidate against
whom a proceeding for disqualification is brought who could be prejudiced because he could be prevented from
assuming office even though in end he prevails.

To summarize, the declaration of ineligibility of a candidate may only be sought in an election protest or action
for quo warranto filed pursuant to § 253 of the Omnibus Election Code within 10 days after his proclamation. With
respect to elective local officials (e.g., Governor, Vice Governor, members of the Sangguniang Panlalawigan,
etc.) such petition must be filed either with the COMELEC, the Regional Trial Courts, or Municipal Trial Courts, as
provided in Art. IX, C, § 2(2) of the Constitution. In the case of the President and Vice President, the petition must
be filed with the Presidential Electoral Tribunal (Art. VII, § 4, last paragraph), and in the case of the Senators, with
the Senate Electoral Tribunal, and in the case of Congressmen, with the House of Representatives Electoral
Tribunal. (Art. VI, § 17) There is greater reason for not allowing before the election the filing of disqualification
proceedings based on alleged ineligibility in the case of candidates for President, Vice President, Senators and
members of the House of Representatives, because of the same policy prohibiting the filing of pre-proclamation
cases against such candidates.

For these reasons, I am of the opinion that the COMELEC had no jurisdiction over SPA No. 95-009; that its
proceedings in that case, including its questioned orders, are void; and that the eligibility of petitioner Imelda
Romualdez-Marcos for the office of Representative of the First District of Leyte may only be inquired into by the
HRET.

Accordingly, I vote to grant the petition and to annul the proceedings of the Commission on Elections in SPA No.
95-009, including its questioned orders doted April 24, 1995, May 7, 1995, May 11, 1995 and May 25, 1995,
declaring petitioner Imelda Romualdez-Marcos ineligible and ordering her proclamation as Representative of the
First District of Leyte suspended. To the extent that Rule 25 of the COMELEC Rules of Procedure authorizes
proceedings for the disqualification of candidates on the ground of ineligibility for the office, it should considered
void.

The provincial board of canvassers should now proceed with the proclamation of petitioner.

Narvasa, C.J., concurs.

PADILLA, J., dissenting:

I regret that I cannot join the majority opinion as expressed in the well-written ponencia of Mr. Justice Kapunan.

As in any controversy arising out of a Constitutional provision, the inquiry must begin and end with the provision
itself. The controversy should not be blurred by what, to me, are academic disquisitions. In this particular
controversy, the Constitutional provision on point states that — "no person shall be a member of the House of
Representatives unless he is a natural-born citizen of the Philippines, and on the day of the election, is at least
twenty-five (25) years of age, able to read and write, and except the party list representatives, a registered voter
in the district in which he shall be elected, and a resident thereof for a period of not less than one year
immediately preceding the day of the election." (Article VI, section 6)

It has been argued that for purposes of our election laws, the term residence has been understood as
synonymous with domicile. This argument has been validated by no less than the Court in numerous
cases1 where significantly the factual circumstances clearly and convincingly proved that a person does not
effectively lose his domicile of origin if the intention to reside therein is manifest with his personal presence in the
place, coupled with conduct indicative of such intention.

With this basic thesis in mind, it would not be difficult to conceive of different modalities within which the phrase
"a resident thereof (meaning, the legislative district) for a period of not less than one year" would fit.

The first instance is where a person's residence and domicile coincide in which case a person only has to prove
that he has been domiciled in a permanent location for not less than a year before the election.

A second situation is where a person maintains a residence apart from his domicile in which case he would have
the luxury of district shopping, provided of course, he satisfies the one-year residence period in the district as the
minimum period for eligibility to the position of congressional representative for the district.
In either case, one would not be constitutionally disqualified for abandoning his residence in order to return to his
domicile of origin, or better still, domicile of choice; neither would one be disqualified for abandoning altogether
his domicile in favor of his residence in the district where he desires to be a candidate.

The most extreme circumstance would be a situation wherein a person maintains several residences in different
districts. Since his domicile of origin continues as an option as long as there is no effective abandonment (animus
non revertendi), he can practically choose the district most advantageous for him.

All these theoretical scenarios, however, are tempered by the unambiguous limitation that "for a period of not less
than one year immediately preceding the day of the election", he must be a resident in the district where he
desires to be elected.

To my mind, the one year residence period is crucial regardless of whether or not the term "residence" is to be
synonymous with "domicile." In other words, the candidate's intent and actual presence in one district must
in all situations satisfy the length of time prescribed by the fundamental law. And this, because of a definite
Constitutional purpose. He must be familiar with the environment and problems of a district he intends to
represent in Congress and the one-year residence in said district would be the minimum period to acquire such
familiarity, if not versatility.

In the case of petitioner Imelda R. Marcos, the operative facts are distinctly set out in the now assailed decision
of the Comelec 2nd Division dated 24 April 1995 (as affirmed by the Comelec en banc) —

In or about 1938 when respondent was a little over 8 years old, she established her domicile in
Tacloban, Leyte (Tacloban City). She studied in the Holy Infant Academy in Tacloban from 1938
to 1948 when she graduated from high school. She pursued her college studies in St. Paul's
College, now Divine Word University of Tacloban, where she earned her degree in Education.
Thereafter, she taught in the Leyte Chinese High School, still in Tacloban City. In 1952 she went
to Manila to work with her cousin, the late Speaker Daniel Z. Romualdez in his office in the House
of Representatives. In 1954, she married ex-president Ferdinand Marcos when he was still a
congressman of Ilocos Norte. She lived with him in Batac, Ilocos Norte and registered there as a
voter. When her husband was elected Senator of the Republic in 1959, she and her husband
lived together in San Juan, Rizal where she registered as a voter. In 1965 when her husband was
elected President of the Republic of the Philippines, she lived with him in Malacanang Palace and
registered as a voter in San Miguel, Manila.

During the Marcos presidency, respondent served as a Member of the Batasang Pambansa,
Minister of Human Settlements and Governor of Metro Manila. She claimed that in February
1986, she and her family were abducted and kidnapped to Honolulu, Hawaii. In November 1991,
she came home to Manila. In 1992 respondent ran for election as President of the Philippines and
filed her Certificate of Candidacy wherein she indicated that she is a resident and registered voter
of San Juan, Metro Manila. On August 24, 1994, respondent filed a letter with the election officer
of San Juan, Metro Manila, requesting for cancellation of her registration in the Permanent List of
Voters in Precinct No. 157 of San Juan, Metro Manila, in order that she may be re-registered or
transferred to Brgy. Olot, Tolosa, Leyte. (Annex 2-B, Answer). On August 31, 1994, respondent
filed her Sworn Application for Cancellation of Voter's Previous Registration (Annex 2-C, Answer)
stating that she is a duly registered voter in 157-A, Brgy. Maytunas, San Juan, Metro that she
intends to register at Brgy. Olot, Tolosa, Leyte.

On January 28, 1995 respondent registered as a voter at Precinct No. 18-A of Olot, Tolosa,
Leyte. She filed with the Board of Election Inspectors CE Form No. 1, Voter Registration Record
No. 94-3349772, wherein she alleged that she has resided in the municipality of Tolosa for a
period of 6 months (Annex A, Petition).

On March 8, 1995, respondent filed with the Office of the Provincial Election Supervisor, Leyte, a
Certificate of Candidacy for the position of Representative of the First District of Leyte wherein
she also alleged that she has been a resident in the constituency where she seeks to be elected
for a period of 7 months. The pertinent entries therein are as follows:

7. PROFESSION OR OCCUPATION: House-wife/ Teacher/ Social


Worker

8. RESIDENCE (complete address): Brgy. Olot, Tolosa, Leyte

Post Office Address for election purposes: Brgy. Olot, Tolosa,


Leyte
9. RESIDENCE IN THE CONSTITUENCY WHEREIN I SEEK TO
BE ELECTED IMMEDIATELY PRECEDING ELECTION:
________ Years Seven Months

10. I AM NOT A PERMANENT RESIDENT OF, OR IMMIGRANT


TO, A FOREIGN COUNTRY.

THAT I AM ELIGIBLE for said office; That I will support and defend the Constitution of the
Republic of the Philippines and will maintain true faith and allegiance thereto; That I will obey the
laws, legal orders and decrees promulgated by the duly-constituted authorities; That the
obligation imposed by my oath is assumed voluntarily, without mental reservation or purpose of
evasion; and That the facts stated herein are true to the best of my knowledge.

(Sgd.) Imelda Romualdez-Marcos


(Signature of Candidate)2

Petitioner's aforestated certificate of candidacy filed on 8 March 1995 contains the decisive component or seed of
her disqualification. It is contained in her answer under oath of "seven months" to the query of "residence in the
constituency wherein I seek to be elected immediately preceding the election."

It follows from all the above that the Comelec committed no grave abuse of discretion in holding that petitioner is
disqualified from the position of representative for the 1st congressional district of Leyte in the elections of 8 May
1995, for failure to meet the "not less than one-year residence in the constituency (1st district, Leyte) immediately
preceding the day of election (8 May 1995)."

Having arrived at petitioner's disqualification to be a representative of the first district of Leyte, the next important
issue to resolve is whether or not the Comelec can order the Board of Canvassers to determine and proclaim the
winner out of the remaining qualified candidates for representative in said district.

I am not unaware of the pronouncement made by this Court in the case of Labo vs. Comelec, G.R. 86564,
August 1, 1989, 176 SCRA 1 which gave the rationale as laid down in the early 1912 case of Topacio
vs. Paredes, 23 Phil. 238 that:

. . . . Sound policy dictates that public elective offices are filled by those who have received the
highest number of votes cast in the election for that office, and it is a fundamental idea in all
republican forms of government that no one can be declared elected and no measure can be
declared carried unless he or it receives a majority or plurality of the legal votes cast in the
election. (20 Corpus Juris 2nd, S 243, p. 676)

The fact that the candidate who obtained the highest number of votes is later declared to be
disqualified or not eligible for the office to which he was elected does not necessarily entitle the
candidate who obtained the second highest number of votes to be declared the winner of the
elective office. The votes cast for a dead, disqualified, or non-eligible person may not be valid to
vote the winner into office or maintain him there. However, in the absence of a statute which
clearly asserts a contrary political and legislative policy on the matter, if the votes were cast in the
sincere belief that the candidate was alive, qualified, or eligible, they should not be treated as
stray, void or meaningless.

Under Sec. 6 RA 6646, (An Act Introducing Additional Reforms in the Electoral System and for other purposes)
(84 O.G. 905, 22 February 1988) it is provided that:

. . . — Any candidate who has been declared by final judgment to be disqualified shall not be
voted for, and the votes cast for him shall not be counted. If for any reason a candidate is not
declared by final judgment before an election to be disqualified and he is voted for and receives
the winning number of votes in such election, the Court or Commission shall continue with the
trial and hearing of the action, inquiry or protest and, upon motion of the complainant or any
intervenor, may, during the pendency thereof order the suspension of the proclamation of such
candidate whenever the evidence of his guilt is strong.

There is no need to indulge in legal hermeneutics to sense the plain and unambiguous meaning of the provision
quoted above. As the law now stands, the legislative policy does not limit its concern with the effect of a final
judgement of disqualification only before the election, but even during or after the election. The law is clear that in
all situations, the votes cast for a disqualified candidate SHALL NOT BE COUNTED. The law has also validated
the jurisdiction of the Court or Commission on Election to continue hearing the petition for disqualification in case
a candidate is voted for and receives the highest number of votes, if for any reason, he is not declared by final
judgment before an election to be disqualified.
Since the present case is an after election scenario, the power to suspend proclamation (when evidence of his
guilt is strong) is also explicit under the law. What happens then when after the elections are over, one is
declared disqualified? Then, votes cast for him "shall not be counted" and in legal contemplation, he no longer
received the highest number of votes.

It stands to reason that Section 6 of RA 6646 does not make the second placer the winner simply because a
"winning candidate is disqualified," but that the law considers him as the candidate who had obtained the highest
number of votes as a result of the votes cast for the disqualified candidate not being counted or considered.

As this law clearly reflects the legislative policy on the matter, then there is no reason why this Court should not
re-examine and consequently abandon the doctrine in the Jun Labo case. It has been stated that "the
qualifications prescribed for elective office cannot be erased by the electorate alone. The will of the people as
expressed through the ballot cannot cure the vice of ineligibility" most especially when it is mandated by no less
than the Constitution.

ACCORDINGLY, I vote to DISMISS the petition and to order the Provincial Board of Canvassers of Leyte to
proclaim the candidate receiving the highest number of votes, from among the qualified candidates, as the duly
elected representative of the 1st district of Leyte.

Hermosisima, Jr. J., dissent.

REGALADO, J., dissenting:

While I agree with same of the factual bases of the majority opinion, I cannot arrive conjointly at the same
conclusion drawn therefrom Hence, this dissent which assuredly is not formulated "on the basis of the personality
of a petitioner in a case."

I go along with the majority in their narration of antecedent facts, insofar as the same are pertinent to this case,
and which I have simplified as follows:

1. Petitioner, although born in Manila, resided during her childhood in the present Tacloban City,
she being a legitimate daughter of parents who appear to have taken up permanent residence
therein. She also went to school there and, for a time, taught in one of the schools in that city.

2. When she married then Rep. Ferdinand E. Marcos who was then domiciled in Batac, Ilocos
Norte, by operation of law she acquired a new domicile in that place in 1954.

3. In the successive years and during the events that happened thereafter, her husband having
been elected as a Senator and then as President, she lived with him and their family in San Juan,
Rizal and then in Malacanang Palace in San Miguel, Manila.

4. Over those years, she registered as a voter and actually voted in Batac, Ilocos Norte, then in
San Juan, Rizal, and also in San Miguel, Manila, all these merely in the exercise of the right of
suffrage.

5. It does not appear that her husband, even after he had assumed those lofty positions
successively, ever abandoned his domicile of origin in Batac, Ilocos Norte where he maintained
his residence and invariably voted in all elections.

6. After the ouster of her husband from the presidency in 1986 and the sojourn of the Marcos
family in Honolulu, Hawaii, U.S.A., she eventually returned to the Philippines in 1991 and resided
in different places which she claimed to have been merely temporary residences.

7. In 1992, petitioner ran for election as President of the Philippines and in her certificate of
candidacy she indicated that she was then a registered voter and resident of San Juan, Metro
Manila.

8. On August 24, 1994, she filed a letter for the cancellation of her registration in the Permanent
List of Voters in Precinct No. 157 of San Juan, Metro Manila in order that she may "be re-
registered or transferred to Brgy. Olot, Tolosa, Leyte." On August 31, 1994, she followed this up
with her Sworn Application for Cancellation of Voter's Previous Registration wherein she stated
that she was a registered voter in Precinct No. 157-A, Brgy. Maytunas, San Juan, Metro Manila
and that she intended to register in Brgy. Olot, Tolosa, Leyte.
9. On January 28, 1995, petitioner registered as a voter at Precinct No. 18-A of Olot, Tolosa,
Leyte, for which purpose she filed with the therein Board of Election Inspectors a voter's
registration record form alleging that she had resided in that municipality for six months.

10. On March 8, 1995, petitioner filed her certificate of candidacy for the position of
Representative of the First District of Leyte wherein she alleged that she had been a resident for
"Seven Months" of the constituency where she sought to be elected.

11. On March 29, 1995, she filed an "Amended/Corrected Certificate of Candidacy" wherein her
answer in the original certificate of candidacy to item "8. RESIDENCE IN THE CONSTITUENCY
WHERE I SEEK, TO BE ELECTED IMMEDIATELY PRECEDING THE ELECTION:" was
changed or replaced with a new entry reading "SINCE CHILDHOOD."

The sole issue for resolution is whether, for purposes of her candidacy, petitioner had complied with the
residency requirement of one year as mandated by no less than Section 6, Article VI of the 1987 Constitution.

I do not intend to impose upon the time of my colleagues with a dissertation on the difference between residence
and domicile. We have had enough of that and I understand that for purposes of political law and, for that matter
of international law, residence is understood to be synonymous with domicile. That is so understood in our
jurisprudence and in American Law, in contradistinction to the concept of residence for purposes of civil,
commercial and procedural laws whenever an issue thereon is relevant or controlling.

Consequently, since in the present case the question of petitioner's residence is integrated in and inseparable
from her domicile, I am addressing the issue from the standpoint of the concept of the latter term, specifically its
permutations into the domicile of origin, domicile of choice and domicile by operation of law, as understood in
American law from which for this case we have taken our jurisprudential bearings.

My readings inform me that the domicile of the parents at the time of birth, or what is termed the "domicile of
origin," constitutes the domicile of an infant until abandoned, or until the acquisition of a new domicile in a
different place.1 In the instant case, we may grant that petitioner's domicile of origin, 2 at least as of 1938, was
what is now Tacloban City.

Now, as I have observed earlier, domicile is said to be of three kinds, that is, domicile by birth, domicile by
choice, and domicile by operation of law. The first is the common case of the place of birth or domicilium originis,
the second is that which is voluntarily acquired by a party or domicilium propio motu; the last which is
consequential, as that of a wife arising from marriage,3 is sometimes called domicilium necesarium. There is no
debate that the domicile of origin can be lost or replaced by a domicile of choice or a domicile by operation of law
subsequently acquired by the party.

When petitioner contracted marriage in 1954 with then Rep. Marcos, by operation of law, not only international or
American but of our own enactment, 4 she acquired her husband's domicile of origin in Batac, Ilocos Norte and
correspondingly lost her own domicile of origin in Tacloban City.

Her subsequent changes of residence — to San Juan, Rizal, then to San Miguel, Manila, thereafter to Honolulu,
Hawaii, and back to now San Juan, Metro Manila — do not appear to have resulted in her thereby acquiring new
domiciles of choice. In fact, it appears that her having resided in those places was by reason of the fortunes or
misfortunes of her husband and his peregrinations in the assumption of new official positions or the loss of them.
Her residence in Honolulu and, of course, those after her return to the Philippines were, as she claimed, against
her will or only for transient purposes which could not have invested them with the status of domiciles of choice. 5

After petitioner's return to the Philippines in 1991 and up to the present imbroglio over her requisite residency in
Tacloban City or Olot, Tolosa, Leyte, there is no showing that she ever attempted to acquire any other domicile of
choice which could have resulted in the abandonment of her legal domicile in Batac, Ilocos Norte. On that score,
we note the majority's own submission 6 that, to successfully effect a change of domicile, one must demonstrate
(a) an actual removal or an actual change of domicile, (b) a bona fide intention of abandoning the former place of
residence and establishing a new one, and (c) acts which correspond with the purpose.

We consequently have to also note that these requirements for the acquisition of a domicile of choice apply
whether what is sought to be changed or substituted is a domicile of origin (domicilium originis) or a domicile by
operation of law (domicilium necesarium). Since petitioner had lost her domicilium originis which had been
replaced by her domicilium necesarium, it is therefore her continuing domicile in Batac, Ilocos Norte which, if at
all, can be the object of legal change under the contingencies of the case at bar.

To get out of this quandary, the majority decision echoes the dissenting opinion of Commissioner Regalado E.
Maambong in SPA 95-009 of the Commission on Elections,7 and advances this novel proposition.
It may be said that petitioner lost her domicile of origin by operation of law as a result of her
marriage to the late President Ferdinand E. Marcos in 1952 (sic, 1954). By operation of law
(domicilium necesarium), her legal domicile at the time of her marriage became Batac, Ilocos
Norte although there were no indications of an intention on her part to abandon her domicile of
origin. Because of her husband's subsequent death and through the operation of the provisions of
the New Family Code already in force at the time, however, her legal domicile automatically
reverted to her domicile of origin. . . . (Emphasis supplied).

Firstly, I am puzzled why although it is conceded that petitioner had acquired a domicilium necesarium in Batac,
Ilocos Norte, the majority insists on making a qualification that she did not intend to abandon her domicile of
origin. I find this bewildering since, in this situation, it is the law that declares where petitioner's domicile is at any
given time, and not her self-serving or putative intent to hold on to her former domicile. Otherwise, contrary to
their own admission that one cannot have more than one domicile at a time, 8 the majority would be suggesting
that petitioner retained Tacloban City as (for lack of a term in law since it does not exist therein) the equivalent of
what is fancied as a reserved, dormant, potential, or residual domicile.

Secondly, domicile once lost in accordance with law can only be recovered likewise in accordance with law.
However, we are here being titillated with the possibility of an automatic reversion to or reacquisition of a domicile
of origin after the termination of the cause for its loss by operation of law. The majority agrees that since
petitioner lost her domicile of origin by her marriage, the termination of the marriage also terminates that effect
thereof. I am impressed by the ingeniousness of this theory which proves that, indeed, necessity is the mother of
inventions. Regretfully, I find some difficulty in accepting either the logic or the validity of this argument.

If a party loses his domicile of origin by obtaining a new domicile of choice, he thereby voluntarily abandons the
former in favor of the latter. If, thereafter, he abandons that chosen domicile, he does not per se recover his
original domicile unless, by subsequent acts legally indicative thereof, he evinces his intent and desire to
establish the same as his new domicile, which is precisely what petitioner belatedly and, evidently just for
purposes of her candidacy, unsuccessfully tried to do.

One's subsequent abandonment of his domicile of choice cannot automatically restore his domicile of origin, not
only because there is no legal authority therefor but because it would be absurd Pursued to its logical
consequence, that theory of ipso jure reversion would rule out the fact that said party could already very well
have obtained another domicile, either of choice or by operation of law, other than his domicile of origin.
Significantly and obviously for this reason, the Family Code, which the majority inexplicably invokes, advisedly
does not regulate this contingency since it would impinge on one's freedom of choice.

Now, in the instant case, petitioner not only voluntarily abandoned her domicile of choice (unless we assume that
she entered into the marital state against her will) but, on top of that, such abandonment was further affirmed
through her acquisition of a new domicile by operation of law. In fact, this is even a case of
both voluntary and legal abandonment of a domicile of origin. With much more reason, therefore, should we
reject the proposition that with the termination of her marriage in 1989, petitioner had supposedly per se and ipso
facto reacquired her domicile of origin which she lost in 1954. Otherwise, this would be tantamount to saying that
during the period of marital coverture, she was simultaneously in possession and enjoyment of a domicile of
origin which was only in a state of suspended animation.

Thus, the American rule is likewise to the effect that while after the husband's death the wife has the right to elect
her own domicile,9 she nevertheless retains the last domicile of her deceased husband until she makes an actual
change. 10 In the absence of affirmative evidence, to the contrary, the presumption is that a wife's domicile or legal
residence follows that of her husband and will continue after his death. 11

I cannot appreciate the premises advanced in support of the majority's theory based on Articles 68 and 69 of the
Family Code. All that is of any relevance therein is that under this new code, the right and power to fix the family
domicile is now shared by the spouses. I cannot perceive how that joint right, which in the first place was never
exercised by the spouses, could affect the domicile fixed by the law for petitioner in 1954 and, for her husband,
long prior thereto. It is true that a wife now has the coordinate power to determine the conjugal or family domicile,
but that has no bearing on this case. With the death of her husband, and each of her children having gotten
married and established their own respective domiciles, the exercise of that joint power was and is no longer
called for or material in the present factual setting of this controversy. Instead, what is of concern in petitioner's
case was the matter of her having acquired or not her own domicile of choice.

I agree with the majority's discourse on the virtues of the growing and expanded participation of women in the
affairs of the nation, with equal rights and recognition by Constitution and statutory conferment. However, I have
searched in vain for a specific law or judicial pronouncement which either expressly or by necessary implication
supports the majority's desired theory of automatic reacquisition of or reversion to the domicilium originis of
petitioner. Definitely, as between the settled and desirable legal norms that should govern this issue, there is a
world of difference; and, unquestionably, this should be resolved by legislative articulation but not by the
eloquence of the well-turned phrase.
In sum, petitioner having lost Tacloban City as her domicile of origin since 1954 and not having automatically
reacquired any domicile therein, she cannot legally claim that her residency in the political constituency of which
it is a part continued since her birth up to the present. Respondent commission was, therefore, correct in rejecting
her pretension to that effect in her amended/corrected certificate of candidacy, and in holding her to her
admission in the original certificate that she had actually resided in that constituency for only seven months prior
to the election. These considerations render it unnecessary to further pass upon the procedural issues raised by
petitioner.

ON THE FOREGOING PREMISES, I vote to DISMISS the petition for lack of merit.

DAVIDE, JR., J., dissenting:

I respectfully dissent from the opinion of the majority written by Mr. Justice Santiago M. Kapunan, more
particularly on the issue of the petitioner's qualification.

Under Section 7, Subdivision A, Article IX of the Constitution, decisions, orders, or rulings of the COMELEC may
be brought to this Court only by the special civil action for certiorari under Rule 65 of the Rules of Court (Aratuc
vs. COMELEC, 88 SCRA 251 [1979]; Dario vs. Mison, 176 SCRA 84 [1989]).

Accordingly, a writ of certiorari may be granted only if the COMELEC has acted without or in excess of
jurisdiction or with grave abuse of discretion (Section 1, Rule 65, Rules of Court). Since the COMELEC has,
undoubtedly, jurisdiction over the private respondent's petition, the only issue left is whether it acted with grave
abuse of discretion in disqualifying the petitioner.

My careful and meticulous perusal of the challenged resolution of 24 April 1995 of the COMELEC Second
Division and the En Banc resolution of 7 May 1995 discloses total absence of abuse of discretion, much less
grave abuse thereof. The resolution of the Second Division dispassionately and objectively discussed in minute
details the facts which established beyond cavil that herein petitioner was disqualified as a candidate on the
ground of lack of residence in the First Congressional District of Leyte. It has not misapplied, miscomprehended,
or misunderstood facts or circumstances of substance pertinent to the issue of her residence.

The majority opinion, however, overturned the COMELEC's findings of fact for lack of proof that the petitioner has
abandoned Tolosa as her domicile of origin, which is allegedly within the First Congressional District of Leyte.

I respectfully submit that the petitioner herself has provided the COMELEC, either by admission or by
documentary evidence, overwhelming proof of the loss or abandonment of her domicile of origin, which is
Tacloban City and not Tolosa, Leyte. Assuming that she decided to live again in her domicile of origin, that
became her second domicile of choice, where her stay, unfortunately, was for only seven months before the day
of the election. She was then disqualified to be a candidate for the position of Representative of the First
Congressional District of Leyte. A holding to the contrary would be arbitrary.

It may indeed be conceded that the petitioner's domicile of choice was either Tacloban City or Tolosa, Leyte.
Nevertheless, she lost it by operation of law sometime in May 1954 upon her marriage to the then Congressman
(later, President) Ferdinand E. Marcos. A domicile by operation of law is that domicile which the law attributes to
a person, independently of his own intention or actual residence, as results from legal domestic relations as that
of the wife arising from marriage (28 C.J.S. Domicile § 7, 11). Under the governing law then, Article 110 of the
Civil Code, her new domicile or her domicile of choice was the domicile of her husband, which was Batac, Ilocos
Norte. Said Article reads as follows:

Art. 110. The husband shall fix the residence of the family. But the court may exempt the wife
from living with the husband if he should live abroad unless in the service of the Republic.

Commenting thereon, civilist Arturo M. Tolentino states:

Although the duty of the spouses to live together is mutual, the husband has a predominant right
because he is empowered by law to fix the family residence. This right even predominates over
some rights recognized by law in the wife. For instance, under article 117 the wife may engage in
business or practice a profession or occupation. But because of the power of the husband to fix
the family domicile he may fix it at such a place as would make it impossible for the wife to
continue in business or in her profession. For justifiable reasons, however, the wife may be
exempted from living in the residence chosen by the husband. The husband cannot validly allege
desertion by the wife who refuses to follow him to a new place of residence, when it appears that
they have lived for years in a suitable home belonging to the wife, and that his choice of a
different home is not made in good faith. (Commentaries and Jurisprudence on the Civil Code of
the Philippines, vol. 1, 1985 ed., 339).
Under common law, a woman upon her marriage loses her own domicile and, by operation of law, acquires that
of her husband, no matter where the wife actually lives or what she believes or intends. Her domicile is fixed in
the sense that it is declared to be the same as his, and subject to certain limitations, he can change her domicile
by changing his own (25 Am Jur 2d Domicile § 48, 37).

It must, however, be pointed out that under Article 69 of the Family Code, the fixing of the family domicile is no
longer the sole prerogative of the husband, but is now a joint decision of the spouses, and in case of
disagreement the court shall decide. The said article uses the term "family domicile," and not family residence, as
"the spouses may have multiple residences, and the wife may elect to remain in one of such residences, which
may destroy the duty of the spouses to live together and its corresponding benefits" (ALICIA V. SEMPIO-DIY,
Handbook on the Family Code of the Philippines, [1988], 102).

The theory of automatic restoration of a woman's domicile of origin upon the death of her husband, which the
majority opinion adopts to overcome the legal effect of the petitioner's marriage on her domicile, is unsupported
by law and by jurisprudence. The settled doctrine is that after the husband's death the wife has a right to elect her
own domicile, but she retains the last domicile of her husband until she makes an actual change (28 C.J.S.
Domicile § 12, 27). Or, on the death of the husband, the power of the wife to acquire her own domicile is revived,
but until she exercises the power her domicile remains that of the husband at the time of his death (25 Am Jur 2d
Domicile § 62, 45). Note that what is revived is not her domicile of origin but her power to acquire her own
domicile.

Clearly, even after the death of her husband, the petitioner's domicile was that of her husband at the time of his
death — which was Batac, Ilocos Norte, since their residences in San Juan, Metro Manila, and San Miguel,
Manila, were their residences for convenience to enable her husband to effectively perform his official duties.
Their residence in San Juan was a conjugal home, and it was there to which she returned in 1991 when she was
already a widow. In her sworn certificate of candidacy for the Office of the President in the synchronized elections
of May 1992, she indicated therein that she was a resident of San Juan, Metro Manila. She also voted in the said
elections in that place.

On the basis of her evidence, it was only on 24 August 1994 when she exercised her right as a widow to acquire
her own domicile in Tolosa, Leyte, through her sworn statement requesting the Election Officer of San Juan,
Metro Manila, to cancel her registration in the permanent list of voters in Precinct 157 thereat and praying that
she be "re-registered or transferred to Brgy. Olot, Tolosa, Leyte, the place of [her] birth and permanent
residence" (photocopy of Exhibit "B," attached as Annex "2" of private respondent Montejo's Comment). Notably,
she contradicted this sworn statement regarding her place of birth when, in her Voter's Affidavit sworn to on 15
March 1992 (photocopy of Exhibit "C," attached as Annex "3," Id.), her Voter Registration Record sworn to on 28
January 1995 (photocopy of Exhibit "E," attached as Annex "5," Id.), and her Certificate of Candidacy sworn to on
8 March 1995 (photocopy of Exhibit "A," attached as Annex "1," Id.), she solemnly declared that she was born in
Manila.

The petitioner is even uncertain as to her domicile of origin. Is it Tacloban City or Tolosa, Leyte? In the affidavit
attached to her Answer to the petition for disqualification (Annex "I" of Petition), she declared under oath that her
"domicile or residence is Tacloban City." If she did intend to return to such domicile or residence of origin why did
she inform the Election Officer of San Juan that she would transfer to Olot, Tolosa, Leyte, and indicate in her
Voter's Registration Record and in her certificate of candidacy that her residence is Olot, Tolosa, Leyte? While
this uncertainty is not important insofar as residence in the congressional district is concerned, it nevertheless
proves that forty-one years had already lapsed since she had lost or abandoned her domicile of origin by virtue of
marriage and that such length of time diminished her power of recollection or blurred her memory.

I find to be misplaced the reliance by the majority opinion on Faypon vs. Quirino (96 Phil. 294 [1954]), and the
subsequent cases which established the principle that absence from original residence or domicile of origin to
pursue studies, practice one's profession, or engage in business in other states does not constitute loss of such
residence or domicile. So is the reliance on Section 117 of the Omnibus Election Code which provides that
transfer of residence to any other place by reason of one's "occupation; profession; employment in private and
public service; educational activities; work in military or naval reservations; service in the army, navy or air force,
the constabulary or national police force; or confinement or detention in government institutions in accordance
with law" is not deemed as loss of original residence. Those cases and legal provision do not include marriage of
a woman. The reason for the exclusion is, of course, Article 110 of the Civil Code. If it were the intention of this
Court or of the legislature to consider the marriage of a woman as a circumstance which would not operate as an
abandonment of domicile (of origin or of choice), then such cases and legal provision should have expressly
mentioned the same.

This Court should not accept as gospel truth the self-serving claim of the petitioner in her affidavit (Annex "A" of
her Answer in COMELEC SPA No. 95-009; Annex "I" of Petition) that her "domicile or residence of origin is
Tacloban City," and that she "never intended to abandon this domicile or residence of origin to which [she]
always intended to return whenever absent." Such a claim of intention cannot prevail over the effect of Article 110
of the Civil Code. Besides, the facts and circumstances or the vicissitudes of the petitioner's life after her
marriage in 1954 conclusively establish that she had indeed abandoned her domicile of origin and had acquired a
new one animo et facto (KOSSUTH KENT KENNAN, A Treatise on Residence and Domicile, [1934], 214, 326).

Neither should this Court place complete trust on the petitioner's claim that she "merely committed an honest
mistake" in writing down the word "seven" in the space provided for the residency qualification requirement in the
certificate of candidacy. Such a claim is self-serving and, in the light of the foregoing disquisitions, would be all
sound and fury signifying nothing. To me, she did not commit any mistake, honest or otherwise; what she stated
was the truth.

The majority opinion also disregards a basic rule in evidence that he who asserts a fact or the affirmative of an
issue has the burden of proving it (Imperial Victory Shipping Agency vs. NLRC, 200 SCRA 178 [1991]; P.T.
Cerna Corp. vs. Court of Appeals, 221 SCRA 19 [1993]). Having admitted marriage to the then Congressman
Marcos, the petitioner could not deny the legal consequence thereof on the change of her domicile to that of her
husband. The majority opinion rules or at least concludes that "[b]y operation of law (domicilium necesarium), her
legal domicile at the time of her marriage automatically became Batac, Ilocos Norte." That conclusion is
consistent with Article 110 of the Civil Code. Since she is presumed to retain her deceased husband's domicile
until she exercises her revived power to acquire her own domicile, the burden is upon her to prove that she has
exercised her right to acquire her own domicile. She miserably failed to discharge that burden.

I vote to deny the petition.

Footnotes

1 Jarrolt v. Mabberly, 103 U.S. 580 (1881).

2 CONST, art. VI, states:

Sec. 6. No person shall be a member of the House of Representatives unless he is a


natural-born citizen of the Philippines and, on the day of the election, is at least twenty-
five years of age, able to read and write, and except the party-list representatives, a
registered voter in the district in which he shall be elected, and a resident thereof for a
period of not less than one year immediately preceding the day of the election.

See, Jarrolt v. Mabberly, supra, note 1.

3 Gallego vs. Vera, 73 Phil. 453 (1941).

4 Rollo, p. 114, Annex "D".

5 Rollo, p. 110, Annex "D".

6 Rollo, p. 113.

7 Rollo, p. 111.

8 Rollo, p. 115, Annex "E".

9 Signed by Virgilo S. Oledan, Provincial Election Supervisor IV, Leyte; Rollo,


p. 116, Annex "F".

10 Rollo, p. 117, Annex "G". Petitioner explained the circumstances surrounding the filling
up of the original certificate thus:

1. On March 8, 1995, I filed my certificate of candidacy for Member of the House of


Representatives (Congresswoman) of the First Legislative District of the province of
Leyte, which was drafted by Mr. Filomeno A. Zeta.

2. I learned lately that Congressman Cirilo Montejo wants to disqualify me as I allegedly


lack residence in the constituency because of the entry of the word "SEVEN" in Item No. 8
of my certificate of candidacy.

3. I read my certificate of candidacy before signing it and thought of the word


"RESIDENCE" to mean actual or physical residence, and the word "SEVEN" merely
reflected my actual and physical residence in Barangay Olot, Tolosa, Leyte.
3.1. The word "SEVEN" was placed on my certificate of candidacy to indicate that at lease
one (1) month had passed from my registration as voter of Tolosa, Leyte, on January 28,
1995, when I wrote "06" months under "PERIOD OF RESIDENCE" as my actual or
physical residence in the town.

4. I thought then that the sense in Item No. 10 of my certificate of candidacy stating
"THAT I AM eligible for said Office" was sufficient to affirm that I possess all the
qualifications, including my residence, for Member of the House of Representatives for
which I am aspiring in the May 8, 1995 elections.

5. The fact, however, is that my domicile or residence of origin is Tacloban City, a


component city of the First Legislative District of Leyte I never intended to abandon this
domicile or residence of origin to which I always intended to return whenever absent;
indeed in 1992, I returned to Tacloban City to live and stay there. On November 5, 1992; I
bought my Residence Certificate No. 15226186L there, which is made an integral part
hereof as Annex "I" (Annex "2" hereof).

11 Id., at p. 120. See also, Rollo, p. 130-133, Annex "I", petitioner's Affidavit explaining
her residence:

13. I established my domicile, however in Tacloban, Leyte (Tacloban City in 1938, when
was little over eight (8) years old. Shortly after my mother died on April 7, 1938, my
widowed father, Vicente Orestes Romualdez, brought me and my brothers. . .and my
sisters to Tacloban, Leyte (now Tacloban City) his hometown.

xxx xxx xxx

18. I have always considered Tacloban City as my permanent residence or residence of


origin have not abandoned and have never intended to abandon my permanent residence
or residence of origin there. To it I always intend to return whenever absent.

19. In 1952, I went to Manila to work with my cousin, the late speaker
Daniel Z. Romualdez in his office in the House of Representatives.

20. In May, 1954, I married President Ferdinand E. Marcos when he was still the
congressman of Ilocos, Norte.

21. As a dutiful wife who loved him deeply, I lived with him in Batac, Ilocos Norte and
registered as a voter there.

22. In 1965, my husband was elected President of the Republic of the Philippines.
Together, we lived in Malacañang Palace and I registered as a voter in San Miguel,
Manila.

23. My registration as voter in Batac, Ilocos Norte; San Juan, Rizal (now San Juan, Metro
Manila); and San Miguel, Manila, was for convenience because I had to live with my
husband to serve him when he was congressman, Senator and President of the Republic
of the Philippines. During those years however, I never intended nor desired to abandon
my domicile or residence of origin in Tacloban City, which I established since I was a
child.

xxx xxx xxx

33. Throughout the Marcos Presidency, I spent most of my birthday anniversaries and
attended the Sto. Nini Fiesta in Tacloban City. I regularly visited my domicile or residence
of origin in Leyte and even held important functions and entertained guests and foreign
dignitaries there.

34. After President Ferdinand E. Marcos and I, together with our children and innocent
grandchildren were abducted and kidnapped to Honolulu, Hawaii, in February, 1986, my
Leyte properties were sequestered by the PCGG, and were destroyed and cannibalized.

xxx xxx xxx


38. Upon my return to the country, I wanted to immediately live and reside in Tacloban
City or in Olot, Tolosa, Leyte even if my residences there were not livable as they had
been destroyed and cannibalized. The PCGG, however, did not permit and allow me.

xxx xxx xxx

40. After the 1992 Presidential Elections, I lived and resided in the residence of my
brother in San Jose, Tacloban City, and pursued my negotiations with PCGG to recover
my sequestered residences in Tacloban City and Barangay Olot, Tolosa, Leyte.

12 Rollo, p. 122.

13 Commissioners Manolo B. Gorospe and Teresita Dy-Liaco Flores formed the majority
opinion. Commissioner Remedies A. Salazar-Fernando dissented.

14 Rollo, p. 64.

15 Rollo, p. 57-64.

16 Petitioner filed a "Motion to Recall Resolution Promulgated on April 24, 1995 and to
Dismiss the Petition Because of Lapse of Jurisdiction; Alternatively, Motion for
Reconsideration." The Commission's May 7, 1995 Resolution treated the same simply as
a Motion for Reconsideration.

17 Commissioners Regalado E. Maambong, Remedios A. Salazar-Fernando and Julio F.


Desamito dissented. All filed separate dissenting opinions. In disqualifying petitioner, the
majority held:

As it stands now, only the Certificate of Candidacy respondent filed on March 8, 1995,
stands, and on the basis of the entries therein, she is disqualified to run for failure to meet
the constitutional requirement of one (1) year of residence in the place where she wanted
to be elected.

18 Rollo, p. 78, Annex "B".

19 Rollo, Annex "D".

20 19 SCRA 966 (1967). See also, Corre v. Corre, 100 Phil. 221 (1956).

21 Id. at 969.

22 Uytengsu v. Republic, 95 Phil. 890 (1954).

23 Id.

24 52 Phil. 645 (1928).

25 Citing People v. Bender 144 N.Y.S., 145.

26 61 Phil. 36 (1934).

27 96 Phil. 294 (1954).

28 Id, see also Ujano v. Republic, 17 SCRA 147 (1966); Nuval v. Guray, supra

note 22.

29 II RECORD OF THE 1987 CONSTITUTIONAL CONVENTION, 110 (July 22, 1986).

30 Id.

31 199 SCRA 692 (1991).

32 Id, at 714.
33 61 Phil. 36 (1934).

34 96 Phil. 294, 299-300 (1954).

35 B.P. 881, sec. 117 states:

xxx xxx xxx

Any person who transfers residence to another city, municipality or country solely by
reason of his occupation; profession; employment in private or public service; educational
activities; work in military or naval reservations; service in the army, navy or air force; the
constabulary or national police force; or confinement or detention in government
institutions in accordance with law shall not be deemed to have lost his original residence.

36 Rollo, p. 38.

37 18 Am Jur 219-220.

38 20 Am Jur 71.

39 TOLENTINO 1 COMMENTARIES & JURISPRUDENCE ON THE CIVIL CODE, 220


(1987).

40 Id.

41 TOLENTINO, 1 COMMENTARIES AND JURISPRUDENCE ON CIVIL CODE, 220


(1987).

42 Under modern laws, it is clear that many exceptions to the rule that the domicile of the
wife is determined by that of her husband must obtain. Accordingly, the wife may acquire
another and separate domicile from that of her husband where the theoretical unity of the
husband and wife is dissolved, as it is by the institution of divorce proceedings; or where
the husband has given cause for divorce; or where there is a separation of the parties by
agreement, or a permanent separation due to desertion of the wife by the husband or
attributable to cruel treatment on the part of the husband; or where there has been a
forfeiture by the wife of the benefit of the husband's domicile. 9 R.C.L., 545, cited in De La
Vina, supra. If the law allows the wife to automatically revert to her original domicile or
acquire a new domicile under these situations, all the more should it sanction a reversion
— or the acquisition of a new domicile by the wife — upon the death of her husband.

43 41 Phi. 13 (1920).

44 The rule that the wife automatically acquires or follows her husband's domicile is not
an absolute one. A specific situation recognized in Spanish jurisprudence involves the
one in which husband acquiesces (1 Manresa 223) or gives his tacit consent (Scaevola,
Civil Code; 354.)

45 42 Phil. 54 (1921).

46 Justice Alicia Sempio-Diy recognizes the same Civil Code distinction. However, taking
another approach, she writes:

(6) The above Article (Article 69, FC) uses the term "family domicile" instead of family
residence because the spouses may have multiple residences, and the wife may elect to
remain in one of such residences, which may destroy the duty of the spouses to live
together and its corresponding benefits. SEMPIO-DIY, HANDBOOK ON THE FAMILY
CODE OF THE PHILIPPINES, 102 (1988).

47 Rollo, pp. 132-133.

48 The provision reads: Section 78. Petition to deny due course or to cancel a certificate
of candidacy. — A verified petition seeking to deny due course or to cancel a certificate of
candidacy may be filed by any person exclusively on the ground that any material
representation contained therein as required under
Section 74 hereof is false. The petition may be filed at any time not later than twenty-five
days from the time of filing of the certificate of candidacy and shall be decided after due
notice and hearing, not later than fifteen days before the election.

49 Marcelino vs. Cruz, 121 SCRA 51 (1983).

50 American Tupe Founders Co. v. Justice's Court, 133 Cal. 819, 65 Pac. 742; Heillen v.
Phillipps, 88 Cal. 557, 26 Pac. 366; Drake v. Bagley, 69 Mo. App. 39; State v. Davis, 194
Mo. 585.

51 Supra, note 39, citing Huffines v. Gold 154 Tenn. 583, 588; 288 S.W. 353, 354.

52 Sec. 6. Effect of Disqualification Case. — Any candidate who has been declared by
final judgment to be disqualified shall not be voted for, and the votes cast for him shall not
be counted. If for any reason a candidate is not declared by final judgment before an
election to be disqualified and he is voted for and receives the winning number of votes in
such election, the Court or Commission shall continue with the trial and hearing of the
action, inquiry or protest and, upon motion of the complainant or any intervenor, may
during the thereof order the suspension of the proclamation of such candidate whenever
the evidence of his guilt is strong.

Sec. 7 Petition to Deny Due Course or to Cancel a Certificate Candidacy.


The procedure hereinabove provided shall apply to petitions to deny due course to or
cancel a certificate of candidacy as provided in Section 78 of Batas Pambansa Blg. 881.

53 CONST., art. VI, sec. 11 states:

The Senate and the House of Representatives shall have an Electoral Tribunal which
shall be the sole judge of all questions relating to the election, returns, and qualifications
of their respective Members. . . .

PUNO, J., concurring:


MARRIAGE
ARTICLE 1
Republic of the Philippines
SUPREME COURT
Manila

EN BANC

G.R. No. L-13274 January 30, 1960

REMEDIOS SACLOLO and ERNESTO PASCUAL, petitioner,


vs.
COURT OF AGRARIAN RELATIONS and SANTIAGO MADLANGSAKAY, respondents.

Ernesto M. Tomaneng for petitioners.


Nora G. Nostratis and Fausto T. Allado for respondent Court.

LABRADOR, J.:

This is a petition for certiorari brought before us to review a decision of the Court of Agrarian Relations, Hon.
Guillermo S. Santos, presiding, dismissing a suit filed by petitioners herein to eject respondent Santiago
Madlangsakay from a four-hectare land belonging to petitioner Remedios Saclolo, in order that the same may be
cultivated by her husband, Ernesto Pascual. The facts are stated by the respondent judge below as follows:

Remedios Saclolo is the owner of a landholding of four (4)hectares, more or less, located at Barrio
Matungao, Bulacan, Bulacan. The said holding is tenanted by respondent Santiago Madlangsakay.
Ernesto Pascual is the husband of Remedios, and the landholding is Remedios' paraphernal property
brought into their marriage. The notice to the respondent required under Sec. 50 (a), of Republic Act No.
1199, dated April 9, 1956, was prepared by counsel for petitioner, Atty. Ernesto M. Tomaneg, to the effect
that Ernesto Pascual desires to farm the land (Exhibit "B") which was served upon respondent on April 9,
1956, and a notice to the Court of said notification was forwarded on the same date. Ernesto Pascual,
who is at present not gainfully employed and who has some experience in farm work, will work the land to
support petitioners' family.

The ground upon which the judge dismissed the petition is stated by him as follows:

We may, and, in fact, do, believe with petitioners that, in cases such as the present, and others cited in
his memorandum (p. 2; case of widow and son) it may be a wise policy to allow the tenant's ejectment
from the landholding, to enable the owner to cultivate the same thru or with the help of her husband. (in
this instant case) or a son, in the other. But the wisdom of the law is not the sphere of this Court. We cannot
... step outside the settled and ordinary meaning of law and by judicial legislation give to the law a meaning not intended. If ... redress is proper...
the complainants must look to the legislature and not to the courts. (per Justice Malcolm in Molina vs. Rafferty, 37 Phil., at page 557.)

The reason given by the court below for denying the petition of the landowner and her husband to cultivate the
landholding is Section 50 (a) of Republic Act No. 1199, which provides that an owner may eject a tenant in order
that he may cultivate it, only when he will personally cultivate his land. The judge below admits that it maybe a
wise policy to permit a tenant to be objected from a landholding to enable the owner to cultivate the same through
or with the help of her husband. But he says that to authorize the ejectment of the tenant, because the wife-
owner desires the property to be cultivated by her husband, is expressly prohibited by law which
requires personal cultivation by the owner, and this prohibition cannot by construction be made inapplicable to
the husband of an owner-wife.

We cannot subscribe to the opinion of the judge of the court below that to authorize the ejectment under the
above circumstances would be a judicial construction of the law beyond the intent thereof. The provisions of the
Agricultural Tenancy Act (Republic Act No. 1199) should be construed in the light of the law, and the legal
principles obtaining in this jurisdiction, especially those that regulate the relation between husband and wife.
Under legal principles, by the contract of marriage, a man and a woman enter a joint life, acting, living and
working as one. Whether under the common law or under the civil law, upon marriage the husband and the wife
become one single moral, spiritual and social being, not only for purposes of procreation but also for the purpose
of mutual help and protection, physically, morally and materially. There is between them a full and complete
community of existence.

Entre las del tercer grupo o de tipo finalistas las hay que atiendena la finalidad estrictamente sexual del
matrimonio, y otras, mas aceptables, que atienden a la finalidad espiritual o integral. En este ultimo
sentido, ya las definiciones de los juristas romanos senalaron la constitucion de una plena comunidad de
la vida como finalidad juridicamente reconocida del matrimonio.... Modernamentese inspira en la misma
idea Ahrens al considerar el matrimonio como in union formada entre dos sexo diferente conel proposito
de una comunidad perfecta de toda su vida moral, espiritual fisica, y de todas las relaciones que son su
consecuencia, y Kipp y Wolff, al definirlo como `la union de un hombre y deuna mejur dirigida al
establecimiento de una plena communidad devida. Todas estas deficiones recogen la idea moral del
matrimonio, propria de la civilizacion cristiana y moderna, y que inspira laslegislaciones positivas.

No faltan, por lo demas, deficiones mixtas. En realdad, lastres notas aludidas de la legalidad, permanecia
y plenitud sonotros tnatos aspectos parciales de la idea del matrimonio. Reuniendolos popdriamos definir
este como la union legal de un hombrey una mujer para la plena y perpetua comunidad de existencia.Y
si quisieramos definirlo en su acepcion de acto, podriamos decirque el matrimonio es el acto solemne por
medio del para la plena yperpetua comunidad de existencia. (Castan, Derecho Civil, Tomo3, 6.a ed.,
paginas 445-446.)

If there is unity and community of existence between husband and wife, then the husband may not be considered
as a being distinct and different from the wife, and the cultivation of the wife's land should be considered as a
joint effort of both. In fact, even if the difficult manual work like plowing and harrowing is usually done by men,
women take part in the planting of the seedlings, in the cleaning of the growing crop, in the harvesting, in the
winnowing — all of which constitute integral parts of the raising of the crop.

Moreover, the law allows a tenant to cultivate piece of agricultural land, held under a contract of tenancy, either
personally or with the aid of labor available from members of this immediate farm household. (Republic Act No.
1199, Section 4, par. 3, as amended by Republic Act No. 2263.) Note that he is not even required to have said
cultivation undertaken by immediate members of his family, but only by his immediate farm household, who may
or may not belong to the immediate members of his family. Surely no reason exists why this same right should be
denied to the landowner herself. The law as it were seeks to extend its protecting arm not only to the tenant but
to the landlord as well.

SEC. 2. Purpose. — It is the purpose of this Act to establish agricultural tenancy relations between
landholders and tenants upon the principle of social justice; to afford adequate protection to the rights of
both tenants and landholders; ... (R.A. No. 1199.)

There is also an express provision of the law which may be construed to give the husband the right to cultivate a
landholding belonging to the wife as paraphernal property. This is Article 137 of the Civil Code of the Philippines,
which says:

ART. 137. The wife have the administration of the paraphernal property, unless she delivers the same to
the husband by means of a public instrument empowering him to administer it ....

The administration of a rice land, for example, is not alone the giving thereof to another and the receiving of the
owner's share in the harvest. It may well include cultivation, or the raising of a crop thereon, should the
administrator deem it better for the spouses to have the cultivation done by the husband principally.

For the foregoing considerations, the order of dismissal sought to be reviewed is hereby set aside and the
petition to eject the respondent from the landholding of Remedios Saclolo is hereby granted. Without costs.

Paras, C.J., Bengzon, Padilla, Montemayor, Bautista Angelo, Concepcion, Endencia, Barrera and Gutierrez
David, JJ., concur.
Republic of the Philippines
SUPREME COURT
Manila

SECOND DIVISION

G.R. No. 118978 May 23, 1997

PHILIPPINE TELEGRAPH AND TELEPHONE COMPANY, * petitioner,


vs.
NATIONAL LABOR RELATIONS COMMISSION and GRACE DE GUZMAN, respondents.

REGALADO, J.:

Seeking relief through the extraordinary writ of certiorari, petitioner Philippine Telegraph and Telephone
Company (hereafter, PT & T) invokes the alleged concealment of civil status and defalcation of company funds
as grounds to terminate the services of an employee. That employee, herein private respondent Grace de
Guzman, contrarily argues that what really motivated PT & T to terminate her services was her having contracted
marriage during her employment, which is prohibited by petitioner in its company policies. She thus claims that
she was discriminated against in gross violation of law, such a proscription by an employer being outlawed by
Article 136 of the Labor Code.

Grace de Guzman was initially hired by petitioner as a reliever, specifically as a "Supernumerary Project Worker,"
for a fixed period from November 21, 1990 until April 20, 1991 vice one C.F. Tenorio who went on maternity
leave.1 Under the Reliever Agreement which she signed with petitioner company, her employment was to be
immediately terminated upon expiration of the agreed period. Thereafter, from June 10, 1991 to July 1, 1991, and
from July 19, 1991 to August 8, 1991, private respondent's services as reliever were again engaged by petitioner,
this time in replacement of one Erlinda F. Dizon who went on leave during both periods. 2 After August 8, 1991,
and pursuant to their Reliever Agreement, her services were terminated.

On September 2, 1991, private respondent was once more asked to join petitioner company as a probationary
employee, the probationary period to cover 150 days. In the job application form that was furnished her to be
filled up for the purpose, she indicated in the portion for civil status therein that she was single although she had
contracted marriage a few months earlier, that is, on May 26, 1991.3

It now appears that private respondent had made the same representation in the two successive reliever
agreements which she signed on June 10, 1991 and July 8, 1991. When petitioner supposedly learned about the
same later, its branch supervisor in Baguio City, Delia M. Oficial, sent to private respondent a memorandum
dated January 15, 1992 requiring her to explain the discrepancy. In that memorandum, she was reminded about
the company's policy of not accepting married women for employment. 4

In her reply letter dated January 17, 1992, private respondent stated that she was not aware of PT&T's policy
regarding married women at the time, and that all along she had not deliberately hidden her true civil
status.5 Petitioner nonetheless remained unconvinced by her explanations. Private respondent was dismissed
from the company effective January 29, 1992, 6 which she readily contested by initiating a complaint for illegal
dismissal, coupled with a claim for non-payment of cost of living allowances (COLA), before the Regional
Arbitration Branch of the National Labor Relations Commission in Baguio City.

At the preliminary conference conducted in connection therewith, private respondent volunteered the information,
and this was incorporated in the stipulation of facts between the parties, that she had failed to remit the amount of
P2,380.75 of her collections. She then executed a promissory note for that amount in favor of petitioner7. All of
these took place in a formal proceeding and with the agreement of the parties and/or their counsel.

On November 23, 1993, Labor Arbiter Irenarco R. Rimando handed down a decision declaring that private
respondent, who had already gained the status of a regular employee, was illegally dismissed by petitioner. Her
reinstatement, plus payment of the corresponding back wages and COLA, was correspondingly ordered, the
labor arbiter being of the firmly expressed view that the ground relied upon by petitioner in dismissing private
respondent was clearly insufficient, and that it was apparent that she had been discriminated against on account
of her having contracted marriage in violation of company rules.

On appeal to the National Labor Relations Commission (NLRC), said public respondent upheld the labor arbiter
and, in its decision dated April 29, 1994, it ruled that private respondent had indeed been the subject of an unjust
and unlawful discrimination by her employer, PT & T. However, the decision of the labor arbiter was modified with
the qualification that Grace de Guzman deserved to be suspended for three months in view of the dishonest
nature of her acts which should not be condoned. In all other respects, the NLRC affirmed the decision of the
labor arbiter, including the order for the reinstatement of private respondent in her employment with PT & T.

The subsequent motion for reconsideration filed by petitioner was rebuffed by respondent NLRC in its resolution
of November 9, 1994, hence this special civil action assailing the aforestated decisions of the labor arbiter and
respondent NLRC, as well as the denial resolution of the latter.

1. Decreed in the Bible itself is the universal norm that women should be regarded with love and respect but,
through the ages, men have responded to that injunction with indifference, on the hubristic conceit that women
constitute the inferior sex. Nowhere has that prejudice against womankind been so pervasive as in the field of
labor, especially on the matter of equal employment opportunities and standards. In the Philippine setting,
women have traditionally been considered as falling within the vulnerable groups or types of workers who must
be safeguarded with preventive and remedial social legislation against discriminatory and exploitative practices in
hiring, training, benefits, promotion and retention.

The Constitution, cognizant of the disparity in rights between men and women in almost all phases of social and
political life, provides a gamut of protective provisions. To cite a few of the primordial ones, Section 14, Article
II8 on the Declaration of Principles and State Policies, expressly recognizes the role of women in nation-building
and commands the State to ensure, at all times, the fundamental equality before the law of women and men.
Corollary thereto, Section 3 of Article XIII9 (the progenitor whereof dates back to both the 1935 and 1973
Constitution) pointedly requires the State to afford full protection to labor and to promote full employment and
equality of employment opportunities for all, including an assurance of entitlement to tenurial security of all
workers. Similarly, Section 14 of Article XIII 10 mandates that the State shall protect working women through
provisions for opportunities that would enable them to reach their full potential.

2. Corrective labor and social laws on gender inequality have emerged with more frequency in the years since
the Labor Code was enacted on May 1, 1974 as Presidential Decree No. 442, largely due to our country's
commitment as a signatory to the United Nations Convention on the Elimination of All Forms of Discrimination
Against Women (CEDAW). 11

Principal among these laws are Republic Act No. 6727 12 which explicitly prohibits discrimination against women
with respect to terms and conditions of employment, promotion, and training opportunities; Republic Act No.
6955 13 which bans the "mail-order-bride" practice for a fee and the export of female labor to countries that cannot
guarantee protection to the rights of women workers; Republic Act No. 7192 14 also known as the "Women in
Development and Nation Building Act," which affords women equal opportunities with men to act and to enter into
contracts, and for appointment, admission, training, graduation, and commissioning in all military or similar
schools of the Armed Forces of the Philippines and the Philippine National Police; Republic Act No.
7322 15 increasing the maternity benefits granted to women in the private sector; Republic Act No. 7877 16 which
outlaws and punishes sexual harassment in the workplace and in the education and training environment; and
Republic Act No. 8042, 17 or the "Migrant Workers and Overseas Filipinos Act of 1995," which prescribes as a
matter of policy, inter alia, the deployment of migrant workers, with emphasis on women, only in countries where
their rights are secure. Likewise, it would not be amiss to point out that in the Family Code, 18 women's rights in
the field of civil law have been greatly enhanced and expanded.

In the Labor Code, provisions governing the rights of women workers are found in Articles 130 to 138 thereof.
Article 130 involves the right against particular kinds of night work while Article 132 ensures the right of women to
be provided with facilities and standards which the Secretary of Labor may establish to ensure their health and
safety. For purposes of labor and social legislation, a woman working in a nightclub, cocktail lounge, massage
clinic, bar or other similar establishments shall be considered as an employee under Article 138. Article 135, on
the other hand, recognizes a woman's right against discrimination with respect to terms and conditions of
employment on account simply of sex. Finally, and this brings us to the issue at hand, Article 136 explicitly
prohibits discrimination merely by reason of the marriage of a female employee.

3. Acknowledged as paramount in the due process scheme is the constitutional guarantee of protection to labor
and security of tenure. Thus, an employer is required, as a condition sine qua non prior to severance of the
employment ties of an individual under his employ, to convincingly establish, through substantial evidence, the
existence of a valid and just cause in dispensing with the services of such employee, one's labor being regarded
as constitutionally protected property.

On the other hand, it is recognized that regulation of manpower by the company falls within the so-called
management prerogatives, which prescriptions encompass the matter of hiring, supervision of workers, work
assignments, working methods and assignments, as well as regulations on the transfer of employees, lay-off of
workers, and the discipline, dismissal, and recall of employees. 19 As put in a case, an employer is free to
regulate, according to his discretion and best business judgment, all aspects of employment, "from hiring to
firing," except in cases of unlawful discrimination or those which may be provided by law. 20
In the case at bar, petitioner's policy of not accepting or considering as disqualified from work any woman worker
who contracts marriage runs afoul of the test of, and the right against, discrimination, afforded all women workers
by our labor laws and by no less than the Constitution. Contrary to petitioner's assertion that it dismissed private
respondent from employment on account of her dishonesty, the record discloses clearly that her ties with the
company were dissolved principally because of the company's policy that married women are not qualified for
employment in PT & T, and not merely because of her supposed acts of dishonesty.

That it was so can easily be seen from the memorandum sent to private respondent by Delia M. Oficial, the
branch supervisor of the company, with the reminder, in the words of the latter, that "you're fully aware that the
company is not accepting married women employee (sic), as it was verbally instructed to you." 21 Again, in the
termination notice sent to her by the same branch supervisor, private respondent was made to understand that
her severance from the service was not only by reason of her concealment of her married status but, over and on
top of that, was her violation of the company's policy against marriage ("and even told you that married women
employees are not applicable [sic] or accepted in our company.") 22 Parenthetically, this seems to be the curious
reason why it was made to appear in the initiatory pleadings that petitioner was represented in this case only by
its said supervisor and not by its highest ranking officers who would otherwise be solidarily liable with the
corporation. 23

Verily, private respondent's act of concealing the true nature of her status from PT & T could not be properly
characterized as willful or in bad faith as she was moved to act the way she did mainly because she wanted to
retain a permanent job in a stable company. In other words, she was practically forced by that very same illegal
company policy into misrepresenting her civil status for fear of being disqualified from work. While loss of
confidence is a just cause for termination of employment, it should not be simulated. 24 It must rest on an actual
breach of duty committed by the employee and not on the employer's caprices. 25 Furthermore, it should never be
used as a subterfuge for causes which are improper, illegal, or unjustified. 26

In the present controversy, petitioner's expostulations that it dismissed private respondent, not because the latter
got married but because she concealed that fact, does have a hollow ring. Her concealment, so it is claimed,
bespeaks dishonesty hence the consequent loss of confidence in her which justified her dismissal.

Petitioner would asseverate, therefore, that while it has nothing against marriage, it nonetheless takes umbrage
over the concealment of that fact. This improbable reasoning, with interstitial distinctions, perturbs the Court since
private respondent may well be minded to claim that the imputation of dishonesty should be the other way
around.

Petitioner would have the Court believe that although private respondent defied its policy against its female
employees contracting marriage, what could be an act of insubordination was inconsequential. What it submits as
unforgivable is her concealment of that marriage yet, at the same time, declaring that marriage as a trivial matter
to which it supposedly has no objection. In other words, PT & T says it gives its blessings to its female employees
contracting marriage, despite the maternity leaves and other benefits it would consequently respond for and
which obviously it would have wanted to avoid. If that employee confesses such fact of marriage, there will be no
sanction; but if such employee conceals the same instead of proceeding to the confessional, she will be
dismissed. This line of reasoning does not impress us as reflecting its true management policy or that we are
being regaled with responsible advocacy.

This Court should be spared the ennui of strained reasoning and the tedium of propositions which confuse
through less than candid arguments. Indeed, petitioner glosses over the fact that it was its unlawful policy against
married women, both on the aspects of qualification and retention, which compelled private respondent to
conceal her supervenient marriage. It was, however, that very policy alone which was the cause of private
respondent's secretive conduct now complained of. It is then apropos to recall the familiar saying that he who is
the cause of the cause is the cause of the evil caused.

Finally, petitioner's collateral insistence on the admission of private respondent that she supposedly
misappropriated company funds, as an additional ground to dismiss her from employment, is somewhat insincere
and self-serving. Concededly, private respondent admitted in the course of the proceedings that she failed to
remit some of her collections, but that is an altogether different story. The fact is that she was dismissed solely
because of her concealment of her marital status, and not on the basis of that supposed defalcation of company
funds. That the labor arbiter would thus consider petitioner's submissions on this supposed dishonesty as a mere
afterthought, just to bolster its case for dismissal, is a perceptive conclusion born of experience in labor cases.
For, there was no showing that private respondent deliberately misappropriated the amount or whether her failure
to remit the same was through negligence and, if so, whether the negligence was in nature simple or grave. In
fact, it was merely agreed that private respondent execute a promissory note to refund the same, which she did,
and the matter was deemed settled as a peripheral issue in the labor case.

Private respondent, it must be observed, had gained regular status at the time of her dismissal. When she was
served her walking papers on January 29, 1992, she was about to complete the probationary period of 150 days
as she was contracted as a probationary employee on September 2, 1991. That her dismissal would be effected
just when her probationary period was winding down clearly raises the plausible conclusion that it was done in
order to prevent her from earning security of tenure. 27 On the other hand, her earlier stints with the company as
reliever were undoubtedly those of a regular employee, even if the same were for fixed periods, as she
performed activities which were essential or necessary in the usual trade and business of PT & T. 28 The primary
standard of determining regular employment is the reasonable connection between the activity performed by the
employee in relation to the business or trade of the employer. 29

As an employee who had therefore gained regular status, and as she had been dismissed without just cause, she
is entitled to reinstatement without loss of seniority rights and other privileges and to full back wages, inclusive of
allowances and other benefits or their monetary equivalent. 30 However, as she had undeniably committed an act
of dishonesty in concealing her status, albeit under the compulsion of an unlawful imposition of petitioner, the
three-month suspension imposed by respondent NLRC must be upheld to obviate the impression or inference
that such act should be condoned. It would be unfair to the employer if she were to return to its fold without any
sanction whatsoever for her act which was not totally justified. Thus, her entitlement to back wages, which shall
be computed from the time her compensation was withheld up to the time of her actual reinstatement, shall be
reduced by deducting therefrom the amount corresponding to her three months suspension.

4. The government, to repeat, abhors any stipulation or policy in the nature of that adopted by petitioner PT & T.
The Labor Code state, in no uncertain terms, as follows:

Art. 136. Stipulation against marriage. — It shall be unlawful for an employer to require as a
condition of employment or continuation of employment that a woman shall not get married, or to
stipulate expressly or tacitly that upon getting married, a woman employee shall be deemed
resigned or separated, or to actually dismiss, discharge, discriminate or otherwise prejudice a
woman employee merely by reason of marriage.

This provision had a studied history for its origin can be traced to Section 8 of Presidential Decree No.
148, 31 better known as the "Women and
Child Labor Law," which amended paragraph (c), Section 12 of Republic Act No. 679, 32 entitled "An Act to
Regulate the Employment of Women and Children, to Provide Penalties for Violations Thereof, and for Other
Purposes." The forerunner to Republic Act No. 679, on the other hand, was Act No. 3071 which became law on
March 16, 1923 and which regulated the employment of women and children in shops, factories, industrial,
agricultural, and mercantile establishments and other places of labor in the then Philippine Islands.

It would be worthwhile to reflect upon and adopt here the rationalization in Zialcita, et al. vs. Philippine Air
Lines, 33 a decision that emanated from the Office of the President. There, a policy of Philippine Air Lines
requiring that prospective flight attendants must be single and that they will be automatically separated from the
service once they marry was declared void, it being violative of the clear mandate in Article 136 of the Labor
Code with regard to discrimination against married women. Thus:

Of first impression is the incompatibility of the respondent's policy or regulation with the codal
provision of law. Respondent is resolute in its contention that Article 136 of the Labor Code
applies only to women employed in ordinary occupations and that the prohibition against marriage
of women engaged in extraordinary occupations, like flight attendants, is fair and reasonable,
considering the pecularities of their chosen profession.

We cannot subscribe to the line of reasoning pursued by respondent. All along, it knew that the
controverted policy has already met its doom as early as March 13, 1973 when Presidential
Decree No. 148, otherwise known as the Women and Child Labor Law, was promulgated. But for
the timidity of those affected or their labor unions in challenging the validity of the policy, the same
was able to obtain a momentary reprieve. A close look at Section 8 of said decree, which
amended paragraph (c) of Section 12 of Republic Act No. 679, reveals that it is exactly the same
provision reproduced verbatim in Article 136 of the Labor Code, which was promulgated on May
1, 1974 to take effect six (6) months later, or on November 1, 1974.

It cannot be gainsaid that, with the reiteration of the same provision in the new Labor Code, all
policies and acts against it are deemed illegal and therefore abrogated. True, Article 132 enjoins
the Secretary of Labor to establish standards that will ensure the safety and health of women
employees and in appropriate cases shall by regulation require employers to determine
appropriate minimum standards for termination in special occupations, such as those of flight
attendants, but that is precisely the factor that militates against the policy of respondent. The
standards have not yet been established as set forth in the first paragraph, nor has the Secretary
of Labor issued any regulation affecting flight attendants.

It is logical to presume that, in the absence of said standards or regulations which are as yet to be
established, the policy of respondent against marriage is patently illegal. This finds support in
Section 9 of the New Constitution, which provides:
Sec. 9. The State shall afford protection to labor, promote full employment and equality in
employment, ensure equal work opportunities regardless of sex, race, or creed, and regulate the
relations between workers and employees. The State shall assure the rights of workers to self-
organization, collective bargaining, security of tenure, and just and humane conditions of work . . .
.

Moreover, we cannot agree to the respondent's proposition that termination from employment of
flight attendants on account of marriage is a fair and reasonable standard designed for their own
health, safety, protection and welfare, as no basis has been laid therefor. Actually, respondent
claims that its concern is not so much against the continued employment of the flight attendant
merely by reason of marriage as observed by the Secretary of Labor, but rather on the
consequence of marriage-pregnancy. Respondent discussed at length in the instant appeal the
supposed ill effects of pregnancy on flight attendants in the course of their employment. We feel
that this needs no further discussion as it had been adequately explained by the Secretary of
Labor in his decision of May 2, 1976.

In a vain attempt to give meaning to its position, respondent went as far as invoking the
provisions of Articles 52 and 216 of the New Civil Code on the preservation of marriage as an
inviolable social institution and the family as a basic social institution, respectively, as bases for its
policy of non-marriage. In both instances, respondent predicates absence of a flight attendant
from her home for long periods of time as contributory to an unhappy married life. This is pure
conjecture not based on actual conditions, considering that, in this modern world, sophisticated
technology has narrowed the distance from one place to another. Moreover, respondent
overlooked the fact that married flight attendants can program their lives to adapt to prevailing
circumstances and events.

Article 136 is not intended to apply only to women employed in ordinary occupations, or it should
have categorically expressed so. The sweeping intendment of the law, be it on special or ordinary
occupations, is reflected in the whole text and supported by Article 135 that speaks of non-
discrimination on the employment of women.

The judgment of the Court of Appeals in Gualberto, et al. vs. Marinduque Mining & Industrial
Corporation 34 considered as void a policy of the same nature. In said case, respondent, in dismissing from the
service the complainant, invoked a policy of the firm to consider female employees in the project it was
undertaking as separated the moment they get married due to lack of facilities for married women. Respondent
further claimed that complainant was employed in the project with an oral understanding that her services would
be terminated when she gets married. Branding the policy of the employer as an example of "discriminatory
chauvinism" tantamount to denying equal employment opportunities to women simply on account of their sex, the
appellate court struck down said employer policy as unlawful in view of its repugnance to the Civil Code,
Presidential Decree No. 148 and the Constitution.

Under American jurisprudence, job requirements which establish employer preference or conditions relating to
the marital status of an employee are categorized as a "sex-plus" discrimination where it is imposed on one sex
and not on the other. Further, the same should be evenly applied and must not inflict adverse effects on a racial
or sexual group which is protected by federal job discrimination laws. Employment rules that forbid or restrict the
employment of married women, but do not apply to married men, have been held to violate Title VII of the United
States Civil Rights Act of 1964, the main federal statute prohibiting job discrimination against employees and
applicants on the basis of, among other things, sex. 35

Further, it is not relevant that the rule is not directed against all women but just against married women. And,
where the employer discriminates against married women, but not against married men, the variable is sex and
the discrimination is unlawful. 36 Upon the other hand, a requirement that a woman employee must remain
unmarried could be justified as a "bona fide occupational qualification," or BFOQ, where the particular
requirements of the job would justify the same, but not on the ground of a general principle, such as the
desirability of spreading work in the workplace. A requirement of that nature would be valid provided it reflects an
inherent quality reasonably necessary for satisfactory job performance. Thus, in one case, a no-marriage rule
applicable to both male and female flight attendants, was regarded as unlawful since the restriction was not
related to the job performance of the flight attendants. 37

5. Petitioner's policy is not only in derogation of the provisions of Article 136 of the Labor Code on the right of a
woman to be free from any kind of stipulation against marriage in connection with her employment, but it likewise
assaults good morals and public policy, tending as it does to deprive a woman of the freedom to choose her
status, a privilege that by all accounts inheres in the individual as an intangible and inalienable right. 38 Hence,
while it is true that the parties to a contract may establish any agreements, terms, and conditions that they may
deem convenient, the same should not be contrary to law, morals, good customs, public order, or public
policy. 39 Carried to its logical consequences, it may even be said that petitioner's policy against legitimate marital
bonds would encourage illicit or common-law relations and subvert the sacrament of marriage.
Parenthetically, the Civil Code provisions on the contract of labor state that the relations between the parties, that
is, of capital and labor, are not merely contractual, impressed as they are with so much public interest that the
same should yield to the common good. 40 It goes on to intone that neither capital nor labor should visit acts of
oppression against the other, nor impair the interest or convenience of the public. 41 In the final reckoning, the
danger of just such a policy against marriage followed by petitioner PT & T is that it strikes at the very essence,
ideals and purpose of marriage as an inviolable social institution and, ultimately, of the family as the foundation of
the nation. 42 That it must be effectively interdicted here in all its indirect, disguised or dissembled forms as
discriminatory conduct derogatory of the laws of the land is not only in order but imperatively required.

ON THE FOREGOING PREMISES, the petition of Philippine Telegraph and Telephone Company is hereby
DISMISSED for lack of merit, with double costs against petitioner.

SO ORDERED.

Romero, Puno, Mendoza and Torres, Jr., JJ., concur.

Footnotes

* The phrase "herein represented by DELIA M. OFICIAL", added hereto in the title of this case as
stated in the petition, has been deleted for being unnecessary and violative of the rules on
pleadings, and is commented upon in the text of this opinion.

1 Rollo, 42; Annex D.

2 Ibid., 44-45, Annexes F and G

3 Ibid., 46-48; Annexes H and I.

4 Ibid., 49; Annex J.

5 Id., 50; Annex K.

6 Id., 51; Annex L.

7 Id., 53; Annex N.

8 The State recognizes the role of women in nation-building, and shall ensure the fundamental
equality before the law of women and men (Sec.14, Art. II).

9 The State shall afford full protection to labor, local and overseas, organized or unorganized, and
promote full employment and equality of employment opportunities for all.

It shall guarantee the rights of all workers to self-organization, collective bargaining and
negotiations, and peaceful concerted activities, including the right to strike in accordance with law.
They shall be entitled to security of tenure, humane conditions of work, and a living wage. They
shall also participate in policy and decision-making processes affecting their rights and benefits
as may be provided by law.

The State shall promote the principle of shared responsibility between workers and employers
and the preferential use of voluntary modes of settling disputes, including conciliation, and shall
enforce their mutual compliance therewith to foster industrial peace.

The State shall regulate the relations between workers and employers, recognizing the right of
labor to its just share in the fruits of production and the right of enterprises to reasonable returns
on investment, and to expansion and growth (Sec. 3, Art. XIII).

10 The State shall protect working women by providing safe and healthful working conditions,
taking into account their maternal functions, and such facilities and opportunities that will enhance
their welfare and enable them to realize their full potential in the service of the nation (Sec. 14,
Art. XIII).

11 Adopted in 1979 by the UN General Assembly, it is regarded as the most comprehensive


international treaty governing the rights of women. The Philippines became a signatory thereto a
year after its adoption by the UN and in 1981, the country ratified it.
The Philippines had likewise been an active participant in all the four U.N. World Conferences on
Women, namely those held in Mexico in 1975, Copenhagen in 1980, Nairobi in 1985, and Beijing
in 1995.

Other relevant international laws to which the Philippines adheres as a member of the
international community include the Universal Declaration of Human Rights, the International
Covenant on Civil and Political Rights, and the International Covenant on Economic, Social and
Cultural Rights.

12 Approved, June 9, 1989.

13 Approved, June 13, 1990.

14 Approved, February 12, 1992.

15 Approved, March 30, 1992.

16 Approved, February 14, 1995.

17 Approved, June 7, 1995.

18 Effective August 3, 1988.

19 Caltex Refinery Employees Association (CREA) vs. National Labor Relations Commission, et
al., G.R. No. 102993, July 14, 1995, 246 SCRA 271; Oriental Mindoro Electric Cooperative, Inc.
vs. National Labor Relations Commission, et al., G.R. No. 111905, July 31, 1995, 246 SCRA 794;
Nuez vs. National Labor Relations Commission, et al., G.R. No. 107574, December 28, 1994, 239
SCRA 518; San Miguel Corporation vs. Ubaldo, et al., G.R. No. 92859, February 1, 1993, 218
SCRA 293.

20 NAFLU vs. National Labor Relations Commission, et al., G.R. No. 90739, October 3, 1991,
202 SCRA 346.

21 Quoted in the Decision of the Third Division, NLRC, in NLRC Case No. RAB-CAR-02-0042-92,
Annex B of petition, Rollo, 35. See also Annex J, supra, Fn. 4.

22 Annex L, id., Rollo, 51.

23 Art. 289, Labor Code; see AC Ransom Labor Union-ACLU vs. National Labor Relations
Commission, et al., G.R. No. 69494, June 10, 1986, 142 SCRA 269; Chua vs. National Labor
Relations Commission, et al., G.R. No. 81450, February 15, 1990, 182 SCRA 353.

24 Mapalo vs. National Labor Relations Commission, et al., G.R. No. 107940, June 17, 1994, 233
SCRA 266; PNOC-Energy Development Corporation vs. National Labor Relations Commission,
et al., G.R. No. 79182, September 11, 1991, 201 SCRA 487.

25 San Antonio vs. National Labor Relations Commission, et al., G.R. No. 100829, November 21,
1995, 250 SCRA 359; Labor vs. National Labor Relations Commission, G.R. No. 110388,
September 14, 1995, 248 SCRA 183.

26 Hospicio de San Jose de Basili vs. National Labor Relations Commission, et al., G.R. No.
75997, August 18, 1988, 164 SCRA 516.

27 Cielo vs. National Labor Relations Commission, et al G.R. No. 78693, January 28, 1991, 193
SCRA 410; Brent School, Inc. vs. Zamora, et al., G.R. No. 48494, February 5, 1990, 181 SCRA
702.

28 Art. 280, Labor Code; see PLDT vs. Montemayor, et al., G.R. No. 88626, October 12, 1990,
190 SCRA 427.

29 De Leon vs. National Labor Relations Commission, et al., G.R. No. 70705, August 21, 1989,
176 SCRA 615.
30 Molave Tours Corp. vs. National Labor Relations Commission, et al., G.R. No. 112909,
November 24, 1995, 250 SCRA 325; see Art. 279, Labor Code, as amended by Republic Act No.
6715.

31 Promulgated on March 13, 1973.

32 Approved on April 15, 1952. It was later amended by Republic Act No. 1131, which in turn was
approved on June 16, 1954.

33 Case No. RO4-3-3398-76; February 20, 1977.

34 CA-G.R. No. 52753-R, June 28, 1978.

35 45A Am. Jur. 2d, Job Discrimination, Sec. 506, p. 486.

36 Ibid., id., id.

37 Ibid., id., Sec. 507.

38 Tolentino, A., Civil Code of the Philippines, Vol. III, 1979 ed., 235; see Art. 874, Civil Code.

39 Art. 1306, Civil Code.

40 Art. 1700, Civil Code; see Macleod &. Co. of the Philippines vs. Progressive Federation of
Labor, 97 Phil. 205 (1955).

41 Art. 1701, Civil Code.

42 The 1987 Constitution provides:

The State recognizes the sanctity of family life and shall protect and strengthen the family as a
basic autonomous social institution. . . . (Sec. 15, Art. II).

The State recognizes the Filipino family as the foundation of the nation. Accordingly, it shall
strengthen its solidarity and actively promote its total development (Sec. 1, Art. XV).

Marriage, as an inviolable social institution, is the foundation of the family and shall be protected
by the State (Sec. 2, Art. XV).
ARTICLES 2-6
Republic of the Philippines
SUPREME COURT
Manila

FIRST DIVISION

G.R. No. 174689 October 22, 2007

ROMMEL JACINTO DANTES SILVERIO, petitioner,


vs.
REPUBLIC OF THE PHILIPPINES, respondent.

DECISION

CORONA, J.:

When God created man, He made him in the likeness of God; He created them male and female.
(Genesis 5:1-2)

Amihan gazed upon the bamboo reed planted by Bathala and she heard voices coming from inside the
bamboo. "Oh North Wind! North Wind! Please let us out!," the voices said. She pecked the reed once,
then twice. All of a sudden, the bamboo cracked and slit open. Out came two human beings; one was a
male and the other was a female. Amihan named the man "Malakas" (Strong) and the woman "Maganda"
(Beautiful). (The Legend of Malakas and Maganda)

When is a man a man and when is a woman a woman? In particular, does the law recognize the changes made
by a physician using scalpel, drugs and counseling with regard to a person’s sex? May a person successfully
petition for a change of name and sex appearing in the birth certificate to reflect the result of a sex reassignment
surgery?

On November 26, 2002, petitioner Rommel Jacinto Dantes Silverio filed a petition for the change of his first name
and sex in his birth certificate in the Regional Trial Court of Manila, Branch 8. The petition, docketed as SP Case
No. 02-105207, impleaded the civil registrar of Manila as respondent.

Petitioner alleged in his petition that he was born in the City of Manila to the spouses Melecio Petines Silverio
and Anita Aquino Dantes on April 4, 1962. His name was registered as "Rommel Jacinto Dantes Silverio" in his
certificate of live birth (birth certificate). His sex was registered as "male."

He further alleged that he is a male transsexual, that is, "anatomically male but feels, thinks and acts as a
female" and that he had always identified himself with girls since childhood. 1 Feeling trapped in a man’s body, he
consulted several doctors in the United States. He underwent psychological examination, hormone treatment and
breast augmentation. His attempts to transform himself to a "woman" culminated on January 27, 2001 when he
underwent sex reassignment surgery2 in Bangkok, Thailand. He was thereafter examined by Dr. Marcelino
Reysio-Cruz, Jr., a plastic and reconstruction surgeon in the Philippines, who issued a medical certificate
attesting that he (petitioner) had in fact undergone the procedure.

From then on, petitioner lived as a female and was in fact engaged to be married. He then sought to have his
name in his birth certificate changed from "Rommel Jacinto" to "Mely," and his sex from "male" to "female."

An order setting the case for initial hearing was published in the People’s Journal Tonight, a newspaper of
general circulation in Metro Manila, for three consecutive weeks. 3 Copies of the order were sent to the Office of
the Solicitor General (OSG) and the civil registrar of Manila.

On the scheduled initial hearing, jurisdictional requirements were established. No opposition to the petition was
made.

During trial, petitioner testified for himself. He also presented Dr. Reysio-Cruz, Jr. and his American fiancé,
Richard P. Edel, as witnesses.

On June 4, 2003, the trial court rendered a decision4 in favor of petitioner. Its relevant portions read:

Petitioner filed the present petition not to evade any law or judgment or any infraction thereof or for any
unlawful motive but solely for the purpose of making his birth records compatible with his present sex.
The sole issue here is whether or not petitioner is entitled to the relief asked for.

The [c]ourt rules in the affirmative.

Firstly, the [c]ourt is of the opinion that granting the petition would be more in consonance with the
principles of justice and equity. With his sexual [re-assignment], petitioner, who has always felt, thought
and acted like a woman, now possesses the physique of a female. Petitioner’s misfortune to be trapped
in a man’s body is not his own doing and should not be in any way taken against him.

Likewise, the [c]ourt believes that no harm, injury [or] prejudice will be caused to anybody or the
community in granting the petition. On the contrary, granting the petition would bring the much-awaited
happiness on the part of the petitioner and her [fiancé] and the realization of their dreams.

Finally, no evidence was presented to show any cause or ground to deny the present petition despite due
notice and publication thereof. Even the State, through the [OSG] has not seen fit to interpose any
[o]pposition.

WHEREFORE, judgment is hereby rendered GRANTING the petition and ordering the Civil Registrar of
Manila to change the entries appearing in the Certificate of Birth of [p]etitioner, specifically for petitioner’s
first name from "Rommel Jacinto" to MELY and petitioner’s gender from "Male" to FEMALE. 5

On August 18, 2003, the Republic of the Philippines (Republic), thru the OSG, filed a petition for certiorari in the
Court of Appeals.6 It alleged that there is no law allowing the change of entries in the birth certificate by reason of
sex alteration.

On February 23, 2006, the Court of Appeals7 rendered a decision8 in favor of the Republic. It ruled that the trial
court’s decision lacked legal basis. There is no law allowing the change of either name or sex in the certificate of
birth on the ground of sex reassignment through surgery. Thus, the Court of Appeals granted the Republic’s
petition, set aside the decision of the trial court and ordered the dismissal of SP Case No. 02-105207. Petitioner
moved for reconsideration but it was denied. 9 Hence, this petition.

Petitioner essentially claims that the change of his name and sex in his birth certificate is allowed under Articles
407 to 413 of the Civil Code, Rules 103 and 108 of the Rules of Court and RA 9048.10

The petition lacks merit.

A Person’s First Name Cannot Be Changed On the Ground of Sex Reassignment

Petitioner invoked his sex reassignment as the ground for his petition for change of name and sex. As found by
the trial court:

Petitioner filed the present petition not to evade any law or judgment or any infraction thereof or for any
unlawful motive but solely for the purpose of making his birth records compatible with his present
sex. (emphasis supplied)

Petitioner believes that after having acquired the physical features of a female, he became entitled to the civil
registry changes sought. We disagree.

The State has an interest in the names borne by individuals and entities for purposes of identification. 11 A change
of name is a privilege, not a right.12 Petitions for change of name are controlled by statutes. 13 In this connection,
Article 376 of the Civil Code provides:

ART. 376. No person can change his name or surname without judicial authority.

This Civil Code provision was amended by RA 9048 (Clerical Error Law). In particular, Section 1 of RA 9048
provides:

SECTION 1. Authority to Correct Clerical or Typographical Error and Change of First Name or
Nickname. – No entry in a civil register shall be changed or corrected without a judicial order, except for
clerical or typographical errors and change of first name or nickname which can be corrected or changed
by the concerned city or municipal civil registrar or consul general in accordance with the provisions of
this Act and its implementing rules and regulations.

RA 9048 now governs the change of first name.14 It vests the power and authority to entertain petitions for
change of first name to the city or municipal civil registrar or consul general concerned. Under the law, therefore,
jurisdiction over applications for change of first name is now primarily lodged with the aforementioned
administrative officers. The intent and effect of the law is to exclude the change of first name from the coverage
of Rules 103 (Change of Name) and 108 (Cancellation or Correction of Entries in the Civil Registry) of the Rules
of Court, until and unless an administrative petition for change of name is first filed and subsequently denied. 15 It
likewise lays down the corresponding venue, 16 form17 and procedure. In sum, the remedy and the proceedings
regulating change of first name are primarily administrative in nature, not judicial.

RA 9048 likewise provides the grounds for which change of first name may be allowed:

SECTION 4. Grounds for Change of First Name or Nickname. – The petition for change of first name or
nickname may be allowed in any of the following cases:

(1) The petitioner finds the first name or nickname to be ridiculous, tainted with dishonor or extremely
difficult to write or pronounce;

(2) The new first name or nickname has been habitually and continuously used by the petitioner and he
has been publicly known by that first name or nickname in the community; or

(3) The change will avoid confusion.

Petitioner’s basis in praying for the change of his first name was his sex reassignment. He intended to make his
first name compatible with the sex he thought he transformed himself into through surgery. However, a change of
name does not alter one’s legal capacity or civil status.18 RA 9048 does not sanction a change of first name on
the ground of sex reassignment. Rather than avoiding confusion, changing petitioner’s first name for his declared
purpose may only create grave complications in the civil registry and the public interest.

Before a person can legally change his given name, he must present proper or reasonable cause or any
compelling reason justifying such change. 19 In addition, he must show that he will be prejudiced by the use of his
true and official name.20 In this case, he failed to show, or even allege, any prejudice that he might suffer as a
result of using his true and official name.

In sum, the petition in the trial court in so far as it prayed for the change of petitioner’s first name was not within
that court’s primary jurisdiction as the petition should have been filed with the local civil registrar concerned,
assuming it could be legally done. It was an improper remedy because the proper remedy was administrative,
that is, that provided under RA 9048. It was also filed in the wrong venue as the proper venue was in the Office of
the Civil Registrar of Manila where his birth certificate is kept. More importantly, it had no merit since the use of
his true and official name does not prejudice him at all. For all these reasons, the Court of Appeals correctly
dismissed petitioner’s petition in so far as the change of his first name was concerned.

No Law Allows The Change of Entry In The Birth Certificate As To Sex On the Ground of Sex
Reassignment

The determination of a person’s sex appearing in his birth certificate is a legal issue and the court must look to
the statutes.21 In this connection, Article 412 of the Civil Code provides:

ART. 412. No entry in the civil register shall be changed or corrected without a judicial order.

Together with Article 376 of the Civil Code, this provision was amended by RA 9048 in so far as clerical or
typographical errors are involved. The correction or change of such matters can now be made through
administrative proceedings and without the need for a judicial order. In effect, RA 9048 removed from the ambit
of Rule 108 of the Rules of Court the correction of such errors.22 Rule 108 now applies only to substantial
changes and corrections in entries in the civil register.23

Section 2(c) of RA 9048 defines what a "clerical or typographical error" is:

SECTION 2. Definition of Terms. – As used in this Act, the following terms shall mean:

xxx xxx xxx

(3) "Clerical or typographical error" refers to a mistake committed in the performance of clerical
work in writing, copying, transcribing or typing an entry in the civil register that is harmless and
innocuous, such as misspelled name or misspelled place of birth or the like, which is visible to the
eyes or obvious to the understanding, and can be corrected or changed only by reference to other
existing record or records: Provided, however, That no correction must involve the change
of nationality, age, status or sex of the petitioner. (emphasis supplied)
Under RA 9048, a correction in the civil registry involving the change of sex is not a mere clerical or typographical
error. It is a substantial change for which the applicable procedure is Rule 108 of the Rules of Court.

The entries envisaged in Article 412 of the Civil Code and correctable under Rule 108 of the Rules of Court are
those provided in Articles 407 and 408 of the Civil Code: 24

ART. 407. Acts, events and judicial decrees concerning the civil status of persons shall be recorded in the
civil register.

ART. 408. The following shall be entered in the civil register:

(1) Births; (2) marriages; (3) deaths; (4) legal separations; (5) annulments of marriage; (6) judgments
declaring marriages void from the beginning; (7) legitimations; (8) adoptions; (9) acknowledgments of
natural children; (10) naturalization; (11) loss, or (12) recovery of citizenship; (13) civil interdiction; (14)
judicial determination of filiation; (15) voluntary emancipation of a minor; and (16) changes of name.

The acts, events or factual errors contemplated under Article 407 of the Civil Code include even those that occur
after birth.25 However, no reasonable interpretation of the provision can justify the conclusion that it covers the
correction on the ground of sex reassignment.

To correct simply means "to make or set aright; to remove the faults or error from" while to change means "to
replace something with something else of the same kind or with something that serves as a substitute." 26 The
birth certificate of petitioner contained no error. All entries therein, including those corresponding to his first name
and sex, were all correct. No correction is necessary.

Article 407 of the Civil Code authorizes the entry in the civil registry of certain acts (such as legitimations,
acknowledgments of illegitimate children and naturalization), events (such as births, marriages, naturalization and
deaths) and judicial decrees (such as legal separations, annulments of marriage, declarations of nullity of
marriages, adoptions, naturalization, loss or recovery of citizenship, civil interdiction, judicial determination of
filiation and changes of name). These acts, events and judicial decrees produce legal consequences that touch
upon the legal capacity, status and nationality of a person. Their effects are expressly sanctioned by the laws. In
contrast, sex reassignment is not among those acts or events mentioned in Article 407. Neither is it recognized
nor even mentioned by any law, expressly or impliedly.

"Status" refers to the circumstances affecting the legal situation (that is, the sum total of capacities and
incapacities) of a person in view of his age, nationality and his family membership. 27

The status of a person in law includes all his personal qualities and relations, more or less permanent in
nature, not ordinarily terminable at his own will, such as his being legitimate or illegitimate, or his
being married or not. The comprehensive term status… include such matters as the beginning and end of
legal personality, capacity to have rights in general, family relations, and its various aspects, such as
birth, legitimation, adoption, emancipation, marriage, divorce, and sometimes even
succession.28 (emphasis supplied)

A person’s sex is an essential factor in marriage and family relations. It is a part of a person’s legal capacity and
civil status. In this connection, Article 413 of the Civil Code provides:

ART. 413. All other matters pertaining to the registration of civil status shall be governed by special laws.

But there is no such special law in the Philippines governing sex reassignment and its effects. This is fatal to
petitioner’s cause.

Moreover, Section 5 of Act 3753 (the Civil Register Law) provides:

SEC. 5. Registration and certification of births. – The declaration of the physician or midwife in
attendance at the birth or, in default thereof, the declaration of either parent of the newborn child, shall be
sufficient for the registration of a birth in the civil register. Such declaration shall be exempt from
documentary stamp tax and shall be sent to the local civil registrar not later than thirty days after the birth,
by the physician or midwife in attendance at the birth or by either parent of the newborn child.

In such declaration, the person above mentioned shall certify to the following facts: (a) date and hour of
birth; (b) sex and nationality of infant; (c) names, citizenship and religion of parents or, in case the father
is not known, of the mother alone; (d) civil status of parents; (e) place where the infant was born; and (f)
such other data as may be required in the regulations to be issued.

xxx xxx xxx (emphasis supplied)


Under the Civil Register Law, a birth certificate is a historical record of the facts as they existed at the time of
birth.29 Thus, the sex of a person is determined at birth, visually done by the birth attendant (the physician or
midwife) by examining the genitals of the infant. Considering that there is no law legally recognizing sex
reassignment, the determination of a person’s sex made at the time of his or her birth, if not attended by
error,30 is immutable.31

When words are not defined in a statute they are to be given their common and ordinary meaning in the absence
of a contrary legislative intent. The words "sex," "male" and "female" as used in the Civil Register Law and laws
concerning the civil registry (and even all other laws) should therefore be understood in their common and
ordinary usage, there being no legislative intent to the contrary. In this connection, sex is defined as "the sum of
peculiarities of structure and function that distinguish a male from a female" 32 or "the distinction between male
and female."33 Female is "the sex that produces ova or bears young" 34 and male is "the sex that has organs to
produce spermatozoa for fertilizing ova."35 Thus, the words "male" and "female" in everyday understanding do not
include persons who have undergone sex reassignment. Furthermore, "words that are employed in a statute
which had at the time a well-known meaning are presumed to have been used in that sense unless the context
compels to the contrary."36 Since the statutory language of the Civil Register Law was enacted in the early 1900s
and remains unchanged, it cannot be argued that the term "sex" as used then is something alterable through
surgery or something that allows a post-operative male-to-female transsexual to be included in the category
"female."

For these reasons, while petitioner may have succeeded in altering his body and appearance through the
intervention of modern surgery, no law authorizes the change of entry as to sex in the civil registry for that
reason. Thus, there is no legal basis for his petition for the correction or change of the entries in his birth
certificate.

Neither May Entries in the Birth Certificate As to First Name or Sex Be Changed on the Ground of Equity

The trial court opined that its grant of the petition was in consonance with the principles of justice and equity. It
believed that allowing the petition would cause no harm, injury or prejudice to anyone. This is wrong.

The changes sought by petitioner will have serious and wide-ranging legal and public policy consequences. First,
even the trial court itself found that the petition was but petitioner’s first step towards his eventual marriage to his
male fiancé. However, marriage, one of the most sacred social institutions, is a special contract of permanent
union between a man and a woman.37 One of its essential requisites is the legal capacity of the contracting
parties who must be a male and a female.38 To grant the changes sought by petitioner will substantially
reconfigure and greatly alter the laws on marriage and family relations. It will allow the union of a man with
another man who has undergone sex reassignment (a male-to-female post-operative transsexual). Second, there
are various laws which apply particularly to women such as the provisions of the Labor Code on employment of
women,39 certain felonies under the Revised Penal Code40 and the presumption of survivorship in case of
calamities under Rule 131 of the Rules of Court, 41 among others. These laws underscore the public policy in
relation to women which could be substantially affected if petitioner’s petition were to be granted.

It is true that Article 9 of the Civil Code mandates that "[n]o judge or court shall decline to render judgment by
reason of the silence, obscurity or insufficiency of the law." However, it is not a license for courts to engage in
judicial legislation. The duty of the courts is to apply or interpret the law, not to make or amend it.

In our system of government, it is for the legislature, should it choose to do so, to determine what guidelines
should govern the recognition of the effects of sex reassignment. The need for legislative guidelines becomes
particularly important in this case where the claims asserted are statute-based.

To reiterate, the statutes define who may file petitions for change of first name and for correction or change of
entries in the civil registry, where they may be filed, what grounds may be invoked, what proof must be presented
and what procedures shall be observed. If the legislature intends to confer on a person who has undergone sex
reassignment the privilege to change his name and sex to conform with his reassigned sex, it has to enact
legislation laying down the guidelines in turn governing the conferment of that privilege.

It might be theoretically possible for this Court to write a protocol on when a person may be recognized as having
successfully changed his sex. However, this Court has no authority to fashion a law on that matter, or on
anything else. The Court cannot enact a law where no law exists. It can only apply or interpret the written word of
its co-equal branch of government, Congress.

Petitioner pleads that "[t]he unfortunates are also entitled to a life of happiness, contentment and [the] realization
of their dreams." No argument about that. The Court recognizes that there are people whose preferences and
orientation do not fit neatly into the commonly recognized parameters of social convention and that, at least for
them, life is indeed an ordeal. However, the remedies petitioner seeks involve questions of public policy to be
addressed solely by the legislature, not by the courts.
WHEREFORE, the petition is hereby DENIED.

Costs against petitioner.

SO ORDERED.

Puno, C.J., Chairperson, Sandoval-Gutierrez, Azcuna, Garcia, JJ., concur.

Footnotes

1 Petitioner went for his elementary and high school, as well as his Bachelor of Science in Statistics and
Master of Arts, in the University of the Philippines. He took up Population Studies Program, Master of Arts
in Sociology and Doctor of Philosophy in Sociology at the University of Hawaii, in Manoa, Hawaii,
U.S.A. Rollo, p. 48.

2 This consisted of "penectomy [surgical removal of penis] bilateral oschiectomy [or orchiectomy which is
the surgical excision of the testes] penile skin inversion vaginoplasty [plastic surgery of the vagina] clitoral
hood reconstruction and augmentation mammoplasty [surgical enhancement of the size and shape of the
breasts]." Id.

3 On January 23, 2003, January 30, 2003 and February 6, 2003.

4 Penned by Judge Felixberto T. Olalia, Jr. Rollo, pp. 51-53.

5 Id., pp. 52-53 (citations omitted).

6 Docketed as CA-G.R. SP No. 78824.

7 Special Sixth Division.

8Penned by Associate Justice Arcangelita M. Romilla-Lontok with Associate Justices Marina L. Buzon
and Aurora Santiago-Lagman concurring. Rollo, pp. 25-33.

9 Resolution dated September 14, 2006, id., pp. 45-46.

10An Act Authorizing the City or Municipal Civil Registrar or the Consul General to Correct a Clerical or
Typographical Error in an Entry and/or Change of First Name or Nickname in the Civil Register Without
Need of a Judicial Order, Amending for the Purpose Articles 376 and 412 of the Civil Code of the
Philippines.

11 Wang v. Cebu City Civil Registrar, G.R. No. 159966, 30 March 2005, 454 SCRA 155.

12 Id.

13 K v. Health Division, Department of Human Resources, 277 Or. 371, 560 P.2d 1070 (1977).

14Under Section 2 (6) of RA 9048, "first name" refers to a name or nickname given to a person which
may consist of one or more names in addition to the middle names and last names. Thus, the term "first
name" will be used here to refer both to first name and nickname.

15 The last paragraph of Section 7 of RA 9048 provides:

SECTION 7. Duties and Powers of the Civil Registrar General. – xxx xxx xxx

Where the petition is denied by the city or municipal civil registrar or the consul general, the
petitioner may either appeal the decision to the civil registrar general or file the appropriate
petition with the proper court.

16SECTION 3. Who May File the Petition and Where. – Any person having direct and personal interest in
the correction of a clerical or typographical error in an entry and/or change of first name or nickname in
the civil register may file, in person, a verified petition with the local civil registry office of the city or
municipality where the record being sought to be corrected or changed is kept.
In case the petitioner has already migrated to another place in the country and it would not be practical
for such party, in terms of transportation expenses, time and effort to appear in person before the local
civil registrar keeping the documents to be corrected or changed, the petition may be filed, in person, with
the local civil registrar of the place where the interested party is presently residing or domiciled. The two
(2) local civil registrars concerned will then communicate to facilitate the processing of the petition.

Citizens of the Philippines who are presently residing or domiciled in foreign countries may file their
petition, in person, with the nearest Philippine Consulates.

The petitions filed with the city or municipal civil registrar or the consul general shall be processed in
accordance with this Act and its implementing rules and regulations.

All petitions for the clerical or typographical errors and/or change of first names or nicknames may be
availed of only once.

17SECTION 5. Form and Contents of the Petition. – The petition shall be in the form of an affidavit,
subscribed and sworn to before any person authorized by the law to administer oaths. The affidavit shall
set forth facts necessary to establish the merits of the petition and shall show affirmatively that the
petitioner is competent to testify to the matters stated. The petitioner shall state the particular erroneous
entry or entries, which are sought to be corrected and/or the change sought to be made.

The petition shall be supported with the following documents:

(1) A certified true machine copy of the certificate or of the page of the registry book containing
the entry or entries sought to be corrected or changed;

(2) At least two (2) public or private documents showing the correct entry or entries upon which
the correction or change shall be based; and

(3) Other documents which the petitioner or the city or municipal civil registrar or the consul
general may consider relevant and necessary for the approval of the petition.

In case of change of first name or nickname, the petition shall likewise be supported with the documents
mentioned in the immediately preceding paragraph. In addition, the petition shall be published at least
once a week for two (2) consecutive weeks in a newspaper of general circulation. Furthermore, the
petitioner shall submit a certification from the appropriate law enforcement agencies that he has no
pending case or no criminal record.

18 Republic v. Court of Appeals, G.R. No. 97906, 21 May 1992, 209 SCRA 189.

19 Supra note 11.

20 Id.

21 In re Ladrach, 32 Ohio Misc.2d 6, 513 N.E.2d 828 (1987).

22 Lee v. Court of Appeals, 419 Phil. 392 (2001).

23 Id.

24 Co v. Civil Register of Manila, G.R. No. 138496, 23 February 2004, 423 SCRA 420.

25 Id.

26 Id.

27 Beduya v. Republic of the Philippines, 120 Phil. 114 (1964).

28 Salonga, Jovito, Private International Law, 1995 Edition, Rex Bookstore, p. 238.

29This, of course, should be taken in conjunction with Articles 407 and 412 of the Civil Code which
authorizes the recording of acts, events and judicial decrees or the correction or change of errors
including those that occur after birth. Nonetheless, in such cases, the entries in the certificates of birth are
not be corrected or changed. The decision of the court granting the petition shall be annotated in the
certificates of birth and shall form part of the civil register in the Office of the Local Civil Registrar. (Co v.
Civil Register of Manila, supra note 24)

30The error pertains to one where the birth attendant writes "male" or "female" but the genitals of the child
are that of the opposite sex.

31Moreover, petitioner’s female anatomy is all man-made. The body that he inhabits is a male body in all
aspects other than what the physicians have supplied.

32 Black’s Law Dictionary, 8th edition (2004), p.1406.

33 Words and Phrases, volume 39, Permanent Edition, p. 106.

In re Application for Marriage License for Nash, 2003-Ohio-7221 (No. 2002-T-0149, slip op., Not
34

Reported in N.E.2d, 2003 WL 23097095 (Ohio App. 11 Dist., December 31, 2003), citing Webster’s II
New College Dictionary (1999).

35 Id.

36 Standard Oil Co. v. United States, 221 U.S. 1 (1911), 31 S.Ct. 502, 55 L.Ed. 619.

37 Article 1, Family Code.

38 Article 2(1), Id.

These are Articles 130 to 138 of the Labor Code which include nightwork prohibition, facilities for
39

women, prohibition on discrimination and stipulation against marriage, among others.

40These include Article 333 on adultery, Articles 337 to 339 on qualified seduction, simple seduction and
acts of lasciviousness with the consent of the offended party and Articles 342 and 343 on forcible and
consented abduction, among others.

41 Section 3(jj)(4).
Republic of the Philippines
SUPREME COURT
Manila

SECOND DIVISION

REPUBLIC OF THE PHILIPPINES, G.R. No. 166676

Petitioner, Present:

- versus - Quisumbing, J., Chairperson,

JENNIFER B. CAGANDAHAN, Carpio Morales,

Respondent. Tinga,

VELASCO, JR., and

BRION, JJ.

Promulgated:

September 12, 2008

x- - - - - - - - - - - - - - - - - - - - - - - - - - - - - - - - - - - - - - - - - - - - - - - - - - -x

DECISION

QUISUMBING, J.:

This is a petition for review under Rule 45 of the Rules of Court raising purely questions of
law and seeking a reversal of the Decision[1] dated January 12, 2005 of the Regional Trial
Court (RTC), Branch 33 of Siniloan, Laguna, which granted the Petition for Correction of
Entries in Birth Certificate filed by Jennifer B. Cagandahan and ordered the following changes
of entries in Cagandahan’s birth certificate: (1) the name "Jennifer Cagandahan" changed to
"Jeff Cagandahan" and (2) gender from "female" to "male."

The facts are as follows.

On December 11, 2003, respondent Jennifer Cagandahan filed a Petition for Correction of
Entries in Birth Certificate2 before the RTC, Branch 33 of Siniloan, Laguna.

In her petition, she alleged that she was born on January 13, 1981 and was registered as a
female in the Certificate of Live Birth but while growing up, she developed secondary male
characteristics and was diagnosed to have Congenital Adrenal Hyperplasia (CAH) which is a
condition where persons thus afflicted possess both male and female characteristics. She
further alleged that she was diagnosed to have clitoral hyperthropy in her early years and at
age six, underwent an ultrasound where it was discovered that she has small ovaries. At age
thirteen, tests revealed that her ovarian structures had minimized, she has stopped growing
and she has no breast or menstrual development. She then alleged that for all interests and
appearances as well as in mind and emotion, she has become a male person. Thus, she
prayed that her birth certificate be corrected such that her gender be changed from female to
male and her first name be changed from Jennifer to Jeff.

The petition was published in a newspaper of general circulation for three (3) consecutive
weeks and was posted in conspicuous places by the sheriff of the court. The Solicitor
General entered his appearance and authorized the Assistant Provincial Prosecutor to appear
in his behalf.

To prove her claim, respondent testified and presented the testimony of Dr. Michael Sionzon
of the Department of Psychiatry, University of the Philippines-Philippine General Hospital. Dr.
Sionzon issued a medical certificate stating that respondent’s condition is known as CAH. He
explained that genetically respondent is female but because her body secretes male
hormones, her female organs did not develop normally and she has two sex organs – female
and male. He testified that this condition is very rare, that respondent’s uterus is not fully
developed because of lack of female hormones, and that she has no monthly period. He
further testified that respondent’s condition is permanent and recommended the change of
gender because respondent has made up her mind, adjusted to her chosen role as male, and
the gender change would be advantageous to her.

The RTC granted respondent’s petition in a Decision dated January 12, 2005 which reads:

The Court is convinced that petitioner has satisfactorily shown that he is entitled to the reliefs
prayed [for]. Petitioner has adequately presented to the Court very clear and convincing
proofs for the granting of his petition. It was medically proven that petitioner’s body produces
male hormones, and first his body as well as his action and feelings are that of a male. He has
chosen to be male. He is a normal person and wants to be acknowledged and identified as a
male.

WHEREFORE, premises considered, the Civil Register of Pakil, Laguna is hereby ordered to
make the following corrections in the birth [c]ertificate of Jennifer Cagandahan upon payment
of the prescribed fees:

a) By changing the name from Jennifer Cagandahan to JEFF CAGANDAHAN; and

b) By changing the gender from female to MALE.

It is likewise ordered that petitioner’s school records, voter’s registry, baptismal certificate,
and other pertinent records are hereby amended to conform with the foregoing corrected
data.

SO ORDERED.[3]

Thus, this petition by the Office of the Solicitor General (OSG) seeking a reversal of the
abovementioned ruling.

The issues raised by petitioner are:

THE TRIAL COURT ERRED IN GRANTING THE PETITION CONSIDERING THAT:

I.

THE REQUIREMENTS OF RULES 103 AND 108 OF THE RULES OF COURT HAVE NOT BEEN
COMPLIED WITH; AND,

II.

CORRECTION OF ENTRY UNDER RULE 108 DOES NOT ALLOW CHANGE OF "SEX" OR
"GENDER" IN THE BIRTH CERTIFICATE, WHILE RESPONDENT’S MEDICAL CONDITION, i.e.,
CONGENITAL ADRENAL HYPERPLASIA DOES NOT MAKE HER A "MALE."4

Simply stated, the issue is whether the trial court erred in ordering the correction of entries in
the birth certificate of respondent to change her sex or gender, from female to male, on the
ground of her medical condition known as CAH, and her name from "Jennifer" to "Jeff,"
under Rules 103 and 108 of the Rules of Court.

The OSG contends that the petition below is fatally defective for non-compliance with Rules
103 and 108 of the Rules of Court because while the local civil registrar is an indispensable
party in a petition for cancellation or correction of entries under Section 3, Rule 108 of the
Rules of Court, respondent’s petition before the court a quo did not implead the local civil
registrar.5 The OSG further contends respondent’s petition is fatally defective since it failed to
state that respondent is a bona fide resident of the province where the petition was filed for at
least three (3) years prior to the date of such filing as mandated under Section 2(b), Rule 103
of the Rules of Court.6 The OSG argues that Rule 108 does not allow change of sex or gender
in the birth certificate and respondent’s claimed medical condition known as CAH does not
make her a male.7

On the other hand, respondent counters that although the Local Civil Registrar of Pakil,
Laguna was not formally named a party in the Petition for Correction of Birth Certificate,
nonetheless the Local Civil Registrar was furnished a copy of the Petition, the Order to
publish on December 16, 2003 and all pleadings, orders or processes in the course of the
proceedings,8 respondent is actually a male person and hence his birth certificate has to be
corrected to reflect his true sex/gender,9 change of sex or gender is allowed under Rule
108,10 and respondent substantially complied with the requirements of Rules 103 and 108 of
the Rules of Court.11

Rules 103 and 108 of the Rules of Court provide:

Rule 103

CHANGE OF NAME

Section 1. Venue. – A person desiring to change his name shall present the petition to the
Regional Trial Court of the province in which he resides, [or, in the City of Manila, to the
Juvenile and Domestic Relations Court].

Sec. 2. Contents of petition. – A petition for change of name shall be signed and verified by
the person desiring his name changed, or some other person on his behalf, and shall set
forth:

(a) That the petitioner has been a bona fide resident of the province where the petition is filed
for at least three (3) years prior to the date of such filing;

(b) The cause for which the change of the petitioner's name is sought;

(c) The name asked for.

Sec. 3. Order for hearing. – If the petition filed is sufficient in form and substance, the court,
by an order reciting the purpose of the petition, shall fix a date and place for the hearing
thereof, and shall direct that a copy of the order be published before the hearing at least once
a week for three (3) successive weeks in some newspaper of general circulation published in
the province, as the court shall deem best. The date set for the hearing shall not be within
thirty (30) days prior to an election nor within four (4) months after the last publication of the
notice.

Sec. 4. Hearing. – Any interested person may appear at the hearing and oppose the petition.
The Solicitor General or the proper provincial or city fiscal shall appear on behalf of the
Government of the Republic.

Sec. 5. Judgment. – Upon satisfactory proof in open court on the date fixed in the order that
such order has been published as directed and that the allegations of the petition are true, the
court shall, if proper and reasonable cause appears for changing the name of the petitioner,
adjudge that such name be changed in accordance with the prayer of the petition.

Sec. 6. Service of judgment. – Judgments or orders rendered in connection with this rule
shall be furnished the civil registrar of the municipality or city where the court issuing the
same is situated, who shall forthwith enter the same in the civil register.

Rule 108

CANCELLATION OR CORRECTION OF ENTRIES

IN THE CIVIL REGISTRY

Section 1. Who may file petition. – Any person interested in any act, event, order or decree
concerning the civil status of persons which has been recorded in the civil register, may file a
verified petition for the cancellation or correction of any entry relating thereto, with the
Regional Trial Court of the province where the corresponding civil registry is located.

Sec. 2. Entries subject to cancellation or correction. – Upon good and valid grounds, the
following entries in the civil register may be cancelled or corrected: (a) births; (b) marriages;
(c) deaths; (d) legal separations; (e) judgments of annulments of marriage; (f) judgments
declaring marriages void from the beginning; (g) legitimations; (h) adoptions; (i)
acknowledgments of natural children; (j) naturalization; (k) election, loss or recovery of
citizenship; (l) civil interdiction; (m) judicial determination of filiation; (n) voluntary
emancipation of a minor; and (o) changes of name.

Sec. 3. Parties. – When cancellation or correction of an entry in the civil register is sought, the
civil registrar and all persons who have or claim any interest which would be affected thereby
shall be made parties to the proceeding.

Sec. 4. Notice and publication. – Upon the filing of the petition, the court shall, by an order, fix
the time and place for the hearing of the same, and cause reasonable notice thereof to be
given to the persons named in the petition. The court shall also cause the order to be
published once a week for three (3) consecutive weeks in a newspaper of general circulation
in the province.

Sec. 5. Opposition. – The civil registrar and any person having or claiming any interest under
the entry whose cancellation or correction is sought may, within fifteen (15) days from notice
of the petition, or from the last date of publication of such notice, file his opposition thereto.

Sec. 6. Expediting proceedings. – The court in which the proceedings is brought may make
orders expediting the proceedings, and may also grant preliminary injunction for the
preservation of the rights of the parties pending such proceedings.

Sec. 7. Order. – After hearing, the court may either dismiss the petition or issue an order
granting the cancellation or correction prayed for. In either case, a certified copy of the
judgment shall be served upon the civil registrar concerned who shall annotate the same in
his record.

The OSG argues that the petition below is fatally defective for non-compliance with Rules 103
and 108 of the Rules of Court because respondent’s petition did not implead the local civil
registrar. Section 3, Rule 108 provides that the civil registrar and all persons who have or
claim any interest which would be affected thereby shall be made parties to the proceedings.
Likewise, the local civil registrar is required to be made a party in a proceeding for the
correction of name in the civil registry. He is an indispensable party without whom no final
determination of the case can be had.[12] Unless all possible indispensable parties were duly
notified of the proceedings, the same shall be considered as falling much too short of the
requirements of the rules.13 The corresponding petition should also implead as respondents
the civil registrar and all other persons who may have or may claim to have any interest that
would be affected thereby.14 Respondent, however, invokes Section 6,[15] Rule 1 of the Rules
of Court which states that courts shall construe the Rules liberally to promote their objectives
of securing to the parties a just, speedy and inexpensive disposition of the matters brought
before it. We agree that there is substantial compliance with Rule 108 when respondent
furnished a copy of the petition to the local civil registrar.

The determination of a person’s sex appearing in his birth certificate is a legal issue and the
court must look to the statutes. In this connection, Article 412 of the Civil Code provides:

ART. 412. No entry in a civil register shall be changed or corrected without a judicial order.

Together with Article 376[16] of the Civil Code, this provision was amended by Republic Act
No. 9048[17] in so far as clerical or typographical errors are involved. The correction or
change of such matters can now be made through administrative proceedings and without
the need for a judicial order. In effect, Rep. Act No. 9048 removed from the ambit of Rule 108
of the Rules of Court the correction of such errors. Rule 108 now applies only to substantial
changes and corrections in entries in the civil register. 18

Under Rep. Act No. 9048, a correction in the civil registry involving the change of sex is not a
mere clerical or typographical error. It is a substantial change for which the applicable
procedure is Rule 108 of the Rules of Court.19

The entries envisaged in Article 412 of the Civil Code and correctable under Rule 108 of the
Rules of Court are those provided in Articles 407 and 408 of the Civil Code:

ART. 407. Acts, events and judicial decrees concerning the civil status of persons shall be
recorded in the civil register.

ART. 408. The following shall be entered in the civil register:


(1) Births; (2) marriages; (3) deaths; (4) legal separations; (5) annulments of marriage; (6)
judgments declaring marriages void from the beginning; (7) legitimations; (8) adoptions; (9)
acknowledgments of natural children; (10) naturalization; (11) loss, or (12) recovery of
citizenship; (13) civil interdiction; (14) judicial determination of filiation; (15) voluntary
emancipation of a minor; and (16) changes of name.

The acts, events or factual errors contemplated under Article 407 of the Civil Code include
even those that occur after birth.20

Respondent undisputedly has CAH. This condition causes the early or "inappropriate"
appearance of male characteristics. A person, like respondent, with this condition produces
too much androgen, a male hormone. A newborn who has XX chromosomes coupled with
CAH usually has a (1) swollen clitoris with the urethral opening at the base, an ambiguous
genitalia often appearing more male than female; (2) normal internal structures of the female
reproductive tract such as the ovaries, uterus and fallopian tubes; as the child grows older,
some features start to appear male, such as deepening of the voice, facial hair, and failure to
menstruate at puberty. About 1 in 10,000 to 18,000 children are born with CAH.

CAH is one of many conditions[21] that involve intersex anatomy. During the twentieth
century, medicine adopted the term "intersexuality" to apply to human beings who cannot be
classified as either male or female.[22] The term is now of widespread use. According to
Wikipedia, intersexuality "is the state of a living thing of a gonochoristic species whose sex
chromosomes, genitalia, and/or secondary sex characteristics are determined to be neither
exclusively male nor female. An organism with intersex may have biological characteristics of
both male and female sexes."

Intersex individuals are treated in different ways by different cultures. In most societies,
intersex individuals have been expected to conform to either a male or female gender
role.[23] Since the rise of modern medical science in Western societies, some intersex people
with ambiguous external genitalia have had their genitalia surgically modified to resemble
either male or female genitals.[24] More commonly, an intersex individual is considered as
suffering from a "disorder" which is almost always recommended to be treated, whether by
surgery and/or by taking lifetime medication in order to mold the individual as neatly as
possible into the category of either male or female.

In deciding this case, we consider the compassionate calls for recognition of the various
degrees of intersex as variations which should not be subject to outright denial. "It has been
suggested that there is some middle ground between the sexes, a ‘no-man’s land’ for those
individuals who are neither truly ‘male’ nor truly ‘female’."[25] The current state of Philippine
statutes apparently compels that a person be classified either as a male or as a female, but
this Court is not controlled by mere appearances when nature itself fundamentally negates
such rigid classification.

In the instant case, if we determine respondent to be a female, then there is no basis for a
change in the birth certificate entry for gender. But if we determine, based on medical
testimony and scientific development showing the respondent to be other than female, then a
change in the

subject’s birth certificate entry is in order.

Biologically, nature endowed respondent with a mixed (neither consistently and categorically
female nor consistently and categorically male) composition. Respondent has female (XX)
chromosomes. However, respondent’s body system naturally produces high levels of male
hormones (androgen). As a result, respondent has ambiguous genitalia and the phenotypic
features of a male.

Ultimately, we are of the view that where the person is biologically or naturally intersex the
determining factor in his gender classification would be what the individual, like respondent,
having reached the age of majority, with good reason thinks of his/her sex. Respondent here
thinks of himself as a male and considering that his body produces high levels of male
hormones (androgen) there is preponderant biological support for considering him as being
male. Sexual development in cases of intersex persons makes the gender classification at
birth inconclusive. It is at maturity that the gender of such persons, like respondent, is fixed.

Respondent here has simply let nature take its course and has not taken unnatural steps to
arrest or interfere with what he was born with. And accordingly, he has already ordered his
life to that of a male. Respondent could have undergone treatment and taken steps, like
taking lifelong medication,[26] to force his body into the categorical mold of a female but he
did not. He chose not to do so. Nature has instead taken its due course in respondent’s
development to reveal more fully his male characteristics.

In the absence of a law on the matter, the Court will not dictate on respondent concerning a
matter so innately private as one’s sexuality and lifestyle preferences, much less on whether
or not to undergo medical treatment to reverse the male tendency due to CAH. The Court will
not consider respondent as having erred in not choosing to undergo treatment in order to
become or remain as a female. Neither will the Court force respondent to undergo treatment
and to take medication in order to fit the mold of a female, as society commonly currently
knows this gender of the human species. Respondent is the one who has to live with his
intersex anatomy. To him belongs the human right to the pursuit of happiness and of health.
Thus, to him should belong the primordial choice of what courses of action to take along the
path of his sexual development and maturation. In the absence of evidence that respondent is
an "incompetent"[27] and in the absence of evidence to show that classifying respondent as
a male will harm other members of society who are equally entitled to protection under the
law, the Court affirms as valid and justified the respondent’s position and his personal
judgment of being a male.

In so ruling we do no more than give respect to (1) the diversity of nature; and (2) how an
individual deals with what nature has handed out. In other words, we respect respondent’s
congenital condition and his mature decision to be a male. Life is already difficult for the
ordinary person. We cannot but respect how respondent deals with his unordinary state and
thus help make his life easier, considering the unique circumstances in this case.

As for respondent’s change of name under Rule 103, this Court has held that a change of
name is not a matter of right but of judicial discretion, to be exercised in the light of the
reasons adduced and the consequences that will follow.[28] The trial court’s grant of
respondent’s change of name from Jennifer to Jeff implies a change of a feminine name to a
masculine name. Considering the consequence that respondent’s change of name merely
recognizes his preferred gender, we find merit in respondent’s change of name. Such a
change will conform with the change of the entry in his birth certificate from female to male.

WHEREFORE, the Republic’s petition is DENIED. The Decision dated January 12, 2005 of the
Regional Trial Court, Branch 33 of Siniloan, Laguna, is AFFIRMED. No pronouncement as to
costs.

SO ORDERED.

LEONARDO A. QUISUMBING

Associate Justice

WE CONCUR:

CONCHITA CARPIO MORALES

Associate Justice
DANTE O. TINGA PRESBITERO J. VELASCO, JR.

Associate Justice Associate Justice


ARTURO D. BRION

Associate Justice

ATTESTATION

I attest that the conclusions in the above Decision had been reached in consultation before
the case was assigned to the writer of the opinion of the Court’s Division.

LEONARDO A. QUISUMBING

Associate Justice
Chairperson

CERTIFICATION

Pursuant to Section 13, Article VIII of the Constitution, and the Division Chairperson’s
Attestation, I certify that the conclusions in the above Decision had been reached in
consultation before the case was assigned to the writer of the opinion of the Court’s Division.

REYNATO S. PUNO

Chief Justice

1
Rollo, pp. 29-32. Penned by Judge Florenio P. Bueser.

2
Id. at 33-37.

3
Id. at 31-32.

4
Id. at 97.

5
Id. at 99.

6
Id. at 103.

7
Id. at 104.

8
Id. at 136.

9
Id. at 127.

10
Id. at 134.

11
Id. at 136.

12
Republic v. Court of Appeals, G.R. No. 103695, March 15, 1996, 255 SCRA 99, 106.

13
Ceruila v. Delantar, G.R. No. 140305, December 9, 2005, 477 SCRA 134, 147.

14
Republic v. Benemerito, G.R. No. 146963, March 15, 2004, 425 SCRA 488, 492.

15
SEC. 6. Construction.- These Rules shall be liberally construed in order to promote their
objective of securing a just, speedy and inexpensive disposition of every action and
proceeding.

16
Art. 376. No person can change his name or surname without judicial authority.

17
An Act Authorizing the City or Municipal Civil Registrar or the Consul General to Correct a
Clerical or Typographical Error in an Entry and/or Change of First Name or Nickname in the
Civil Registrar Without Need of a Judicial Order, Amending for this Purpose Articles 376 and
412 of the Civil Code of the Philippines. Approved, March 22, 2001.

18
Silverio v. Republic of the Philippines, G.R. No. 174689, October 19, 2007, 537 SCRA 373,
388.

19
Id. at 389.

20
Id. at 389.

21
(1) 5-alpha reductase deficiency; (2) androgen insensitivity syndrome; (3) aphallia; (4)
clitoromegaly; (5) congenital adrenal hyperplasia; (6) gonadal dysgenesis (partial &
complete); (7) hypospadias; (8) Kallmann syndrome; (9) Klinefelter syndrome; (10)
micropenis; (11) mosaicism involving sex chromosomes; (12) MRKH (mullerian agenesis;
vaginal agenesis; congenital absence of vagina); (13) ovo-testes (formerly called "true
hermaphroditism"); (14) partial androgen insensitivity syndrome; (15) progestin induced
virilization; (16) Swyer syndrome; (17) Turner syndrome. [Intersexuality
<http://en.wikipedia.org/wiki/Intersexual> (visited August 15, 2008).]

22
Intersexuality <http://en.wikipedia.org/wiki/Intersexual> (visited August 15, 2008).

23
Intersexuality <http://en.wikipedia.org/wiki/Intersexual> (visited August 15, 2008), citing
Gagnon and Simon 1973.

24
Intersexuality <http://en.wikipedia.org/wiki/Intersexual> (visited August 15, 2008).

25
M.T. v. J.T. 140 N.J. Super 77 355 A. 2d 204.

26
The goal of treatment is to return hormone levels to normal. This is done by taking a form
of cortisol (dexamethasone), fludrocortisone, or hydrocortisone) every day. Additional doses
of medicine are needed during times of stress, such as severe illness or surgery.

xxxx

Parents of children with congenital adrenal hyperplasia should be aware of the side effects of
steroid therapy. They should report signs of infection and stress to their health care provider
because increases in medication may be required. In additional, steroid medications cannot
be stopped suddenly, or adrenal insufficiency will result.

xxxx

The outcome is usually associated with good health, but short stature may result even with
treatment. Males have normal fertility. Females may have a smaller opening of the vagina and
lower fertility. Medication to treat this disorder must be continued for life. (Congenital Adrenal
Hyperplasia <http://www.nlm.nih.gov/medlineplus/encyclopedia.html>.)

27
The word "incompetent" includes persons suffering the penalty of civil interdiction or who
are hospitalized lepers, prodigals, deaf and dumb who are unable to read and write, those
who are of unsound mind, even though they have lucid intervals, and persons not being of
unsound mind, but by reason of age, disease, weak mind, and other similar causes, cannot,
without outside aid, take care of themselves and manage their property, becoming thereby an
easy prey for deceit and exploitation. (See Sec. 2 of Rule 92 of the Rules of Court)

28
Yu v. Republic of the Philippines, 123 Phil. 1106, 1110 (1966).
Republic of the Philippines
SUPREME COURT
Manila

EN BANC

G.R. No. L-4904 February 5, 1909

ROSALIA MARTINEZ, plaintiff-appellant,


vs.
ANGEL TAN, defendant-appellee.

Domingo Franco, for appellant.


Doroteo Karagdag, for appellee.

WILLARD, J.:

The only question in this case is whether or not the plaintiff and the defendant were married on the 25th day of
September, 1907, before the justice of the peace, Jose Ballori, in the town of Palompon in the Province of Leyte.

There was received in evidence at the trial what is called an expediente de matrimonio civil. It is written in
Spanish and consists, first, of a petition directed to the justice of the peace, dated on the 25th of September,
1907, signed by the plaintiff and the defendant, in which they state that they have mutually agreed to enter into a
contract of marriage before the justice of the peace, and ask that the justice solemnize the marriage. Following
this is a document dated on the same day, signed by the justice of the peace, by the plaintiff, by the defendant,
and by Zacarias Esmero and Pacita Ballori. It states the presentation of the petition above mentioned; that the
persons who signed it where actually present in the office of the justice on the same day named; that they ratified
under oath the contents of the petition, and that they insisted in what they had there asked for. It also stated that
being required to produce witnesses of the marriage, the presented Zacarias Esmero as a witness for the
husband and Pacita Ballori as a witness for the wife. Following this is a certificate of marriage signed by the
justice of the peace and the witnesses Zacarias Esmero and Pacita Ballori, dated the 25th day of September,
1907, in which it is stated that the plaintiff and the defendant were legally married by the justice of the peace in
the presence of the witnesses on that day.

The court below decided the case in favor of the defendant, holding that the parties were legally married on the
day named. The evidence in support of that decision is: First. The document itself, which the plaintiff admits that
she signed. Second. The evidence of the defendant, who testifies that he and said plaintiff appeared before the
justice of the peace at the time named, together with the witness Zacarias Esmero and Pacita Ballori, and that
they all signed the document above mentioned. Third. The evidence of Zacarias Esmero, one of the above-
named witnesses, who testifies that the plaintiff, the defendant, and Pacita Ballori appeared before the justice at
the time named and did sign the document referred to. Fourth. The evidence of Pacita Ballori, who testified to the
same effect. Fifth. The evidence of Jose Santiago, the bailiff of the court of the justice of the peace, who testified
that the plaintiff, the defendant, the two witnesses above-named, and the justice of the peace were all present in
the office of the justice of the peace at the time mentioned.

The only direct evidence in favor of the plaintiff is her own testimony that she never appeared before the justice of
the peace and never was married to the defendant. She admits that she signed the document in question, but
says that she signed it in her own home, without reading it, and at the request of the defendant, who told her that
it was a paper authorizing him to ask the consent of her parents to the marriage.

There is some indirect evidence which the plaintiff claims supports her case, but which we think, when properly
considered, is not entitled to much weight. The plaintiff at the time was visiting, in the town of Palompon, her
married brother and was there for about two weeks. The wife of her brother, Rosario Bayot, testified that the
plaintiff never left the house except in her company. But she admitted on cross-examination that she herself went
to school every morning and that on one occasion the plaintiff had gone to church unaccompanied. The
testimony of this witness loses its force when the testimony of Pacita Ballori is considered. She says that at the
request of the defendant on the day named, about 5 o'clock in the afternoon, she went to the store of a Chinese
named Veles; that there she met the plaintiff and her mother; that she asked the mother of the plaintiff to allow
the plaintiff to accompany her, the witness, to her own house for the purpose of examining some dress patterns;
that the mother gave her consent and the two rights left the store, but instead of going to the house of the witness
they went directly to the office of the justice of the peace where the ceremony took place; that after the ceremony
had taken place, one came advising them that the mother was approaching, and that they thereupon hurriedly left
the office of the justice and went to the house of Pacita Ballori, where the mother later found them.

The other testimony of the plaintiff relating to certain statements made by the justice of the peace, who died after
the ceremony was performed and before the trial, and certain statements made by Pacita Ballori, is not sufficient
to overcome the positive testimony of the witnesses for the defendant.
The other testimony of Pacita Ballori is severely criticized by counsel for the appellant in his brief. It appears that
during her first examination she was seized with an hysterical attack and practically collapsed at the trial. Her
examination was adjourned to a future day and was completed in her house where she was sick in bed. It is
claimed by counsel that her collapse was due to the fact that she recognized that she testified falsely in stating
the office of the justice of the peace was at the time in the municipal building, when, in fact, it was in a private
house. We do not think that the record justifies the claim of the appellant. The statement as to the location of the
office of the justice of the peace was afterwards corrected by the witness and we are satisfied that she told the
facts substantially as they occurred.

There is, moreover, in the case written evidence which satisfies us that the plaintiff was not telling the truth when
she said she did not appear before the justice of the peace. This evidence consists of eight letters, which the
defendant claims were all written by the plaintiff. The plaintiff admits that she wrote letters numbered 2 and 9. The
authenticity of the others was proven. No. 9 is as follows:

ANGEL: Up to this time I did not see my father; but I know that he is very angry and if he be informed that
we have been married civilly, I am sure that he will turn me out of the house.

Do what you may deem convenient, as I don't know what to do.

Should I be able to go to-morrow to Merida, I shall do so, because I can not remain here.

Yours, ROSAL.

Letter No. 6, which bears no date, but which undoubtedly was written on the morning of the 25th of September, is
as follows:

Sr. D. ANGEL, TAN.

ANGEL: It is impossible for me to go to the house of Veles this morning because my sister in law will not
let me go there; if it suits you, I believe that this afternoon, about 5 or 6 o'clock, is the best hour.

Arrange everything, as I shall go there only for the purpose of signing, and have Pacita wait for me at the
Chinese store, because I don't like to go without Pacita.

The house must be one belonging to prudent people, and no one should know anything about it.

Yours, ROSAL.

It will be noticed that this corroborates completely the testimony of Pacita Ballori as to her meeting the plaintiff in
the afternoon at the store of the Chinese, Veles. Letter No. 7 is also undated, but was evidently written after the
marriage before the justice of the peace. It is as follows:

Sr. D. ANGEL, TAN.

ANGEL: If you want to speak to my mother, who is also yours, come here by and by, at about 9 or 10,
when you see that the tide is high because my brother will have to go to the boat for the purpose of
loading lumber.

Don't tell her that we have been civilly married, but tell her at first that you are willing to celebrate the
marriage at this time, because I don't like her to know to-day that we have been at the court-house,
inasmuch as she told me this morning that she heard that we would go to the court, and that we must not
cause her to be ashamed, and that if I insist on being married I must do it right.

Tell her also that you have asked me to carry you.

I send you herewith the letter of your brother, in order that you may do what he wishes.

Yours, ROSAL.

Letter No. 8 was also evidently written after the marriage and is in part as follows:

Sr. D. ANGEL TAN.


ANGEL: I believe it is better for you to go to Ormoc on Sunday of the steamer Rosa, for the purpose of
asking my father's permission for our marriage, and in case he fails to give it, then we shall do what we
deem proper, and, if he does not wish us to marry without his permission, you must request his consent.

Tell me who said that my sister in law knows that we are civilly married; my brother ill treatment is a
matter of no importance, as every thing may be carried out, with patience.

It was proven at the trial that the defendant did go to Ormoc on the steamer Rosa as indicated in this letter, and
that the plaintiff was on the same boat. The plaintiff testified, however, that she had no communication with the
defendant during the voyage. The plaintiff and the defendant never lived together as husband and wife, and upon
her arrival in Ormoc, after consulting with her family, she went to Cebu and commenced this action, which was
brought for the purpose of procuring the cancellation of the certificate of marriage and for damages. The
evidence strongly preponderates in favor of the decision of the court below to the effect that the plaintiff appeared
before the justice of the peace at the time named.

It is claimed by the plaintiff that what took place before the justice of the peace, even admitting all that the
witnesses for the defendant testified to, did not constitute a legal marriage. General orders, No. 68, section 6, is
as follows:

No particular form from the ceremony of marriage is required, but the parties must declare in the
presence of the person solemnizing the marriage, that they take each other as husband and wife.

Zacarias Esmero, one of the witnesses, testified that upon the occasion in question the justice of the peace said
nothing until after the document was signed and then addressing himself to the plaintiff and the defendant said,
"You are married." The petition signed the plaintiff and defendant contained a positive statement that they had
mutually agreed to be married and they asked the justice of the peace to solemnize the marriage. The document
signed by the plaintiff, the defendant, and the justice of the peace, stated that they ratified under oath, before the
justice, the contents of the petition and that witnesses of the marriage were produced. A mortgage took place as
shown by the certificate of the justice of the peace, signed by both contracting parties, which certificates gives
rise to the presumption that the officer authorized the marriage in due form, the parties before the justice of the
peace declaring that they took each other as husband and wife, unless the contrary is proved, such presumption
being corroborated in this case by the admission of the woman to the effect that she had contracted the marriage
certified to in the document signed by her, which admission can only mean the parties mutually agreed to unite in
marriage when they appeared and signed the said document which so states before the justice of the peace who
authorized the same. It was proven that both the plaintiff and the defendant were able to read and write the
Spanish language, and that they knew the contents of the document which they signed; and under the
circumstances in this particular case were satisfied, and so hold, that what took place before the justice of the
peace on this occasion amounted to a legal marriage.

The defendant's original answer was a general denial of the allegations contained in the complaint. Among these
allegations was a statement that the parties had obtain previously the consent of the plaintiff's parents. The
defendant was afterwards allowed to amend his answer so that it was a denial of the allegations of the complaint
except that relating to the condition in regard to the consent of the parents. The plaintiff objected to the allowance
of this amendment. After the trial had commenced the defendant was again allowed to amend his answer so that
it should be an admission of paragraphs 2 and 3 of the complaint, except that part which related to the consent of
the parents. It will be seen that this second amendment destroyed completely the first amendment and the
defendants lawyer stated that what he intended to allege in his first amendment, but by reason of the haste with
which the first amendment was drawn he had unintentionally made it exactly the opposite of what he had
intended to state. After argument the court allowed the second amendment. We are satisfied that in this
allowance there was no abuse of discretion and we do not see how the plaintiff was in any way prejudiced. She
proceeded with the trial of the case without asking for a continuance.

The judgment of the court below acquitting the defendant of the complaint is affirmed, with the costs of this
instance against the appellant.

Arellano, C.J., Torres, Mapa, Johnson, and Carson, JJ., concur.


FIRST DIVISION

[G.R. No. L-3474. September 20, 1907. ]

RAFAEL ENRIQUEZ, ET AL., Plaintiffs-Appellees, v. FRANCISCO ENRIQUEZ, ET


AL., Defendants-Appellants.

W. A. Kincaid, for Appellants.

Hartigan, Rohde & Gutierrez, for Appellees.

SYLLABUS

1. APPEAL; ASSIGNMENT OF ERRORS. — Where upon an appeal by both the plaintiff and the
defendant it appears that the plaintiff made no assignment of errors against the rulings or against
the decision of the lower court. so much of the judgment as may be adverse to the plaintiff can not
be considered by this court.

2. MARRIAGE; SUFFICIENCY OF PROOF. — When a marriage ceremony is duly celebrated between


two persons upon a certain date, in order to show that the same persons are lawfully married
before that time it is necessary to prove that a marriage ceremony had theretofore been
celebrated between them. The fact of having had children prior to the celebration of the latter
ceremony does not prove a former marriage.

3. COMMON-LAW MARRIAGE. — What are known as common-law marriages in England and the
United States were never recognized by the Spanish law in force in these Islands. No valid
marriage could exist under the Spanish law unless some ecclesiastical or civil functionary
intervened in its celebration.

DECISION

WILLARD, J. :

The plaintiffs brought this action in the Court of First Instance of Manila on the 2d day of June,
1902, asking that a deed made by Antonio Enriquez on the 27th of March, 1883, conveying to the
defendant Carmen de la Cavada certain real estate in the city of Manila, be annulled and set aside.
Judgment was rendered in the court below to the effect that the plaintiffs were the owners of an
undivided half of the said real estate, and that the defendant Carmen de la Cavada should pay to
the plaintiffs upward of 1,300 pesos, as rents and profits thereof. Both parties moved for a new
trial on the ground of the insufficiency of the evidence, by the plaintiffs in this court have neither
assigned as errors the rulings made against them, by the lower court nor have they discussed any
such rulings in their brief. So much of the decision, therefore, as is adverse to the plaintiffs we can
not consider, and the questions to be resolved are those presented by the appeal of the
defendants.

The decision of the court below was based upon the following facts, deemed to be established by
the evidence, namely, that Antonio Enriquez and Doña Ciriaca Villanueva were legally married
prior to the year 1860; that in 1861 the property in question was acquired by Antonio Enriquez;
that it thereby became a part of the property belonging to the conjugal partnership; that Doña
Ciriaca Villanueva died in 1882; that upon her death an undivided half of the property passed to
her heirs, the plaintiffs; that when, in 1883, Antonio Enriquez undertook to convey the entire
property to the defendant Doña Carmen de la Cavada he, as matter of law, conveyed one half
thereof, and that the other half remained and now is the property of the plaintiffs.

The correctness of this decision depends upon the question as to whether Antonio Enriquez and
Doña Ciriaca Villanueva were legally married in 1861. The court below found and, the evidence
sustains that finding, that a marriage ceremony was duly performed between these persons in
1865, but held that the fact that prior to 1861 they had lived together as husband and wife, had
been recognized as such, and had children who were baptized as the legitimate children of their
lawful marriage was sufficient evidence to raise the presumption that they were at the time legally
married.
A marriage ceremony having been duly celebrated between these persons in 1865, it is necessary,
in order to show that they were legally married before that time, to prove that the same kind of a
marriage ceremony had theretofore been celebrated. Although, as held by the Supreme Court of
the United States, by the common law of England, a valid marriage might be contacted without the
intervention of any ecclesiastical or civil functionary (Traverse v. Rheinhardt, 27 Sup. Ct. Rep.,
563, decided April 15, 1907), yet such was never the law in these Islands during the Spanish
domination here. During the entire period of that domination no valid marriage could exist unless
some ecclesiastical or civil functionary intervened in its celebration, and the intervention of civil
functionaries was limited to the short time elapsing between the 8th day of December, 1889, when
the Civil Code took effect here, and the 29th day of the same month, when the provisions of Title
IV, Book I, of that code were suspended. During the time covered by the lives of Antonio Enriquez
and Doña Ciriaca Villanueva no valid marriage between them could be contracted by their mere
agreement to live together as husband and wife.

There is no proof in this case that a marriage, valid in accordance with the laws then in force in
these Islands, was celebrated between these persons in 1865. In order to show that they were
before that time husband and wife, it was necessary to prove that a marriage ceremony in which
an ecclesiastical functionary intervened was duly celebrated. No proof of any such marriage was
offered. As has been said, the fact that prior to 1865 they lived together as husband and wife and
had children is not evidence in this case to show that they were married prior to that time. Nor is
the fact that in the certificates of baptism of these children it is stated that they were the
legitimate children of the lawful marriage of their parents.

The court below said: jgc:chanrobles.com.ph

"Loss of the record of the first marriage, or some like reason, might have made the second
ceremony seem necessary and for that reason it was celebrated." cralaw virtua1aw library

This consideration is to our minds entirely insufficient to explain the celebration of the second
marriage. If the former marriage had taken place, it must have been celebrated before some priest
or other officer of the Roman Catholic Church. The law required that a record of such marriages
should be kept in the parish registry, and if such marriage in fact had been performed, it probably
would have been easy to have obtained a certified copy of such record. No evidence was offered in
this case of any attempt to obtain such record or that the records of the church were the ceremony
had been performed had been destroyed. In fact, no proof whatever was offered in the case to
show the celebration of such prior marriage, except the facts hereinbefore stated, that the parties
lived together as husband and wife and had children who were baptized as aforesaid. We hold that
this evidence is insufficient to prove in this case a prior marriage, where it appears that a marriage
ceremony was duly performed between the parties at a later date; and we therefore hold that
Antonio Enriquez and Doña Ciriaca Villanueva were not legally married prior to 1865, and that,
therefore, when this property was acquired by Antonio in 1861 it did not become a part of the
property belonging to the conjugal partnership, but on the contrary was a part of the capital which
he brought to the marriage. Being a part of the capital brought to the marriage by the husband,
upon the death of the wife — the husband surviving her — no interest whatever therein passed to
her heirs.

The judgment of the court below, which rests solely upon the proposition that at the time of the
death of Doña Ciriaca Villanueva one-half of this property passed to her heirs, can not, therefore,
be sustained. That judgment is reversed, without costs to either in this court, judgment is entered
acquitting the defendants of the complaint, with the costs of the first instance against the
plaintiffs. So ordered.

Arellano, C.J., Torres, Johnson, and Tracey, JJ., concur.


Republic of the Philippines
SUPREME COURT
Manila

THIRD DIVISION

G.R. No. 198780 October 16, 2013

REPUBLIC OF THE PHILIPPINES, Petitioner,


vs.
LIBERTY D. ALBIOS, Respondent.

DECISION

MENDOZA, J.:

This is a petition for review on certiorari under Rule 45 of the Rules t of Court assailing the September 29, 2011
Decision1 of the Court of Appeals (CA), in CA-G.R. CV No. 95414, which affirmed the April 25, 2008Decision2 of
the Regional Trial Court, Imus, Cavite (RTC). declaring the marriage of Daniel Lee Fringer (Fringer) and
respondent Liberty Albios (A/bios) as void from the beginning.

The facts

On October 22, 2004, Fringer, an American citizen, and Albios were married before Judge Ofelia I. Calo of the
Metropolitan Trial Court, Branch59, Mandaluyong City (MeTC), as evidenced by a Certificate of Marriage with
Register No. 2004-1588.3

On December 6, 2006, Albios filed with the RTC a petition for declaration of nullity 4 of her marriage with Fringer.
She alleged that immediately after their marriage, they separated and never lived as husband and wife because
they never really had any intention of entering into a married state or complying with any of their essential marital
obligations. She described their marriage as one made in jest and, therefore, null and void ab initio .

Summons was served on Fringer but he did not file his answer. On September 13, 2007, Albios filed a motion to
set case for pre-trial and to admit her pre-trial brief. The RTC ordered the Assistant Provincial Prosecutor to
conduct an investigation and determine the existence of a collusion. On October 2, 2007, the Assistant
Prosecutor complied and reported that she could not make a determination for failure of both parties to appear at
the scheduled investigation.

At the pre-trial, only Albios, her counsel and the prosecutor appeared. Fringer did not attend the hearing despite
being duly notified of the schedule. After the pre-trial, hearing on the merits ensued.

Ruling of the RTC

In its April 25, 2008 Decision,5 the RTC declared the marriage void ab initio, the dispositive portion of which
reads:

WHEREFORE, premises considered, judgment is hereby rendered declaring the marriage of Liberty Albios and
Daniel Lee Fringer as void from the very beginning. As a necessary consequence of this pronouncement,
petitioner shall cease using the surname of respondent as she never acquired any right over it and so as to avoid
a misimpression that she remains the wife of respondent.

xxxx

SO ORDERED.6

The RTC was of the view that the parties married each other for convenience only. Giving credence to the
testimony of Albios, it stated that she contracted Fringer to enter into a marriage to enable her to acquire
American citizenship; that in consideration thereof, she agreed to pay him the sum of $2,000.00; that after the
ceremony, the parties went their separate ways; that Fringer returned to the United States and never again
communicated with her; and that, in turn, she did not pay him the $2,000.00 because he never processed her
petition for citizenship. The RTC, thus, ruled that when marriage was entered into for a purpose other than the
establishment of a conjugal and family life, such was a farce and should not be recognized from its inception.

Petitioner Republic of the Philippines, represented by the Office of the Solicitor General (OSG), filed a motion for
reconsideration. The RTC issued the Order, 7 dated February 5, 2009, denying the motion for want of merit. It
explained that the marriage was declared void because the parties failed to freely give their consent to the
marriage as they had no intention to be legally bound by it and used it only as a means to acquire American
citizenship in consideration of $2,000.00.

Not in conformity, the OSG filed an appeal before the CA.

Ruling of the CA

In its assailed decision, dated September 29, 2011, the CA affirmed the RTC ruling which found that the essential
requisite of consent was lacking. The CA stated that the parties clearly did not understand the nature and
consequence of getting married and that their case was similar to a marriage in jest. It further explained that the
parties never intended to enter into the marriage contract and never intended to live as husband and wife or build
a family. It concluded that their purpose was primarily for personal gain, that is, for Albios to obtain foreign
citizenship, and for Fringer, the consideration of $2,000.00.

Hence, this petition.

Assignment of Error

THE COURT OF APPEALS ERRED ON A QUESTION OF LAWWHEN IT HELD THAT A MARRIAGE


CONTRACTED FOR THEPURPOSE OF OBTAINING FOREIGN CITIZENSHIP WAS DONEIN JEST, HENCE,
LACKING IN THE ESSENTIAL ELEMENT OFCONSENT.8

The OSG argues that albeit the intention was for Albios to acquire American citizenship and for Fringer to be paid
$2,000.00, both parties freely gave their consent to the marriage, as they knowingly and willingly entered into that
marriage and knew the benefits and consequences of being bound by it. According to the OSG, consent should
be distinguished from motive, the latter being inconsequential to the validity of marriage.

The OSG also argues that the present case does not fall within the concept of a marriage in jest. The parties here
intentionally consented to enter into a real and valid marriage, for if it were otherwise, the purpose of Albios to
acquire American citizenship would be rendered futile.

On October 29, 2012, Albios filed her Comment9 to the petition, reiterating her stand that her marriage was
similar to a marriage by way of jest and, therefore, void from the beginning.

On March 22, 2013, the OSG filed its Reply10 reiterating its arguments in its petition for review on certiorari.

Ruling of the Court

The resolution of this case hinges on this sole question of law: Is a marriage, contracted for the sole purpose of
acquiring American citizenship in consideration of $2,000.00, void ab initio on the ground of lack of consent?

The Court resolves in the negative.

Before the Court delves into its ruling, It shall first examine the phenomenon of marriage fraud for the purposes of
immigration.

Marriage Fraud in Immigration

The institution of marriage carries with it concomitant benefits. This has led to the development of marriage fraud
for the sole purpose of availing of particular benefits. In the United States, marriages where a couple marries only
to achieve a particular purpose or acquire specific benefits, have been referred to as "limited purpose"
marriages.11 A common limited purpose marriage is one entered into solely for the legitimization of a
child.12 Another, which is the subject of the present case, is for immigration purposes. Immigration law is usually
concerned with the intention of the couple at the time of their marriage, 13 and it attempts to filter out those who
use marriage solely to achieve immigration status.14

In 1975, the seminal case of Bark v. Immigration and Naturalization Service, 15 established the principal test for
determining the presence of marriage fraud in immigration cases. It ruled that a "marriage is a sham if the bride
and groom did not intend to establish a life together at the time they were married. "This standard was modified
with the passage of the Immigration Marriage Fraud Amendment of 1986 (IMFA), which now requires the couple
to instead demonstrate that the marriage was not "entered into for the purpose of evading the immigration laws of
the United States." The focus, thus, shifted from determining the intention to establish a life together, to
determining the intention of evading immigration laws. 16 It must be noted, however, that this standard is used
purely for immigration purposes and, therefore, does not purport to rule on the legal validity or existence of a
marriage.
The question that then arises is whether a marriage declared as a sham or fraudulent for the limited purpose of
immigration is also legally void and in existent. The early cases on limited purpose marriages in the United States
made no definitive ruling. In 1946, the notable case of

United States v. Rubenstein17 was promulgated, wherein in order to allow an alien to stay in the country, the
parties had agreed to marry but not to live together and to obtain a divorce within six months. The Court, through
Judge Learned Hand, ruled that a marriage to convert temporary into permanent permission to stay in the country
was not a marriage, there being no consent, to wit:

x x x But, that aside, Spitz and Sandler were never married at all. Mutual consent is necessary to every contract;
and no matter what forms or ceremonies the parties may go through indicating the contrary, they do not contract
if they do not in fact assent, which may always be proved. x x x Marriage is no exception to this rule: a marriage
in jest is not a marriage at all. x x x It is quite true that a marriage without subsequent consummation will be valid;
but if the spouses agree to a marriage only for the sake of representing it as such to the outside world and with
the understanding that they will put an end to it as soon as it has served its purpose to deceive, they have never
really agreed to be married at all. They must assent to enter into the relation as it is ordinarily understood, and it
is not ordinarily understood as merely a pretence, or cover, to deceive others. 18

(Italics supplied)

On the other end of the spectrum is the 1969 case of Mpiliris v. Hellenic Lines, 19 which declared as valid a
marriage entered into solely for the husband to gain entry to the United States, stating that a valid marriage could
not be avoided "merely because the marriage was entered into for a limited purpose." 20 The 1980 immigration
case of Matter of McKee,21 further recognized that a fraudulent or sham marriage was intrinsically different from a
non subsisting one.

Nullifying these limited purpose marriages for lack of consent has, therefore, been recognized as problematic.
The problem being that in order to obtain an immigration benefit, a legal marriage is first necessary. 22 At present,
United States courts have generally denied annulments involving" limited purpose" marriages where a couple
married only to achieve a particular purpose, and have upheld such marriages as valid.23

The Court now turns to the case at hand.

Respondent’s marriage not void

In declaring the respondent’s marriage void, the RTC ruled that when a marriage was entered into for a purpose
other than the establishment of a conjugal and family life, such was a farce and should not be recognized from its
inception. In its resolution denying the OSG’s motion for reconsideration, the RTC went on to explain that the
marriage was declared void because the parties failed to freely give their consent to the marriage as they had no
intention to be legally bound by it and used it only as a means for the respondent to acquire American citizenship.
Agreeing with the RTC, the CA ruled that the essential requisite of consent was lacking. It held that the parties
clearly did not understand the nature and consequence of getting married. As in the Rubenstein case, the CA
found the marriage to be similar to a marriage in jest considering that the parties only entered into the marriage
for the acquisition of American citizenship in exchange of $2,000.00. They never intended to enter into a
marriage contract and never intended to live as husband and wife or build a family.

The CA’s assailed decision was, therefore, grounded on the parties’ supposed lack of consent. Under Article 2 of
the Family Code, consent is an essential requisite of marriage. Article 4 of the same Code provides that the
absence of any essential requisite shall render a marriage void ab initio.

Under said Article 2, for consent to be valid, it must be (1) freely given and (2) made in the presence of a
solemnizing officer. A "freely given" consent requires that the contracting parties willingly and deliberately enter
into the marriage. Consent must be real in the sense that it is not vitiated nor rendered defective by any of the
vices of consent under Articles45 and 46 of the Family Code, such as fraud, force, intimidation, and undue
influence.24 Consent must also be conscious or intelligent, in that the parties must be capable of intelligently
understanding the nature of, and both the beneficial or unfavorable consequences of their act.25 Their
understanding should not be affected by insanity, intoxication, drugs, or hypnotism. 26

Based on the above, consent was not lacking between Albios and Fringer. In fact, there was real consent
because it was not vitiated nor rendered defective by any vice of consent. Their consent was also conscious and
intelligent as they understood the nature and the beneficial and inconvenient consequences of their marriage, as
nothing impaired their ability to do so. That their consent was freely given is best evidenced by their conscious
purpose of acquiring American citizenship through marriage. Such plainly demonstrates that they willingly and
deliberately contracted the marriage. There was a clear intention to enter into a real and valid marriage so as to
fully comply with the requirements of an application for citizenship. There was a full and complete understanding
of the legal tie that would be created between them, since it was that precise legal tie which was necessary to
accomplish their goal.
In ruling that Albios’ marriage was void for lack of consent, the CA characterized such as akin to a marriage by
way of jest. A marriage in jest is a pretended marriage, legal in form but entered into as a joke, with no real
intention of entering into the actual marriage status, and with a clear understanding that the parties would not be
bound. The ceremony is not followed by any conduct indicating a purpose to enter into such a relation. 27 It is a
pretended marriage not intended to be real and with no intention to create any legal ties whatsoever, hence, the
absence of any genuine consent. Marriages in jest are void ab initio, not for vitiated, defective, or unintelligent
consent, but for a complete absence of consent. There is no genuine consent because the parties have
absolutely no intention of being bound in any way or for any purpose.

The respondent’s marriage is not at all analogous to a marriage in jest. Albios and Fringer had an undeniable
1âwphi1

intention to be bound in order to create the very bond necessary to allow the respondent to acquire American
citizenship. Only a genuine consent to be married would allow them to further their objective, considering that
only a valid marriage can properly support an application for citizenship. There was, thus, an apparent intention
to enter into the actual marriage status and to create a legal tie, albeit for a limited purpose. Genuine consent
was, therefore, clearly present.

The avowed purpose of marriage under Article 1 of the Family Code is for the couple to establish a conjugal and
family life. The possibility that the parties in a marriage might have no real intention to establish a life together is,
however, insufficient to nullify a marriage freely entered into in accordance with law. The same Article 1 provides
that the nature, consequences, and incidents of marriage are governed by law and not subject to stipulation. A
marriage may, thus, only be declared void or voidable under the grounds provided by law. There is no law that
declares a marriage void if it is entered into for purposes other than what the Constitution or law declares, such
as the acquisition of foreign citizenship. Therefore, so long as all the essential and formal requisites prescribed by
law are present, and it is not void or voidable under the grounds provided by law, it shall be declared valid. 28

Motives for entering into a marriage are varied and complex. The State does not and cannot dictate on the kind of
life that a couple chooses to lead. Any attempt to regulate their lifestyle would go into the realm of their right to
privacy and would raise serious constitutional questions. 29 The right to marital privacy allows married couples to
structure their marriages in almost any way they see fit, to live together or live apart, to have children or no
children, to love one another or not, and so on.30 Thus, marriages entered into for other purposes, limited or
otherwise, such as convenience, companionship, money, status, and title, provided that they comply with all the
legal requisites,31 are equally valid. Love, though the ideal consideration in a marriage contract, is not the only
valid cause for marriage. Other considerations, not precluded by law, may validly support a marriage.

Although the Court views with disdain the respondent’s attempt to utilize marriage for dishonest purposes, It
cannot declare the marriage void. Hence, though the respondent’s marriage may be considered a sham or
fraudulent for the purposes of immigration, it is not void ab initio and continues to be valid and subsisting.

Neither can their marriage be considered voidable on the ground of fraud under Article 45 (3) of the Family Code.
Only the circumstances listed under Article 46 of the same Code may constitute fraud, namely, (1) non-
disclosure of a previous conv1ctwn involving moral turpitude; (2) concealment by the wife of a pregnancy by
another man; (3) concealment of a sexually transmitted disease; and (4) concealment of drug addiction,
alcoholism, or homosexuality. No other misrepresentation or deceit shall constitute fraud as a ground for an
action to annul a marriage. Entering into a marriage for the sole purpose of evading immigration laws does not
qualify under any of the listed circumstances. Furthermore, under Article 47 (3), the ground of fraud may only be
brought by the injured or innocent party. In the present case, there is no injured party because Albios and Fringer
both conspired to enter into the sham marriage.

Albios has indeed made a mockery of the sacred institution of marriage. Allowing her marriage with Fringer to be
declared void would only further trivialize this inviolable institution. The Court cannot declare such a marriage
void in the event the parties fail to qualify for immigration benefits, after they have availed of its benefits, or simply
have no further use for it. These unscrupulous individuals cannot be allowed to use the courts as instruments in
their fraudulent schemes. Albios already misused a judicial institution to enter into a marriage of convenience;
she should not be allowed to again abuse it to get herself out of an inconvenient situation.

No less than our Constitution declares that marriage, as an in violable social institution, is the foundation of the
family and shall be protected by the State. 32 It must, therefore, be safeguarded from the whims and caprices of
the contracting parties. This Court cannot leave the impression that marriage may easily be entered into when it
suits the needs of the parties, and just as easily nullified when no longer needed.

WHEREFORE, the petition is GRANTED. The September 29, 2011 Decision of the Court of Appeals in CA-G.R.
CV No. 95414 is ANNULLED, and Civil Case No. 1134-06 is DISMISSED for utter lack of merit.

SO ORDERED.

JOSE CATRAL MENDOZA


Associate Justice
WE CONCUR:

PRESBITERO J. VELASCO, JR.


Associate Justice
Chairperson

TERESITA J. LEONARDO-DE CASTRO* ARTURO D. BRION**


Associate Justice Associate Justice

DIOSDADO M. PERALTA
Associate Justice

ATTESTATION

I attest that the conclusions in the above Decision had been reached in consultation before the case was
assigned to the writer of the opinion of the Court's Division.

PRESBITER J. VELASCO, JR.


Associate Justice
Chairperson, Third Division

CERTIFICATION

Pursuant to Section 13, Article VIII of the Constitution and the Division Chairperson's Attestation, I certify that the
conclusions in the above Decision had been reached in consultation before the case was assigned to the writer
of the opinion of the Court's Division.

MARIA LOURDES P. A. SERENO


Chief Justice

Footnotes

* Designated Acting Member in lieu of Associate Justice Marvic Mario Victor F. Leonen per Special Order
No. 1570 dated October 14. 2013.

** Designated Acting Member in lieu of Associate Justice Roberto A. Abad. Per Special Order No.
1554dated September 19, 2013.

1Rollo. pp. 26-32; penned by Associate Justice Juan Q. Enriquez. Jr. and concurred in by Associate
Justice Ramon M. Bato. Jr. and Associate Justice Fiorito S. Macalino of the Fifth Division. Manila.

2 Id. at 38-39.

3 Id. at 37.

4 Id. at 33-35.

5 Id. at 38-39.

6 Id. at 39.

7 Id. at 48-49.

8 Id. at 13.

9 Id. at 61-71.

10 Id. at 89-95.
11Abrams, Kerry. Marriage Fraud . 100 Cal. L. Rev. 1
(2012);http://papers.ssrn.com/sol3/papers.cfm?abstract_id=2000956. Lutwak v. United States , 344 U.S.
604, 612-613 (U.S. 1953).

12Abrams, Kerry. Marriage Fraud . 100 Cal. L. Rev. 1


(2012);http://papers.ssrn.com/sol3/papers.cfm?abstract_id=2000956; citing Schibi v. Schibi , 69 A.2d 831
(Conn. 1949) (denying annulment where parties married only to give a name to a prospective child);
Bishop v. Bishop , 308 N.Y.S.2d 998 (Sup. Ct. 1970); Erickson v. Erickson , 48 N.Y.S.2d 588 (Sup. Ct.
1944) (holding similarly to Schibi ); Delfino v.Delfino , 35 N.Y.S.2d 693 (Sup. Ct. 1942) (denying
annulment where purpose of marriage was to protect the girl’s name and there was an understanding that
the parties would not live together as man and wife); Bove v. Pinciotti , 46 Pa. D. & C. 159 (1942);
Campbell v. Moore , 189 S.E.2d 497 (S.C.1939) (refusing an annulment where parties entered marriage
for the purpose of legitimizing a child); Chander v. Chander , No.2937-98-4, 1999 WL 1129721 (Va. Ct.
App. June 22, 1999) (denying annulment where wife married husband to get his pension with no intention
to consummate marriage because husband knew that was the purpose of the marriage).

13Abrams, Kerry. Immigration Law and the Regulation of Marriage; 91 Minn. L. Rev. 1625
(2007);http://www.minnesotalawreview.org/wp-content/uploads/2012/01/Abrams_Final.pdf; citing
Immigration and Nationality Act (INA), § 237(a)(1)(G), 8 U.S.C. § 1227(a)(1)(G) (2000).

14Abrams, Kerry. Immigration Law and the Regulation of Marriage ; 91 Minn. L. Rev. 1625
(2007);http://www.minnesotalawreview.org/wp-content/uploads/2012/01/Abrams_Final.pdf; citing 132
CONG.REC. 27,012, 27,015 (1986) (statement of Rep Mc Collum) (promoting the Immigration Marriage
Fraud Amendments of 1986).

15 511 F.2d 1200, 1201 (9th Cir. 1975).

16Abrams, Kerry. Immigration Law and the Regulation of Marriage; 91 Minn. L. Rev. 1625
(2007);http://www.minnesotalawreview.org/wp-content/uploads/2012/01/Abrams_Final.pdf.

17 151 F.2d 915 (2d Cir. 1945).

18 United States v. Rubenstein , 151 F.2d 915 (2d Cir. 1945).

19 Mpiliris v. Hellenic Lines, Ltd. , 323 F. Supp. 865 (S.D. Tex. 1969), aff’d , 440 F.2d 1163 (5th Cir. 1971).

20 Abrams, Kerry. Marriage Fraud . 100 Cal. L. Rev. 1


(2012);http://papers.ssrn.com/sol3/papers.cfm?abstract_id=2000956; citing Mpiliris v. Hellenic Lines, Ltd.
, 323 F. Supp. 865 (S.D. Tex. 1969), aff’d, 440F.2d 1163 (5th Cir. 1971).

21 Matter of McKee, 17 I. & N. Dec. 332, 333 (B.I.A. 1980).

22Lynn D. Wardle and Laurence C. Nolan, Family Law in the USA, (The Netherlands: Kluwer Law
International, 2011) p. 86.

23Abrams, Kerry. Marriage Fraud. 100 Cal. L. Rev. 1


(2012);http://papers.ssrn.com/sol3/papers.cfm?abstract_id=2000956.

24Alicia V. Sempio-Diy, Handbook on the Family Code of the Philippines, (Quezon City, Philippines: Joer
Printing Services, 2005), p. 4.

Melencio S. Sta. Maria, Jr., Persons and Family Relations Law, (Quezon City, Philippines: Rex Printing
25

Company, Inc., 2010), Fifth Edition, p. 121.

Arturo M. Tolentino, Commentaries and Jurisprudence on the Civil Code of the Philippines, (Manila,
26

Philippines: Central Book Supply, Inc., 2004), Volume I, p. 231.

Arturo M. Tolentino, Commentaries and Jurisprudence on the Civil Code of the Philippines, (Manila,
27

Philippines: Central Book Supply, Inc., 2004), Volume I, p. 231; citing McClurg v. Terry, 21 N.J. 225.

28 Article 4, Family Code.

29 Bark v. Immigration & Naturalization Service, 511 F.2d 1200, 1201 (9th Cir. 1975).
30Abrams, Kerry. Immigration Law and the Regulation of Marriage; 91 Minn. L. Rev. 1625
(2007);http://www.minnesotalawreview.org/wp-content/uploads/2012/01/Abrams_Final.pdf; citing
McGuire v. McGuire , 59 N.W.2d 336, 337 (Neb. 1953). Griswold v. Connecticut, 381 U.S. 479, 485–86
(1965).

31 Article 4, Family Code.

32 Const. ( 1987), Article XV, Section 2.


Republic of the Philippines
SUPREME COURT
Manila

EN BANC

G.R. No. L-20089 December 26, 1964

BEATRIZ P. WASSMER, plaintiff-appellee,


vs.
FRANCISCO X. VELEZ, defendant-appellant.

Jalandoni & Jamir for defendant-appellant.


Samson S. Alcantara for plaintiff-appellee.

BENGZON, J.P., J.:

The facts that culminated in this case started with dreams and hopes, followed by appropriate planning and
serious endeavors, but terminated in frustration and, what is worse, complete public humiliation.

Francisco X. Velez and Beatriz P. Wassmer, following their mutual promise of love, decided to get married and
set September 4, 1954 as the big day. On September 2, 1954 Velez left this note for his bride-to-be:

Dear Bet —

Will have to postpone wedding — My mother opposes it. Am leaving on the Convair today.

Please do not ask too many people about the reason why — That would only create a scandal.

Paquing

But the next day, September 3, he sent her the following telegram:

NOTHING CHANGED REST ASSURED RETURNING VERY SOON APOLOGIZE MAMA PAPA
LOVE .

PAKING

Thereafter Velez did not appear nor was he heard from again.

Sued by Beatriz for damages, Velez filed no answer and was declared in default. Plaintiff adduced evidence
before the clerk of court as commissioner, and on April 29, 1955, judgment was rendered ordering defendant to
pay plaintiff P2,000.00 as actual damages; P25,000.00 as moral and exemplary damages; P2,500.00 as
attorney's fees; and the costs.

On June 21, 1955 defendant filed a "petition for relief from orders, judgment and proceedings and motion for new
trial and reconsideration." Plaintiff moved to strike it cut. But the court, on August 2, 1955, ordered the parties and
their attorneys to appear before it on August 23, 1955 "to explore at this stage of the proceedings the possibility
of arriving at an amicable settlement." It added that should any of them fail to appear "the petition for relief and
the opposition thereto will be deemed submitted for resolution."

On August 23, 1955 defendant failed to appear before court. Instead, on the following day his counsel filed a
motion to defer for two weeks the resolution on defendants petition for relief. The counsel stated that he would
confer with defendant in Cagayan de Oro City — the latter's residence — on the possibility of an amicable
element. The court granted two weeks counted from August 25, 1955.

Plaintiff manifested on June 15, 1956 that the two weeks given by the court had expired on September 8, 1955
but that defendant and his counsel had failed to appear.

Another chance for amicable settlement was given by the court in its order of July 6, 1956 calling the parties and
their attorneys to appear on July 13, 1956. This time. however, defendant's counsel informed the court that
chances of settling the case amicably were nil.

On July 20, 1956 the court issued an order denying defendant's aforesaid petition. Defendant has appealed to
this Court. In his petition of June 21, 1955 in the court a quo defendant alleged excusable negligence as ground
to set aside the judgment by default. Specifically, it was stated that defendant filed no answer in the belief that an
amicable settlement was being negotiated.

A petition for relief from judgment on grounds of fraud, accident, mistake or excusable negligence, must be duly
supported by an affidavit of merits stating facts constituting a valid defense. (Sec. 3, Rule 38, Rules of Court.)
Defendant's affidavit of merits attached to his petition of June 21, 1955 stated: "That he has a good and valid
defense against plaintiff's cause of action, his failure to marry the plaintiff as scheduled having been due to
fortuitous event and/or circumstances beyond his control." An affidavit of merits like this stating mere conclusions
or opinions instead of facts is not valid. (Cortes vs. Co Bun Kim, L-3926, Oct. 10, 1951; Vaswani vs. P.
Tarrachand Bros., L-15800, December 29, 1960.)

Defendant, however, would contend that the affidavit of merits was in fact unnecessary, or a mere surplusage,
because the judgment sought to be set aside was null and void, it having been based on evidence adduced
before the clerk of court. In Province of Pangasinan vs. Palisoc, L-16519, October 30, 1962, this Court pointed
out that the procedure of designating the clerk of court as commissioner to receive evidence is sanctioned by
Rule 34 (now Rule 33) of the Rules of Court. Now as to defendant's consent to said procedure, the same did not
have to be obtained for he was declared in default and thus had no standing in court (Velez vs. Ramas, 40 Phil.
787; Alano vs. Court of First Instance, L-14557, October 30, 1959).

In support of his "motion for new trial and reconsideration," defendant asserts that the judgment is contrary to
law. The reason given is that "there is no provision of the Civil Code authorizing" an action for breach of promise
to marry. Indeed, our ruling in Hermosisima vs. Court of Appeals (L-14628, Sept. 30, 1960), as reiterated
in Estopa vs. Biansay (L-14733, Sept. 30, 1960), is that "mere breach of a promise to marry" is not an actionable
wrong. We pointed out that Congress deliberately eliminated from the draft of the new Civil Code the provisions
that would have it so.

It must not be overlooked, however, that the extent to which acts not contrary to law may be perpetrated with
impunity, is not limitless for Article 21 of said Code provides that "any person who wilfully causes loss or injury to
another in a manner that is contrary to morals, good customs or public policy shall compensate the latter for the
damage."

The record reveals that on August 23, 1954 plaintiff and defendant applied for a license to contract marriage,
which was subsequently issued (Exhs. A, A-1). Their wedding was set for September 4, 1954. Invitations were
printed and distributed to relatives, friends and acquaintances (Tsn., 5; Exh. C). The bride-to-be's trousseau,
party drsrses and other apparel for the important occasion were purchased (Tsn., 7-8). Dresses for the maid of
honor and the flower girl were prepared. A matrimonial bed, with accessories, was bought. Bridal showers were
given and gifts received (Tsn., 6; Exh. E). And then, with but two days before the wedding, defendant, who was
then 28 years old,: simply left a note for plaintiff stating: "Will have to postpone wedding — My mother opposes it
... " He enplaned to his home city in Mindanao, and the next day, the day before the wedding, he wired plaintiff:
"Nothing changed rest assured returning soon." But he never returned and was never heard from again.

Surely this is not a case of mere breach of promise to marry. As stated, mere breach of promise to marry is not
an actionable wrong. But to formally set a wedding and go through all the above-described preparation and
publicity, only to walk out of it when the matrimony is about to be solemnized, is quite different. This is palpably
and unjustifiably contrary to good customs for which defendant must be held answerable in damages in
accordance with Article 21 aforesaid.

Defendant urges in his afore-stated petition that the damages awarded were excessive. No question is raised as
to the award of actual damages. What defendant would really assert hereunder is that the award of moral and
exemplary damages, in the amount of P25,000.00, should be totally eliminated.

Per express provision of Article 2219 (10) of the New Civil Code, moral damages are recoverable in the cases
mentioned in Article 21 of said Code. As to exemplary damages, defendant contends that the same could not be
adjudged against him because under Article 2232 of the New Civil Code the condition precedent is that "the
defendant acted in a wanton, fraudulent, reckless, oppressive, or malevolent manner." The argument is devoid of
merit as under the above-narrated circumstances of this case defendant clearly acted in a "wanton ... , reckless
[and] oppressive manner." This Court's opinion, however, is that considering the particular circumstances of this
case, P15,000.00 as moral and exemplary damages is deemed to be a reasonable award.

PREMISES CONSIDERED, with the above-indicated modification, the lower court's judgment is hereby affirmed,
with costs.

Bengzon, C.J., Bautista Angelo, Reyes, J.B.L., Barrera, Paredes, Dizon, Regala, Makalintal, and Zaldivar,
JJ., concur.
ARTICLE 7
Republic of the Philippines
SUPREME COURT
Manila

SECOND DIVISION

A.M. No. MTJ-96-1088 July 19, 1996

RODOLFO G. NAVARRO, complainant,

vs.

JUDGE HERNANDO C. DOMAGTOY, respondent.

ROMERO, J.:p

The complainant in this administrative case is the Municipal Mayor of Dapa, Surigao del Norte, Rodolfo G. Navarro. He has submitted evidence in relation to
two specific acts committed by respondent Municipal Circuit Trial Court Judge Hernando Domagtoy, which, he contends, exhibits gross misconduct as well as
inefficiency in office and ignorance of the law.

First, on September 27, 1994, respondent judge solemnized the wedding between Gaspar A. Tagadan and Arlyn
F. Borga, despite the knowledge that the groom is merely separated from his first wife.

Second, it is alleged that he performed a marriage ceremony between Floriano Dador Sumaylo and Gemma G.
del Rosario outside his court's jurisdiction on October 27, 1994. Respondent judge holds office and has
jurisdiction in the Municipal Circuit Trial Court of Sta. Monica-Burgos, Surigao del Norte. The wedding was
solemnized at the respondent judge's residence in the municipality of Dapa, which does not fall within his
jurisdictional area of the municipalities of Sta. Monica and Burgos, located some 40 to 45 kilometers away from
the municipality of Dapa, Surigao del Norte.

In his letter-comment to the office of the Court Administrator, respondent judge avers that the office and name of
the Municipal Mayor of Dapa have been used by someone else, who, as the mayor's "lackey," is overly
concerned with his actuations both as judge and as a private person. The same person had earlier filed
Administrative Matter No 94-980-MTC, which was dismissed for lack of merit on September 15, 1994, and
Administrative Matter No. OCA-IPI-95-16, "Antonio Adapon v. Judge Hernando C. Domagtoy," which is still
pending.

In relation to the charges against him, respondent judge seeks exculpation from his act of having solemnized the
marriage between Gaspar Tagadan, a married man separated from his wife, and Arlyn F. Borga by stating that
he merely relied on the Affidavit issued by the Municipal Trial Judge of Basey, Samar, confirming the fact that Mr.
Tagadan and his first wife have not seen each other for almost seven years. 1 With respect to the second charge,
he maintains that in solemnizing the marriage between Sumaylo and del Rosario, he did not violate Article 7,
paragraph 1 of the Family Code which states that: "Marriage may be solemnized by: (1) Any incumbent member
of the judiciary within the court's jurisdiction;" and that article 8 thereof applies to the case in question.

The complaint was not referred, as is usual, for investigation, since the pleadings submitted were considered
sufficient for a resolution of the case. 2

Since the countercharges of sinister motives and fraud on the part of complainant have not been sufficiently
proven, they will not be dwelt upon. The acts complained of and respondent judge's answer thereto will suffice
and can be objectively assessed by themselves to prove the latter's malfeasance.

The certified true copy of the marriage contract between Gaspar Tagadan and Arlyn Borga states that Tagadan's
civil status is "separated." Despite this declaration, the wedding ceremony was solemnized by respondent judge.
He presented in evidence a joint affidavit by Maurecio A. Labado, Sr. and Eugenio Bullecer, subscribed and
sworn to before Judge Demosthenes C. Duquilla, Municipal Trial Judge of Basey, Samar. 3 The affidavit was not
issued by the latter judge, as claimed by respondent judge, but merely acknowledged before him. In their
affidavit, the affiants stated that they knew Gaspar Tagadan to have been civilly married to Ida D. Peñaranda in
September 1983; that after thirteen years of cohabitation and having borne five children, Ida Peñaranda left the
conjugal dwelling in Valencia, Bukidnon and that she has not returned nor been heard of for almost seven years,
thereby giving rise to the presumption that she is already dead.

In effect, Judge Domagtoy maintains that the aforementioned joint affidavit is sufficient proof of Ida Peñaranda's
presumptive death, and ample reason for him to proceed with the marriage ceremony. We do not agree.

Article 41 of the Family Code expressly provides:

A marriage contracted by any person during the subsistence of a previous marriage shall be null
and void, unless before the celebration of the subsequent marriage, the prior spouse had been
absent for four consecutive years and the spouse present had a well-founded belief that the
absent spouse was already dead. In case of disappearance where there is danger of death under
the circumstances set forth in the provisions of Articles 391 of the Civil Code, an absence of only
two years shall be sufficient.

For the purpose of contracting the subsequent marriage under the preceding paragraph, the
spouse present must institute a summary proceeding as provided in this Code for the declaration
of presumptive death of the absentee, without prejudice to the effect of reappearance of the
absent spouse. (Emphasis added.)

There is nothing ambiguous or difficult to comprehend in this provision. In fact, the law is clear and simple. Even
if the spouse present has a well-founded belief that the absent spouse was already dead, a summary proceeding
for the declaration of presumptive death is necessary in order to contract a subsequent marriage, a mandatory
requirement which has been precisely incorporated into the Family Code to discourage subsequent marriages
where it is not proven that the previous marriage has been dissolved or a missing spouse is factually or
presumptively dead, in accordance with pertinent provisions of law.

In the case at bar, Gaspar Tagadan did not institute a summary proceeding for the declaration of his first wife's
presumptive death. Absent this judicial declaration, he remains married to Ida Peñaranda. Whether wittingly or
unwittingly, it was manifest error on the part of respondent judge to have accepted the joint affidavit submitted by
the groom. Such neglect or ignorance of the law has resulted in a bigamous, and therefore void, marriage. Under
Article 35 of the Family Code, " The following marriage shall be void from the beginning: (4) Those bigamous . . .
marriages not falling under Article 41."

The second issue involves the solemnization of a marriage ceremony outside the court's jurisdiction, covered by
Articles 7 and 8 of the Family Code, thus:

Art. 7. Marriage may be solemnized by :

(1) Any incumbent member of the judiciary within the court's jurisdiction;

xxx xxx xxx (Emphasis supplied.)

Art. 8. The marriage shall be solemnized publicly in the chambers the judge or in open court, in
the church, chapel or temple, or in the office of the consul-general, consul or vice-consul, as the
case may be, and not elsewhere, except in cases of marriages contracted on the point of death or
in remote places in accordance with Article 29 of this Code, or where both parties request the
solemnizing officer in writing in which case the marriage may be solemnized at a house or place
designated by them in a sworn statement to that effect.

Respondent judge points to Article 8 and its exceptions as the justification for his having solemnized the marriage
between Floriano Sumaylo and Gemma del Rosario outside of his court's jurisdiction. As the aforequoted
provision states, a marriage can be held outside of the judge's chambers or courtroom only in the following
instances: (1) at the point of death, (2) in remote places in accordance with Article 29 or (3) upon request of both
parties in writing in a sworn statement to this effect. There is no pretense that either Sumaylo or del Rosario was
at the point of death or in the remote place. Moreover, the written request presented addressed to the respondent
judge was made by only one party, Gemma del Rosario. 4

More importantly, the elementary principle underlying this provision is the authority of the solemnizing judge.
Under Article 3, one of the formal requisites of marriage is the "authority of the solemnizing officer." Under Article
7, marriage may be solemnized by, among others, "any incumbent member of the judiciary within the court's
jurisdiction." Article 8, which is a directory provision, refers only to the venue of the marriage ceremony and does
not alter or qualify the authority of the solemnizing officer as provided in the preceding provision. Non-compliance
herewith will not invalidate the marriage.
A priest who is commissioned and allowed by his local ordinary to marry the faithful, is authorized to do so only
within the area of the diocese or place allowed by his Bishop. An appellate court Justice or a Justice of this Court
has jurisdiction over the entire Philippines to solemnize marriages, regardless of the venue, as long as the
requisites of the law are complied with. However, judges who are appointed to specific jurisdictions, may officiate
in weddings only within said areas and not beyond. Where a judge solemnizes a marriage outside his court's
jurisdiction, there is a resultant irregularity in the formal requisite laid down in Article 3, which while it may not
affect the validity of the marriage, may subject the officiating official to administrative liability. 5

Inasmuch as respondent judge's jurisdiction covers the municipalities of Sta. Monica and Burgos, he was not
clothed with authority to solemnize a marriage in the municipality of Dapa, Surigao del Norte. By citing Article 8
and the exceptions therein as grounds for the exercise of his misplaced authority, respondent judge again
demonstrated a lack of understanding of the basic principles of civil law.

Accordingly, the Court finds respondent to have acted in gross ignorance of the law. The legal principles
applicable in the cases brought to our attention are elementary and uncomplicated, prompting us to conclude that
respondent's failure to apply them is due to a lack of comprehension of the law.

The judiciary should be composed of persons who, if not experts, are at least, proficient in the law they are sworn
to apply, more than the ordinary laymen. They should be skilled and competent in understanding and applying
the law. It is imperative that they be conversant with basic legal principles like the ones involved in instant
case. 6 It is not too much to expect them to know and apply the law intelligently. 7 Otherwise, the system of justice
rests on a shaky foundation indeed, compounded by the errors committed by those not learned in the law. While
magistrates may at times make mistakes in judgment, for which they are not penalized, the respondent judge
exhibited ignorance of elementary provisions of law, in an area which has greatly prejudiced the status of married
persons.

The marriage between Gaspar Tagadan and Arlyn Borga is considered bigamous and void, there being a
subsisting marriage between Gaspar Tagadan and Ida Peñaranda.

The Office of the Court Administrator recommends, in its Memorandum to the Court, a six-month suspension and
a stern warning that a repetition of the same or similar acts will be dealt with more severely. Considering that one
of the marriages in question resulted in a bigamous union and therefore void, and the other lacked the necessary
authority of respondent judge, the Court adopts said recommendation. Respondent is advised to be more
circumspect in applying the law and to cultivate a deeper understanding of the law.

IN VIEW OF THE FOREGOING, respondent Judge Hernando C. Domagtoy is hereby SUSPENDED for a period
of six (6) months and given a STERN WARNING that a repetition of the same or similar acts will be dealt with
more severely.

Regalado, Puno, Mendoza and Torres, Jr., JJ., concur.

Footnotes

1 Rollo, pp. 7-8.

2 Uy v. Dizon-Capulong, A.M. No. RTJ-91-766, April 7, 1993; Montemayor v. Collado, A.M. No. 2519-
MTJ, September 10, 1981; Ubongon v. Mayo, A.M. No. 1255-CTJ, August 6, 1980, 99 SCRA 30.

3 Rollo, p. 12.

4 Rollo, pp. 10-11.

5 Article 4, Family Code.

6 Lim v. Domogas, A.M. No. RTJ-92-899, October 15, 1993, 227 SCRA 258, 263 citing Ubongan v.
Mayor, 99 SCRA 30 and Ajeno v. Inserto, 71 SCRA 166.

7 . . . . Realty Co. v. Arranz, A.M. No. MTJ-93-978 October 27, 1994, 237 SCRA 771.
Republic of the Philippines
SUPREME COURT
Manila

EN BANC

G.R. No. L-11598 January 27, 1959

THE PEOPLE OF THE PHILIPPINES, plaintiff-appelee,


vs.
FEDERIC BUSTAMANTE, defendant-appellant.

Assistant Solicitor General Antonio A. Torres and Atty. Eduardo C. Abaya for appellee.
Ramon S. Milo for appellant.

REYES, J.B.L., J.:

Charged and convicted of the crime of bigamy in the Court of First Instance of Pangasinan, Federico Bustamante
appealed to this Court on points of law.

The records disclose that defendant-appellant Bustamante was united in wedlock to one Maria Perez on August
9, 1954, before the Justice of the Peace of Binalonan, Pangasinan (Exh. "A", pp. 9-11, t.s.n.). A little over a year
later, or on September 16, 1955, he contracted a second marriage with Demetria Tibayan, solemnized before
Vice-Mayor Francisco B. Nato of Mapandan, Pangasinan, who was then acting as Mayor of the said Municipality
(Exh. "B"), while the first marriage was still subsisting. Defendant dwelt with Demetria and her parents for about a
month, after which a time he returned to Calasiao, Pangasinan to live with the first wife, Maria Perez. In the
course of her search for him, Demetria discovered from the Binalonan municipal authorities the previous
marriage of defendant Bustamante. Hence, this accusation.

Defendant did not testify in his behalf during the trial. the main problem poised in this appeal concerns the
authority of Francisco Nato to solemnize the second marriage.

It appears that Enrique Aquino and Francisco Nato were the duly elected mayor and vice-mayor, respectively, of
the municipality of Mapandan, Pangasinan in the elections of 1951. On September 16, 1955, Aquino went on
leave of absence for one month. In view of this, the vice-mayor was designed by the mayor to take over the rein
of municipal government during his absence; and Nato was acting in this capacity when he performed the second
marriage of Bustamante with Demetria Tibayan.

Appellant, relying upon article 56 of the Civil Code of the Philippines —

Art. 56. Marriage may be solemnized by:

(1) The Chief Justice and Associate Justices of the Supreme Court;

(2) The Presiding Justice and the Justice of the Court of Appeals;

(3) Judges of the Courts of First Instance;

(4) Mayors of cities and municipalities;

(5) Municipal judges and justices of the Peace;

(6) Priests, rabbis, ministers of the gospel of any denominations, church, religion or sect, duly registered,
as provided in article 92; and

(7) Ship captains, airplanes chiefs, military commanders, and consuls and vice-consuls in special cases
provided in provided in articles 74 and 75.

contends that there could not have been a second marriage to speak of, as Nato was merely acting as mayor
when he celebrated the same, hence, without authority of law to do so. He lays stress on the distinction made by
this court in the case Salaysay vs. Hon. Fred Ruiz Castro, et al. * 52 Off. Gaz., No. 2, 809, between "Acting
Mayor" and "Vice-Mayor acting as Mayor", urging that while the former may solemnize marriages, the latter could
not.
We find this connection untenable. When the issue involves the assumption of powers and duties of the office of
the mayor by the vice-mayor, when proper, it is immaterial whether it because the latter is the Acting Mayor or
merely acting as Mayor, for in both instances, he discharges all the duties and wields the power appurtenant to
said office (Laxamana vs. Baltazar,1 48 Off. Gaz., No. 9, 3869; Sec. 2195, Revised Administrative Code). The
case of Salaysay vs. Castro (supra) cited by the appellant, which revolves upon the interpretation of section 27 of
the Revised Election Code, is entirely distinct from the one at bar. This instance does not involve a question of
title to the office, but the performance of the functions thereunto appertaining by one who is admitted to be
temporarily vested with it. As correctly observed by the lower court, that case even concedes and recognizes the
powers and duties of the Mayor to devolve upon the Vice-Mayor whenever the latter is in an acting capacity. The
word "acting" as held in the case of Austria vs. Amante,2 45 Off. Gaz., 2809, when preceding the title of an office
connotes merely the temporary character or nature of the same.

The information charges that the appellant contracted the second marriage before the Justice of the Peace of
Mapandan, Pangasinan, while the marriage certificate, Exh. "B", and the testimonies of witnesses indicate clearly
that it was performed by Francisco Nato. Appellant assigns as error the admission by the lower court of the said
evidence, notwithstanding counsel's objection. This is not reversible error. The wrong averment, if at all, was
unsubstantial and immaterial that need not even be alleged, for it matters not who solemnized the marriage, it
being sufficient that the information charging bigamy alleges that a second marriage was contracted while the
first still remained undissolved. The information filed in this case which properly states the time and place of the
second wedding, was sufficient to apprise the defendant of the crime imputed. Neither procedural prejudice nor
error was committed by the lower court in finding appellant guilty.

Article 349 of the Revised Penal Code attaches the penalty of prision mayor to the crime of bigamy. Pursuant to
the Indeterminate Sentence Law, the court must impose an indeterminate penalty, the maximum term of which
shall be that which, in view of the attending circumstances, could be properly imposed under the Code (in this
case the medium period of prision mayor, there being no aggravating nor mitigating circumstances), and the
minimum which shall be within the range of the penalty next lower to that prescribed for the offense (or prision
correccional medium) (People vs. Gonzales, 73 Phil., 549).

The penalty imposed by the lower court (imprisonment for not less than two (2) years, four (4) months and one
(1) day of prision correccional and not more than eight (8) years and one (1) day of prision mayor), being in
accordance with law, is affirmed. Costs against appellant.

So ordered.

Paras, C.J., Bengzon, Padilla, Montemayor, Bautista Angelo, Labrador, Concepcion and Endencia, JJ., concur.

Footnotes

* 98 Phil., 364.

1 92 Phil., 32.

2 79 Phil., 780.
ARTICLE 9
Republic of the Philippines
SUPREME COURT
Manila

EN BANC

G.R. No. L-31763 December 27, 1929

THE PEOPLE OF THE PHILIPPINE ISLANDS, plaintiff-appellee,


vs.
H. JANSSEN, defendant-appellant.

W. E. Greenbaum and Luis G. Hofileña for appellant.


Attorney-General Jaranilla for appellee.

VILLA-REAL, J.:

H. Janssen appeals to this court from the judgment of the Court of First Instance of Antique convicting him of a
violation of section 2 of Act No. 3412, and sentencing him to pay a fine of P200, with subsidiary imprisonment in
case of insolvency at the rate of one day for every 12 ½ pesetas, and to pay the court costs.

In support of his appeal, the appellant assigns the following alleged errors as committed by the court a quo in its
decision, to wit:

The trial court erred:

1. In holding that it is the duty of the accused to inquire into and determine the residence of the bride
before solemnizing marriage.

2. In finding that the habitual residence of the bride, Juana S. del Rosario is the municipality of Banga,
Province of Capiz, and not the municipality of San Jose, Province of Antique.

3. In holding that the accused cannot solemnize marriage without publishing or proclaiming such
marriage 10 days prior to the celebration thereof.

4. In holding that the accused has violated section 2 of Act No. 3412.

5. In convicting the accused.

The following facts were proved at the trial beyond a reasonable doubt.

On December 26, 1928, Pedro N. Cerdeña and Juana S. del Rosario appeared before Reverend Father H.
Janssen, a Catholic parish priest of the municipality of San Jose, Antique, to have their names inscribed in the
marriage registry, Exhibit 3, which was done. On December 30, 1928, the banns were published in his parish in
San Jose, Antique.

As the classes opened on January 7, 1929, the contracting parties asked the defendant-appellant to marry them
before that date. Upon petition of the defendant-appellant, the Bishop of Jaro issued the following dispensation
on December 29, 1928:

In view of the exposition and petition contained in the foregoing document, and with the understanding
that no obstacle has been discovered in the investigation made or to be made of the status and liberty of
the contracting parties, Pedro Cerdeña, single, of age, a resident of San Jose, Antique, and Juana S. del
Rosario, a native of Banga, Province of Capiz, residing in San Jose, Antique, single, of age; dispensation
is granted from one call of the banns, as prayed for, subject to alms --------pesos, to be applied to
charitable work and the expenses of divine worship, enjoining the Reverend Parish Priest of San Jose,
Province of Antique, to whom a copy of this decree shall be transmitted, not to solemnize the marriage
under consideration, without being certain of the status and liberty of both contracting parties, and that in
the realization of said act, no complaint of any kind shall be made on any legal ground; and that
otherwise, it is our will that the dispensation be granted.
Causes: Urgent business of both parties, who being Government employees, cannot await the last call
without serious prejudice.

On the 1st of January, 1929, another proclamation was made to that effect.

On January 4, 1929, the municipal secretary of San Jose, Antique, issued the following authority to solemnize
marriage:

To all those authorized to celebrate marriage:

You are hereby authorized to solemnize the marriage of Pedro N. Cerdeña and Juana S. del Rosario, in
accordance with the rites and ceremonies of your Church, sect, or religion, and with the laws of the
Philippine Islands. lawphi1.net

Given this day, January 4, 1929, in the municipality of San Jose, Antique, P. I.

Attached hereto is a copy of the petition filed by the contracting parties.

By virtue of the above-quoted dispensation, and in view of said authority of the municipal secretary of San Jose,
Antique, the defendant-appellant on January 6, 1929, solemnized the marriage of Pedro N. Cerdeña to Juana S.
del Rosario.

The only question to be decided in this appeal is whether or not the defendant-appellant violated section 2 of Act
No. 3412, the pertinent part of which reads as follows:

The municipal secretary or clerk of the municipal court of Manila, as the case may be, shall post during
ten days in a conspicuous place in the building where he has his office, a notice setting forth the full
names and domiciles of the applicants for marriage licenses, their respective ages, and the names of
their parents if living or of their guardians if otherwise. At the expiration of this term, a license shall
issue: Provided, however, That in case any such applicant states in writing and under oath that the rules
and practices of the church, sect, or religion under which such applicant desires to contract marriage
require banns or publications prior to the solemnization of the marriage, it shall not be necessary for the
municipal secretary to make the publication required in this paragraph, and in this case the license shall
issue immediately after the filing of the application and shall state the church, sect, or religion in which the
marriage is to be solemnized.

While it is true that section 2 of Act No. 3412 quoted above, requires the municipal secretary to post a notice for
ten days upon a conspiciuous place of the building where he has his office, setting forth the names, surnames,
and residence of applicants for a license to contract marriage, their age, the names of their parents, if alive, or of
their guardians, as the case may be, before issuing the license applied for, the same section contains a proviso
to the effect that when the contracting parties desire to marry in a church which requires previous proclamation
before the celebration of the marriage, there is no need of said publication. The only doubt is whether said
proclamation must be made during ten days, as in the publication in case the marriage is not celebrated in a
church. The law simply says that if the marriage takes place in a church whose rules and practices require
proclamation, the license applied for shall at once be issued, and it does not say that the proclamation required
by said church is to be made during ten days. As section 2 of Act No. 3412 is penal in character, it should be
strictly construed. And as said section does not require that the proclamation be made during ten days, but that it
is sufficient that the church in which the marriage is to take place requires a proclamation, it is immaterial how
many days said proclamation is made in.

For the foregoing considerations, we are of opinion and so hold that the defendant-appellant did not violate
section 2 of Act No. 3412 in solemnizing the marriage of Pedro N. Cerdeña and Juana S. del Rosario after two
proclamations, before ten days were up, the third proclamation having been dispensed with by a competent
ecclesiastical authority.
lawphi1.net

The trial court was also of opinion that the defendant-appellant was bound to investigate whether the license was
issued by an official duly authorized by law, that is, by the municipal secretary of the municipality where the
woman habitually resides.

The law does not impose this duty upon priest or ministers of religion. It is sufficient to know that the license has
been issued by a competent official, and it may be presumed from the issuance of said license that said official
has complied with his duty of ascertaining whether the woman who desires to get married resides habitually in his
municipality. (Act N. 190, sec. 334, No. 14.)
Wherefore, we are of opinion and so hold, that when a marriage is solemnized by a church, sect, or religion
whose rules and practices require proclamation or publicity, it is not necessary that said proclamation be made
during ten days, unless said rules or practices so require.

By virtue whereof, the appealed judgment is reversed, and the defendant is absolved from the information, with
costs de oficio. So ordered.

Avanceña, C.J., Street, Malcolm, Ostrand and Johns, JJ., concur.


ARTICLE 21
Garcia vs. Recio GR No. 138322

THIRD DIVISION

G.R. No. 138322 October 2, 2001

GRACE J. GARCIA, a.k.a. GRACE J. GARCIA-RECIO, petitioner,


vs.
REDERICK A. RECIO, respondents.

PANGANIBAN, J.:

A divorce obtained abroad by an alien may be recognized in our jurisdiction, provided such decree is valid
according to the national law of the foreigner. However, the divorce decree and the governing personal law of the
alien spouse who obtained the divorce must be proven. Our courts do not take judicial notice of foreign laws and
judgment; hence, like any other facts, both the divorce decree and the national law of the alien must be alleged
and proven according to our law on evidence.

The Case

Before us is a Petition for Review under Rule 45 of the Rules of Court, seeking to nullify the January 7, 1999
Decision1 and the March 24, 1999 Order2 of the Regional Trial Court of Cabanatuan City, Branch 28, in Civil
Case No. 3026-AF. The assailed Decision disposed as follows:

"WHEREFORE, this Court declares the marriage between Grace J. Garcia and Rederick A. Recio
solemnized on January 12, 1994 at Cabanatuan City as dissolved and both parties can now remarry
under existing and applicable laws to any and/or both parties." 3

The assailed Order denied reconsideration of the above-quoted Decision.

The Facts

Rederick A. Recio, a Filipino, was married to Editha Samson, an Australian citizen, in Malabon, Rizal, on March
1, 1987.4 They lived together as husband and wife in Australia. On May 18, 1989, 5 a decree of divorce,
purportedly dissolving the marriage, was issued by an Australian family court.

On June 26, 1992, respondent became an Australian citizen, as shown by a "Certificate of Australian Citizenship"
issued by the Australian government. 6 Petitioner – a Filipina – and respondent were married on January 12, 1994
in Our Lady of Perpetual Help Church in Cabanatuan City.7 In their application for a marriage license, respondent
was declared as "single" and "Filipino."8

Starting October 22, 1995, petitioner and respondent lived separately without prior judicial dissolution of their
marriage. While the two were still in Australia, their conjugal assets were divided on May 16, 1996, in accordance
with their Statutory Declarations secured in Australia. 9

On March 3, 1998, petitioner filed a Complaint for Declaration of Nullity of Marriage 10 in the court a quo, on the
ground of bigamy – respondent allegedly had a prior subsisting marriage at the time he married her on January
12, 1994. She claimed that she learned of respondent's marriage to Editha Samson only in November, 1997.

In his Answer, respondent averred that, as far back as 1993, he had revealed to petitioner his prior
marriage and its subsequent dissolution.11 He contended that his first marriage to an Australian citizen had been
validly dissolved by a divorce decree obtained in Australian in 1989; 12 thus, he was legally capacitated to marry
petitioner in 1994.
1âwphi1.nêt

On July 7, 1998 – or about five years after the couple's wedding and while the suit for the declaration of nullity
was pending – respondent was able to secure a divorce decree from a family court in Sydney, Australia because
the "marriage ha[d] irretrievably broken down." 13

Respondent prayed in his Answer that the Complained be dismissed on the ground that it stated no cause of
action.14 The Office of the Solicitor General agreed with respondent. 15 The court marked and admitted the
documentary evidence of both parties. 16 After they submitted their respective memoranda, the case was
submitted for resolution.17

Thereafter, the trial court rendered the assailed Decision and Order.

Ruling of the Trial Court

The trial court declared the marriage dissolved on the ground that the divorce issued in Australia was valid and
recognized in the Philippines. It deemed the marriage ended, but not on the basis of any defect in an essential
element of the marriage; that is, respondent's alleged lack of legal capacity to remarry. Rather, it based its
Decision on the divorce decree obtained by respondent. The Australian divorce had ended the marriage; thus,
there was no more martial union to nullify or annual.

Hence, this Petition.18

Issues

Petitioner submits the following issues for our consideration:

"I

The trial court gravely erred in finding that the divorce decree obtained in Australia by the respondent ipso
facto terminated his first marriage to Editha Samson thereby capacitating him to contract a second
marriage with the petitioner.

"2

The failure of the respondent, who is now a naturalized Australian, to present a certificate of legal
capacity to marry constitutes absence of a substantial requisite voiding the petitioner' marriage to the
respondent.

"3

The trial court seriously erred in the application of Art. 26 of the Family Code in this case.

"4

The trial court patently and grievously erred in disregarding Arts. 11, 13, 21, 35, 40, 52 and 53 of the
Family Code as the applicable provisions in this case.

"5

The trial court gravely erred in pronouncing that the divorce gravely erred in pronouncing that the divorce
decree obtained by the respondent in Australia ipso facto capacitated the parties to remarry, without first
securing a recognition of the judgment granting the divorce decree before our courts." 19

The Petition raises five issues, but for purposes of this Decision, we shall concentrate on two pivotal ones: (1)
whether the divorce between respondent and Editha Samson was proven, and (2) whether respondent was
proven to be legally capacitated to marry petitioner. Because of our ruling on these two, there is no more
necessity to take up the rest.

The Court's Ruling

The Petition is partly meritorious.

First Issue:

Proving the Divorce Between Respondent and Editha Samson

Petitioner assails the trial court's recognition of the divorce between respondent and Editha Samson.
Citing Adong v. Cheong Seng Gee,20 petitioner argues that the divorce decree, like any other foreign judgment,
may be given recognition in this jurisdiction only upon proof of the existence of (1) the foreign law allowing
absolute divorce and (2) the alleged divorce decree itself. She adds that respondent miserably failed to establish
these elements.
Petitioner adds that, based on the first paragraph of Article 26 of the Family Code, marriages solemnized abroad
are governed by the law of the place where they were celebrated (the lex loci celebrationist). In effect, the Code
requires the presentation of the foreign law to show the conformity of the marriage in question to the legal
requirements of the place where the marriage was performed.

At the outset, we lay the following basic legal principles as the take-off points for our discussion. Philippine law
does not provide for absolute divorce; hence, our courts cannot grant it. 21 A marriage between two Filipinos
cannot be dissolved even by a divorce obtained abroad, because of Articles 15 22 and 1723 of the Civil Code.24 In
mixed marriages involving a Filipino and a foreigner, Article 2625 of the Family Code allows the former to contract
a subsequent marriage in case the divorce is "validly obtained abroad by the alien spouse capacitating him or her
to remarry."26 A divorce obtained abroad by a couple, who are both aliens, may be recognized in the Philippines,
provided it is consistent with their respective national laws. 27

A comparison between marriage and divorce, as far as pleading and proof are concerned, can be made. Van
Dorn v. Romillo Jr. decrees that "aliens may obtain divorces abroad, which may be recognized in the Philippines,
provided they are valid according to their national law." 28 Therefore, before a foreign divorce decree can be
recognized by our courts, the party pleading it must prove the divorce as a fact and demonstrate its conformity to
the foreign law allowing it.29 Presentation solely of the divorce decree is insufficient.

Divorce as a Question of Fact

Petitioner insists that before a divorce decree can be admitted in evidence, it must first comply with the
registration requirements under Articles 11, 13 and 52 of the Family Code. These articles read as follows:

"ART. 11. Where a marriage license is required, each of the contracting parties shall file separately a
sworn application for such license with the proper local civil registrar which shall specify the following:

xxx xxx xxx

"(5) If previously married, how, when and where the previous marriage was dissolved or annulled;

xxx xxx xxx

"ART. 13. In case either of the contracting parties has been previously married, the applicant shall be
required to furnish, instead of the birth of baptismal certificate required in the last preceding article, the
death certificate of the deceased spouse or the judicial decree of annulment or declaration of nullity of his
or her previous marriage. x x x.

"ART. 52. The judgment of annulment or of absolute nullity of the marriage, the partition and distribution
of the properties of the spouses, and the delivery of the children's presumptive legitimes shall be
recorded in the appropriate civil registry and registries of property; otherwise, the same shall not affect
their persons."

Respondent, on the other hand, argues that the Australian divorce decree is a public document – a written official
act of an Australian family court. Therefore, it requires no further proof of its authenticity and due execution.

Respondent is getting ahead of himself. Before a foreign judgment is given presumptive evidentiary value, the
document must first be presented and admitted in evidence. 30 A divorce obtained abroad is proven by the divorce
decree itself. Indeed the best evidence of a judgment is the judgment itself. 31 The decree purports to be a written
act or record of an act of an officially body or tribunal of a foreign country.32

Under Sections 24 and 25 of Rule 132, on the other hand, a writing or document may be proven as a public or
official record of a foreign country by either (1) an official publication or (2) a copy thereof attested 33 by the officer
having legal custody of the document. If the record is not kept in the Philippines, such copy must be (a)
accompanied by a certificate issued by the proper diplomatic or consular officer in the Philippine foreign service
stationed in the foreign country in which the record is kept and (b) authenticated by the seal of his office. 34

The divorce decree between respondent and Editha Samson appears to be an authentic one issued by an
Australian family court.35 However, appearance is not sufficient; compliance with the aforemetioned rules on
evidence must be demonstrated.

Fortunately for respondent's cause, when the divorce decree of May 18, 1989 was submitted in evidence,
counsel for petitioner objected, not to its admissibility, but only to the fact that it had not been registered in the
Local Civil Registry of Cabanatuan City.36 The trial court ruled that it was admissible, subject to petitioner's
qualification.37 Hence, it was admitted in evidence and accorded weight by the judge. Indeed, petitioner's failure
to object properly rendered the divorce decree admissible as a written act of the Family Court of Sydney,
Australia.38

Compliance with the quoted articles (11, 13 and 52) of the Family Code is not necessary; respondent was no
longer bound by Philippine personal laws after he acquired Australian citizenship in 1992. 39 Naturalization is the
legal act of adopting an alien and clothing him with the political and civil rights belonging to a
citizen.40 Naturalized citizens, freed from the protective cloak of their former states, don the attires of their
adoptive countries. By becoming an Australian, respondent severed his allegiance to the Philippines and
the vinculum juris that had tied him to Philippine personal laws.

Burden of Proving Australian Law

Respondent contends that the burden to prove Australian divorce law falls upon petitioner, because she is the
party challenging the validity of a foreign judgment. He contends that petitioner was satisfied with the original of
the divorce decree and was cognizant of the marital laws of Australia, because she had lived and worked in that
country for quite a long time. Besides, the Australian divorce law is allegedly known by Philippine courts: thus,
judges may take judicial notice of foreign laws in the exercise of sound discretion.

We are not persuaded. The burden of proof lies with "the party who alleges the existence of a fact or thing
necessary in the prosecution or defense of an action." 41 In civil cases, plaintiffs have the burden of proving the
material allegations of the complaint when those are denied by the answer; and defendants have the burden of
proving the material allegations in their answer when they introduce new matters. 42 Since the divorce was a
defense raised by respondent, the burden of proving the pertinent Australian law validating it falls squarely upon
him.

It is well-settled in our jurisdiction that our courts cannot take judicial notice of foreign laws. 43 Like any other facts,
they must be alleged and proved. Australian marital laws are not among those matters that judges are supposed
to know by reason of their judicial function. 44 The power of judicial notice must be exercised with caution, and
every reasonable doubt upon the subject should be resolved in the negative.

Second Issue:

Respondent's Legal Capacity to Remarry

Petitioner contends that, in view of the insufficient proof of the divorce, respondent was legally incapacitated to
marry her in 1994.

Hence, she concludes that their marriage was void ab initio.

Respondent replies that the Australian divorce decree, which was validly admitted in evidence, adequately
established his legal capacity to marry under Australian law.

Respondent's contention is untenable. In its strict legal sense, divorce means the legal dissolution of a lawful
union for a cause arising after marriage. But divorces are of different types. The two basic ones are (1) absolute
divorce or a vinculo matrimonii and (2) limited divorce or a mensa et thoro. The first kind terminates the marriage,
while the second suspends it and leaves the bond in full force.45 There is no showing in the case at bar which
type of divorce was procured by respondent.

Respondent presented a decree nisi or an interlocutory decree – a conditional or provisional judgment of divorce.
It is in effect the same as a separation from bed and board, although an absolute divorce may follow after the
lapse of the prescribed period during which no reconciliation is effected. 46

Even after the divorce becomes absolute, the court may under some foreign statutes and practices, still restrict
remarriage. Under some other jurisdictions, remarriage may be limited by statute; thus, the guilty party in a
divorce which was granted on the ground of adultery may be prohibited from remarrying again. The court may
allow a remarriage only after proof of good behavior. 47

On its face, the herein Australian divorce decree contains a restriction that reads:

"1. A party to a marriage who marries again before this decree becomes absolute (unless the other party
has died) commits the offence of bigamy."48

This quotation bolsters our contention that the divorce obtained by respondent may have been restricted. It did
not absolutely establish his legal capacity to remarry according to his national law. Hence, we find no basis for
the ruling of the trial court, which erroneously assumed that the Australian divorce ipso facto restored
respondent's capacity to remarry despite the paucity of evidence on this matter.
We also reject the claim of respondent that the divorce decree raises a disputable presumption or presumptive
evidence as to his civil status based on Section 48, Rule 39 49 of the Rules of Court, for the simple reason that no
proof has been presented on the legal effects of the divorce decree obtained under Australian laws.

Significance of the Certificate of Legal Capacity

Petitioner argues that the certificate of legal capacity required by Article 21 of the Family Code was not submitted
together with the application for a marriage license. According to her, its absence is proof that respondent did not
have legal capacity to remarry.

We clarify. To repeat, the legal capacity to contract marriage is determined by the national law of the party
concerned. The certificate mentioned in Article 21 of the Family Code would have been sufficient to establish the
legal capacity of respondent, had he duly presented it in court. A duly authenticated and admitted certificate is
prima facie evidence of legal capacity to marry on the part of the alien applicant for a marriage license. 50

As it is, however, there is absolutely no evidence that proves respondent's legal capacity to marry petitioner. A
review of the records before this Court shows that only the following exhibits were presented before the lower
court: (1) for petitioner: (a) Exhibit "A" – Complaint;51 (b) Exhibit "B" – Certificate of Marriage Between Rederick A.
Recto (Filipino-Australian) and Grace J. Garcia (Filipino) on January 12, 1994 in Cabanatuan City, Nueva
Ecija;52 (c) Exhibit "C" – Certificate of Marriage Between Rederick A. Recio (Filipino) and Editha D. Samson
(Australian) on March 1, 1987 in Malabon, Metro Manila; 53 (d) Exhibit "D" – Office of the City Registrar of
Cabanatuan City Certification that no information of annulment between Rederick A. Recto and Editha D.
Samson was in its records;54 and (e) Exhibit "E" – Certificate of Australian Citizenship of Rederick A. Recto; 55 (2)
for respondent: (Exhibit "1" – Amended Answer;56 (b) Exhibit "S" – Family Law Act 1975 Decree Nisi of
Dissolution of Marriage in the Family Court of Australia;57 (c) Exhibit "3" – Certificate of Australian Citizenship of
Rederick A. Recto;58 (d) Exhibit "4" – Decree Nisi of Dissolution of Marriage in the Family Court of Australia
Certificate;59 and Exhibit "5" – Statutory Declaration of the Legal Separation Between Rederick A. Recto and
Grace J. Garcia Recio since October 22, 1995.60

Based on the above records, we cannot conclude that respondent, who was then a naturalized Australian citizen,
was legally capacitated to marry petitioner on January 12, 1994. We agree with petitioner's contention that the
court a quo erred in finding that the divorce decree ipso facto clothed respondent with the legal capacity to
remarry without requiring him to adduce sufficient evidence to show the Australian personal law governing his
status; or at the very least, to prove his legal capacity to contract the second marriage.

Neither can we grant petitioner's prayer to declare her marriage to respondent null and void on the ground of
bigamy. After all, it may turn out that under Australian law, he was really capacitated to marry petitioner as a
direct result of the divorce decree. Hence, we believe that the most judicious course is to remand this case to the
trial court to receive evidence, if any, which show petitioner's legal capacity to marry petitioner. Failing in that,
then the court a quo may declare a nullity of the parties' marriage on the ground of bigamy, there being already in
evidence two existing marriage certificates, which were both obtained in the Philippines, one in Malabon, Metro
Manila dated March 1, 1987 and the other, in Cabanatuan City dated January 12, 1994.

WHEREFORE, in the interest of orderly procedure and substantial justice, we REMAND the case to the court a
quo for the purpose of receiving evidence which conclusively show respondent's legal capacity to marry
petitioner; and failing in that, of declaring the parties' marriage void on the ground of bigamy, as above discussed.
No costs.

SO ORDERED.

Melo, Puno, Vitug, and Sandoval-Gutierrez, JJ., concur.

Footnotes

1 Penned by Judge Feliciano V. Buenaventura; rollo, pp. 7-9.

2 Rollo, p. 10.

3 Ibid, p. 9.

4 Rollo, p. 37.

5 Ibid., p. 47.

6 Id., p. 44.
7 Id., p. 36.

8 Annex "I"; temporary rollo, p. 9.

9The couple secured an Australian "Statutory Declaration" of their legal separation and division of
conjugal assets. See Annexes "3" and "4" of Respondent's Comment; rollo, p. 48.

10 Id., pp. 33-35.

11 Id., p. 39.

12 Amended Answer, p. 2; rollo, p. 39.

13 Id., pp. 77-78.

14 Id., p. 43.

15 Rollo, pp. 48-51.

16 TSN, December 16, 1998, pp. 1-8; records, pp. 172-179.

17 RTC Order of December 16, 1998; ibid., p. 203.

18The case was deemed submitted for decision on January 11, 2000, upon this Court's receipt of the
Memorandum for petitioner, signed by Atty. Olivia Velasco-Jacoba. The Memorandum for respondent,
signed by Atty. Gloria V. Gomez of Gomez and Associates, had been filed on December 10, 1999.

19 Petitioner's Memorandum, pp. 8-9; rollo, pp. 242-243.

20 43 Phil. 43, 49, March 3, 1922.

21Ruben F. Balane, "Family Courts and Significant Jurisprudence in Family Law," Journal of the
Integrated Bar of the Philippines, 1st & 2nd Quarters, 2001, Vol. XXVII, No. 1, p. 25.

22"ART. 15. Laws relating to family rights and duties, or to the status, condition and legal capacity of
persons are binding upon citizens of the Philippines, even though living abroad."

23"ART. 17. The forms and solemnities of contracts, wills, and other public instruments shall be governed
by the laws of the country in which they are executed.

xxx xxx xxx

"Prohibitive laws concerning persons, their acts or property, and those which have for their object
public order, public policy and good customs shall not be rendered ineffective by laws or
judgments promulgated, or by determinations or conventions agreed upon in a foreign country."

25Tenchaves v. Escano 15 SCRA 355, 362, November 29, 1965; Barretto Gonzalez v. Gonzales, 58 Phil.
67, 71-72, March 7, 1933.

"Art. 26. All marriages solemnized outside the Philippines in accordance with the laws in force in the
country where they were solemnized, and valid there as such, shall also be valid in this country, except
those prohibited under Articles 35(1), (4), (5), and (6), 36, 37, and 38. (71a).

"Where a marriage between a Filipino citizen and a foreigner is validly celebrated and a divorce is
thereafter validly obtained abroad by the alien spouse capacitating him or her to remarry, the Filipino
spouse shall have capacity to remarry under Philippine law." (As amended by EO 227, prom. July 27,
1987).

Cf. Van Dorn v. Romillo Jr., 139 SCRA 139, 143-144, October 8, 1985; and Pilapil v. Ibay-Somera, 174
26

SCRA 653, 663, June 30, 1989. 1âwphi1.nêt

27 Van Dorn v. Romillo Jr., supra.

28 Ibid., p. 143.
29For a detailed discussion of Van Dorn, see Salonga, Private International Law, 1995 ed. pp. 295-
300. See also Jose C. Vitug, Compendium of Civil Law and Jurisprudence, 1993 ed., p. 16;

30 "SEC. 19. Classes of documents. – For the purpose of their presentation in evidence, documents are
either public or private.

"Public documents are:

"(a) The written official acts, or records of the official acts of the sovereign authority, official bodies and
tribunals, and public officers, whether in the Philippines, or of a foreign country.

xxx xxx x x x."

31Burr W. Jones, Commentaries on the Law of Evidence in Civil Cases, Vol. IV, 1926 ed., p. 3511; §3,
Rule 130 of the Rules on Evidence provides that "when the subject of inquiry is the contents of a
document, no evidence shall be admissible other than the original document itself."

32 "SEC. 19. Classes of documents. – For the purpose of their presentation in evidence, documents are
either public or private.

Public documents are:

"(a) The written official acts, or records of the official acts of the sovereign authority, official bodies and
tribunals, and public officers, whether in the Philippines, or of a foreign country.

xxx xxx x x x."

33"Sec. 25. What attestation of copy must state. – Whenever a copy of a document or record is attested
for the purpose of evidence, the attestation must state, in substance, that the copy is a correct copy of the
original, or a specific part thereof, as the case may be. The attestation must be under the official seal of
the attesting officer, if there be any, or if he be the clerk of a court having a seal, under the seal of such
court."

34"Sec. 24. Proof of official record. – The record of public documents referred to in paragraph (a) of
Section 19, when admissible for any purpose, may be evidenced by an official publication thereof or by a
copy attested by the officer having the legal custody of the record, or by his deputy, and accompanied, if
the record is not kept in the Philippines, with a certificate that such officer has the custody. If the office in
which the record is kept is in a foreign country, the certificate may be made by a secretary of the
embassy or legation, consul general, consul, vice-consul, or consular agent or by any officer in the
foreign service of the Philippines stationed in the foreign country in which the record is kept, and
authenticated by the seal of his office."

See also Asiavest Ltd. v. Court of Appeals, 296 SCRA 539, 550-551, September 25, 1998; Pacific Asia
Overseas Shipping Corp. v. National Labor Relations Commission, 161 SCRA 122, 133-134, May 6,
1988.

35The transcript of stenographic notes states that the original copies of the divorce decrees were
presented in court (TSN, December 16, 1998, p. 5; records, p. 176), but only photocopies of the same
documents were attached to the records (Records, Index of Exhibit, p. 1.).

36 TSN, December 15, 1998, p. 7; records, p. 178.

37 TSN, December 16, 1998, p. 7; records, p. 178.

38People v. Yatco, 97 Phil. 941, 945, November 28, 1955; Marella v. Reyes, 12 Phil. 1, 3, November 10,
1908; People v. Diaz, 271 SCRA 504, 516, April 18, 1997; De la Torre v. Court of Appeals, 294 SCRA
196, 203-204, August 14, 1998, Maunlad Savings & Loan Asso., Inc. v. Court of Appeals, GR No.
114942, November 27, 2000, pp. 8-9.

39 Art. 15, Civil Code.

40Joaquin Bernas, The 1987 Constitution of the Republic of the Philippines: A Commentary, 1996 ed., p.
566.

41 Ricardo J. Francisco, Evidence: Rules of Court in the Philippines, second edition, p. 382.
42 Ibid., p. 384.

43 Wildvalley Shipping Co., Ltd. v. Court of Appeals, GR No. 119602, October 56, 2000, p. 7.

44 Francisco, p. 29, citing De los Angeles v. Cabahug, 106 839, December 29, 1959.

45 274 CJS, 15-17, §1.

46 Ibid., p. 611-613, §161.

47 27A CJS, 625, §162.

48 Rollo, p. 36.

49 "SEC. 48. Effect of foreign judgments or final orders. – The effect of a judgment or final order of a
tribunal of a foreign country, having jurisdiction to render the judgment or final order is as follows:

xxx xxx xxx

"(b) In case of a judgment or final order against a person, the judgment or final order is
presumptive evidence of a right as between the parties and their successors in interest by a
subsequent title.

"In either case, the judgment or final order may be repelled by evidence of a want of jurisdiction,
want of notice to the party, collusion, fraud, or clear mistake of law or fact."

50 In passing, we note that the absence of the said certificate is merely an irregularity in complying with
the formal requirement for procuring a marriage license. Under Article 4 of the Family Code, an
irregularity will not affect the validity of a marriage celebrated on the basis of a marriage license issued
without that certificate. (Vitug, Compendium, pp. 120-126); Sempio-Diy, Handbook on the Family Code of
the Philippines, 197 reprint, p. 17; Rufus Rodriguez, The Family Code of the Philippines Annotated, 1990
ed., p. 42; Melencio Sta. Maria Jr., Persons and Family Relations Law, 1999 ed., p. 146.).
ARTICLE 22

Delgado vs. Heirs of Mariciana Rustia GR. No. 155733


SECOND DIVISION

G.R. No. 155733 January 27, 2006

IN THE MATTER OF THE INTESTATE ESTATES OF THE DECEASED JOSEFA DELGADO AND
GUILLERMO RUSTIA CARLOTA DELGADO VDA. DE DE LA ROSA and other HEIRS OF LUIS DELGADO,
namely, HEIRS OF CONCHA VDA. DE AREVALO, HEIRS OF LUISA DELGADO VDA. DE DANAO, ANGELA
DELGADO ARESPACOCHAGA, TERESA DELGADO PERLAS, CAROLINA DELGADO-ARESPACOCHAGA,
RODOLFO DELGADO, BENJAMIN DELGADO, GLICERIA DELGADO and CLEOFAS DELGADO; and HEIRS
OF GORGONIO DELGADO, namely, RAMON DELGADO CAMPO, CARLOS DELGADO CAMPO, CLARITA
DELGADO CAMPO-REIZA, YOLANDA DELGADO ENCINAS, FELISA DELGADO CAMPO-ENCINAS and
MELINDA DELGADO CAMPO-MADARANG, Petitioners,
vs.
HEIRS OF MARCIANA RUSTIA VDA. DE DAMIAN, namely, GUILLERMO R. DAMIAN and JOSE R. DAMIAN;
HEIRS OF HORTENCIA RUSTIA CRUZ, namely, TERESITA CRUZ-SISON, HORACIO R. CRUZ, JOSEFINA
CRUZ-RODIL, AMELIA CRUZ-ENRIQUEZ and FIDEL R. CRUZ, JR.; HEIRS OF ROMAN RUSTIA, SR.,
namely, JOSEFINA RUSTIA ALBANO, VIRGINIA RUSTIA PARAISO, ROMAN RUSTIA, JR., SERGIO
RUSTIA, FRANCISCO RUSTIA, LETICIA RUSTIA-MIRANDA; and GUILLERMINA RUSTIA, as
Oppositors;1 and GUILLERMA RUSTIA, as Intervenor,2 Respondents.3

DECISION

CORONA, J.:

In this petition for review on certiorari, petitioners seek to reinstate the May 11, 1990 decision of the Regional
Trial Court (RTC) of Manila, Branch 55,4 in SP Case No. 97668, which was reversed and set aside by the Court
of Appeals in its decision5 dated October 24, 2002.

FACTS OF THE CASE

This case concerns the settlement of the intestate estates of Guillermo Rustia and Josefa Delgado. 6 The main
issue in this case is relatively simple: who, between petitioners and respondents, are the lawful heirs of the
decedents. However, it is attended by several collateral issues that complicate its resolution.

The claimants to the estates of Guillermo Rustia and Josefa Delgado may be divided into two groups: (1) the
alleged heirs of Josefa Delgado, consisting of her half- and full-blood siblings, nephews and nieces, and
grandnephews and grandnieces, and (2) the alleged heirs of Guillermo Rustia, particularly, his sisters, 7 his
nephews and nieces,8 his illegitimate child,9 and the de facto adopted child10 (ampun-ampunan) of the decedents.

The alleged heirs of Josefa Delgado

The deceased Josefa Delgado was the daughter of Felisa11 Delgado by one Lucio Campo. Aside from Josefa,
five other children were born to the couple, namely, Nazario, Edilberta, Jose, Jacoba, and Gorgonio, all
surnamed Delgado. Felisa Delgado was never married to Lucio Campo, hence, Josefa and her full-blood siblings
were all natural children of Felisa Delgado.

However, Lucio Campo was not the first and only man in Felisa Delgado’s life. Before him was Ramon
Osorio12 with whom Felisa had a son, Luis Delgado. But, unlike her relationship with Lucio Campo which was
admittedly one without the benefit of marriage, the legal status of Ramon Osorio’s and Felisa Delgado’s union is
in dispute.

The question of whether Felisa Delgado and Ramon Osorio ever got married is crucial to the claimants because
the answer will determine whether their successional rights fall within the ambit of the rule against reciprocal
intestate succession between legitimate and illegitimate relatives.13 If Ramon Osorio and Felisa Delgado had
been validly married, then their only child Luis Delgado was a legitimate half-blood brother of Josefa Delgado and
therefore excluded from the latter’s intestate estate. He and his heirs would be barred by the principle of absolute
separation between the legitimate and illegitimate families. Conversely, if the couple were never married, Luis
Delgado and his heirs would be entitled to inherit from Josefa Delgado’s intestate estate, as they would all be
within the illegitimate line.
Petitioners allege that Ramon Osorio and Felisa Delgado were never married. In support thereof, they assert that
no evidence was ever presented to establish it, not even so much as an allegation of the date or place of the
alleged marriage. What is clear, however, is that Felisa retained the surname Delgado. So did Luis, her son with
Ramon Osorio. Later on, when Luis got married, his Partida de Casamiento14 stated that he was "hijo natural de
Felisa Delgado" (the natural child of Felisa Delgado),15 significantly omitting any mention of the name and other
circumstances of his father.16 Nevertheless, oppositors (now respondents) insist that the absence of a record of
the alleged marriage did not necessarily mean that no marriage ever took place.

Josefa Delgado died on September 8, 1972 without a will. She was survived by Guillermo Rustia and some
collateral relatives, the petitioners herein. Several months later, on June 15, 1973, Guillermo Rustia executed an
affidavit of self-

adjudication of the remaining properties comprising her estate.

The marriage of Guillermo Rustia and Josefa Delgado

Sometime in 1917, Guillermo Rustia proposed marriage to Josefa Delgado17 but whether a marriage in fact took
place is disputed. According to petitioners, the two eventually lived together as husband and wife but were never
married. To prove their assertion, petitioners point out that no record of the contested marriage existed in the civil
registry. Moreover, a baptismal certificate naming Josefa Delgado as one of the sponsors referred to her as
"Señorita" or unmarried woman.

The oppositors (respondents here), on the other hand, insist that the absence of a marriage certificate did not of
necessity mean that no marriage transpired. They maintain that Guillermo Rustia and Josefa Delgado were
married on June 3, 1919 and from then on lived together as husband and wife until the death of Josefa on
September 8, 1972. During this period spanning more than half a century, they were known among their relatives
and friends to have in fact been married. To support their proposition, oppositors presented the following pieces
of evidence:

1. Certificate of Identity No. 9592 dated [December 1, 1944] issued to Mrs. Guillermo J. Rustia by Carlos
P. Romulo, then Resident Commissioner to the United States of the Commonwealth of the Philippines;

2. Philippine Passport No. 4767 issued to Josefa D. Rustia on June 25, 1947;

3. Veterans Application for Pension or Compensation for Disability Resulting from Service in the Active
Military or Naval Forces of the United States- Claim No. C-4, 004, 503 (VA Form 526) filed with the
Veterans Administration of the United States of America by Dr. Guillermo J. Rustia wherein Dr. Guillermo
J. Rustia himself [swore] to his marriage to Josefa Delgado in Manila on 3 June 1919; 18

4. Titles to real properties in the name of Guillermo Rustia indicated that he was married to Josefa
Delgado.

The alleged heirs of Guillermo Rustia

Guillermo Rustia and Josefa Delgado never had any children. With no children of their own, they took into their
home the youngsters Guillermina Rustia Rustia and Nanie Rustia. These children, never legally adopted by the
couple, were what was known in the local dialect as ampun-ampunan.

During his life with Josefa, however, Guillermo Rustia did manage to father an illegitimate child, 19 the intervenor-
respondent Guillerma Rustia, with one Amparo Sagarbarria. According to Guillerma, Guillermo Rustia treated her
as his daughter, his own flesh and blood, and she enjoyed open and continuous possession of that status from
her birth in 1920 until her father’s demise. In fact, Josefa Delgado’s obituary which was prepared by Guillermo
Rustia, named the intervenor-respondent as one of their children. Also, her report card from the University of
Santo Tomas identified Guillermo Rustia as her parent/guardian. 20

Oppositors (respondents here) nonetheless posit that Guillerma Rustia has no interest in the intestate estate of
Guillermo Rustia as she was never duly acknowledged as an illegitimate child. They contend that her right to
compulsory acknowledgement prescribed when Guillermo died in 1974 and that she cannot claim voluntary
acknowledgement since the documents she presented were not the authentic writings prescribed by the new Civil
Code.21

On January 7, 1974, more than a year after the death of Josefa Delgado, Guillermo Rustia filed a petition for the
adoption22 of their ampun-ampunan Guillermina Rustia. He stated under oath "[t]hat he ha[d] no legitimate,
legitimated, acknowledged natural children or natural children by legal fiction." 23 The petition was overtaken by
his death on February 28, 1974.
Like Josefa Delgado, Guillermo Rustia died without a will. He was survived by his sisters Marciana Rustia vda.
de Damian and Hortencia Rustia-Cruz, and by the children of his predeceased brother Roman Rustia Sr.,
namely, Josefina Rustia Albano, Virginia Rustia Paraiso, Roman Rustia, Jr., Sergio Rustia, Francisco Rustia and
Leticia Rustia Miranda.24

ANTECEDENT PROCEEDINGS

On May 8, 1975, Luisa Delgado vda. de Danao, the daughter of Luis Delgado, filed the original petition for letters
of administration of the intestate estates of the "spouses Josefa Delgado and Guillermo Rustia" with the RTC of
Manila, Branch 55.25 This petition was opposed by the following: (1) the sisters of Guillermo Rustia, namely,
Marciana Rustia vda. de Damian and Hortencia Rustia-Cruz;26 (2) the heirs of Guillermo Rustia’s late brother,
Roman Rustia, Sr., and (3) the ampun-ampunan Guillermina Rustia Rustia. The opposition was grounded on the
theory that Luisa Delgado vda. de Danao and the other claimants were barred under the law from inheriting from
their illegitimate half-blood relative Josefa Delgado.

In November of 1975, Guillerma Rustia filed a motion to intervene in the proceedings, claiming she was the only
surviving descendant in the direct line of Guillermo Rustia. Despite the objections of the oppositors (respondents
herein), the motion was granted.

On April 3, 1978, the original petition for letters of administration was amended to state that Josefa Delgado and
Guillermo Rustia were never married but had merely lived together as husband and wife.

On January 24, 1980, oppositors (respondents herein) filed a motion to dismiss the petition in the RTC insofar as
the estate of Guillermo Rustia was concerned. The motion was denied on the ground that the interests of the
petitioners and the other claimants remained in issue and should be properly threshed out upon submission of
evidence.

On March 14, 1988, Carlota Delgado vda. de de la Rosa substituted for her sister, Luisa Delgado vda. de Danao,
who had died on May 18, 1987.

On May 11, 1990, the RTC appointed Carlota Delgado vda. de de la Rosa as administratrix of both estates.27 The
dispositive portion of the decision read:

WHEREFORE, in view of all the foregoing, petitioner and her co-claimants to the estate of the late Josefa
Delgado listed in the Petitions, and enumerated elsewhere in this Decision, are hereby declared as the only legal
heirs of the said Josefa Delgado who died intestate in the City of Manila on September 8, 1972, and entitled to
partition the same among themselves in accordance with the proportions referred to in this Decision.

Similarly, the intervenor Guillerma S. Rustia is hereby declared as the sole and only surviving heir of the late Dr.
Guillermo Rustia, and thus, entitled to the entire estate of the said decedent, to the exclusion of the oppositors
and the other parties hereto.

The Affidavit of Self-Adjudication of the estate of Josefa Delgado executed by the late Guillermo J. Rustia on
June 15, 1973 is hereby SET ASIDE and declared of no force and effect.

As the estates of both dece[d]ents have not as yet been settled, and their settlement [is] considered consolidated
in this proceeding in accordance with law, a single administrator therefor is both proper and necessary, and, as
the petitioner Carlota Delgado Vda. de dela Rosa has established her right to the appointment as administratrix
of the estates, the Court hereby APPOINTS her as the ADMINISTRATRIX of the intestate estate of the decedent
JOSEFA DELGADO in relation to the estate of DR. GUILLERMO J. RUSTIA.

Accordingly, let the corresponding LETTERS OF ADMINISTRATION issue to the petitioner CARLOTA
DELGADO VDA. DE DE LA ROSA upon her filing of the requisite bond in the sum of FIVE HUNDRED
THOUSAND PESOS (P500,000.00).

Finally, oppositor GUILLERMINA RUSTIA RUSTIA is hereby ordered to cease and desist from her acts of
administration of the subject estates, and is likewise ordered to turn over to the appointed administratix all her
collections of the rentals and income due on the assets of the estates in question, including all documents,
papers, records and titles pertaining to such estates to the petitioner and appointed administratix CARLOTA
DELGADO VDA. DE DE LA ROSA, immediately upon receipt of this Decision. The same oppositor is hereby
required to render an accounting of her actual administration of the estates in controversy within a period of sixty
(60) days from receipt hereof.

SO ORDERED.28
On May 20, 1990, oppositors filed an appeal which was denied on the ground that the record on appeal was not
filed on time.29 They then filed a petition for certiorari and mandamus 30 which was dismissed by the Court of
Appeals.31 However, on motion for reconsideration and after hearing the parties’ oral arguments, the Court of
Appeals reversed itself and gave due course to oppositors’ appeal in the interest of substantial justice. 32

In a petition for review to this Court, petitioners assailed the resolution of the Court of Appeals, on the ground that
oppositors’ failure to file the record on appeal within the reglementary period was a jurisdictional defect which
nullified the appeal. On October 10, 1997, this Court allowed the continuance of the appeal. The pertinent portion
of our decision33 read:

As a rule, periods prescribed to do certain acts must be followed. However, under exceptional circumstances, a
delay in the filing of an appeal may be excused on grounds of substantial justice.

xxx xxx xxx

The respondent court likewise pointed out the trial court’s pronouncements as to certain matters of substance,
relating to the determination of the heirs of the decedents and the party entitled to the administration of their
estate, which were to be raised in the appeal, but were barred absolutely by the denial of the record on appeal
upon too technical ground of late filing.

xxx xxx xxx

In this instance, private respondents’ intention to raise valid issues in the appeal is apparent and should not have
been construed as an attempt to delay or prolong the administration proceedings.

xxx xxx xxx

A review of the trial court’s decision is needed.

xxx xxx xxx

WHEREFORE, in view of the foregoing considerations, the Court hereby AFFIRMS the Resolution dated
November 27, 1991 of the Court of Appeals in CA-G.R. SP No. 23415, for the APPROVAL of the private
respondents’ Record on Appeal and the CONTINUANCE of the appeal from the Manila, Branch LV Regional
Trial Court’s May 11, 1990 decision.

SO ORDERED.

Acting on the appeal, the Court of Appeals 34 partially set aside the trial court’s decision. Upon motion for
reconsideration,35 the Court of Appeals amended its earlier decision. 36 The dispositive portion of the amended
decision read:

With the further modification, our assailed decision is RECONSIDERED and VACATED. Consequently, the
decision of the trial court is REVERSED and SET ASIDE. A new one is hereby RENDERED declaring: 1.) Dr.
Guillermo Rustia and Josefa Delgado Rustia to have been legally married; 2.) the intestate estate of Dr.
Guillermo Rustia, Jacoba Delgado-Encinas and the children of Gorgonio Delgado (Campo) entitled to partition
among themselves the intestate estate of Josefa D. Rustia in accordance with the proportion referred to in this
decision; 3.) the oppositors-appellants as the legal heirs of the late Dr. Guillermo Rustia and thereby entitled to
partition his estate in accordance with the proportion referred to herein; and 4.) the intervenor-appellee Guillerma
S. Rustia as ineligible to inherit from the late Dr. Guillermo Rustia; thus revoking her appointment as
administratrix of his estate.

The letters of administration of the intestate estate of Dr. Guillermo Rustia in relation to the intestate estate of
Josefa Delgado shall issue to the nominee of the oppositors-appellants upon his or her qualification and filing of
the requisite bond in the sum of FIVE HUNDRED THOUSAND PESOS (P500,000.00).

Oppositor-appellant Guillermina Rustia Rustia is hereby ordered to cease and desist from her acts of
administration of the subject estates and to turn over to the appointed administrator all her collections of the
rentals and incomes due on the assets of the estates in question, including all documents, papers, records and
titles pertaining to such estates to the appointed administrator, immediately upon notice of his qualification and
posting of the requisite bond, and to render an accounting of her (Guillermina Rustia Rustia) actual administration
of the estates in controversy within a period of sixty (60) days from notice of the administrator’s qualification and
posting of the bond.
The issue of the validity of the affidavit of self-adjudication executed by Dr. Guillermo Rustia on June 15, 1973
is REMANDED to the trial court for further proceedings to determine the extent of the shares of Jacoba Delgado-
Encinas and the children of Gorgonio Delgado (Campo) affected by the said adjudication.

Hence, this recourse.

The issues for our resolution are:

1. whether there was a valid marriage between Guillermo Rustia and Josefa Delgado;

2. who the legal heirs of the decedents Guillermo Rustia and Josefa Delgado are;

3. who should be issued letters of administration.

The marriage of Guillermo Rustia and Josefa Delgado

A presumption is an inference of the existence or non-existence of a fact which courts are permitted to draw from
proof of other facts. Presumptions are classified into presumptions of law and presumptions of fact. Presumptions
of law are, in turn, either conclusive or disputable. 37

Rule 131, Section 3 of the Rules of Court provides:

Sec. 3. Disputable presumptions. — The following presumptions are satisfactory if uncontradicted, but may be
contradicted and overcome by other evidence:

xxx xxx xxx

(aa) That a man and a woman deporting themselves as husband and wife have entered into a lawful contract of
marriage;

xxx xxx xxx

In this case, several circumstances give rise to the presumption that a valid marriage existed between Guillermo
Rustia and Josefa Delgado. Their cohabitation of more than 50 years cannot be doubted. Their family and friends
knew them to be married. Their reputed status as husband and wife was such that even the original petition for
letters of administration filed by Luisa Delgado vda. de Danao in 1975 referred to them as "spouses."

Yet, petitioners maintain that Josefa Delgado and Guillermo Rustia had simply lived together as husband and
wife without the benefit of marriage. They make much of the absence of a record of the contested marriage, the
testimony of a witness38 attesting that they were not married, and a baptismal certificate which referred to Josefa
Delgado as "Señorita" or unmarried woman.39

We are not persuaded.

First, although a marriage contract is considered a primary evidence of marriage, its absence is not always proof
that no marriage in fact took place.40 Once the presumption of marriage arises, other evidence may be presented
in support thereof. The evidence need not necessarily or directly establish the marriage but must at least be
enough to strengthen the presumption of marriage. Here, the certificate of identity issued to Josefa Delgado as
Mrs. Guillermo Rustia,41 the passport issued to her as Josefa D. Rustia,42 the declaration under oath of no less
than Guillermo Rustia that he was married to Josefa Delgado 43 and the titles to the properties in the name of
"Guillermo Rustia married to Josefa Delgado," more than adequately support the presumption of marriage. These
are public documents which are prima facie evidence of the facts stated therein. 44 No clear and convincing
evidence sufficient to overcome the presumption of the truth of the recitals therein was presented by petitioners.

Second, Elisa vda. de Anson, petitioners’ own witness whose testimony they primarily relied upon to support their
position, confirmed that Guillermo Rustia had proposed marriage to Josefa Delgado and that eventually, the two
had "lived together as husband and wife." This again could not but strengthen the presumption of marriage.

Third, the baptismal certificate45 was conclusive proof only of the baptism administered by the priest who baptized
the child. It was no proof of the veracity of the declarations and statements contained therein, 46 such as the
alleged single or unmarried ("Señorita") civil status of Josefa Delgado who had no hand in its preparation.

Petitioners failed to rebut the presumption of marriage of Guillermo Rustia and Josefa Delgado. In this
jurisdiction, every intendment of the law leans toward legitimizing matrimony. Persons dwelling together
apparently in marriage are presumed to be in fact married. This is the usual order of things in society and, if the
parties are not what they hold themselves out to be, they would be living in constant violation of the common
rules of law and propriety. Semper praesumitur pro matrimonio. Always presume marriage.47

The Lawful Heirs Of Josefa Delgado

To determine who the lawful heirs of Josefa Delgado are, the questioned status of the cohabitation of her mother
Felisa Delgado with Ramon Osorio must first be addressed.

As mentioned earlier, presumptions of law are either conclusive or disputable. Conclusive presumptions are
inferences which the law makes so peremptory that no contrary proof, no matter how strong, may overturn
them.48 On the other hand, disputable presumptions, one of which is the presumption of marriage, can be relied
on only in the absence of sufficient evidence to the contrary.

Little was said of the cohabitation or alleged marriage of Felisa Delgado and Ramon Osorio. The oppositors (now
respondents) chose merely to rely on the disputable presumption of marriage even in the face of such
countervailing evidence as (1) the continued use by Felisa and Luis (her son with Ramon Osorio) of the surname
Delgado and (2) Luis Delgado’s and Caridad Concepcion’s Partida de Casamiento49 identifying Luis as "hijo
natural de Felisa Delgado" (the natural child of Felisa Delgado).50

All things considered, we rule that these factors sufficiently overcame the rebuttable presumption of marriage.
Felisa Delgado and Ramon Osorio were never married. Hence, all the children born to Felisa Delgado out of her
relations with Ramon Osorio and Lucio Campo, namely, Luis and his half-blood siblings Nazario, Edilberta, Jose,
Jacoba, Gorgonio and the decedent Josefa, all surnamed Delgado, 51 were her natural children.52

Pertinent to this matter is the following observation:

Suppose, however, that A begets X with B, and Y with another woman, C; then X and Y would be natural
brothers and sisters, but of half-blood relationship. Can they succeed each other reciprocally?

The law prohibits reciprocal succession between illegitimate children and legitimate children of the same parent,
even though there is unquestionably a tie of blood between them. It seems that to allow an illegitimate child to
succeed ab intestato (from) another illegitimate child begotten with a parent different from that of the former,
would be allowing the illegitimate child greater rights than a legitimate child. Notwithstanding this, however, we
submit that

succession should be allowed, even when the illegitimate brothers and sisters are only of the half-blood. The
reason impelling the prohibition on reciprocal successions between legitimate and illegitimate families does not
apply to the case under consideration. That prohibition has for its basis the difference in category between
illegitimate and legitimate relatives. There is no such difference when all the children are illegitimate children of
the same parent, even if begotten with different persons. They all stand on the same footing before the law, just
like legitimate children of half-blood relation. We submit, therefore, that the rules regarding succession of
legitimate brothers and sisters should be applicable to them. Full blood illegitimate brothers and sisters should
receive double the portion of half-blood brothers and sisters; and if all are either of the full blood or of the half-
blood, they shall share equally.53

Here, the above-named siblings of Josefa Delgado were related to her by full-blood, except Luis Delgado, her
half-brother. Nonetheless, since they were all illegitimate, they may inherit from each other. Accordingly, all of
them are entitled to inherit from Josefa Delgado.

We note, however, that the petitioners before us are already the nephews, nieces, grandnephews and
grandnieces of Josefa Delgado. Under Article 972 of the new Civil Code, the right of representation in the
collateral line takes place only in favor of the children of brothers and sisters (nephews and nieces).
Consequently, it cannot be exercised by grandnephews and grandnieces. 54 Therefore, the only collateral relatives
of Josefa Delgado who are entitled to partake of her intestate estate are her brothers and sisters, or their children
who were still alive at the time of her death on September 8, 1972. They have a vested right to participate in the
inheritance.55 The records not being clear on this matter, it is now for the trial court to determine who were the
surviving brothers and sisters (or their children) of Josefa Delgado at the time of her death. Together with
Guillermo Rustia,56 they are entitled to inherit from Josefa Delgado in accordance with Article 1001 of the new
Civil Code:57

Art. 1001. Should brothers and sisters or their children survive with the widow or widower, the latter shall be
entitled to one-half of the inheritance and the brothers and sisters or their children to the other one-half.

Since Josefa Delgado had heirs other than Guillermo Rustia, Guillermo could not have validly adjudicated
Josefa’s estate all to himself. Rule 74, Section 1 of the Rules of Court is clear. Adjudication by an heir of the
decedent’s entire estate to himself by means of an affidavit is allowed only if he is the sole heir to the estate:
SECTION 1. Extrajudicial settlement by agreement between heirs. – If the decedent left no will and no debts and
the heirs are all of age, or the minors are represented by their judicial or legal representatives duly authorized for
the purpose, the parties may, without securing letters of administration, divide the estate among themselves as
they see fit by means of a public instrument filed in the office of the register of deeds, and should they disagree,
they may do so in an ordinary action of partition. If there is only one heir, he may adjudicate to himself the
estate by means of an affidavit filed in the office of the register of deeds. x x x (emphasis supplied)

The Lawful Heirs Of Guillermo Rustia

Intervenor (now co-respondent) Guillerma Rustia is an illegitimate child58 of Guillermo Rustia. As such, she may
be entitled to successional rights only upon proof of an admission or recognition of paternity. 59 She, however,
claimed the status of an acknowledged illegitimate child of Guillermo Rustia only after the death of the latter on
February 28, 1974 at which time it was already the new Civil Code that was in effect.

Under the old Civil Code (which was in force till August 29, 1950), illegitimate children absolutely had no
hereditary rights. This draconian edict was, however, later relaxed in the new Civil Code which granted certain
successional rights to illegitimate children but only on condition that they were first recognized or acknowledged
by the parent.

Under the new law, recognition may be compulsory or voluntary. 60 Recognition is compulsory in any of the
following cases:

(1) in cases of rape, abduction or seduction, when the period of the offense coincides more or less with
that of the conception;

(2) when the child is in continuous possession of status of a child of the alleged father (or mother) 61 by the
direct acts of the latter or of his family;

(3) when the child was conceived during the time when the mother cohabited with the supposed father;

(4) when the child has in his favor any evidence or proof that the defendant is his father. 62

On the other hand, voluntary recognition may be made in the record of birth, a will, a statement before a court of
record or in any authentic writing.63

Intervenor Guillerma sought recognition on two grounds: first, compulsory recognition through the open and
continuous possession of the status of an illegitimate child and second, voluntary recognition through authentic
writing.

There was apparently no doubt that she possessed the status of an illegitimate child from her birth until the death
of her putative father Guillermo Rustia. However, this did not constitute acknowledgment but a mere ground by
which she could have compelled acknowledgment through the courts.64 Furthermore, any (judicial) action for
compulsory acknowledgment has a dual limitation: the lifetime of the child and the lifetime of the putative
parent.65 On the death of either, the action for compulsory recognition can no longer be filed. 66 In this case,
intervenor Guillerma’s right to claim compulsory acknowledgment prescribed upon the death of Guillermo Rustia
on February 28, 1974.

The claim of voluntary recognition (Guillerma’s second ground) must likewise fail. An authentic writing, for
purposes of voluntary recognition, is understood as a genuine or indubitable writing of the parent (in this case,
Guillermo Rustia). This includes a public instrument or a private writing admitted by the father to be his. 67 Did
intervenor’s report card from the University of Santo Tomas and Josefa Delgado’s obituary prepared by Guillermo
Rustia qualify as authentic writings under the new Civil Code? Unfortunately not. The report card of intervenor
Guillerma did not bear the signature of Guillermo Rustia. The fact that his name appears there as intervenor’s
parent/guardian holds no weight since he had no participation in its preparation. Similarly, while witnesses
testified that it was Guillermo Rustia himself who drafted the notice of death of Josefa Delgado which was
published in the Sunday Times on September 10, 1972, that published obituary was not the authentic writing
contemplated by the law. What could have been admitted as an authentic writing was the original manuscript of
the notice, in the handwriting of Guillermo Rustia himself and signed by him, not the newspaper clipping of the
obituary. The failure to present the original signed manuscript was fatal to intervenor’s claim.

The same misfortune befalls the ampun-ampunan, Guillermina Rustia Rustia, who was never adopted in
accordance with law. Although a petition for her adoption was filed by Guillermo Rustia, it never came to fruition
and was dismissed upon the latter’s death. We affirm the ruling of both the trial court and the Court of Appeals
holding her a legal stranger to the deceased spouses and therefore not entitled to inherit from them ab intestato.
We quote:
Adoption is a juridical act, a proceeding in rem, which [created] between two persons a relationship similar to that
which results from legitimate paternity and filiation. Only an adoption made through the court, or in pursuance
with the procedure laid down under Rule 99 of the Rules of Court is valid in this jurisdiction. It is not of natural law
at all, but is wholly and entirely artificial. To establish the relation, the statutory requirements must be strictly
carried out, otherwise, the adoption is an absolute nullity. The fact of adoption is never presumed, but must be
affirmatively [proven] by the person claiming its existence. 68

Premises considered, we rule that two of the claimants to the estate of Guillermo Rustia, namely, intervenor
Guillerma Rustia and the ampun-ampunan Guillermina Rustia Rustia, are not lawful heirs of the decedent. Under
Article 1002 of the new Civil Code, if there are no descendants, ascendants, illegitimate children, or surviving
spouse, the collateral relatives shall succeed to the entire estate of the deceased. Therefore, the lawful heirs of
Guillermo Rustia are the remaining claimants, consisting of his sisters, 69 nieces and nephews.70

Entitlement To Letters Of Administration

An administrator is a person appointed by the court to administer the intestate estate of the decedent. Rule 78,
Section 6 of the Rules of Court prescribes an order of preference in the appointment of an administrator:

Sec. 6. When and to whom letters of administration granted. – If no executor is named in the will, or the executor
or executors are incompetent, refuse the trust, or fail to give a bond, or a person dies intestate, administration
shall be granted:

(a) To the surviving husband or wife, as the case may be, or next of kin, or both, in the discretion of the
court, or to such person as such surviving husband or wife, or next of kin, requests to have appointed, if
competent and willing to serve;

(b) If such surviving husband or wife, as the case may be, or next of kin, or the person selected by them,
be incompetent or unwilling, or if the husband or widow or next of kin, neglects for thirty (30) days after
the death of the person to apply for administration or to request that the administration be granted to
some other person, it may be granted to one or more of the principal creditors, if competent and willing to
serve;

(c) If there is no such creditor competent and willing to serve, it may be granted to such other person as
the court may select.

In the appointment of an administrator, the principal consideration is the interest in the estate of the one to be
appointed.71 The order of preference does not rule out the appointment of co-administrators, specially in cases
where

justice and equity demand that opposing parties or factions be represented in the management of the estates, 72 a
situation which obtains here.

It is in this light that we see fit to appoint joint administrators, in the persons of Carlota Delgado vda. de de la
Rosa and a nominee of the nephews and nieces of Guillermo Rustia. They are the next of kin of the deceased
spouses Josefa Delgado and Guillermo Rustia, respectively.

WHEREFORE, the petition (which seeks to reinstate the May 11, 1990 decision of the RTC Manila, Branch 55) is
hereby DENIED. The assailed October 24, 2002 decision of the Court of Appeals is AFFIRMED with the following
modifications:

1. Guillermo Rustia’s June 15, 1973 affidavit of self-adjudication is hereby ANNULLED.

2. the intestate estate of Guillermo Rustia shall inherit half of the intestate estate of Josefa Delgado. The
remaining half shall pertain to (a) the full and half-siblings of Josefa Delgado who survived her and (b) the
children of any of Josefa Delgado’s full- or half-siblings who may have predeceased her, also surviving at
the time of her death. Josefa Delgado’s grandnephews and grandnieces are excluded from her estate. In
this connection, the trial court is hereby ordered to determine the identities of the relatives of Josefa
Delgado who are entitled to share in her estate.

3. Guillermo Rustia’s estate (including its one-half share of Josefa Delgado’s estate) shall be inherited by
Marciana Rustia vda. de Damian and Hortencia Rustia Cruz (whose respective shares shall
be per capita) and the children of the late Roman Rustia, Sr. (who survived Guillermo Rustia and whose
respective shares shall be per stirpes). Considering that Marciana Rustia vda. de Damian and Hortencia
Rustia Cruz are now deceased, their respective shares shall pertain to their estates.
4. Letters of administration over the still unsettled intestate estates of Guillermo Rustia and Josefa
Delgado shall issue to Carlota Delgado vda. de de la Rosa and to a nominee from among the heirs of
Guillermo Rustia, as joint administrators, upon their qualification and filing of the requisite bond in such
amount as may be determined by the trial court.

No pronouncement as to costs.

SO ORDERED.

RENATO C. CORONA
Associate Justice

WE CONCUR:

REYNATO S. PUNO
Associate Justice
Chairman

ANGELINA SANDOVAL-GUTIERREZ ADOLFO S. AZCUNA


Associate Justice Asscociate Justice

CANCIO C. GARCIA
Associate Justice

ATTESTATION

I attest that the conclusions in the above decision were reached in consultation before the case was assigned to
the writer of the opinion of the Court’s Division.

REYNATO S. PUNO
Associate Justice
Chairman, Second Division

CERTIFICATION

Pursuant to Article VIII, Section 13 of the Constitution, and the Division Chairman’s Attestation, I hereby certify
that the conclusions in the above decision were reached in consultation before the case was assigned to the
writer of the opinion of the Court.

ARTEMIO V. PANGANIBAN
Chief Justice

Footnotes

1 Oppositors in SP Case No. 97668 with the RTC Manila, Branch 55.

2 Intervenor in SP Case No. 97668 with the RTC Manila, Branch 55.

3 In the petition for review on certiorari filed by petitioners, the oppositors were identified as "oppositors-
respondents," while intervenor was identified as "intervenor-respondent." For clarity, we shall refer to
them collectively as "respondents" in this decision. The Court of Appeals was also impleaded as public
respondent but this was not necessary since this is a petition for review under Rule 45 of the Rules of
Court.

4 Judge Hermogenes Liwag, Rollo, pp. 92-106.

5Penned by Associate Justice Jose L. Sabio, Jr., and concurred in by Associate Justices Oswaldo D.
Agcaoili and Sergio L. Pestaño of the former 15th Division, Rollo, pp. 75-90.

6The original action was a petition for letters of administration of the intestate estates of Guillermo Rustia
and Josefa Delgado, Rollo, p. 92.
7 Marciana Rustia vda. de Damian and Hortencia Rustia Cruz, both deceased and now substituted by
their respective heirs.

8 The children of Guillermo Rustia’s deceased brother Roman Rustia, Sr.

9 Intervenor Guillerma Rustia.

10 Oppositor Guillermina Rustia Rustia.

11 In some pleadings, this was spelled as "Feliza."

12 In some pleadings, this was spelled as "Osario" and in others, "Oscorro."

13Art. 992, new Civil Code. An illegitimate child has no right to inherit ab intestato from the legitimate
children and relatives of his father or mother; nor shall such children or relatives inherit in the same
manner from the illegitimate child.

14 Rollo, p. 1262.

15 Id., pp. 1200-1201.

16 In relation, the Civil Code of Spain (the old Civil Code) provided that when the acknowledgment was
made separately by either parent, the name of the other parent shall not be revealed. Nor shall any
circumstance be mentioned by which such person might be recognized (Article 132). This showed the
intent of the said Code to protect the identity of the non-acknowledging parent.

17 One of the children of Felisa Delgado with Lucio Campo.

18 CA decision, Rollo, pp. 77-78.

19 Under the old Civil Code, which was in effect at the time of Guillerma Rustia’s birth in 1920, she was an
illegitimate child, not a natural child, since she was born of parents who at the time of conception were
disqualified to marry each other.

20 Rollo, p. 920.

21 Law in effect at the time of the death of Guillermo Rustia.

22 Filed before the then Juvenile and Domestic Relations Court of Manila.

23 Rollo, p. 1149.

24 Most of the respondents herein.

25Filed on behalf of the surviving brothers, sisters, nephews, nieces, grandnephews and grandnieces of
Josefa Delgado.

26 Now represented by their heirs as respondents.

27 Id.

28 Rollo, pp. 105-106.

29 Dated September 25, 1990.

30 This petition was initially filed with the Supreme Court but was referred to the Court of Appeals, the
latter having concurrent jurisdiction with the Supreme Court over the petition.

31Penned by Associate Justice Artemon Luna, and concurred in by Associate Justices Serafin Camilon
and Celso Magsino of the Seventh Division, dated March 20, 1991, Rollo, pp. 627-644.

32 Resolution dated November 27, 1991, Rollo, pp. 656-671.

33 De la Rosa v. Court of Appeals, 345 Phil. 678 (1997).


34Decision penned by Associate Justice Jose L. Sabio, Jr., and concurred in by Associate Justices
Oswaldo D. Agcaoili and Sergio L. Pestaño of the 15th Division, dated January 31, 2002, Rollo, pp. 46-
63.

35Both the petitioner and the oppositors filed a motion for reconsideration of the January 31, 2002
decision of the Court of Appeals.

36 Dated October 24, 2002.

37 II Florenz D. Regalado, Remedial Law Compendium 672 (9th rev. ed. 2001).

38 Elisa vda. de Anson.

39 Rollo, p. 1266.

40 Balogbog v. Court of Appeals, 336 Phil. 252 (1997).

Certificate of Identity No. 9592 dated December 1, 1944 issued to Mrs. Guillermo J. Rustia by Carlos P.
41

Romulo, then Resident Commissioner to the United States of the Commonwealth of the Philippines.

42 Philippine Passport No. 4767 issued to Josefa D. Rustia on June 25, 1947.

43 Veterans Application for Pension or Compensation for Disability Resulting from Service in the Active
Military or Naval Forces of the United States- Claim No. C-4, 004, 503 (VA Form 526) filed with the
Veterans Administration of the United States of America by Dr. Guillermo J. Rustia wherein Dr. Guillermo
J. Rustia himself stated under oath to his marriage to Josefa Delgado in Manila on June 3, 1919.

44 Rule 132, Section 23, Rules of Court.

45Josefa Delgado stood as sponsor in the baptism of Luisa Delgado on September 14, 1919, Rollo, p.
1266. In 1975, Luisa Delgado vda. de Danao filed a petition for letters of administration for the intestate
estate of Josefa Delgado; supra, note 25.

46 Acebedo v. Arquero, 447 Phil. 76 (2003).

Vda. de Jacob v. Court of Appeals, 371 Phil. 693 (1999), citing Perido v. Perido, No. L-28248, 12
47

March 1975, 63 SCRA 97.

48 Ricardo Francisco, Evidence 400 (3rd ed. 1996).

49 Rollo, p. 1262.

50 Id., pp. 1200-1201.

51 Old Civil Code, art. 134. An acknowledged natural child is entitled:

1. To bear the surname of the person acknowledging it.


2. To receive support from such person, in accordance with article 143.
3. To receive the hereditary portion, if available, determined by this Code.

52The records do not indicate the dates of birth of Felisa Delgado’s children. The dates when Felisa
Delgado cohabited with Ramon Osorio and Lucio Campo were likewise not stated. From the limited facts
of the case on this issue, it is safe to assume that they were all born during the effectivity of the old Civil
Code. Under the said Code, children born out of wedlock of parents who, at the time of conception, could
have married, were natural children.

53III Arturo M. Tolentino, Commentaries and Jurisprudence on the Civil Code of the Philippines 493-494
(1979 ed.) citing 7 Manresa 139.

54 Desiderio P. Jurado, Comments and Jurisprudence on Succession 391 (8th ed. 1991).

55In case the surviving collateral relatives are already deceased at the time of execution of this judgment,
their shares in the inheritance of Josefa Delgado shall accrue to their respective estates.

56 Then surviving spouse, now represented by his intestate estate.


57 Law in effect at the time of the death of Josefa Delgado.

58 Under the old Civil Code, which was in effect at the time of Guillerma Rustia’s birth in 1920, she is an
illegitimate child, not a natural child, since she was born of parents who, at the time of conception, were
disqualified to marry each other.

59 Paterno v. Paterno, No. L- 23060, 30 June 1967, 20 SCRA 585.

60I Arturo M. Tolentino, Commentaries and Jurisprudence on the Civil Code of the Philippines 577 (1985
ed.).

61Art. 284 of the new Civil Code provided that the mother is obliged to recognize her natural child in any
of the cases referred to in Art. 283.

62 New Civil Code, Art. 283.

63 New Civil Code, Art. 278.

64 Supra, note 60, at 283.

This was provided in Article 285 of the new Civil Code and carried over to Article 175 of the Family
65

Code. While there are exceptions to this rule, Guillerma’s case does not fall within the exceptions.

66 Subject to exceptions provided in paragraphs (1) and (2) of Article 285 of the new Civil Code.

67 I Tolentino, supra note 60, at 585-586.

68 RTC decision, Rollo, p. 104.

69 Marciana Rustia vda. de Damian and Hortencia Rustia Cruz, represented by their heirs in this petition.

70 Children of his predeceased brother Roman Rustia, Sr.

71 II Regalado, supra note 37, at 39.

72 Gabriel et al. v. Court of Appeals, G.R. No. 101512, 7 August 1992, 212 SCRA 413.
ARTICLE 26
Republic of the Philippines
SUPREME COURT

FIRST DIVISION

G.R. No. 154380 October 5, 2005

REPUBLIC OF THE PHILIPPINES, Petitioner,


vs.
CIPRIANO ORBECIDO III, Respondent.

DECISION

QUISUMBING, J.:

Given a valid marriage between two Filipino citizens, where one party is later naturalized as a foreign citizen and
obtains a valid divorce decree capacitating him or her to remarry, can the Filipino spouse likewise remarry under
Philippine law?

Before us is a case of first impression that behooves the Court to make a definite ruling on this apparently novel
question, presented as a pure question of law.

In this petition for review, the Solicitor General assails the Decision1 dated May 15, 2002, of the Regional Trial
Court of Molave, Zamboanga del Sur, Branch 23 and its Resolution2 dated July 4, 2002 denying the motion for
reconsideration. The court a quo had declared that herein respondent Cipriano Orbecido III is capacitated to
remarry. The fallo of the impugned Decision reads:

WHEREFORE, by virtue of the provision of the second paragraph of Art. 26 of the Family Code and by reason of
the divorce decree obtained against him by his American wife, the petitioner is given the capacity to remarry
under the Philippine Law.

IT IS SO ORDERED.3

The factual antecedents, as narrated by the trial court, are as follows.

On May 24, 1981, Cipriano Orbecido III married Lady Myros M. Villanueva at the United Church of Christ in the
Philippines in Lam-an, Ozamis City. Their marriage was blessed with a son and a daughter, Kristoffer Simbortriz
V. Orbecido and Lady Kimberly V. Orbecido.

In 1986, Cipriano’s wife left for the United States bringing along their son Kristoffer. A few years later, Cipriano
discovered that his wife had been naturalized as an American citizen.

Sometime in 2000, Cipriano learned from his son that his wife had obtained a divorce decree and then married a
certain Innocent Stanley. She, Stanley and her child by him currently live at 5566 A. Walnut Grove Avenue, San
Gabriel, California.

Cipriano thereafter filed with the trial court a petition for authority to remarry invoking Paragraph 2 of Article 26 of
the Family Code. No opposition was filed. Finding merit in the petition, the court granted the same. The Republic,
herein petitioner, through the Office of the Solicitor General (OSG), sought reconsideration but it was denied.

In this petition, the OSG raises a pure question of law:

WHETHER OR NOT RESPONDENT CAN REMARRY UNDER ARTICLE 26 OF THE FAMILY CODE4

The OSG contends that Paragraph 2 of Article 26 of the Family Code is not applicable to the instant case
because it only applies to a valid mixed marriage; that is, a marriage celebrated between a Filipino citizen and an
alien. The proper remedy, according to the OSG, is to file a petition for annulment or for legal
separation.5 Furthermore, the OSG argues there is no law that governs respondent’s situation. The OSG posits
that this is a matter of legislation and not of judicial determination. 6

For his part, respondent admits that Article 26 is not directly applicable to his case but insists that when his
naturalized alien wife obtained a divorce decree which capacitated her to remarry, he is likewise capacitated by
operation of law pursuant to Section 12, Article II of the Constitution. 7
At the outset, we note that the petition for authority to remarry filed before the trial court actually constituted a
petition for declaratory relief. In this connection, Section 1, Rule 63 of the Rules of Court provides:

RULE 63

DECLARATORY RELIEF AND SIMILAR REMEDIES

Section 1. Who may file petition—Any person interested under a deed, will, contract or other written instrument,
or whose rights are affected by a statute, executive order or regulation, ordinance, or other governmental
regulation may, before breach or violation thereof, bring an action in the appropriate Regional Trial Court to
determine any question of construction or validity arising, and for a declaration of his rights or duties, thereunder.

...

The requisites of a petition for declaratory relief are: (1) there must be a justiciable controversy; (2) the
controversy must be between persons whose interests are adverse; (3) that the party seeking the relief has a
legal interest in the controversy; and (4) that the issue is ripe for judicial determination. 8

This case concerns the applicability of Paragraph 2 of Article 26 to a marriage between two Filipino citizens
where one later acquired alien citizenship, obtained a divorce decree, and remarried while in the U.S.A. The
interests of the parties are also adverse, as petitioner representing the State asserts its duty to protect the
institution of marriage while respondent, a private citizen, insists on a declaration of his capacity to remarry.
Respondent, praying for relief, has legal interest in the controversy. The issue raised is also ripe for judicial
determination inasmuch as when respondent remarries, litigation ensues and puts into question the validity of his
second marriage.

Coming now to the substantive issue, does Paragraph 2 of Article 26 of the Family Code apply to the case of
respondent? Necessarily, we must dwell on how this provision had come about in the first place, and what was
the intent of the legislators in its enactment?

Brief Historical Background

On July 6, 1987, then President Corazon Aquino signed into law Executive Order No. 209, otherwise known as
the "Family Code," which took effect on August 3, 1988. Article 26 thereof states:

All marriages solemnized outside the Philippines in accordance with the laws in force in the country where they
were solemnized, and valid there as such, shall also be valid in this country, except those prohibited under
Articles 35, 37, and 38.

On July 17, 1987, shortly after the signing of the original Family Code, Executive Order No. 227 was likewise
signed into law, amending Articles 26, 36, and 39 of the Family Code. A second paragraph was added to Article
26. As so amended, it now provides:

ART. 26. All marriages solemnized outside the Philippines in accordance with the laws in force in the country
where they were solemnized, and valid there as such, shall also be valid in this country, except those prohibited
under Articles 35(1), (4), (5) and (6), 36, 37 and 38.

Where a marriage between a Filipino citizen and a foreigner is validly celebrated and a divorce is thereafter
validly obtained abroad by the alien spouse capacitating him or her to remarry, the Filipino spouse shall have
capacity to remarry under Philippine law. (Emphasis supplied)

On its face, the foregoing provision does not appear to govern the situation presented by the case at hand. It
seems to apply only to cases where at the time of the celebration of the marriage, the parties are a Filipino citizen
and a foreigner. The instant case is one where at the time the marriage was solemnized, the parties were two
Filipino citizens, but later on, the wife was naturalized as an American citizen and subsequently obtained a
divorce granting her capacity to remarry, and indeed she remarried an American citizen while residing in the
U.S.A.

Noteworthy, in the Report of the Public Hearings 9 on the Family Code, the Catholic Bishops’ Conference of the
Philippines (CBCP) registered the following objections to Paragraph 2 of Article 26:

1. The rule is discriminatory. It discriminates against those whose spouses are Filipinos who divorce them
abroad. These spouses who are divorced will not be able to re-marry, while the spouses of foreigners who validly
divorce them abroad can.
2. This is the beginning of the recognition of the validity of divorce even for Filipino citizens. For those whose
foreign spouses validly divorce them abroad will also be considered to be validly divorced here and can re-marry.
We propose that this be deleted and made into law only after more widespread consultation. (Emphasis
supplied.)

Legislative Intent

Records of the proceedings of the Family Code deliberations showed that the intent of Paragraph 2 of Article 26,
according to Judge Alicia Sempio-Diy, a member of the Civil Code Revision Committee, is to avoid the absurd
situation where the Filipino spouse remains married to the alien spouse who, after obtaining a divorce, is no
longer married to the Filipino spouse.

Interestingly, Paragraph 2 of Article 26 traces its origin to the 1985 case of Van Dorn v. Romillo, Jr.10 The Van
Dorn case involved a marriage between a Filipino citizen and a foreigner. The Court held therein that a divorce
decree validly obtained by the alien spouse is valid in the Philippines, and consequently, the Filipino spouse is
capacitated to remarry under Philippine law.

Does the same principle apply to a case where at the time of the celebration of the marriage, the parties were
Filipino citizens, but later on, one of them obtains a foreign citizenship by naturalization?

The jurisprudential answer lies latent in the 1998 case of Quita v. Court of Appeals.11 In Quita, the parties were,
as in this case, Filipino citizens when they got married. The wife became a naturalized American citizen in 1954
and obtained a divorce in the same year. The Court therein hinted, by way of obiter dictum, that a Filipino
divorced by his naturalized foreign spouse is no longer married under Philippine law and can thus remarry.

Thus, taking into consideration the legislative intent and applying the rule of reason, we hold that Paragraph 2 of
Article 26 should be interpreted to include cases involving parties who, at the time of the celebration of the
marriage were Filipino citizens, but later on, one of them becomes naturalized as a foreign citizen and obtains a
divorce decree. The Filipino spouse should likewise be allowed to remarry as if the other party were a foreigner at
the time of the solemnization of the marriage. To rule otherwise would be to sanction absurdity and injustice.
Where the interpretation of a statute according to its exact and literal import would lead to mischievous results or
contravene the clear purpose of the legislature, it should be construed according to its spirit and reason,
disregarding as far as necessary the letter of the law. A statute may therefore be extended to cases not within the
literal meaning of its terms, so long as they come within its spirit or intent. 12

If we are to give meaning to the legislative intent to avoid the absurd situation where the Filipino spouse remains
married to the alien spouse who, after obtaining a divorce is no longer married to the Filipino spouse, then the
instant case must be deemed as coming within the contemplation of Paragraph 2 of Article 26.

In view of the foregoing, we state the twin elements for the application of Paragraph 2 of Article 26 as follows:

1. There is a valid marriage that has been celebrated between a Filipino citizen and a foreigner; and

2. A valid divorce is obtained abroad by the alien spouse capacitating him or her to remarry.

The reckoning point is not the citizenship of the parties at the time of the celebration of the marriage, but their
citizenship at the time a valid divorce is obtained abroad by the alien spouse capacitating the latter to remarry.

In this case, when Cipriano’s wife was naturalized as an American citizen, there was still a valid marriage that
has been celebrated between her and Cipriano. As fate would have it, the naturalized alien wife subsequently
obtained a valid divorce capacitating her to remarry. Clearly, the twin requisites for the application of Paragraph 2
of Article 26 are both present in this case. Thus Cipriano, the "divorced" Filipino spouse, should be allowed to
remarry.

We are also unable to sustain the OSG’s theory that the proper remedy of the Filipino spouse is to file either a
petition for annulment or a petition for legal separation. Annulment would be a long and tedious process, and in
this particular case, not even feasible, considering that the marriage of the parties appears to have all the badges
of validity. On the other hand, legal separation would not be a sufficient remedy for it would not sever the
marriage tie; hence, the legally separated Filipino spouse would still remain married to the naturalized alien
spouse.

However, we note that the records are bereft of competent evidence duly submitted by respondent concerning
the divorce decree and the naturalization of respondent’s wife. It is settled rule that one who alleges a fact has
the burden of proving it and mere allegation is not evidence. 13
Accordingly, for his plea to prosper, respondent herein must prove his allegation that his wife was naturalized as
an American citizen. Likewise, before a foreign divorce decree can be recognized by our own courts, the party
pleading it must prove the divorce as a fact and demonstrate its conformity to the foreign law allowing it.14 Such
foreign law must also be proved as our courts cannot take judicial notice of foreign laws. Like any other fact, such
laws must be alleged and proved.15 Furthermore, respondent must also show that the divorce decree allows his
former wife to remarry as specifically required in Article 26. Otherwise, there would be no evidence sufficient to
declare that he is capacitated to enter into another marriage.

Nevertheless, we are unanimous in our holding that Paragraph 2 of Article 26 of the Family Code (E.O. No. 209,
as amended by E.O. No. 227), should be interpreted to allow a Filipino citizen, who has been divorced by a
spouse who had acquired foreign citizenship and remarried, also to remarry. However, considering that in the
present petition there is no sufficient evidence submitted and on record, we are unable to declare, based on
respondent’s bare allegations that his wife, who was naturalized as an American citizen, had obtained a divorce
decree and had remarried an American, that respondent is now capacitated to remarry. Such declaration could
only be made properly upon respondent’s submission of the aforecited evidence in his favor.

ACCORDINGLY, the petition by the Republic of the Philippines is GRANTED. The assailed Decision dated May
15, 2002, and Resolution dated July 4, 2002, of the Regional Trial Court of Molave, Zamboanga del Sur, Branch
23, are hereby SET ASIDE.

No pronouncement as to costs.

SO ORDERED.

LEONARDO A. QUISUMBING

Associate Justice

WE CONCUR:

HILARIO G. DAVIDE, JR.

Chief Justice

Chairman

CONSUELO YNARES-SANTIAGO, ANTONIO T. CARPIO

Associate Justice Associate Justice

ADOLFO S. AZCUNA

Associate Justice

CERTIFICATION

Pursuant to Section 13, Article VIII of the Constitution, it is hereby certified that the conclusions in the above
Decision were reached in consultation before the case was assigned to the writer of the opinion of the Court’s
Division.

HILARIO G. DAVIDE, JR.

Chief Justice

Footnotes

1 Rollo, pp. 20-22.

2 Id. at 27-29.

3 Id. at 21-22.
4 Id. at 105.

5 Id. at 106-110.

6 Id. at 110.

7 Sec. 12. The State recognizes the sanctity of family life and shall protect and strengthen the family as a
basic autonomous social institution. It shall equally protect the life of the mother and the life of the unborn
from conception. The natural and primary right and duty of parents in the rearing of the youth for civic
efficiency and the development of moral character shall receive the support of the Government.

8Office of the Ombudsman v. Ibay, G.R. No. 137538, 3 September 2001, 364 SCRA 281,
286, citing Galarosa v. Valencia, G.R. No. 109455, 11 November 1993, 227 SCRA 729, 737.

9 Held on January 27 and 28, 1988 and February 3, 1988.

10 No. L-68470, 8 October 1985, 139 SCRA 139.

11 G.R. No. 124862, 22 December 1998, 300 SCRA 406.

12 Lopez & Sons, Inc. v. Court of Tax Appeals, No. L-9274, 1 February 1957, 100 Phil. 850, 855.

13 Cortes v. Court of Appeals, G.R. No. 121772, 13 January 2003, 395 SCRA 33, 38.

14 Garcia v. Recio, G.R. No. 138322, 2 October 2001, 366 SCRA 437, 447.

15 Id. at 451.
Republic of the Philippines
SUPREME COURT
Manila

SECOND DIVISION

G.R. No. 188289 August 20, 2014

DAVID A. NOVERAS, Petitioner,


vs.
LETICIA T. NOVERAS, Respondent.

DECISION

PEREZ, J.:

Before the Court is a petition for review assailing the 9 May 2008 Decision1 of the Court of Appeals in CA-G.R ..
CV No. 88686, which affirmed in part the 8 December 2006 Decision2 of the Regional Trial Court (RTC) of Baler,
Aurora, Branch 96.

The factual antecedents are as follow:

David A. Noveras (David) and Leticia T. Noveras (Leticia) were married on 3 December 1988 in Quezon City,
Philippines. They resided in California, United States of America (USA) where they eventually acquired American
citizenship. They then begot two children, namely: Jerome T.

Noveras, who was born on 4 November 1990 and JenaT. Noveras, born on 2 May 1993. David was engaged in
courier service business while Leticia worked as a nurse in San Francisco, California.

During the marriage, they acquired the following properties in the Philippines and in the USA:

PHILIPPINES
PROPERTY FAIR MARKET VALUE
House and Lot with an area of 150 sq. m. ₱1,693,125.00
located at 1085 Norma Street, Sampaloc,
Manila (Sampaloc property)
Agricultural land with an area of 20,742 sq. ₱400,000.00
m. located at Laboy, Dipaculao, Aurora
A parcel of land with an area of 2.5 hectares ₱490,000.00
located at Maria Aurora, Aurora
₱175,000.00
3
A parcel of land with an area of 175 sq.m.
located at Sabang Baler, Aurora
3-has. coconut plantation in San Joaquin ₱750,000.00
Maria Aurora, Aurora
USA
PROPERTY FAIR MARKET VALUE
House and Lot at 1155 Hanover Street, Daly
City, California
$550,000.00
(unpaid debt of $285,000.00)
Furniture and furnishings $3,000
Jewelries (ring and watch) $9,000
2000 Nissan Frontier 4x4 pickup truck $13,770.00
Bank of America Checking Account $8,000
Bank of America Cash Deposit
Life Insurance (Cash Value) $100,000.00
4
Retirement, pension, profit-sharing, annuities $56,228.00

The Sampaloc property used to beowned by David’s parents. The parties herein secured a loan from a bank and
mortgaged the property. When said property was about to be foreclosed, the couple paid a total of ₱1.5 Million
for the redemption of the same.

Due to business reverses, David left the USA and returned to the Philippines in 2001. In December 2002,Leticia
executed a Special Power of Attorney (SPA) authorizing David to sell the Sampaloc property for ₱2.2 Million.
According to Leticia, sometime in September 2003, David abandoned his family and lived with Estrellita Martinez
in Aurora province. Leticia claimed that David agreed toand executed a Joint Affidavit with Leticia in the presence
of David’s father, Atty. Isaias Noveras, on 3 December 2003 stating that: 1) the ₱1.1Million proceeds from the
sale of the Sampaloc property shall be paid to and collected by Leticia; 2) that David shall return and pay to
Leticia ₱750,000.00, which is equivalent to half of the amount of the redemption price of the Sampaloc property;
and 3) that David shall renounce and forfeit all his rights and interest in the conjugal and real properties situated
in the Philippines.5 David was able to collect ₱1,790,000.00 from the sale of the Sampaloc property, leaving an
unpaid balance of ₱410,000.00.

Upon learning that David had an extra-marital affair, Leticia filed a petition for divorce with the Superior Court of
California, County of San Mateo, USA. The California court granted the divorce on 24 June 2005 and judgment
was duly entered on 29 June 2005.6 The California court granted to Leticia the custody of her two children, as
well as all the couple’s properties in the USA. 7

On 8 August 2005, Leticia filed a petition for Judicial Separation of Conjugal Property before the RTC of Baler,
Aurora. She relied on the 3 December 2003 Joint Affidavit and David’s failure to comply with his obligation under
the same. She prayed for: 1) the power to administer all conjugal properties in the Philippines; 2) David and his
partner to cease and desist from selling the subject conjugal properties; 3) the declaration that all conjugal
properties be forfeited in favor of her children; 4) David to remit half of the purchase price as share of Leticia from
the sale of the Sampaloc property; and 5) the payment of₱50,000.00 and ₱100,000.00 litigation expenses. 8

In his Answer, David stated that a judgment for the dissolution of their marriage was entered on 29 June 2005 by
the Superior Court of California, County of San Mateo. He demanded that the conjugal partnership properties,
which also include the USA properties, be liquidated and that all expenses of liquidation, including attorney’s fees
of both parties be charged against the conjugal partnership. 9

The RTC of Baler, Aurora simplified the issues as follow:

1. Whether or not respondent David A. Noveras committed acts of abandonment and marital infidelity
which can result intothe forfeiture of the parties’ properties in favor of the petitioner and their two (2)
children.

2. Whether or not the Court has jurisdiction over the properties in California, U.S.A. and the same can be
included in the judicial separation prayed for.

3. Whether or not the "Joint Affidavit" x x x executed by petitioner Leticia T. Noveras and respondent
David A. Noveras will amount to a waiver or forfeiture of the latter’s property rights over their conjugal
properties.

4. Whether or not Leticia T. Noveras isentitled to reimbursement of onehalf of the ₱2.2 [M]illion sales
proceeds of their property in Sampaloc, Manila and one-half of the ₱1.5 [M]illion used to redeem the
property of Atty. Isaias Noveras, including interests and charges.

5. How the absolute community properties should be distributed.

6. Whether or not the attorney’s feesand litigation expenses of the parties were chargeable against their
conjugal properties.

Corollary to the aboveis the issue of:

Whether or not the two common children of the parties are entitled to support and presumptive legitimes.10

On 8 December 2006, the RTC rendered judgment as follows:

1. The absolute community of property of the parties is hereby declared DISSOLVED;


2. The net assets of the absolute community of property ofthe parties in the Philippines are hereby
ordered to be awarded to respondent David A. Noveras only, with the properties in the United States of
America remaining in the sole ownership of petitioner Leticia Noveras a.k.a. Leticia Tacbiana pursuant to
the divorce decree issuedby the Superior Court of California, County of San Mateo, United States of
America, dissolving the marriage of the parties as of June 24, 2005. The titles presently covering said
properties shall be cancelled and new titles be issued in the name of the party to whom said properties
are awarded;

3. One-half of the properties awarded to respondent David A. Noveras in the preceding paragraph are
hereby given to Jerome and Jena, his two minor children with petitioner LeticiaNoveras a.k.a. Leticia
Tacbiana as their presumptive legitimes and said legitimes must be annotated on the titles covering the
said properties.Their share in the income from these properties shall be remitted to them annually by the
respondent within the first half of January of each year, starting January 2008;

4. One-half of the properties in the United States of America awarded to petitioner Leticia Noveras a.k.a.
Leticia Tacbiana in paragraph 2 are hereby given to Jerome and Jena, her two minor children with
respondent David A. Noveras as their presumptive legitimes and said legitimes must be annotated on the
titles/documents covering the said properties. Their share in the income from these properties, if any,
shall be remitted to them annually by the petitioner within the first half of January of each year, starting
January 2008;

5. For the support of their two (2) minor children, Jerome and Jena, respondent David A. Noveras shall
give them US$100.00 as monthly allowance in addition to their income from their presumptive legitimes,
while petitioner Leticia Tacbiana shall take care of their food, clothing, education and other needs while
they are in her custody in the USA. The monthly allowance due from the respondent shall be increased in
the future as the needs of the children require and his financial capacity can afford;

6. Of the unpaid amount of ₱410,000.00 on the purchase price of the Sampaloc property, the Paringit
Spouses are hereby ordered to pay ₱5,000.00 to respondent David A. Noveras and ₱405,000.00 to the
two children. The share of the respondent may be paid to him directly but the share of the two children
shall be deposited with a local bank in Baler, Aurora, in a joint account tobe taken out in their names,
withdrawal from which shall only be made by them or by their representative duly authorized with a
Special Power of Attorney. Such payment/deposit shall be made withinthe period of thirty (30) days after
receipt of a copy of this Decision, with the passbook of the joint account to be submitted to the custody of
the Clerk of Court of this Court within the same period. Said passbook can be withdrawn from the Clerk of
Court only by the children or their attorney-in-fact; and

7. The litigation expenses and attorney’s fees incurred by the parties shall be shouldered by them
individually.11

The trial court recognized that since the parties are US citizens, the laws that cover their legal and personalstatus
are those of the USA. With respect to their marriage, the parties are divorced by virtue of the decree of
dissolution of their marriage issued by the Superior Court of California, County of San Mateo on 24June 2005.
Under their law, the parties’ marriage had already been dissolved. Thus, the trial court considered the petition
filed by Leticia as one for liquidation of the absolute community of property regime with the determination of the
legitimes, support and custody of the children, instead of an action for judicial separation of conjugal property.

With respect to their property relations, the trial court first classified their property regime as absolute community
of property because they did not execute any marriage settlement before the solemnization of their marriage
pursuant to Article 75 of the Family Code. Then, the trial court ruled that in accordance with the doctrine of
processual presumption, Philippine law should apply because the court cannot take judicial notice of the US law
since the parties did not submit any proof of their national law. The trial court held that as the instant petition does
not fall under the provisions of the law for the grant of judicial separation of properties, the absolute community
properties cannot beforfeited in favor of Leticia and her children. Moreover, the trial court observed that Leticia
failed to prove abandonment and infidelity with preponderant evidence.

The trial court however ruled that Leticia is not entitled to the reimbursements she is praying for considering that
she already acquired all of the properties in the USA. Relying still on the principle of equity, the Court also
adjudicated the Philippine properties to David, subject to the payment of the children’s presumptive legitimes.
The trial court held that under Article 89 of the Family Code, the waiver or renunciation made by David of his
property rights in the Joint Affidavit is void.

On appeal, the Court of Appeals modified the trial court’s Decision by directing the equal division of the Philippine
properties between the spouses. Moreover with respect to the common children’s presumptive legitime, the
appellate court ordered both spouses to each pay their children the amount of ₱520,000.00, thus:
WHEREFORE, the instant appeal is PARTLY GRANTED. Numbers 2, 4 and 6 of the assailedDecision dated
December 8, 2006 of Branch 96, RTC of Baler, Aurora Province, in Civil Case No. 828 are hereby MODIFIED to
read as follows:

2. The net assets of the absolute community of property of the parties in the Philippines are hereby
divided equally between petitioner Leticia Noveras a.k.a. Leticia Tacbiana (sic) and respondent David A.
Noveras;

xxx

4. One-half of the properties awarded to petitioner Leticia Tacbiana (sic) in paragraph 2 shall pertain to
her minor children, Jerome and Jena, as their presumptive legitimes which shall be annotated on the
titles/documents covering the said properties. Their share in the income therefrom, if any, shall be
remitted to them by petitioner annually within the first half of January, starting 2008;

xxx

6. Respondent David A. Noveras and petitioner Leticia Tacbiana (sic) are each ordered to pay the
amount of₱520,000.00 to their two children, Jerome and Jena, as their presumptive legitimes from the
sale of the Sampaloc property inclusive of the receivables therefrom, which shall be deposited to a local
bank of Baler, Aurora, under a joint account in the latter’s names. The payment/deposit shall be made
within a period of thirty (30) days from receipt ofa copy of this Decision and the corresponding passbook
entrusted to the custody ofthe Clerk of Court a quowithin the same period, withdrawable only by the
children or their attorney-in-fact.

A number 8 is hereby added, which shall read as follows:

8. Respondent David A. Noveras is hereby ordered to pay petitioner Leticia Tacbiana (sic) the amount of
₱1,040,000.00 representing her share in the proceeds from the sale of the Sampaloc property.

The last paragraph shall read as follows:

Send a copy of this Decision to the local civil registry of Baler, Aurora; the local civil registry of Quezon City; the
Civil RegistrarGeneral, National Statistics Office, Vibal Building, Times Street corner EDSA, Quezon City; the
Office of the Registry of Deeds for the Province of Aurora; and to the children, Jerome Noveras and Jena
Noveras.

The rest of the Decision is AFFIRMED.12

In the present petition, David insists that the Court of Appeals should have recognized the California Judgment
which awarded the Philippine properties to him because said judgment was part of the pleading presented and
offered in evidence before the trial court. David argues that allowing Leticia to share in the Philippine properties is
tantamount to unjust enrichment in favor of Leticia considering that the latter was already granted all US
properties by the California court.

In summary and review, the basic facts are: David and Leticia are US citizens who own properties in the USA
and in the Philippines. Leticia obtained a decree of divorce from the Superior Court of California in June 2005
wherein the court awarded all the properties in the USA to Leticia. With respect to their properties in the
Philippines, Leticiafiled a petition for judicial separation ofconjugal properties.

At the outset, the trial court erred in recognizing the divorce decree which severed the bond of marriage between
the parties. In Corpuz v. Sto. Tomas,13 we stated that:

The starting point in any recognition of a foreign divorce judgment is the acknowledgment that our courts do not
take judicial notice of foreign judgments and laws. Justice Herrera explained that, as a rule, "no sovereign is
bound to give effect within its dominion to a judgment rendered by a tribunal of another country." This means that
the foreign judgment and its authenticity must beproven as facts under our rules on evidence, together with the
alien’s applicable national law to show the effect of the judgment on the alien himself or herself. The recognition
may be made in an action instituted specifically for the purpose or in another action where a party invokes the
foreign decree as an integral aspect of his claim or defense. 14

The requirements of presenting the foreign divorce decree and the national law of the foreigner must comply with
our Rules of Evidence. Specifically, for Philippine courts to recognize a foreign judgment relating to the status of
a marriage, a copy of the foreign judgment may be admitted in evidence and proven as a fact under Rule 132,
Sections 24 and 25, in relation to Rule 39, Section 48(b) of the Rules of Court. 15
Under Section 24 of Rule 132, the record of public documents of a sovereign authority or tribunal may be proved
by: (1) an official publication thereof or (2) a copy attested by the officer having the legal custody thereof. Such
official publication or copy must beaccompanied, if the record is not kept in the Philippines, with a certificate that
the attesting officer has the legal custody thereof. The certificate may be issued by any of the authorized
Philippine embassy or consular officials stationed in the foreign country in which the record is kept, and
authenticated by the seal of his office. The attestation must state, in substance, that the copy is a correct copy of
the original, or a specific part thereof, asthe case may be, and must be under the official seal of the attesting
officer.

Section 25 of the same Rule states that whenever a copy of a document or record is attested for the purpose of
evidence, the attestation must state, in substance, that the copy is a correct copy of the original, or a specific part
thereof, as the case may be. The attestation must be under the official seal of the attesting officer, if there be any,
or if hebe the clerk of a court having a seal, under the seal of such court.

Based on the records, only the divorce decree was presented in evidence. The required certificates to prove its
authenticity, as well as the pertinent California law on divorce were not presented.

It may be noted that in Bayot v. Court of Appeals,16 we relaxed the requirement on certification where we held that
"[petitioner therein] was clearly an American citizenwhen she secured the divorce and that divorce is recognized
and allowed in any of the States of the Union, the presentation of a copy of foreign divorce decree duly
authenticatedby the foreign court issuing said decree is, as here, sufficient." In this case however, it appears that
there is no seal from the office where the divorce decree was obtained.

Even if we apply the doctrine of processual presumption17 as the lower courts did with respect to the property
regime of the parties, the recognition of divorce is entirely a different matter because, to begin with, divorce is not
recognized between Filipino citizens in the Philippines. Absent a valid recognition of the divorce decree, it follows
that the parties are still legally married in the Philippines. The trial court thus erred in proceeding directly to
liquidation.

As a general rule, any modification in the marriage settlements must be made before the celebration of marriage.
An exception to this rule is allowed provided that the modification isjudicially approved and refers only to the
instances provided in Articles 66,67, 128, 135 and 136 of the Family Code. 18

Leticia anchored the filing of the instant petition for judicial separation of property on paragraphs 4 and 6 of Article
135 of the Family Code, to wit:

Art. 135. Any of the following shall be considered sufficient cause for judicial separation of property:

(1) That the spouse of the petitioner has been sentenced to a penalty which carries with it civil
interdiction;

(2) That the spouse of the petitioner has been judicially declared an absentee;

(3) That loss of parental authority ofthe spouse of petitioner has been decreed by the court;

(4) That the spouse of the petitioner has abandoned the latter or failed to comply with his or her
obligations to the family as provided for in Article 101;

(5) That the spouse granted the power of administration in the marriage settlements has abused that
power; and

(6) That at the time of the petition, the spouses have been separated in fact for at least one year and
reconciliation is highly improbable.

In the cases provided for in Numbers (1), (2), and (3), the presentation of the final judgment against the guiltyor
absent spouse shall be enough basis for the grant of the decree ofjudicial separation of property. (Emphasis
supplied).

The trial court had categorically ruled that there was no abandonment in this case to necessitate judicial
separation of properties under paragraph 4 of Article 135 of the Family Code. The trial court ratiocinated:

Moreover, abandonment, under Article 101 of the Family Code quoted above, must be for a valid cause and the
spouse is deemed to have abandoned the other when he/she has left the conjugal dwelling without intention of
returning. The intention of not returning is prima facie presumed if the allegedly [sic] abandoning spouse failed to
give any information as to his or her whereabouts within the period of three months from such abandonment.
In the instant case, the petitioner knows that the respondent has returned to and stayed at his hometown in Maria
Aurora, Philippines, as she even went several times to visit him there after the alleged abandonment. Also, the
respondent has been going back to the USA to visit her and their children until the relations between them
worsened. The last visit of said respondent was in October 2004 when he and the petitioner discussed the filing
by the latter of a petition for dissolution of marriage with the California court. Such turn for the worse of their
relationship and the filing of the saidpetition can also be considered as valid causes for the respondent to stay in
the Philippines.19

Separation in fact for one year as a ground to grant a judicial separation of property was not tackled in the trial
court’s decision because, the trial court erroneously treated the petition as liquidation of the absolute community
of properties.

The records of this case are replete with evidence that Leticia and David had indeed separated for more than a
year and that reconciliation is highly improbable. First, while actual abandonment had not been proven, it is
undisputed that the spouses had been living separately since 2003 when David decided to go back to the
Philippines to set up his own business. Second, Leticia heard from her friends that David has been cohabiting
with Estrellita Martinez, who represented herself as Estrellita Noveras. Editha Apolonio, who worked in the
hospital where David was once confined, testified that she saw the name of Estrellita listed as the wife of David in
the Consent for Operation form.20 Third and more significantly, they had filed for divorce and it was granted by the
California court in June 2005.

Having established that Leticia and David had actually separated for at least one year, the petition for judicial
separation of absolute community of property should be granted.

The grant of the judicial separation of the absolute community property automatically dissolves the absolute
community regime, as stated in the 4th paragraph of Article 99 ofthe Family Code, thus:

Art. 99. The absolute community terminates:

(1) Upon the death of either spouse;

(2) When there is a decree of legal separation;

(3) When the marriage is annulled or declared void; or

(4) In case of judicial separation of property during the marriage under Articles 134 to 138. (Emphasis
supplied).

Under Article 102 of the same Code, liquidation follows the dissolution of the absolute community regime and the
following procedure should apply:

Art. 102. Upon dissolution of the absolute community regime, the following procedure shall apply:

(1) An inventory shall be prepared, listing separately all the properties of the absolute community and the
exclusive properties of each spouse.

(2) The debts and obligations of the absolute community shall be paid out of its assets. In case of
insufficiency of said assets, the spouses shall be solidarily liable for the unpaid balance with their
separate properties in accordance with the provisions of the second paragraph of Article 94.

(3) Whatever remains of the exclusive properties of the spouses shall thereafter be delivered to each of
them.

(4) The net remainder of the properties of the absolute community shall constitute its net assets, which
shall be divided equally between husband and wife, unless a different proportion or division was agreed
upon in the marriage settlements, or unless there has been a voluntary waiver of such share provided in
this Code. For purposes of computing the net profits subject to forfeiture in accordance with Articles 43,
No. (2) and 63, No. (2),the said profits shall be the increase in value between the market value of the
community property at the time of the celebration of the marriage and the market value at the time of its
dissolution.

(5) The presumptive legitimes of the common children shall be delivered upon partition, in accordance
with Article 51.

(6) Unless otherwise agreed upon by the parties, in the partition of the properties, the conjugal dwelling
and the lot on which it is situated shall be adjudicated tothe spouse with whom the majority of the
common children choose to remain. Children below the age of seven years are deemed to have chosen
the mother, unless the court has decided otherwise. In case there is no such majority, the court shall
decide, taking into consideration the best interests of said children. At the risk of being repetitious, we will
not remand the case to the trial court. Instead, we shall adopt the modifications made by the Court of
Appeals on the trial court’s Decision with respect to liquidation.

We agree with the appellate court that the Philippine courts did not acquire jurisdiction over the California
properties of David and Leticia. Indeed, Article 16 of the Civil Code clearly states that real property as well as
personal property is subject to the law of the country where it is situated. Thus, liquidation shall only be limited to
the Philippine properties.

We affirm the modification madeby the Court of Appeals with respect to the share of the spouses in the
absolutecommunity properties in the Philippines, as well as the payment of their children’s presumptive legitimes,
which the appellate court explained in this wise:

Leticia and David shall likewise have an equal share in the proceeds of the Sampaloc property. While both
1âwphi1

claimed to have contributed to the redemption of the Noveras property, absent a clear showing where their
contributions came from, the same is presumed to have come from the community property. Thus, Leticia is not
entitled to reimbursement of half of the redemption money.

David's allegation that he used part of the proceeds from the sale of the Sampaloc property for the benefit of the
absolute community cannot be given full credence. Only the amount of ₱120,000.00 incurred in going to and from
the U.S.A. may be charged thereto. Election expenses in the amount of ₱300,000.00 when he ran as municipal
councilor cannot be allowed in the absence of receipts or at least the Statement of Contributions and
Expenditures required under Section 14 of Republic Act No. 7166 duly received by the Commission on Elections.
Likewise, expenses incurred to settle the criminal case of his personal driver is not deductible as the same had
not benefited the family. In sum, Leticia and David shall share equally in the proceeds of the sale net of the
amount of ₱120,000.00 or in the respective amounts of ₱1,040,000.00.

xxxx

Under the first paragraph of Article 888 of the Civil Code, "(t)he legitime of legitimate children and descendants
consists of one-half or the hereditary estate of the father and of the mother." The children arc therefore entitled to
half of the share of each spouse in the net assets of the absolute community, which shall be annotated on the
titles/documents covering the same, as well as to their respective shares in the net proceeds from the sale of the
Sampaloc property including the receivables from Sps. Paringit in the amount of ₱410,000.00. Consequently,
David and Leticia should each pay them the amount of ₱520,000.00 as their presumptive legitimes therefrom. 21

WHEREFORE, the petition is DENIED. The assailed Decision of the Court of Appeals in CA G.R. CV No. 88686
is AFFIRMED.

SO ORDERED.

JOSE PORTUGAL PEREZ


Associate Justice

WE CONCUR:

MARIA LOURDES P. A. SERENO*


Chief Justice

ANTONIO T. CARPIO
PRESBITERO J. VELASCO, JR.**
Associate Justice
Associate Justice
Chairperson

MARIANO C. DEL CASTILLO


Associate Justice

ATTESTATION

I attest that the conclusions in the above Decision had been reached in consultation before the case was
assigned to the writer of the opinion of the Court's Division.

ANTONIO T. CARPIO
Associate Justice
Second Division Chairperson
CERTIFICATION

Pursuant to Section 13, Article VIII of the Constitution and the Division Chairperson's Attestation, I certify that the
conclusions in the above Decision had been reached in consultation before the case was assigned to the writer
of the opinion of the Court's Division.

MARIA LOURDES P. A. SERENO


Chief Justice

Footnotes

* Per Raffle dated 28 July 2014.

** Per Special Order No. 1757 dated 20 August 2014.

1
Penned by Associate Justice Estela M. Perlas-Bernabe (now Supreme Court Associate Justice) with
Associate Justices Portia Aliflo-Hormachuelos and Rosmari D. Carandang, concurring. Rollo, pp. 26-37.

2
Presided by Judge Corazon D. Soluren. Records, pp. 262-288.

3
Id. at 2.

4
Id. at 27-28.

5
Id. at 16.

6
Id. at 77.

7
Id. at 79-81.

8
Id. at 4-5.

9
Id. at 23-26.

10
Id. at 267.

11
Id. at 287-288.

12
Rollo, pp. 36-37.

13
G.R. No. 186571, 11 August 2010, 628 SCRA 266.

14
Id. at 281-282.

15
Fujiki v. Marinay, G.R. No. 196049, 26 June 2013.

16
591 Phil. 452, 470 (2008).

17
Processual presumption means that where a foreign law is not pleaded or, even if pleaded, is not
proved, the presumption is that foreign law is the same as ours. See EDI-Staffbuilders Int’l. Inc. v. NLRC,
563 Phil. 1, 22 (2007).

18
Sta. Maria, Persons and Family Relations Law, Fourth Edition, 2004, p. 396.

19
Records, p. 280.

20
TSN, 9 March 2006, p. 13.

21
Rollo, pp. 34-35.
Republic of the Philippines
SUPREME COURT
Manila

THIRD DIVISION

G.R. No. 186571 August 11, 2010

GERBERT R. CORPUZ, Petitioner,


vs.
DAISYLYN TIROL STO. TOMAS and The SOLICITOR GENERAL, Respondents.

DECISION

BRION, J.:

Before the Court is a direct appeal from the decision1 of the Regional Trial Court (RTC) of Laoag City, Branch 11,
elevated via a petition for review on certiorari2 under Rule 45 of the Rules of Court (present petition).

Petitioner Gerbert R. Corpuz was a former Filipino citizen who acquired Canadian citizenship through
naturalization on November 29, 2000.3 On January 18, 2005, Gerbert married respondent Daisylyn T. Sto.
Tomas, a Filipina, in Pasig City.4 Due to work and other professional commitments, Gerbert left for Canada soon
after the wedding. He returned to the Philippines sometime in April 2005 to surprise Daisylyn, but was shocked to
discover that his wife was having an affair with another man. Hurt and disappointed, Gerbert returned to Canada
and filed a petition for divorce. The Superior Court of Justice, Windsor, Ontario, Canada granted Gerbert’s
petition for divorce on December 8, 2005. The divorce decree took effect a month later, on January 8, 2006. 5

Two years after the divorce, Gerbert has moved on and has found another Filipina to love. Desirous of marrying
his new Filipina fiancée in the Philippines, Gerbert went to the Pasig City Civil Registry Office and registered the
Canadian divorce decree on his and Daisylyn’s marriage certificate. Despite the registration of the divorce
decree, an official of the National Statistics Office (NSO) informed Gerbert that the marriage between him and
Daisylyn still subsists under Philippine law; to be enforceable, the foreign divorce decree must first be judicially
recognized by a competent Philippine court, pursuant to NSO Circular No. 4, series of 1982. 6

Accordingly, Gerbert filed a petition for judicial recognition of foreign divorce and/or declaration of marriage as
dissolved (petition) with the RTC. Although summoned, Daisylyn did not file any responsive pleading but
submitted instead a notarized letter/manifestation to the trial court. She offered no opposition to Gerbert’s petition
and, in fact, alleged her desire to file a similar case herself but was prevented by financial and personal
circumstances. She, thus, requested that she be considered as a party-in-interest with a similar prayer to
Gerbert’s.

In its October 30, 2008 decision,7 the RTC denied Gerbert’s petition. The RTC concluded that Gerbert was not
the proper party to institute the action for judicial recognition of the foreign divorce decree as he is a naturalized
Canadian citizen. It ruled that only the Filipino spouse can avail of the remedy, under the second paragraph of
Article 26 of the Family Code,8 in order for him or her to be able to remarry under Philippine law. 9 Article 26 of the
Family Code reads:

Art. 26. All marriages solemnized outside the Philippines, in accordance with the laws in force in the country
where they were solemnized, and valid there as such, shall also be valid in this country, except those prohibited
under Articles 35(1), (4), (5) and (6), 36, 37 and 38.

Where a marriage between a Filipino citizen and a foreigner is validly celebrated and a divorce is thereafter
validly obtained abroad by the alien spouse capacitating him or her to remarry, the Filipino spouse shall likewise
have capacity to remarry under Philippine law.

This conclusion, the RTC stated, is consistent with the legislative intent behind the enactment of the second
paragraph of Article 26 of the Family Code, as determined by the Court in Republic v. Orbecido III;10 the provision
was enacted to "avoid the absurd situation where the Filipino spouse remains married to the alien spouse who,
after obtaining a divorce, is no longer married to the Filipino spouse." 11

THE PETITION

From the RTC’s ruling,12 Gerbert filed the present petition.13

Gerbert asserts that his petition before the RTC is essentially for declaratory relief, similar to that filed in
Orbecido; he, thus, similarly asks for a determination of his rights under the second paragraph of Article 26 of the
Family Code. Taking into account the rationale behind the second paragraph of Article 26 of the Family Code, he
contends that the provision applies as well to the benefit of the alien spouse. He claims that the RTC ruling
unduly stretched the doctrine in Orbecido by limiting the standing to file the petition only to the Filipino spouse –
an interpretation he claims to be contrary to the essence of the second paragraph of Article 26 of the Family
Code. He considers himself as a proper party, vested with sufficient legal interest, to institute the case, as there is
a possibility that he might be prosecuted for bigamy if he marries his Filipina fiancée in the Philippines since two
marriage certificates, involving him, would be on file with the Civil Registry Office. The Office of the Solicitor
General and Daisylyn, in their respective Comments, 14 both support Gerbert’s position.

Essentially, the petition raises the issue of whether the second paragraph of Article 26 of the Family Code
extends to aliens the right to petition a court of this jurisdiction for the recognition of a foreign divorce decree.

THE COURT’S RULING

The alien spouse can claim no right under the second paragraph of Article 26 of the Family Code as the
substantive right it establishes is in favor of the Filipino spouse

The resolution of the issue requires a review of the legislative history and intent behind the second paragraph of
Article 26 of the Family Code.

The Family Code recognizes only two types of defective marriages – void15 and voidable16 marriages. In both
cases, the basis for the judicial declaration of absolute nullity or annulment of the marriage exists before or at the
time of the marriage. Divorce, on the other hand, contemplates the dissolution of the lawful union for cause
arising after the marriage.17 Our family laws do not recognize absolute divorce between Filipino citizens. 18

Recognizing the reality that divorce is a possibility in marriages between a Filipino and an alien, President
Corazon C. Aquino, in the exercise of her legislative powers under the Freedom Constitution, 19 enacted
Executive Order No. (EO) 227, amending Article 26 of the Family Code to its present wording, as follows:

Art. 26. All marriages solemnized outside the Philippines, in accordance with the laws in force in the country
where they were solemnized, and valid there as such, shall also be valid in this country, except those prohibited
under Articles 35(1), (4), (5) and (6), 36, 37 and 38.

Where a marriage between a Filipino citizen and a foreigner is validly celebrated and a divorce is thereafter
validly obtained abroad by the alien spouse capacitating him or her to remarry, the Filipino spouse shall likewise
have capacity to remarry under Philippine law.

Through the second paragraph of Article 26 of the Family Code, EO 227 effectively incorporated into the law this
Court’s holding in Van Dorn v. Romillo, Jr.20 and Pilapil v. Ibay-Somera.21 In both cases, the Court refused to
acknowledge the alien spouse’s assertion of marital rights after a foreign court’s divorce decree between the
alien and the Filipino. The Court, thus, recognized that the foreign divorce had already severed the marital bond
between the spouses. The Court reasoned in Van Dorn v. Romillo that:

To maintain x x x that, under our laws, [the Filipino spouse] has to be considered still married to [the alien
spouse] and still subject to a wife's obligations x x x cannot be just. [The Filipino spouse] should not be obliged to
live together with, observe respect and fidelity, and render support to [the alien spouse]. The latter should not
continue to be one of her heirs with possible rights to conjugal property. She should not be discriminated against
in her own country if the ends of justice are to be served.22

As the RTC correctly stated, the provision was included in the law "to avoid the absurd situation where the
Filipino spouse remains married to the alien spouse who, after obtaining a divorce, is no longer married to the
Filipino spouse."23 The legislative intent is for the benefit of the Filipino spouse, by clarifying his or her marital
status, settling the doubts created by the divorce decree. Essentially, the second paragraph of Article 26 of the
Family Code provided the Filipino spouse a substantive right to have his or her marriage to the alien spouse
considered as dissolved, capacitating him or her to remarry. 24 Without the second paragraph of Article 26 of the
Family Code, the judicial recognition of the foreign decree of divorce, whether in a proceeding instituted precisely
for that purpose or as a related issue in another proceeding, would be of no significance to the Filipino spouse
since our laws do not recognize divorce as a mode of severing the marital bond; 25 Article 17 of the Civil Code
provides that the policy against absolute divorces cannot be subverted by judgments promulgated in a foreign
country. The inclusion of the second paragraph in Article 26 of the Family Code provides the direct exception to
this rule and serves as basis for recognizing the dissolution of the marriage between the Filipino spouse and his
or her alien spouse.

Additionally, an action based on the second paragraph of Article 26 of the Family Code is not limited to the
recognition of the foreign divorce decree. If the court finds that the decree capacitated the alien spouse to
remarry, the courts can declare that the Filipino spouse is likewise capacitated to contract another marriage. No
court in this jurisdiction, however, can make a similar declaration for the alien spouse (other than that already
established by the decree), whose status and legal capacity are generally governed by his national law. 26

Given the rationale and intent behind the enactment, and the purpose of the second paragraph of Article 26 of
the Family Code, the RTC was correct in limiting the applicability of the provision for the benefit of the Filipino
spouse. In other words, only the Filipino spouse can invoke the second paragraph of Article 26 of the Family
Code; the alien spouse can claim no right under this provision.

The foreign divorce decree is presumptive evidence of a right that clothes the party with legal interest to petition
for its recognition in this jurisdiction

We qualify our above conclusion – i.e., that the second paragraph of Article 26 of the Family Code bestows no
rights in favor of aliens – with the complementary statement that this conclusion is not sufficient basis to dismiss
Gerbert’s petition before the RTC. In other words, the unavailability of the second paragraph of Article 26 of the
Family Code to aliens does not necessarily strip Gerbert of legal interest to petition the RTC for the recognition of
his foreign divorce decree. The foreign divorce decree itself, after its authenticity and conformity with the alien’s
national law have been duly proven according to our rules of evidence, serves as a presumptive evidence of right
in favor of Gerbert, pursuant to Section 48, Rule 39 of the Rules of Court which provides for the effect of foreign
judgments. This Section states:

SEC. 48. Effect of foreign judgments or final orders.—The effect of a judgment or final order of a tribunal of a
foreign country, having jurisdiction to render the judgment or final order is as follows:

(a) In case of a judgment or final order upon a specific thing, the judgment or final order is conclusive
upon the title of the thing; and

(b) In case of a judgment or final order against a person, the judgment or final order is presumptive
evidence of a right as between the parties and their successors in interest by a subsequent title.

In either case, the judgment or final order may be repelled by evidence of a want of jurisdiction, want of notice to
the party, collusion, fraud, or clear mistake of law or fact.

To our mind, direct involvement or being the subject of the foreign judgment is sufficient to clothe a party with the
requisite interest to institute an action before our courts for the recognition of the foreign judgment. In a divorce
situation, we have declared, no less, that the divorce obtained by an alien abroad may be recognized in the
Philippines, provided the divorce is valid according to his or her national law. 27

The starting point in any recognition of a foreign divorce judgment is the acknowledgment that our courts do not
take judicial notice of foreign judgments and laws. Justice Herrera explained that, as a rule, "no sovereign is
bound to give effect within its dominion to a judgment rendered by a tribunal of another country." 28 This means
that the foreign judgment and its authenticity must be proven as facts under our rules on evidence, together with
the alien’s applicable national law to show the effect of the judgment on the alien himself or herself. 29 The
recognition may be made in an action instituted specifically for the purpose or in another action where a party
invokes the foreign decree as an integral aspect of his claim or defense.

In Gerbert’s case, since both the foreign divorce decree and the national law of the alien, recognizing his or her
capacity to obtain a divorce, purport to be official acts of a sovereign authority, Section 24, Rule 132 of the Rules
of Court comes into play. This Section requires proof, either by (1) official publications or (2) copies attested by
the officer having legal custody of the documents. If the copies of official records are not kept in the Philippines,
these must be (a) accompanied by a certificate issued by the proper diplomatic or consular officer in the
Philippine foreign service stationed in the foreign country in which the record is kept and (b) authenticated by the
seal of his office.

The records show that Gerbert attached to his petition a copy of the divorce decree, as well as the required
certificates proving its authenticity,30 but failed to include a copy of the Canadian law on divorce. 31 Under this
situation, we can, at this point, simply dismiss the petition for insufficiency of supporting evidence, unless we
deem it more appropriate to remand the case to the RTC to determine whether the divorce decree is consistent
with the Canadian divorce law.

We deem it more appropriate to take this latter course of action, given the Article 26 interests that will be served
and the Filipina wife’s (Daisylyn’s) obvious conformity with the petition. A remand, at the same time, will allow
other interested parties to oppose the foreign judgment and overcome a petitioner’s presumptive evidence of a
right by proving want of jurisdiction, want of notice to a party, collusion, fraud, or clear mistake of law or fact.
Needless to state, every precaution must be taken to ensure conformity with our laws before a recognition is
made, as the foreign judgment, once recognized, shall have the effect of res judicata 32 between the parties, as
provided in Section 48, Rule 39 of the Rules of Court. 33
In fact, more than the principle of comity that is served by the practice of reciprocal recognition of foreign
judgments between nations, the res judicata effect of the foreign judgments of divorce serves as the deeper basis
for extending judicial recognition and for considering the alien spouse bound by its terms. This same effect, as
discussed above, will not obtain for the Filipino spouse were it not for the substantive rule that the second
paragraph of Article 26 of the Family Code provides.

Considerations beyond the recognition of the foreign divorce decree

As a matter of "housekeeping" concern, we note that the Pasig City Civil Registry Office has already recorded the
divorce decree on Gerbert and Daisylyn’s marriage certificate based on the mere presentation of the
decree.34 We consider the recording to be legally improper; hence, the need to draw attention of the bench and
the bar to what had been done.

Article 407 of the Civil Code states that "[a]cts, events and judicial decrees concerning the civil status of persons
shall be recorded in the civil register." The law requires the entry in the civil registry of judicial decrees that
produce legal consequences touching upon a person’s legal capacity and status, i.e., those affecting "all his
personal qualities and relations, more or less permanent in nature, not ordinarily terminable at his own will, such
as his being legitimate or illegitimate, or his being married or not." 35

A judgment of divorce is a judicial decree, although a foreign one, affecting a person’s legal capacity and status
that must be recorded. In fact, Act No. 3753 or the Law on Registry of Civil Status specifically requires the
registration of divorce decrees in the civil registry:

Sec. 1. Civil Register. – A civil register is established for recording the civil status of persons, in which shall be
entered:

(a) births;

(b) deaths;

(c) marriages;

(d) annulments of marriages;

(e) divorces;

(f) legitimations;

(g) adoptions;

(h) acknowledgment of natural children;

(i) naturalization; and

(j) changes of name.

xxxx

Sec. 4. Civil Register Books. — The local registrars shall keep and preserve in their offices the following books, in
which they shall, respectively make the proper entries concerning the civil status of persons:

(1) Birth and death register;

(2) Marriage register, in which shall be entered not only the marriages solemnized but also divorces and
dissolved marriages.

(3) Legitimation, acknowledgment, adoption, change of name and naturalization register.

But while the law requires the entry of the divorce decree in the civil registry, the law and the submission of the
decree by themselves do not ipso facto authorize the decree’s registration. The law should be read in relation
with the requirement of a judicial recognition of the foreign judgment before it can be given res judicata effect. In
the context of the present case, no judicial order as yet exists recognizing the foreign divorce decree. Thus, the
Pasig City Civil Registry Office acted totally out of turn and without authority of law when it annotated the
Canadian divorce decree on Gerbert and Daisylyn’s marriage certificate, on the strength alone of the foreign
decree presented by Gerbert.
Evidently, the Pasig City Civil Registry Office was aware of the requirement of a court recognition, as it cited NSO
Circular No. 4, series of 1982,36 and Department of Justice Opinion No. 181, series of 198237 – both of which
required a final order from a competent Philippine court before a foreign judgment, dissolving a marriage, can be
registered in the civil registry, but it, nonetheless, allowed the registration of the decree. For being contrary to law,
the registration of the foreign divorce decree without the requisite judicial recognition is patently void and cannot
produce any legal effect. 1avvphi1

Another point we wish to draw attention to is that the recognition that the RTC may extend to the Canadian
divorce decree does not, by itself, authorize the cancellation of the entry in the civil registry. A petition for
recognition of a foreign judgment is not the proper proceeding, contemplated under the Rules of Court, for the
cancellation of entries in the civil registry.

Article 412 of the Civil Code declares that "no entry in a civil register shall be changed or corrected, without
judicial order." The Rules of Court supplements Article 412 of the Civil Code by specifically providing for a special
remedial proceeding by which entries in the civil registry may be judicially cancelled or corrected. Rule 108 of the
Rules of Court sets in detail the jurisdictional and procedural requirements that must be complied with before a
judgment, authorizing the cancellation or correction, may be annotated in the civil registry. It also requires, among
others, that the verified petition must be filed with the RTC of the province where the corresponding civil registry
is located;38 that the civil registrar and all persons who have or claim any interest must be made parties to the
proceedings;39 and that the time and place for hearing must be published in a newspaper of general
circulation.40 As these basic jurisdictional requirements have not been met in the present case, we cannot
consider the petition Gerbert filed with the RTC as one filed under Rule 108 of the Rules of Court.

We hasten to point out, however, that this ruling should not be construed as requiring two separate proceedings
for the registration of a foreign divorce decree in the civil registry – one for recognition of the foreign decree and
another specifically for cancellation of the entry under Rule 108 of the Rules of Court. The recognition of the
foreign divorce decree may be made in a Rule 108 proceeding itself, as the object of special proceedings (such
as that in Rule 108 of the Rules of Court) is precisely to establish the status or right of a party or a particular fact.
Moreover, Rule 108 of the Rules of Court can serve as the appropriate adversarial proceeding 41 by which the
applicability of the foreign judgment can be measured and tested in terms of jurisdictional infirmities, want of
notice to the party, collusion, fraud, or clear mistake of law or fact.

WHEREFORE, we GRANT the petition for review on certiorari, and REVERSE the October 30, 2008 decision of
the Regional Trial Court of Laoag City, Branch 11, as well as its February 17, 2009 order. We order the REMAND
of the case to the trial court for further proceedings in accordance with our ruling above. Let a copy of this
Decision be furnished the Civil Registrar General. No costs.

SO ORDERED.

ARTURO D. BRION
Associate Justice

WE CONCUR:

CONCHITA CARPIO MORALES


Associate Justice

LUCAS P. BERSAMIN ROBERTO A. ABAD


Associate Justice Associate Justice

MARTIN S. VILLARAMA, JR.


Associate Justice

ATTESTATION

I attest that the conclusions in the above Decision had been reached in consultation before the case was
assigned to the writer of the opinion of the Court’s Division.

CONCHITA CARPIO MORALES


Associate Justice
Chairperson

CERTIFICATION
Pursuant to Section 13, Article VIII of the Constitution, and the Division Chairperson’s Attestation, it is hereby
certified that the conclusions in the above Decision had been reached in consultation before the case was
assigned to the writer of the opinion of the Court’s Division.

RENATO C. CORONA
Chief Justice

Footnotes

* Designated additional Member of the Third Division, in view of the retirement of Chief Justice Reynato
S. Puno, per Special Order No. 843 dated May 17, 2010.

1 Dated October 30, 2008, penned by Judge Perla B. Querubin; rollo, pp. 24-31.

2 Id. at 3-20.

3 Id. at 27.

4 Marriage Certificate, id. at 37.

5 Certificate of Divorce, id. at 38.

6 Id. at 47-50; the pertinent portion of NSO Circular No. 4, series of 1982, states:

It would therefore be premature to register the decree of annulment in the Register of Annulment
of Marriages in Manila, unless and until final order of execution of such foreign judgment is issued
by competent Philippine court.

7 Supra note 1.

8 Executive Order No. 209, enacted on July 6, 1987.

9 Rollo, p. 31.

10 G.R. No. 154380, October 5, 2005, 472 SCRA 114.

11 Id. at 121.

12Gerbert’s motion for reconsideration of the RTC’s October 30, 2008 decision was denied in an order
dated February 17, 2009; rollo, p. 32.

13 Supra note 2.

14 Rollo, pp. 79-87 and 125-142, respectively.

15The void marriages are those enumerated under Articles 35, 36, 37, 38, 40, 41, 44, and 53 in relation to
Article 52 of the Family Code.

16 The voidable marriages are those enumerated under Article 45 of the Family Code.

17 Garcia v. Recio, G.R. No. 138322, October 2, 2001, 366 SCRA 437, 452.

Ibid. See A. Tolentino, Commentaries and Jurisprudence on the Civil Code of the Philippines, Volume
18

One, with the Family Code of the Philippines (2004 ed.), p. 262.

19 Proclamation No. 3, issued on March 25, 1996.

20 G.R. No. L-68470, October 8, 1985, 139 SCRA 139.

21 G.R. No. 80116, June 30, 1989, 174 SCRA 653.


22 Van Dorn v. Romillo, supra note 20 at 144.

23 Republic v. Orbecido, supra note 10 at 121.

24The capacity of the Filipino spouse to remarry, however, depends on whether the foreign divorce
decree capacitated the alien spouse to do so.

25 See Article 17 in relation to Article 15 of the Civil Code:

Art. 15. Laws relating to family rights and duties, or to the status, condition and legal capacity of
persons are binding upon citizens of the Philippines, even though living abroad.

xxxx

Art. 17. x x x Prohibitive laws concerning persons, their acts or property, and those which have for
their object public order, public policy and good customs shall not be rendered ineffective by laws
or judgments promulgated, or by determinations or conventions agreed upon in a foreign country.

26 Parenthetically, we add that an alien’s legal capacity to contract is evidenced by a certificate issued by
his or her respective diplomatic and consular officials, which he or she must present to secure a marriage
license (Article 21, Family Code). The Filipino spouse who seeks to remarry, however, must still resort to
a judicial action for a declaration of authority to remarry.

27 Garcia v. Recio, supra note 17 at 447; citing Van Dorn v. Romillo, supra note 20.

28 Remedial Law, Volume II, Rules 23-56 (2007 ed.), p. 529.

Republic v. Orbecido III, supra note 10 at 123 and Garcia v. Recio, supra note 17 at 448; see also
29

Bayot v. Court of Appeals, G.R. No. 155635, November 7, 2008, 570 SCRA 472.

30 Rollo, pp. 38-41.

31The foreign divorce decree only stated that the marriage between Gerbert and Daisylyn was dissolved
by the Canadian court. The full text of the court’s judgment was not included.

32 Literally means "a thing adjudged," Black’s Law Dictionary (5th ed.), p. 1178; it establishes a rule that a
final judgment or decree on the merits by a court of competent jurisdiction is conclusive of the rights of
the parties or their privies in all later suits, on points and matters determined in the former. Supra note 28
at 462.

33See Philsec Investment Corporation v. Court of Appeals, G.R. No. 103493, June 19, 1997, 274 SCRA
102, 110, where the Court said:

While this Court has given the effect of res judicata to foreign judgments in several cases, it was
after the parties opposed to the judgment had been given ample opportunity to repel them on
grounds allowed under the law. It is not necessary for this purpose to initiate a separate action or
proceeding for enforcement of the foreign judgment. What is essential is that there is opportunity
to challenge the foreign judgment, in order for the court to properly determine its efficacy. This is
because in this jurisdiction, with respect to actions in personam, as distinguished from actions in
rem, a foreign judgment merely constitutes prima facie evidence of the justness of the claim of a
party and, as such, is subject to proof to the contrary.

34On the face of the marriage certificate, the word "DIVORCED" was written in big, bold letters; rollo, p.
37.

Silverio v. Republic, G.R. No. 174689, October 22, 2007, 537 SCRA 373, 390, citing Beduya v.
35

Republic, 120 Phil. 114 (1964).

36 Rollo, pp. 47-50.

37 Id. at 51.

38 Section 1, Rule 108, Rules of Court.

39 Section 3, Rule 108, Rules of Court.


40 Section 4, Rule 108, Rules of Court.

41When the entry sought to be corrected is substantial (i.e., the civil status of a person), a Rule 108
proceeding is deemed adversarial in nature. See Co v. Civil Register of Manila, G.R. No. 138496,
February 23, 2004, 423 SCRA 420, 430.
Republic of the Philippines
SUPREME COURT
Manila

SECOND DIVISION

G.R. No. 155635 November 7, 2008

MARIA REBECCA MAKAPUGAY BAYOT, petitioner,


vs.
THE HONORABLE COURT OF APPEALS and VICENTE MADRIGAL BAYOT, respondents.

x-------------------------------------------x

G.R. No. 163979 November 7, 2008

MARIA REBECCA MAKAPUGAY BAYOT, petitioner,


vs.
VICENTE MADRIGAL BAYOT, respondent.

DECISION

VELASCO, JR., J.:

The Case

Before us are these two petitions interposed by petitioner Maria Rebecca Makapugay Bayot impugning certain
issuances handed out by the Court of Appeals (CA) in CA-G.R. SP No. 68187.

In the first, a petition for certiorari 1 under Rule 65 and docketed as G.R. No. 155635, Rebecca assails and seeks to
nullify the April 30, 2002 Resolution2 of the CA, as reiterated in another Resolution of September 2, 2002, 3 granting a
writ of preliminary injunction in favor of private respondent Vicente Madrigal Bayot staving off the trial court's grant of
support pendente lite to Rebecca.

The second, a petition for review under Rule 45, 4 docketed G.R. No. 163979, assails the March 25, 2004 Decision5 of
the CA, (1) dismissing Civil Case No. 01-094, a suit for declaration of absolute nullity of marriage with application for
support commenced by Rebecca against Vicente before the Regional Trial Court (RTC) in Muntinlupa City; and (2)
setting aside certain orders and a resolution issued by the RTC in the said case.

Per its Resolution of August 11, 2004, the Court ordered the consolidation of both cases.

The Facts

Vicente and Rebecca were married on April 20, 1979 in Sanctuario de San Jose, Greenhills, Mandaluyong City. On its
face, the Marriage Certificate6 identified Rebecca, then 26 years old, to be an American citizen 7 born in Agaña, Guam,
USA to Cesar Tanchiong Makapugay, American, and Helen Corn Makapugay, American.

On November 27, 1982 in San Francisco, California, Rebecca gave birth to Marie Josephine Alexandra or Alix. From
then on, Vicente and Rebecca's marital relationship seemed to have soured as the latter, sometime in 1996, initiated
divorce proceedings in the Dominican Republic. Before the Court of the First Instance of the Judicial District of Santo
Domingo, Rebecca personally appeared, while Vicente was duly represented by counsel. On February 22, 1996, the
Dominican court issued Civil Decree No. 362/96,8 ordering the dissolution of the couple's marriage and "leaving them
to remarry after completing the legal requirements," but giving them joint custody and guardianship over Alix. Over a
year later, the same court would issue Civil Decree No. 406/97,9 settling the couple's property relations pursuant to an
Agreement10 they executed on December 14, 1996. Said agreement specifically stated that the "conjugal property
which they acquired during their marriage consist[s] only of the real property and all the improvements and personal
properties therein contained at 502 Acacia Avenue, Alabang, Muntinlupa." 11

Meanwhile, on March 14, 1996, or less than a month from the issuance of Civil Decree No. 362/96, Rebecca filed with
the Makati City RTC a petition12 dated January 26, 1996, with attachments, for declaration of nullity of
marriage, docketed as Civil Case No. 96-378. Rebecca, however, later moved13 and secured approval 14 of the motion
to withdraw the petition.

On May 29, 1996, Rebecca executed an Affidavit of Acknowledgment15 stating under oath that she is an American
citizen; that, since 1993, she and Vicente have been living separately; and that she is carrying a child not of Vicente.

On March 21, 2001, Rebecca filed another petition, this time before the Muntinlupa City RTC, for declaration of
absolute nullity of marriage16 on the ground of Vicente's alleged psychological incapacity. Docketed as Civil Case No.
01-094 and entitled as Maria Rebecca Makapugay Bayot v. Vicente Madrigal Bayot, the petition was eventually raffled
to Branch 256 of the court. In it, Rebecca also sought the dissolution of the conjugal partnership of gains with
application for support pendente lite for her and Alix. Rebecca also prayed that Vicente be ordered to pay a permanent
monthly support for their daughter Alix in the amount of PhP 220,000.

On June 8, 2001, Vicente filed a Motion to Dismiss17 on, inter alia, the grounds of lack of cause of action and that the
petition is barred by the prior judgment of divorce. Earlier, on June 5, 2001, Rebecca filed and moved for the allowance
of her application for support pendente lite.

To the motion to dismiss, Rebecca interposed an opposition, insisting on her Filipino citizenship, as affirmed by the
Department of Justice (DOJ), and that, therefore, there is no valid divorce to speak of.

Meanwhile, Vicente, who had in the interim contracted another marriage, and Rebecca commenced several criminal
complaints against each other. Specifically, Vicente filed adultery and perjury complaints against Rebecca. Rebecca,
on the other hand, charged Vicente with bigamy and concubinage.

Ruling of the RTC on the Motion to Dismiss


and Motion for Support Pendente Lite

On August 8, 2001, the RTC issued an Order 18 denying Vicente's motion to dismiss Civil Case No. 01-094 and
granting Rebecca's application for support pendente lite, disposing as follows:

Wherefore, premises considered, the Motion to Dismiss filed by the respondent is DENIED. Petitioner's
Application in Support of the Motion for Support Pendente Lite is hereby GRANTED. Respondent is hereby
ordered to remit the amount of TWO HUNDRED AND TWENTY THOUSAND PESOS (Php 220,000.00) a
month to Petitioner as support for the duration of the proceedings relative to the instant Petition.

SO ORDERED.19

The RTC declared, among other things, that the divorce judgment invoked by Vicente as bar to the petition for
declaration of absolute nullity of marriage is a matter of defense best taken up during actual trial. As to the grant of
support pendente lite, the trial court held that a mere allegation of adultery against Rebecca does not operate to
preclude her from receiving legal support.

Following the denial 20 of his motion for reconsideration of the above August 8, 2001 RTC order, Vicente went to the CA
on a petition for certiorari, with a prayer for the issuance of a temporary restraining order (TRO) and/or writ of
preliminary injunction.21 His petition was docketed as CA-G.R. SP No. 68187.

Grant of Writ of Preliminary Injunction by the CA

On January 9, 2002, the CA issued the desired TRO. 22 On April 30, 2002, the appellate court granted, via a Resolution,
the issuance of a writ of preliminary injunction, the decretal portion of which reads:

IN VIEW OF ALL THE FOREGOING, pending final resolution of the petition at bar, let the Writ of Preliminary
Injunction be ISSUED in this case, enjoining the respondent court from implementing the assailed Omnibus
Order dated August 8, 2001 and the Order dated November 20, 2001, and from conducting further
proceedings in Civil Case No. 01-094, upon the posting of an injunction bond in the amount of P250,000.00.

SO ORDERED.23

Rebecca moved24 but was denied reconsideration of the aforementioned April 30, 2002 resolution. In the meantime, on
May 20, 2002, the preliminary injunctive writ 25 was issued. Rebecca also moved for reconsideration of this issuance,
but the CA, by Resolution dated September 2, 2002, denied her motion.

The adverted CA resolutions of April 30, 2002 and September 2, 2002 are presently being assailed in Rebecca's
petition for certiorari, docketed under G.R. No. 155635.

Ruling of the CA

Pending resolution of G.R. No. 155635, the CA, by a Decision dated March 25, 2004, effectively dismissed Civil Case
No. 01-094, and set aside incidental orders the RTC issued in relation to the case. The fallo of the presently assailed
CA Decision reads:

IN VIEW OF THE FOREGOING, the petition is GRANTED. The Omnibus Order dated August 8, 2001 and the
Order dated November 20, 2001 are REVERSED and SET ASIDE and a new one entered DISMISSING Civil
Case No. 01-094, for failure to state a cause of action. No pronouncement as to costs.

SO ORDERED.26
To the CA, the RTC ought to have granted Vicente's motion to dismiss on the basis of the following premises:

(1) As held in China Road and Bridge Corporation v. Court of Appeals, the hypothetical-admission rule applies in
determining whether a complaint or petition states a cause of action. 27 Applying said rule in the light of the essential
elements of a cause of action,28 Rebecca had no cause of action against Vicente for declaration of nullity of marriage.

(2) Rebecca no longer had a legal right in this jurisdiction to have her marriage with Vicente declared void, the union
having previously been dissolved on February 22, 1996 by the foreign divorce decree she personally secured as an
American citizen. Pursuant to the second paragraph of Article 26 of the Family Code, such divorce restored Vicente's
capacity to contract another marriage.

(3) Rebecca's contention about the nullity of a divorce, she being a Filipino citizen at the time the foreign divorce
decree was rendered, was dubious. Her allegation as to her alleged Filipino citizenship was also doubtful as it was not
shown that her father, at the time of her birth, was still a Filipino citizen. The Certification of Birth of Rebecca issued by
the Government of Guam also did not indicate the nationality of her father.

(4) Rebecca was estopped from denying her American citizenship, having professed to have that nationality status and
having made representations to that effect during momentous events of her life, such as: (a) during her marriage; (b)
when she applied for divorce; and (c) when she applied for and eventually secured an American passport on January
18, 1995, or a little over a year before she initiated the first but later withdrawn petition for nullity of her marriage (Civil
Case No. 96-378) on March 14, 1996.

(5) Assuming that she had dual citizenship, being born of a purportedly Filipino father in Guam, USA which follows
the jus soli principle, Rebecca's representation and assertion about being an American citizen when she secured her
foreign divorce precluded her from denying her citizenship and impugning the validity of the divorce.

Rebecca seasonably filed a motion for reconsideration of the above Decision, but this recourse was denied in the
equally assailed June 4, 2004 Resolution.29 Hence, Rebecca's Petition for Review on Certiorari under Rule 45,
docketed under G.R. No. 163979.

The Issues

In G.R. No. 155635, Rebecca raises four (4) assignments of errors as grounds for the allowance of her petition, all of
which converged on the proposition that the CA erred in enjoining the implementation of the RTC's orders which would
have entitled her to support pending final resolution of Civil Case No. 01-094.

In G.R. No. 163979, Rebecca urges the reversal of the assailed CA decision submitting as follows:

THE COURT OF APPEALS GRAVELY ERRED IN NOT MENTIONING AND NOT TAKING INTO
CONSIDERATION IN ITS APPRECIATION OF THE FACTS THE FACT OF PETITIONER'S FILIPINO
CITIZENSHIP AS CATEGORICALLY STATED AND ALLEGED IN HER PETITION BEFORE THE COURT A
QUO.

II

THE COURT OF APPEALS GRAVELY ERRED IN RELYING ONLY ON ANNEXES TO THE PETITION IN
RESOLVING THE MATTERS BROUGHT BEFORE IT.

III

THE COURT OF APPEALS GRAVELY ERRED IN FAILING TO CONSIDER THAT RESPONDENT IS


ESTOPPED FROM CLAIMING THAT HIS MARRIAGE TO PETITIONER HAD ALREADY BEEN DISSOLVED
BY VIRTUE OF HIS SUBSEQUENT AND CONCURRENT ACTS.

IV

THE COURT OF APPEALS GRAVELY ERRED IN RULING THAT THERE WAS ABUSE OF DISCRETION
ON THE PART OF THE TRIAL COURT, MUCH LESS A GRAVE ABUSE.30

We shall first address the petition in G.R. No. 163979, its outcome being determinative of the success or failure of the
petition in G.R. No. 155635.

Three legal premises need to be underscored at the outset. First, a divorce obtained abroad by an alien married to a
Philippine national may be recognized in the Philippines, provided the decree of divorce is valid according to the
national law of the foreigner.31 Second, the reckoning point is not the citizenship of the divorcing parties at birth or at
the time of marriage, but their citizenship at the time a valid divorce is obtained abroad. And third, an absolute divorce
secured by a Filipino married to another Filipino is contrary to our concept of public policy and morality and shall not be
recognized in this jurisdiction.32

Given the foregoing perspective, the determinative issue tendered in G.R. No. 155635, i.e., the propriety of the
granting of the motion to dismiss by the appellate court, resolves itself into the questions of: first, whether petitioner
Rebecca was a Filipino citizen at the time the divorce judgment was rendered in the Dominican Republic on February
22, 1996; and second, whether the judgment of divorce is valid and, if so, what are its consequent legal effects?

The Court's Ruling

The petition is bereft of merit.

Rebecca an American Citizen in the Purview of This Case

There can be no serious dispute that Rebecca, at the time she applied for and obtained her divorce from Vicente, was
an American citizen and remains to be one, absent proof of an effective repudiation of such citizenship. The following
are compelling circumstances indicative of her American citizenship: (1) she was born in Agaña, Guam, USA; (2) the
principle of jus soli is followed in this American territory granting American citizenship to those who are born there; and
(3) she was, and may still be, a holder of an American passport.33

And as aptly found by the CA, Rebecca had consistently professed, asserted, and represented herself as an American
citizen, particularly: (1) during her marriage as shown in the marriage certificate; (2) in the birth certificate of Alix; and
(3) when she secured the divorce from the Dominican Republic. Mention may be made of the Affidavit of
Acknowledgment34 in which she stated being an American citizen.

It is true that Rebecca had been issued by the Bureau of Immigration (Bureau) of Identification (ID) Certificate No. RC
9778 and a Philippine Passport. On its face, ID Certificate No. RC 9778 would tend to show that she has indeed been
recognized as a Filipino citizen. It cannot be over-emphasized, however, that such recognition was given only on June
8, 2000 upon the affirmation by the Secretary of Justice of Rebecca's recognition pursuant to the Order of Recognition
issued by Bureau Associate Commissioner Edgar L. Mendoza.

For clarity, we reproduce in full the contents of ID Certificate No. RC 9778:

To Whom It May Concern:

This is to certify that *MARIA REBECCA MAKAPUGAY BAYOT* whose photograph and thumbprints are
affixed hereto and partially covered by the seal of this Office, and whose other particulars are as follows:

Place of Birth: Guam, USA Date of Birth: March 5, 1953

Sex: female Civil Status: married Color of Hair: brown

Color of Eyes: brown Distinguishing marks on face: none

was - r e c o g n i z e d - as a citizen of the Philippines as per pursuant to Article IV, Section 1, Paragraph 3 of
the 1935 Constitution per order of Recognition JBL 95-213 signed by Associate Commissioner Jose B. Lopez
dated October 6, 1995, and duly affirmed by Secretary of Justice Artemio G. Tuquero in his 1 st Indorsement
dated June 8, 2000.

Issued for identification purposes only. NOT VALID for travel purposes.

Given under my hand and seal this 11th day of October, 1995

(SGD) EDGAR L. MENDOZA


ASSO. COMMISSIONER

Official Receipt No. 5939988


issued at Manila
dated Oct. 10, 1995 for P 2,000

From the text of ID Certificate No. RC 9778, the following material facts and dates may be deduced: (1) Bureau
Associate Commissioner Jose B. Lopez issued the Order of Recognition on October 6, 1995; (2) the 1st Indorsement
of Secretary of Justice Artemio G. Tuquero affirming Rebecca's recognition as a Filipino citizen was issued on June 8,
2000 or almost five years from the date of the order of recognition; and (3) ID Certificate No. RC 9778 was purportedly
issued on October 11, 1995 after the payment of the PhP 2,000 fee on October 10, 1995 per OR No. 5939988.
What begs the question is, however, how the above certificate could have been issued by the Bureau on October 11,
1995 when the Secretary of Justice issued the required affirmation only on June 8, 2000. No explanation was given for
this patent aberration. There seems to be no error with the date of the issuance of the 1 st Indorsement by Secretary of
Justice Tuquero as this Court takes judicial notice that he was the Secretary of Justice from February 16, 2000 to
January 22, 2001. There is, thus, a strong valid reason to conclude that the certificate in question must be spurious.

Under extant immigration rules, applications for recognition of Filipino citizenship require the affirmation by the DOJ of
the Order of Recognition issued by the Bureau. Under Executive Order No. 292, also known as the 1987
Administrative Code, specifically in its Title III, Chapter 1, Sec. 3(6), it is the DOJ which is tasked to "provide
immigration and naturalization regulatory services and implement the laws governing citizenship and the admission
and stay of aliens." Thus, the confirmation by the DOJ of any Order of Recognition for Filipino citizenship issued by the
Bureau is required.

Pertinently, Bureau Law Instruction No. RBR-99-00235 on Recognition as a Filipino Citizen clearly provides:

The Bureau [of Immigration] through its Records Section shall automatically furnish the Department of Justice
an official copy of its Order of Recognition within 72 days from its date of approval by the way of indorsement
for confirmation of the Order by the Secretary of Justice pursuant to Executive Order No. 292. No
Identification Certificate shall be issued before the date of confirmation by the Secretary of Justice and
any Identification Certificate issued by the Bureau pursuant to an Order of Recognition shall prominently
indicate thereon the date of confirmation by the Secretary of Justice. (Emphasis ours.)

Not lost on the Court is the acquisition by Rebecca of her Philippine passport only on June 13, 2000, or five days after
then Secretary of Justice Tuquero issued the 1 st Indorsement confirming the order of recognition. It may be too much to
attribute to coincidence this unusual sequence of close events which, to us, clearly suggests that prior to said
affirmation or confirmation, Rebecca was not yet recognized as a Filipino citizen. The same sequence would also imply
that ID Certificate No. RC 9778 could not have been issued in 1995, as Bureau Law Instruction No. RBR-99-002
mandates that no identification certificate shall be issued before the date of confirmation by the Secretary of Justice.
Logically, therefore, the affirmation or confirmation of Rebecca's recognition as a Filipino citizen through the
1st Indorsement issued only on June 8, 2000 by Secretary of Justice Tuquero corresponds to the eventual issuance of
Rebecca's passport a few days later, or on June 13, 2000 to be exact.

When Divorce Was Granted Rebecca, She Was not a


Filipino Citizen and Was not Yet Recognized as One

The Court can assume hypothetically that Rebecca is now a Filipino citizen. But from the foregoing disquisition, it is
indubitable that Rebecca did not have that status of, or at least was not yet recognized as, a Filipino citizen when she
secured the February 22, 1996 judgment of divorce from the Dominican Republic.

The Court notes and at this juncture wishes to point out that Rebecca voluntarily withdrew her original petition for
declaration of nullity (Civil Case No. 96-378 of the Makati City RTC) obviously because she could not show proof of her
alleged Filipino citizenship then. In fact, a perusal of that petition shows that, while bearing the date January 26, 1996,
it was only filed with the RTC on March 14, 1996 or less than a month after Rebecca secured, on February 22, 1996,
the foreign divorce decree in question. Consequently, there was no mention about said divorce in the petition.
Significantly, the only documents appended as annexes to said original petition were: the Vicente-Rebecca Marriage
Contract (Annex "A") and Birth Certificate of Alix (Annex "B"). If indeed ID Certificate No. RC 9778 from the Bureau
was truly issued on October 11, 1995, is it not but logical to expect that this piece of document be appended to form
part of the petition, the question of her citizenship being crucial to her case?

As may be noted, the petition for declaration of absolute nullity of marriage under Civil Case No. 01-094, like the
withdrawn first petition, also did not have the ID Certificate from the Bureau as attachment. What were attached
consisted of the following material documents: Marriage Contract (Annex "A") and Divorce Decree. It was only through
her Opposition (To Respondent's Motion to Dismiss dated 31 May 2001) 36 did Rebecca attach as Annex "C" ID
Certificate No. RC 9778.

At any rate, the CA was correct in holding that the RTC had sufficient basis to dismiss the petition for declaration of
absolute nullity of marriage as said petition, taken together with Vicente's motion to dismiss and Rebecca's opposition
to motion, with their respective attachments, clearly made out a case of lack of cause of action, which we will expound
later.

Validity of Divorce Decree

Going to the second core issue, we find Civil Decree Nos. 362/96 and 406/97 valid.

First, at the time of the divorce, as above elucidated, Rebecca was still to be recognized, assuming for argument that
she was in fact later recognized, as a Filipino citizen, but represented herself in public documents as an American
citizen. At the very least, she chose, before, during, and shortly after her divorce, her American citizenship to govern
her marital relationship. Second, she secured personally said divorce as an American citizen, as is evident in the text
of the Civil Decrees, which pertinently declared:
IN THIS ACTION FOR DIVORCE in which the parties expressly submit to the jurisdiction of this court, by
reason of the existing incompatibility of temperaments x x x. The parties MARIA REBECCA M. BAYOT, of
United States nationality, 42 years of age, married, domiciled and residing at 502 Acacia Ave., Ayala
Alabang, Muntin Lupa, Philippines, x x x, who personally appeared before this court, accompanied by DR.
JUAN ESTEBAN OLIVERO, attorney, x x x and VICENTE MADRIGAL BAYOT, of Philippine nationality, of 43
years of age, married and domiciled and residing at 502 Acacia Ave., Ayala Alabang, Muntin Lupa, Filipino,
appeared before this court represented by DR. ALEJANDRO TORRENS, attorney, x x x, revalidated by special
power of attorney given the 19th of February of 1996, signed before the Notary Public Enrico L. Espanol of the
City of Manila, duly legalized and authorizing him to subscribe all the acts concerning this case. 37 (Emphasis
ours.)

Third, being an American citizen, Rebecca was bound by the national laws of the United States of America, a country
which allows divorce. Fourth, the property relations of Vicente and Rebecca were properly adjudicated through their
Agreement38 executed on December 14, 1996 after Civil Decree No. 362/96 was rendered on February 22, 1996, and
duly affirmed by Civil Decree No. 406/97 issued on March 4, 1997. Veritably, the foreign divorce secured by Rebecca
was valid.

To be sure, the Court has taken stock of the holding in Garcia v. Recio that a foreign divorce can be recognized here,
provided the divorce decree is proven as a fact and as valid under the national law of the alien spouse. 39 Be this as it
may, the fact that Rebecca was clearly an American citizen when she secured the divorce and that divorce is
recognized and allowed in any of the States of the Union,40 the presentation of a copy of foreign divorce decree duly
authenticated by the foreign court issuing said decree is, as here, sufficient.

It bears to stress that the existence of the divorce decree has not been denied, but in fact admitted by both parties. And
neither did they impeach the jurisdiction of the divorce court nor challenge the validity of its proceedings on the ground
of collusion, fraud, or clear mistake of fact or law, albeit both appeared to have the opportunity to do so. The same
holds true with respect to the decree of partition of their conjugal property. As this Court explained in Roehr v.
Rodriguez:

Before our courts can give the effect of res judicata to a foreign judgment [of divorce] x x x, it must be shown
that the parties opposed to the judgment had been given ample opportunity to do so on grounds allowed under
Rule 39, Section 50 of the Rules of Court (now Rule 39, Section 48, 1997 Rules of Civil Procedure), to wit:

SEC. 50. Effect of foreign judgments.--The effect of a judgment of a tribunal of a foreign country,
having jurisdiction to pronounce the judgment is as follows:

(a) In case of a judgment upon a specific thing, the judgment is conclusive upon the title to the thing;

(b) In case of a judgment against a person, the judgment is presumptive evidence of a right as
between the parties and their successors in interest by a subsequent title; but the judgment may be
repelled by evidence of a want of jurisdiction, want of notice to the party, collusion, fraud, or clear
mistake of law or fact.

It is essential that there should be an opportunity to challenge the foreign judgment, in order for the court in this
jurisdiction to properly determine its efficacy. In this jurisdiction, our Rules of Court clearly provide that with
respect to actions in personam, as distinguished from actions in rem, a foreign judgment |merely
constitutes prima facie evidence of the justness of the claim of a party and, as such, is subject to proof to the
contrary.41

As the records show, Rebecca, assisted by counsel, personally secured the foreign divorce while Vicente was duly
represented by his counsel, a certain Dr. Alejandro Torrens, in said proceedings. As things stand, the foreign divorce
decrees rendered and issued by the Dominican Republic court are valid and, consequently, bind both Rebecca and
Vicente.

Finally, the fact that Rebecca may have been duly recognized as a Filipino citizen by force of the June 8, 2000
affirmation by Secretary of Justice Tuquero of the October 6, 1995 Bureau Order of Recognition will not, standing
alone, work to nullify or invalidate the foreign divorce secured by Rebecca as an American citizen on February 22,
1996. For as we stressed at the outset, in determining whether or not a divorce secured abroad would come within the
pale of the country's policy against absolute divorce, the reckoning point is the citizenship of the parties at the time a
valid divorce is obtained.42

Legal Effects of the Valid Divorce

Given the validity and efficacy of divorce secured by Rebecca, the same shall be given a res judicata effect in this
jurisdiction. As an obvious result of the divorce decree obtained, the marital vinculum between Rebecca and Vicente is
considered severed; they are both freed from the bond of matrimony. In plain language, Vicente and Rebecca are no
longer husband and wife to each other. As the divorce court formally pronounced: "[T]hat the marriage between MARIA
REBECCA M. BAYOT and VICENTE MADRIGAL BAYOT is hereby dissolved x x x leaving them free to remarry
after completing the legal requirements."43
Consequent to the dissolution of the marriage, Vicente could no longer be subject to a husband's obligation under the
Civil Code. He cannot, for instance, be obliged to live with, observe respect and fidelity, and render support to
Rebecca.44

The divorce decree in question also brings into play the second paragraph of Art. 26 of the Family Code, providing as
follows:

Art. 26. x x x x

Where a marriage between a Filipino citizen and a foreigner is validly celebrated and a divorce is thereafter
validly obtained abroad by the alien spouse capacitating him or her to remarry, the Filipino spouse shall
likewise have capacity to remarry under Philippine law. (As amended by E.O. 227)

In Republic v. Orbecido III, we spelled out the twin elements for the applicability of the second paragraph of Art. 26,
thus:

x x x [W]e state the twin elements for the application of Paragraph 2 of Article 26 as follows:

1. There is a valid marriage that has been celebrated between a Filipino citizen and a foreigner; and

2. A valid divorce is obtained abroad by the alien spouse capacitating him or her to remarry.

The reckoning point is not the citizenship of the parties at the time of the celebration of the marriage, but their
citizenship at the time a valid divorce is obtained abroad by the alien spouse capacitating the latter to
remarry.45

Both elements obtain in the instant case. We need not belabor further the fact of marriage of Vicente and Rebecca,
their citizenship when they wed, and their professed citizenship during the valid divorce proceedings.

Not to be overlooked of course is the fact that Civil Decree No. 406/97 and the Agreement executed on December 14,
1996 bind both Rebecca and Vicente as regards their property relations. The Agreement provided that the ex-couple's
conjugal property consisted only their family home, thus:

9. That the parties stipulate that the conjugal property which they acquired during their marriage
consists only of the real property and all the improvements and personal properties therein contained at 502
Acacia Avenue, Ayala Alabang, Muntinlupa, covered by TCT No. 168301 dated Feb. 7, 1990 issued by the
Register of Deeds of Makati, Metro Manila registered in the name of Vicente M. Bayot, married to Rebecca M.
Bayot, x x x.46 (Emphasis ours.)

This property settlement embodied in the Agreement was affirmed by the divorce court which, per its second divorce
decree, Civil Decree No. 406/97 dated March 4, 1997, ordered that, "THIRD: That the agreement entered into between
the parties dated 14th day of December 1996 in Makati City, Philippines shall survive in this Judgment of divorce by
reference but not merged and that the parties are hereby ordered and directed to comply with each and every
provision of said agreement."47

Rebecca has not repudiated the property settlement contained in the Agreement. She is thus estopped by her
representation before the divorce court from asserting that her and Vicente's conjugal property was not limited to their
family home in Ayala Alabang.48

No Cause of Action in the Petition for Nullity of Marriage

Upon the foregoing disquisitions, it is abundantly clear to the Court that Rebecca lacks, under the premises, cause of
action. Philippine Bank of Communications v. Trazo explains the concept and elements of a cause of action, thus:

A cause of action is an act or omission of one party in violation of the legal right of the other. A motion to
dismiss based on lack of cause of action hypothetically admits the truth of the allegations in the complaint. The
allegations in a complaint are sufficient to constitute a cause of action against the defendants if, hypothetically
admitting the facts alleged, the court can render a valid judgment upon the same in accordance with the prayer
therein. A cause of action exists if the following elements are present, namely: (1) a right in favor of the
plaintiff by whatever means and under whatever law it arises or is created; (2) an obligation on the part of the
named defendant to respect or not to violate such right; and (3) an act or omission on the part of such
defendant violative of the right of the plaintiff or constituting a breach of the obligation of the defendant to the
plaintiff for which the latter may maintain an action for recovery of damages. 49

One thing is clear from a perusal of Rebecca's underlying petition before the RTC, Vicente's motion to dismiss and
Rebecca's opposition thereof, with the documentary evidence attached therein: The petitioner lacks a cause of action
for declaration of nullity of marriage, a suit which presupposes the existence of a marriage.
To sustain a motion to dismiss for lack of cause of action, the movant must show that the claim for relief does not exist
rather than that a claim has been defectively stated or is ambiguous, indefinite, or uncertain. 50 With the valid foreign
divorce secured by Rebecca, there is no more marital tie binding her to Vicente. There is in fine no more marriage to
be dissolved or nullified.

The Court to be sure does not lose sight of the legal obligation of Vicente and Rebecca to support the needs of their
daughter, Alix. The records do not clearly show how he had discharged his duty, albeit Rebecca alleged that the
support given had been insufficient. At any rate, we do note that Alix, having been born on November 27, 1982,
reached the majority age on November 27, 2000, or four months before her mother initiated her petition for declaration
of nullity. She would now be 26 years old. Hence, the issue of back support, which allegedly had been partly
shouldered by Rebecca, is best litigated in a separate civil action for reimbursement. In this way, the actual figure for
the support of Alix can be proved as well as the earning capacity of both Vicente and Rebecca. The trial court can thus
determine what Vicente owes, if any, considering that support includes provisions until the child concerned shall have
finished her education.

Upon the foregoing considerations, the Court no longer need to delve into the issue tendered in G.R. No. 155635, that
is, Rebecca's right to support pendente lite. As it were, her entitlement to that kind of support hinges on the tenability of
her petition under Civil Case No. 01-094 for declaration of nullity of marriage. The dismissal of Civil Case No. 01-094
by the CA veritably removed any legal anchorage for, and effectively mooted, the claim for support pendente lite.

WHEREFORE, the petition for certiorari in G.R. No. 155635 is hereby DISMISSED on the ground of mootness, while
the petition for review in G.R. No. 163979 is hereby DENIED for lack of merit. Accordingly, the March 25, 2004
Decision and June 4, 2004 Resolution of the CA in CA-G.R. SP No. 68187 are hereby AFFIRMED. Costs against
petitioner.

SO ORDERED.

PRESBITERO J. VELASCO, JR.


Associate Justice

WE CONCUR:

LEONARDO A. QUISUMBING
Associate Justice
Chairperson

CONCHITA CARPIO MORALES DANTE O. TINGA


Associate Justice Associate Justice

ARTURO D. BRION
Associate Justice

ATTESTATION

I attest that the conclusions in the above Decision had been reached in consultation before the case was assigned to
the writer of the opinion of the Court's Division.

LEONARDO A. QUISUMBING
Associate Justice
Chairperson

CERTIFICATION

Pursuant to Section 13, Article VIII of the Constitution, and the Division Chairperson's Attestation, it is hereby certified
that the conclusions in the above Decision were reached in consultation before the case was assigned to the writer of
the opinion of the Court's Division.
REYNATO S. PUNO
Chief Justice

Footnotes

1
Rollo (G.R. No. 155635), pp. 3-34.

2
Id. at 36-38. Penned by Associate, now Presiding, Justice Conrado M. Vasquez, Jr. and concurred in by
Associate Justices Andres B. Reyes, Jr. and Mario L. Guariña III.

3
Id. at 40-41.

4
Rollo (G.R. No. 163979), pp. 10-43.

5
Id. at 575-583.

6
Id. at 145.

7
See Certification of Birth from the Government of Guam issued on June 1, 2000; rollo (G.R. No. 155635), p.
213.

8
Rollo (G.R. No. 163979), pp. 146-150.

9
Id. at 214-217.

10
Rollo (G.R. No. 155635), pp. 151-158.

11
Id. at 154.

12
Rollo (G.R. No. 163979), pp. 206-212.

13
Id. at 305-306. Per a motion to withdraw dated November 8, 1996.

14
Id. at 213. Per Order of Judge Josefina Guevara Salonga dated November 14, 1996.

15
Id. at 236-237.

16
Id. at 126-144.

17
Id. at 156-204.

18
Id. at 123-124. Penned by Presiding Judge Alberto L. Lerma.

19
Id. at 338.

20
Id. at 125. Per Order dated November 20, 2001.

27
G.R. No. 137898, December 15, 2000, 348 SCRA 401, 409.

28
Enumerated in San Lorenzo Village Association, Inc. v. Court of Appeals, G.R. No. 116825 March 26, 1998,
288 SCRA 115, 125: (1) the legal right of the plaintiff, (2) the correlative obligation of the defendant, and (3) the
act or omission of the defendant in violation of said legal right.
ARTICLE 27-34
Republic of the Philippines
SUPREME COURT
Manila

THIRD DIVISION

G.R. No. 175581 March 28, 2008

REPUBLIC OF THE PHILIPPINES, Petitioner,


vs.
JOSE A. DAYOT, Respondent.

x - - - - - - - - - - - - - - - - - - - - - - -x

G.R. No. 179474

FELISA TECSON-DAYOT, Petitioner,


vs.
JOSE A. DAYOT, Respondent.

DECISION

CHICO-NAZARIO, J.:

Before us are two consolidated petitions. G.R. No. 175581 and G.R. No. 179474 are Petitions for Review under
Rule 45 of the Rules of Court filed by the Republic of the Philippines and Felisa Tecson-Dayot (Felisa),
respectively, both challenging the Amended Decision1 of the Court of Appeals, dated 7 November 2006, in CA-
G.R. CV No. 68759, which declared the marriage between Jose Dayot (Jose) and Felisa void ab initio.

The records disclose that on 24 November 1986, Jose and Felisa were married at the Pasay City Hall. The
marriage was solemnized by Rev. Tomas V. Atienza. 2 In lieu of a marriage license, Jose and Felisa executed a
sworn affidavit,3 also dated 24 November 1986, attesting that both of them had attained the age of maturity, and
that being unmarried, they had lived together as husband and wife for at least five years.

On 7 July 1993, Jose filed a Complaint4 for Annulment and/or Declaration of Nullity of Marriage with the Regional
Trial Court (RTC), Biñan, Laguna, Branch 25. He contended that his marriage with Felisa was a sham, as no
marriage ceremony was celebrated between the parties; that he did not execute the sworn affidavit stating that
he and Felisa had lived as husband and wife for at least five years; and that his consent to the marriage was
secured through fraud.

In his Complaint, Jose gave his version of the events which led to his filing of the same. According to Jose, he
was introduced to Felisa in 1986. Immediately thereafter, he came to live as a boarder in Felisa’s house, the
latter being his landlady. Some three weeks later, Felisa requested him to accompany her to the Pasay City Hall,
ostensibly so she could claim a package sent to her by her brother from Saudi Arabia. At the Pasay City Hall,
upon a pre-arranged signal from Felisa, a man bearing three folded pieces of paper approached them. They were
told that Jose needed to sign the papers so that the package could be released to Felisa. He initially refused to
do so. However, Felisa cajoled him, and told him that his refusal could get both of them killed by her brother who
had learned about their relationship. Reluctantly, he signed the pieces of paper, and gave them to the man who
immediately left. It was in February 1987 when he discovered that he had contracted marriage with Felisa. He
alleged that he saw a piece of paper lying on top of the table at the sala of Felisa’s house. When he perused the
same, he discovered that it was a copy of his marriage contract with Felisa. When he confronted Felisa, the latter
feigned ignorance.

In opposing the Complaint, Felisa denied Jose’s allegations and defended the validity of their marriage. She
declared that they had maintained their relationship as man and wife absent the legality of marriage in the early
part of 1980, but that she had deferred contracting marriage with him on account of their age difference. 5 In her
pre-trial brief, Felisa expounded that while her marriage to Jose was subsisting, the latter contracted marriage
with a certain Rufina Pascual (Rufina) on 31 August 1990. On 3 June 1993, Felisa filed an action for bigamy
against Jose. Subsequently, she filed an administrative complaint against Jose with the Office of the
Ombudsman, since Jose and Rufina were both employees of the National Statistics and Coordinating
Board.6 The Ombudsman found Jose administratively liable for disgraceful and immoral conduct, and meted out
to him the penalty of suspension from service for one year without emolument. 7

On 26 July 2000, the RTC rendered a Decision8 dismissing the Complaint. It disposed:
WHEREFORE, after a careful evaluation and analysis of the evidence presented by both parties, this Court finds
and so holds that the [C]omplaint does not deserve a favorable consideration. Accordingly, the above-entitled
case is hereby ordered DISMISSED with costs against [Jose].9

The RTC ruled that from the testimonies and evidence presented, the marriage celebrated between Jose and
Felisa on 24 November 1986 was valid. It dismissed Jose’s version of the story as implausible, and rationalized
that:

Any person in his right frame of mind would easily suspect any attempt to make him or her sign a blank sheet of
paper. [Jose] could have already detected that something was amiss, unusual, as they were at Pasay City Hall to
get a package for [Felisa] but it [was] he who was made to sign the pieces of paper for the release of the said
package. Another indirect suggestion that could have put him on guard was the fact that, by his own admission,
[Felisa] told him that her brother would kill them if he will not sign the papers. And yet it took him, more or less,
three months to "discover" that the pieces of paper that he signed was [sic] purportedly the marriage contract.
[Jose] does not seem to be that ignorant, as perceived by this Court, to be "taken in for a ride" by [Felisa.]

[Jose’s] claim that he did not consent to the marriage was belied by the fact that he acknowledged Felisa Tecson
as his wife when he wrote [Felisa’s] name in the duly notarized statement of assets and liabilities he filled up on
May 12, 1988, one year after he discovered the marriage contract he is now claiming to be sham and false.
[Jose], again, in his company I.D., wrote the name of [Felisa] as the person to be contacted in case of
emergency. This Court does not believe that the only reason why her name was written in his company I.D. was
because he was residing there then. This is just but a lame excuse because if he really considers her not his
lawfully wedded wife, he would have written instead the name of his sister.

When [Jose’s] sister was put into the witness stand, under oath, she testified that she signed her name voluntarily
as a witness to the marriage in the marriage certificate (T.S.N., page 25, November 29, 1996) and she further
testified that the signature appearing over the name of Jose Dayot was the signature of his [sic] brother that he
voluntarily affixed in the marriage contract (page 26 of T.S.N. taken on November 29, 1996), and when she was
asked by the Honorable Court if indeed she believed that Felisa Tecson was really chosen by her brother she
answered yes. The testimony of his sister all the more belied his claim that his consent was procured through
fraud.10

Moreover, on the matter of fraud, the RTC ruled that Jose’s action had prescribed. It cited Article 87 11 of the New
Civil Code which requires that the action for annulment of marriage must be commenced by the injured party
within four years after the discovery of the fraud. Thus:

That granting even for the sake of argument that his consent was obtained by [Felisa] through fraud, trickery and
machinations, he could have filed an annulment or declaration of nullity of marriage at the earliest possible
opportunity, the time when he discovered the alleged sham and false marriage contract. [Jose] did not take any
action to void the marriage at the earliest instance. x x x. 12

Undeterred, Jose filed an appeal from the foregoing RTC Decision to the Court of Appeals. In a Decision dated
11 August 2005, the Court of Appeals found the appeal to be without merit. The dispositive portion of the
appellate court’s Decision reads:

WHEREFORE, the Decision appealed from is AFFIRMED.13

The Court of Appeals applied the Civil Code to the marriage between Jose and Felisa as it was solemnized prior
to the effectivity of the Family Code. The appellate court observed that the circumstances constituting fraud as a
ground for annulment of marriage under Article 8614 of the Civil Code did not exist in the marriage between the
parties. Further, it ruled that the action for annulment of marriage on the ground of fraud was filed beyond the
prescriptive period provided by law. The Court of Appeals struck down Jose’s appeal in the following manner:

Nonetheless, even if we consider that fraud or intimidation was employed on Jose in giving his consent to the
marriage, the action for the annulment thereof had already prescribed. Article 87 (4) and (5) of the Civil Code
provides that the action for annulment of marriage on the ground that the consent of a party was obtained by
fraud, force or intimidation must be commenced by said party within four (4) years after the discovery of the fraud
and within four (4) years from the time the force or intimidation ceased. Inasmuch as the fraud was allegedly
discovered by Jose in February, 1987 then he had only until February, 1991 within which to file an action for
annulment of marriage. However, it was only on July 7, 1993 that Jose filed the complaint for annulment of his
marriage to Felisa.15

Likewise, the Court of Appeals did not accept Jose’s assertion that his marriage to Felisa was void ab initio for
lack of a marriage license. It ruled that the marriage was solemnized under Article 76 16 of the Civil Code as one of
exceptional character, with the parties executing an affidavit of marriage between man and woman who have
lived together as husband and wife for at least five years. The Court of Appeals concluded that the falsity in the
affidavit to the effect that Jose and Felisa had lived together as husband and wife for the period required by
Article 76 did not affect the validity of the marriage, seeing that the solemnizing officer was misled by the
statements contained therein. In this manner, the Court of Appeals gave credence to the good-faith reliance of
the solemnizing officer over the falsity of the affidavit. The appellate court further noted that on the dorsal side of
said affidavit of marriage, Rev. Tomas V. Atienza, the solemnizing officer, stated that he took steps to ascertain
the ages and other qualifications of the contracting parties and found no legal impediment to their marriage.
Finally, the Court of Appeals dismissed Jose’s argument that neither he nor Felisa was a member of the sect to
which Rev. Tomas V. Atienza belonged. According to the Court of Appeals, Article 5617 of the Civil Code did not
require that either one of the contracting parties to the marriage must belong to the solemnizing officer’s church
or religious sect. The prescription was established only in Article 7 18 of the Family Code which does not govern
the parties’ marriage.

Differing with the ruling of the Court of Appeals, Jose filed a Motion for Reconsideration thereof. His central
1avvphi1

opposition was that the requisites for the proper application of the exemption from a marriage license under
Article 76 of the Civil Code were not fully attendant in the case at bar. In particular, Jose cited the legal condition
that the man and the woman must have been living together as husband and wife for at least five years before
the marriage. Essentially, he maintained that the affidavit of marital cohabitation executed by him and Felisa was
false.

The Court of Appeals granted Jose’s Motion for Reconsideration and reversed itself. Accordingly, it rendered an
Amended Decision, dated 7 November 2006, the fallo of which reads:

WHEREFORE, the Decision dated August 11, 2005 is RECALLED and SET ASIDE and another one entered
declaring the marriage between Jose A. Dayot and Felisa C. Tecson void ab initio.

Furnish a copy of this Amended Decision to the Local Civil Registrar of Pasay City. 19

In its Amended Decision, the Court of Appeals relied on the ruling of this Court in Niñal v. Bayadog, 20 and
reasoned that:

In Niñal v. Bayadog, where the contracting parties to a marriage solemnized without a marriage license on the
basis of their affidavit that they had attained the age of majority, that being unmarried, they had lived together for
at least five (5) years and that they desired to marry each other, the Supreme Court ruled as follows:

"x x x In other words, the five-year common-law cohabitation period, which is counted back from the date of
celebration of marriage, should be a period of legal union had it not been for the absence of the marriage. This 5-
year period should be the years immediately before the day of the marriage and it should be a period of
cohabitation characterized by exclusivity – meaning no third party was involved at any time within the 5 years and
continuity – that is unbroken. Otherwise, if that continuous 5-year cohabitation is computed without any distinction
as to whether the parties were capacitated to marry each other during the entire five years, then the law would be
sanctioning immorality and encouraging parties to have common law relationships and placing them on the same
footing with those who lived faithfully with their spouse. Marriage being a special relationship must be respected
as such and its requirements must be strictly observed. The presumption that a man and a woman deporting
themselves as husband and wife is based on the approximation of the requirements of the law. The parties
should not be afforded any excuse to not comply with every single requirement and later use the same missing
element as a pre-conceived escape ground to nullify their marriage. There should be no exemption from securing
a marriage license unless the circumstances clearly fall within the ambit of the exception. It should be noted that
a license is required in order to notify the public that two persons are about to be united in matrimony and that
anyone who is aware or has knowledge of any impediment to the union of the two shall make it known to the
local civil registrar.

Article 80(3) of the Civil Code provides that a marriage solemnized without a marriage license, save marriages of
exceptional character, shall be void from the beginning. Inasmuch as the marriage between Jose and Felisa is
not covered by the exception to the requirement of a marriage license, it is, therefore, void ab initio because of
the absence of a marriage license.21

Felisa sought reconsideration of the Amended Decision, but to no avail. The appellate court rendered a
Resolution22 dated 10 May 2007, denying Felisa’s motion.

Meanwhile, the Republic of the Philippines, through the Office of the Solicitor General (OSG), filed a Petition for
Review before this Court in G.R. No. 175581, praying that the Court of Appeals’ Amended Decision dated 7
November 2006 be reversed and set aside for lack of merit, and that the marriage between Jose and Felisa be
declared valid and subsisting. Felisa filed a separate Petition for Review, docketed as G.R. No. 179474, similarly
assailing the appellate court’s Amended Decision. On 1 August 2007, this Court resolved to consolidate the two
Petitions in the interest of uniformity of the Court rulings in similar cases brought before it for resolution. 23

The Republic of the Philippines propounds the following arguments for the allowance of its Petition, to wit:
I

RESPONDENT FAILED TO OVERTHROW THE PRESUMPTION OF THE VALIDITY OF HIS


MARRIAGE TO FELISA.

II

RESPONDENT DID NOT COME TO THE COURT WITH CLEAN HANDS AND SHOULD NOT BE
ALLOWED TO PROFIT FROM HIS OWN FRAUDULENT CONDUCT.

III

RESPONDENT IS ESTOPPED FROM ASSAILING THE LEGALITY OF HIS MARRIAGE FOR LACK OF
MARRIAGE LICEN[S]E.24

Correlative to the above, Felisa submits that the Court of Appeals misapplied Niñal. 25 She differentiates the case
at bar from Niñal by reasoning that one of the parties therein had an existing prior marriage, a circumstance
which does not obtain in her cohabitation with Jose. Finally, Felisa adduces that Jose only sought the annulment
of their marriage after a criminal case for bigamy and an administrative case had been filed against him in order
to avoid liability. Felisa surmises that the declaration of nullity of their marriage would exonerate Jose from any
liability.

For our resolution is the validity of the marriage between Jose and Felisa. To reach a considered ruling on the
issue, we shall jointly tackle the related arguments vented by petitioners Republic of the Philippines and Felisa.

The Republic of the Philippines asserts that several circumstances give rise to the presumption that a valid
marriage exists between Jose and Felisa. For her part, Felisa echoes the claim that any doubt should be
resolved in favor of the validity of the marriage by citing this Court’s ruling in Hernandez v. Court of Appeals. 26 To
buttress its assertion, the Republic points to the affidavit executed by Jose and Felisa, dated 24 November 1986,
attesting that they have lived together as husband and wife for at least five years, which they used in lieu of a
marriage license. It is the Republic’s position that the falsity of the statements in the affidavit does not affect the
validity of the marriage, as the essential and formal requisites were complied with; and the solemnizing officer
was not required to investigate as to whether the said affidavit was legally obtained. The Republic opines that as
a marriage under a license is not invalidated by the fact that the license was wrongfully obtained, so must a
marriage not be invalidated by the fact that the parties incorporated a fabricated statement in their affidavit that
they cohabited as husband and wife for at least five years. In addition, the Republic posits that the parties’
marriage contract states that their marriage was solemnized under Article 76 of the Civil Code. It also bears the
signature of the parties and their witnesses, and must be considered a primary evidence of marriage. To further
fortify its Petition, the Republic adduces the following documents: (1) Jose’s notarized Statement of Assets and
Liabilities, dated 12 May 1988 wherein he wrote Felisa’s name as his wife; (2) Certification dated 25 July 1993
issued by the Barangay Chairman 192, Zone ZZ, District 24 of Pasay City, attesting that Jose and Felisa had
lived together as husband and wife in said barangay; and (3) Jose’s company ID card, dated 2 May 1988,
indicating Felisa’s name as his wife.

The first assignment of error compels this Court to rule on the issue of the effect of a false affidavit under Article
76 of the Civil Code. A survey of the prevailing rules is in order.

It is beyond dispute that the marriage of Jose and Felisa was celebrated on 24 November 1986, prior to the
effectivity of the Family Code. Accordingly, the Civil Code governs their union. Article 53 of the Civil Code spells
out the essential requisites of marriage as a contract:

ART. 53. No marriage shall be solemnized unless all these requisites are complied with:

(1) Legal capacity of the contracting parties;

(2) Their consent, freely given;

(3) Authority of the person performing the marriage; and

(4) A marriage license, except in a marriage of exceptional character. (Emphasis ours.)

Article 5827 makes explicit that no marriage shall be solemnized without a license first being issued by the local
civil registrar of the municipality where either contracting party habitually resides, save marriages of an
exceptional character authorized by the Civil Code, but not those under Article 75. 28 Article 80(3)29 of the Civil
Code makes it clear that a marriage performed without the corresponding marriage license is void, this being
nothing more than the legitimate consequence flowing from the fact that the license is the essence of the
marriage contract.30 This is in stark contrast to the old Marriage Law, 31 whereby the absence of a marriage
license did not make the marriage void. The rationale for the compulsory character of a marriage license under
the Civil Code is that it is the authority granted by the State to the contracting parties, after the proper
government official has inquired into their capacity to contract marriage. 32

Under the Civil Code, marriages of exceptional character are covered by Chapter 2, Title III, comprising Articles
72 to 79. To wit, these marriages are: (1) marriages in articulo mortis or at the point of death during peace or war,
(2) marriages in remote places, (2) consular marriages,33 (3) ratification of marital cohabitation, (4) religious
ratification of a civil marriage, (5) Mohammedan or pagan marriages, and (6) mixed marriages. 34

The instant case pertains to a ratification of marital cohabitation under Article 76 of the Civil Code, which
provides:

ART. 76. No marriage license shall be necessary when a man and a woman who have attained the age of
majority and who, being unmarried, have lived together as husband and wife for at least five years, desire to
marry each other. The contracting parties shall state the foregoing facts in an affidavit before any person
authorized by law to administer oaths. The official, priest or minister who solemnized the marriage shall also state
in an affidavit that he took steps to ascertain the ages and other qualifications of the contracting parties and that
he found no legal impediment to the marriage.

The reason for the law,35 as espoused by the Code Commission, is that the publicity attending a marriage license
may discourage such persons who have lived in a state of cohabitation from legalizing their status.36

It is not contested herein that the marriage of Jose and Felisa was performed without a marriage license. In lieu
thereof, they executed an affidavit declaring that "they have attained the age of maturity; that being unmarried,
they have lived together as husband and wife for at least five years; and that because of this union, they desire to
marry each other."37 One of the central issues in the Petition at bar is thus: whether the falsity of an affidavit of
marital cohabitation, where the parties have in truth fallen short of the minimum five-year requirement, effectively
renders the marriage void ab initio for lack of a marriage license.

We answer in the affirmative.

Marriages of exceptional character are, doubtless, the exceptions to the rule on the indispensability of the formal
requisite of a marriage license. Under the rules of statutory construction, exceptions, as a general rule, should be
strictly38 but reasonably construed.39 They extend only so far as their language fairly warrants, and all doubts
should be resolved in favor of the general provisions rather than the exception. 40 Where a general rule is
established by statute with exceptions, the court will not curtail the former or add to the latter by implication. 41 For
the exception in Article 76 to apply, it is a sine qua non thereto that the man and the woman must have attained
the age of majority, and that, being unmarried, they have lived together as husband and wife for at least five
years.

A strict but reasonable construction of Article 76 leaves us with no other expediency but to read the law as it is
plainly written. The exception of a marriage license under Article 76 applies only to those who have lived together
as husband and wife for at least five years and desire to marry each other. The Civil Code, in no ambiguous
terms, places a minimum period requirement of five years of cohabitation. No other reading of the law can be
had, since the language of Article 76 is precise. The minimum requisite of five years of cohabitation is an
indispensability carved in the language of the law. For a marriage celebrated under Article 76 to be valid, this
material fact cannot be dispensed with. It is embodied in the law not as a directory requirement, but as one that
partakes of a mandatory character. It is worthy to mention that Article 76 also prescribes that the contracting
parties shall state the requisite facts42 in an affidavit before any person authorized by law to administer oaths; and
that the official, priest or minister who solemnized the marriage shall also state in an affidavit that he took steps to
ascertain the ages and other qualifications of the contracting parties and that he found no legal impediment to the
marriage.

It is indubitably established that Jose and Felisa have not lived together for five years at the time they executed
their sworn affidavit and contracted marriage. The Republic admitted that Jose and Felisa started living together
only in June 1986, or barely five months before the celebration of their marriage. 43 The Court of Appeals also
noted Felisa’s testimony that Jose was introduced to her by her neighbor, Teresita Perwel, sometime in February
or March 1986 after the EDSA Revolution.44 The appellate court also cited Felisa’s own testimony that it was only
in June 1986 when Jose commenced to live in her house. 45

Moreover, it is noteworthy that the question as to whether they satisfied the minimum five-year requisite is factual
in nature. A question of fact arises when there is a need to decide on the truth or falsehood of the alleged
facts.46 Under Rule 45, factual findings are ordinarily not subject to this Court’s review. 47 It is already well-settled
that:
The general rule is that the findings of facts of the Court of Appeals are binding on this Court. A recognized
exception to this rule is when the Court of Appeals and the trial court, or in this case the administrative body,
make contradictory findings. However, the exception does not apply in every instance that the Court of Appeals
and the trial court or administrative body disagree. The factual findings of the Court of Appeals remain conclusive
on this Court if such findings are supported by the record or based on substantial evidence. 48

Therefore, the falsity of the affidavit dated 24 November 1986, executed by Jose and Felisa to exempt them from
the requirement of a marriage license, is beyond question.

We cannot accept the insistence of the Republic that the falsity of the statements in the parties’ affidavit will not
affect the validity of marriage, since all the essential and formal requisites were complied with. The argument
deserves scant merit. Patently, it cannot be denied that the marriage between Jose and Felisa was celebrated
without the formal requisite of a marriage license. Neither did Jose and Felisa meet the explicit legal requirement
in Article 76, that they should have lived together as husband and wife for at least five years, so as to be
excepted from the requirement of a marriage license.

Anent petitioners’ reliance on the presumption of marriage, this Court holds that the same finds no applicability to
the case at bar. Essentially, when we speak of a presumption of marriage, it is with reference to the prima facie
presumption that a man and a woman deporting themselves as husband and wife have entered into a lawful
contract of marriage.49 Restated more explicitly, persons dwelling together in apparent matrimony are presumed,
in the absence of any counter-presumption or evidence special to the case, to be in fact married. 50 The present
case does not involve an apparent marriage to which the presumption still needs to be applied. There is no
question that Jose and Felisa actually entered into a contract of marriage on 24 November 1986, hence,
compelling Jose to institute a Complaint for Annulment and/or Declaration of Nullity of Marriage, which spawned
the instant consolidated Petitions.

In the same vein, the declaration of the Civil Code51 that every intendment of law or fact leans towards the validity
of marriage will not salvage the parties’ marriage, and extricate them from the effect of a violation of the law. The
marriage of Jose and Felisa was entered into without the requisite marriage license or compliance with the
stringent requirements of a marriage under exceptional circumstance. The solemnization of a marriage without
prior license is a clear violation of the law and would lead or could be used, at least, for the perpetration of fraud
against innocent and unwary parties, which was one of the evils that the law sought to prevent by making a prior
license a prerequisite for a valid marriage.52 The protection of marriage as a sacred institution requires not just
the defense of a true and genuine union but the exposure of an invalid one as well. 53 To permit a false affidavit to
take the place of a marriage license is to allow an abject circumvention of the law. If this Court is to protect the
fabric of the institution of marriage, we must be wary of deceptive schemes that violate the legal measures set
forth in our laws.

Similarly, we are not impressed by the ratiocination of the Republic that as a marriage under a license is not
invalidated by the fact that the license was wrongfully obtained, so must a marriage not be invalidated by a
fabricated statement that the parties have cohabited for at least five years as required by law. The contrast is
flagrant. The former is with reference to an irregularity of the marriage license, and not to the absence of one.
Here, there is no marriage license at all. Furthermore, the falsity of the allegation in the sworn affidavit relating to
the period of Jose and Felisa’s cohabitation, which would have qualified their marriage as an exception to the
requirement for a marriage license, cannot be a mere irregularity, for it refers to a quintessential fact that the law
precisely required to be deposed and attested to by the parties under oath. If the essential matter in the sworn
affidavit is a lie, then it is but a mere scrap of paper, without force and effect. Hence, it is as if there was no
affidavit at all.

In its second assignment of error, the Republic puts forth the argument that based on equity, Jose should be
denied relief because he perpetrated the fabrication, and cannot thereby profit from his wrongdoing. This is a
misplaced invocation. It must be stated that equity finds no room for application where there is a law. 54 There is a
law on the ratification of marital cohabitation, which is set in precise terms under Article 76 of the Civil Code.
Nonetheless, the authorities are consistent that the declaration of nullity of the parties’ marriage is without
prejudice to their criminal liability.55

The Republic further avers in its third assignment of error that Jose is deemed estopped from assailing the
legality of his marriage for lack of a marriage license. It is claimed that Jose and Felisa had lived together from
1986 to 1990, notwithstanding Jose’s subsequent marriage to Rufina Pascual on 31 August 1990, and that it took
Jose seven years before he sought the declaration of nullity; hence, estoppel had set in.

This is erroneous. An action for nullity of marriage is imprescriptible.56 Jose and Felisa’s marriage was celebrated
sans a marriage license. No other conclusion can be reached except that it is void ab initio. In this case, the right
to impugn a void marriage does not prescribe, and may be raised any time.

Lastly, to settle all doubts, jurisprudence has laid down the rule that the five-year common-law cohabitation
period under Article 76 means a five-year period computed back from the date of celebration of marriage, and
refers to a period of legal union had it not been for the absence of a marriage. 57 It covers the years immediately
preceding the day of the marriage, characterized by exclusivity - meaning no third party was involved at any time
within the five years - and continuity that is unbroken.58

WHEREFORE, the Petitions are DENIED. The Amended Decision of the Court of Appeals, dated 7 November
2006 in CA-G.R. CV No. 68759, declaring the marriage of Jose Dayot to Felisa Tecson-Dayot void ab initio, is
AFFIRMED, without prejudice to their criminal liability, if any. No costs.

SO ORDERED.

MINITA V. CHICO-NAZARIO
Associate Justice

WE CONCUR:

MA. ALICIA AUSTRIA-MARTINEZ


Associate Justice
Acting Chairperson

DANTE O. TINGA* PRESBITERO J. VELASCO, JR.**


Associate Justice Associate Justice

RUBEN T. REYES
Associate Justice

ATTESTATION

I attest that the conclusions in the above Decision were reached in consultation before the case was assigned to
the writer of the opinion of the Court’s Division.

MA. ALICIA AUSTRIA-MARTINEZ


Associate Justice
Acting Chairperson, Third Division

CERTIFICATION

Pursuant to Section 13, Article VIII of the Constitution, and the Division Acting Chairperson’s attestation, it is
hereby certified that the conclusions in the above Decision were reached in consultation before the case was
assigned to the writer of the opinion of the Court’s Division.

REYNATO S. PUNO
Chief Justice

Footnotes

* Per Special Order No. 497, dated 14 March 2008, signed by Chief Justice Reynato S. Puno designating
Associate Justice Dante O. Tinga to replace Associate Justice Consuelo Ynares-Santiago, who is on
official leave under the Court’s Wellness Program and assigning Associate Justice Alicia Austria-Martinez
as Acting Chairperson.

** Justice Presbitero J. Velasco, Jr. was designated to sit as additional member replacing Justice Antonio
Eduardo B. Nachura per Raffle dated 12 September 2007.

1Penned by Associate Justice Marina L. Buzon with Associate Justices Mario L. Guariña III and Santiago
Javier Ranada, concurring; rollo (G.R. No. 175581), pp. 65-70; rollo, (G.R. No. 179474), pp. 156-161.

2 Records, p. 170.

3 Id.

4 Id. at 1-8.
5The marriage contract shows that at the time of the celebration of the parties’ marriage, Jose was 27
years old, while Felisa was 37.

6The Administrative complaint before the Administrative Adjudication Bureau of the Office of the
Ombudsman was docketed as OMB-ADM-0-93-0466; Records, pp. 252-258.

7 Id. at 257.

8 Id. at 313-323.

9 Id. at 323.

10 Id. at 321-322.

11ART. 87. - The action for annulment of marriage must be commenced by the parties and within the
periods as follows:

(1) For causes mentioned in Number 1 of Article 85, by the party whose parent or guardian did
not give his or her consent, within four years after attaining the age of twenty or eighteen years,
as the case may be; or by the parent or guardian or person having legal charge, at any time
before such party has arrived at the age of twenty or eighteen years;

(2) For causes mentioned in Number 2 of Article 85, by the spouse who has been absent, during
his or her lifetime; or by either spouse of the subsequent marriage during the lifetime of the other;

(3) For causes mentioned in Number 3 of Article 85, by the sane spouse, who had no knowledge
of the other's insanity; or by any relative or guardian of the party of unsound mind, at any time
before the death of either party;

(4) For causes mentioned in Number 4, by the injured party, within four years after the discovery
of the fraud;

(5) For causes mentioned in Number 5, by the injured party, within four years from the time the
force or intimidation ceased;

(6) For causes mentioned in Number 6, by the injured party, within eight years after the marriage.

12 Records, p. 322.

13 Rollo (G.R. No. 179474), p. 125.

14ART. 86. Any of the following circumstances shall constitute fraud referred to in number 4 of the
preceding article:

(1) Misrepresentation as to the identity of one of the contracting parties;

(2) Nondisclosure of the previous conviction of the other party of a crime involving moral
turpitude, and the penalty imposed was imprisonment for two years or more;

(3) Concealment by the wife of the fact that at the time of the marriage, she was pregnant by a
man other than her husband;

No other misrepresentation or deceit as to character, rank, fortune or chastity shall constitute


such fraud as will give grounds for action for the annulment of marriage.

15 Rollo (G.R. No. 179474), p. 122.

16ART. 76. No marriage license shall be necessary when a man and a woman who have attained the age
of majority and who, being unmarried, have lived together as husband and wife for at least five years,
desire to marry each other. The contracting parties shall state the foregoing facts in an affidavit before
any person authorized by law to administer oaths. The official, priest or minister who solemnized the
marriage shall also state in an affidavit that he took steps to ascertain the ages and other qualifications of
the contracting parties and that he found no legal impediment to the marriage.

17 ART. 56. Marriage may be solemnized by:


(1) The Chief Justice and Associate Justices of the Supreme Court;

(2) The Presiding Justice and the Justices of the Court of Appeals;

(3) Judges of the Courts of First Instance;

(4) Mayors of cities and municipalities;

(5) Municipal judges and justices of the peace;

(6) Priests, rabbis, ministers of the gospel of any denomination, church, religion or sect, duly
registered, as provided in Article 92; and

(7) Ship captains, airplane chiefs, military commanders, and consuls and vice-consuls in special
cases provided in Articles 74 and 75.

18 ART. 7. Marriage may be solemnized by:

(1) Any incumbent member of the judiciary within the court’s jurisdiction;

(2) Any priest, rabbi, imam, or minister of any church or religious sect duly authorized by his
church or religious sect and registered with the civil registrar general, acting within the limits of the
written authority granted him by his church or religious sect and provided that at least one of the
contracting parties belongs to the solemnizing officer's church or religious sect;

(3) Any ship captain or airplane chief only in the cases mentioned in Article 31;

(4) Any military commander of a unit to which a chaplain is assigned, in the absence of the latter,
during a military operation, likewise only in the cases mentioned in Article 32; or

(5) Any consul-general, consul or vice-consul in the case provided in Article 10.

19 CA rollo, p. 279.

20 384 Phil. 661 (2000).

21 CA rollo, pp. 278-279.

22 Rollo (G.R. No. 179474), pp. 173-174.

23 Rollo (G.R. No. 179474), p. 180.

24 Rollo (G.R. No. 175581), pp. 44-45.

25 Erroneously cited as Niño v. Bayadog; rollo (G.R. No. 179474), p. 18.

26 377 Phil. 919 (1999).

27ART. 58. Save marriages of an exceptional character authorized in Chapter 2 of this Title, but not those
under Article 75, no marriage shall be solemnized without a license first being issued by the local civil
registrar of the municipality where either contracting party habitually resides.

28ART. 75. Marriages between Filipino citizens abroad may be solemnized by consuls and vice-consuls
of the Republic of the Philippines. The duties of the local civil registrar and of a judge or justice of the
peace or mayor with regard to the celebration of marriage shall be performed by such consuls and vice-
consuls.

29 ART. 80. The following marriages shall be void from the beginning:

xxxx

(3) Those solemnized without a marriage license, save marriages of exceptional character.

30 People v. De Lara, No. 12583-R, 14 February 1955, 51 O.G. 4079, 4082.


31The Marriage Law, otherwise known as Act No. 3613, requires the following essential requisites: (1)
legal capacity of the contracting parties; and (2) their mutual consent.

Report of the Code Commission, pp. 79-80; see also Ambrosio Padilla, Civil Code Annotated, 1956
32

Edition, Vol. I, p. 195.

33 Must be read with Article 58 of the Civil Code which provides:

ART. 58. Save marriages of an exceptional character authorized in Chapter 2 of this Title, but not
those under Article 75, no marriage shall be solemnized without a license first being issued by the
local civil registrar of the municipality where either contracting party habitually resides.

34 Edgardo L. Paras, Civil Code of the Philippines Annotated (1984 Eleventh Ed.), pp. 302-310.

35In Niñal v. Bayadog (supra note 20 at 668-669), this Court articulated the spirit behind Article 76 of the
Civil Code, thus:

"However, there are several instances recognized by the Civil Code wherein a marriage license is
dispensed with, one of which is that provided in Article 76, referring to the marriage of a man and
a woman who have lived together and exclusively with each other as husband and wife for a
continuous and unbroken period of at least five years before the marriage. The rationale why no
license is required in such case is to avoid exposing the parties to humiliation, shame and
embarrassment concomitant with the scandalous cohabitation of persons outside a valid marriage
due to the publication of every applicant’s name for a marriage license. The publicity attending the
marriage license may discourage such persons from legitimizing their status. To preserve peace
in the family, avoid the peeping and suspicious eye of public exposure and contain the source of
gossip arising from the publication of their names, the law deemed it wise to preserve their
privacy and exempt them from that requirement."

36The Report of the Code Commission states that "No marriage license shall be necessary when a man
and a woman who have attained the age of majority and who, being unmarried, have lived together as
husband and wife for at least five years desire to marry each other. In such case, the publicity attending a
marriage license may discourage such persons from legalizing their status," Report of the Code
Commission, p. 80.

Records, p. 49. The affidavit was denominated by the parties as an "Affidavit on (sic) Marriage Between
37

Man and Woman Who Haved (sic) Lived Together as Husband and Wife for at Least Five Years."

38 Benedicto v. Court of Appeals, 416 Phil. 722, 744 (2001).

39 Commissioner of Internal Revenue v. Court of Appeals, 363 Phil. 130, 137 (1999).

40 Id.

41 Id. citing Samson v. Court of Appeals, G.R. No. L-43182, 25 November 1986, 145 SCRA 654, 659.

42The first part of Article 76 states, "No marriage license shall be necessary when a man and a woman
who have attained the age of majority and who, being unmarried, have lived together as husband and
wife for at least five years, desire to marry each other x x x."

43 Rollo (G.R. No. 175581), p. 38.

44 Rollo (G.R. No. 179474), p. 158, citing TSN (Civil Case No. B-4143), 15 April 1999.

45 Id. at 159.

46First Dominion Resources Corporation v. Peñaranda, G.R. No. 166616, 27 January 2006, 480 SCRA
504, 508.

47 Civil Service Commission v. Ledesma, G.R. No. 154521, 30 September 2005, 471 SCRA 589, 605.

48 Id.

49 Vda. de Jacob v. Court of Appeals, 371 Phil. 693, 708 (1999).


50 Id.

51ART. 220. In case of doubt, all presumptions favor the solidarity of the family. Thus, every intendment
of law or fact leans toward the validity of marriage, the indissolubility of the marriage bonds, the
legitimacy of children, the community of property during marriage, the authority of parents over their
children, and the validity of defense for any member of the family in case of unlawful aggression.

52 People v. De Lara, supra note 30 at 4083.

53 Malcampo-Sin v. Sin, 407 Phil. 583, 588 (2001).

Salavarria v. Letran College, 357 Phil. 189, 196 (1998); Aparente, Sr. v. National Labor Relations
54

Commission, 387 Phil. 96, 108 (2000).

55 Supra note 33 at 306. Alicia V. Sempio-Diy in A Handbook on the Family Code of the Philippines (1995
Ed., p. 38) wrote that "If the parties falsify their affidavit in order to have an instant marriage, although the
truth is that they have not been cohabiting for five years, their marriage will be void for lack of a marriage
license, and they will also be criminally liable." Article 76 of the Civil Code is now Article 34 of the Family
Code, which reads:

ART. 34. No license shall be necessary for the marriage of a man and a woman who have lived
together as husband and wife for at least five years and without any legal impediment to marry
each other. The contracting parties shall state the foregoing facts in an affidavit before any person
authorized by law to administer oaths. The solemnizing officer shall also state under oath that he
ascertained the qualifications of the contracting parties and found no legal impediment to the
marriage.

56 Niñal v. Bayadog, supra note 20 at 134.

57 Id. at 130-131.

58 Id.
Niñal vs. Badayog GR No. 133778

FIRST DIVISION

G.R. No. 133778 March 14, 2000

ENGRACE NIÑAL for Herself and as Guardian ad Litem of the minors BABYLINE NIÑAL, INGRID NIÑAL,
ARCHIE NIÑAL & PEPITO NIÑAL, JR., petitioners,
vs.
NORMA BAYADOG, respondent.

YNARES-SANTIAGO, J.:

May the heirs of a deceased person file a petition for the declaration of nullity of his marriage after his death?

Pepito Niñal was married to Teodulfa Bellones on September 26, 1974. Out of their marriage were born herein
petitioners. Teodulfa was shot by Pepito resulting in her death on April 24, 1985. One year and 8 months
thereafter or on December 11, 1986, Pepito and respondent Norma Badayog got married without any marriage
license. In lieu thereof, Pepito and Norma executed an affidavit dated December 11, 1986 stating that they had
lived together as husband and wife for at least five years and were thus exempt from securing a marriage license.
On February 19, 1997, Pepito died in a car accident. After their father's death, petitioners filed a petition for
declaration of nullity of the marriage of Pepito to Norma alleging that the said marriage was void for lack of a
marriage license. The case was filed under the assumption that the validity or invalidity of the second marriage
would affect petitioner's successional rights. Norma filed a motion to dismiss on the ground that petitioners have
no cause of action since they are not among the persons who could file an action for "annulment of marriage"
under Article 47 of the Family Code.

Judge Ferdinand J. Marcos of the Regional Trial Court of Toledo City, Cebu, Branch 59, dismissed the petition
after finding that the Family Code is "rather silent, obscure, insufficient" to resolve the following issues:

(1) Whether or not plaintiffs have a cause of action against defendant in asking for the declaration of the
nullity of marriage of their deceased father, Pepito G. Niñal, with her specially so when at the time of the
filing of this instant suit, their father Pepito G. Niñal is already dead;

(2) Whether or not the second marriage of plaintiffs' deceased father with defendant is null and void ab
initio;

(3) Whether or not plaintiffs are estopped from assailing the validity of the second marriage after it was
dissolved due to their father's death. 1

Thus, the lower court ruled that petitioners should have filed the action to declare null and void their father's
marriage to respondent before his death, applying by analogy Article 47 of the Family Code which enumerates
the time and the persons who could initiate an action for annulment of marriage. 2 Hence, this petition for review
with this Court grounded on a pure question of law.

This petition was originally dismissed for non-compliance with Section 11, Rule 13 of the 1997 Rules of Civil
Procedure, and because "the verification failed to state the basis of petitioner's averment that the allegations in
the petition are "true and correct"." It was thus treated as an unsigned pleading which produces no legal effect
under Section 3, Rule 7, of the 1997 Rules. 3 However, upon motion of petitioners, this Court reconsidered the
dismissal and reinstated the petition for review. 4

The two marriages involved herein having been solemnized prior to the effectivity of the Family Code (FC), the
applicable law to determine their validity is the Civil Code which was the law in effect at the time of their
celebration. 5 A valid marriage license is a requisite of marriage under Article 53 of the Civil Code, 6 the absence of
which renders the marriage void ab initio pursuant to Article 80(3) 7 in relation to Article 58. 8 The requirement and
issuance of marriage license is the State's demonstration of its involvement and participation in every marriage,
in the maintenance of which the general public is interested. 9 This interest proceeds from the constitutional
mandate that the State recognizes the sanctity of family life and of affording protection to the family as a basic
"autonomous social institution." 10 Specifically, the Constitution considers marriage as an "inviolable social
institution," and is the foundation of family life which shall be protected by the State. 11 This is why the Family
Code considers marriage as "a special contract of permanent union" 12 and case law considers it "not just an
adventure but a lifetime commitment." 13

However, there are several instances recognized by the Civil Code wherein a marriage license is dispensed with,
one of which is that provided in Article 76, 14 referring to the marriage of a man and a woman who have lived
together and exclusively with each other as husband and wife for a continuous and unbroken period of at least
five years before the marriage. The rationale why no license is required in such case is to avoid exposing the
parties to humiliation, shame and embarrassment concomitant with the scandalous cohabitation of persons
outside a valid marriage due to the publication of every applicant's name for a marriage license. The publicity
attending the marriage license may discourage such persons from legitimizing their status. 15 To preserve peace
in the family, avoid the peeping and suspicious eye of public exposure and contain the source of gossip arising
from the publication of their names, the law deemed it wise to preserve their privacy and exempt them from that
requirement.

There is no dispute that the marriage of petitioners' father to respondent Norma was celebrated without any
marriage license. In lieu thereof, they executed an affidavit stating that "they have attained the age of majority,
and, being unmarried, have lived together as husband and wife for at least five years, and that we now desire to
marry each other." 16 The only issue that needs to be resolved pertains to what nature of cohabitation is
contemplated under Article 76 of the Civil Code to warrant the counting of the five year period in order to exempt
the future spouses from securing a marriage license. Should it be a cohabitation wherein both parties are
capacitated to marry each other during the entire five-year continuous period or should it be a cohabitation
wherein both parties have lived together and exclusively with each other as husband and wife during the entire
five-year continuous period regardless of whether there is a legal impediment to their being lawfully married,
which impediment may have either disappeared or intervened sometime during the cohabitation period?

Working on the assumption that Pepito and Norma have lived together as husband and wife for five years without
the benefit of marriage, that five-year period should be computed on the basis of a cohabitation as "husband and
wife" where the only missing factor is the special contract of marriage to validate the union. In other words, the
five-year common-law cohabitation period, which is counted back from the date of celebration of marriage, should
be a period of legal union had it not been for the absence of the marriage. This 5-year period should be the years
immediately before the day of the marriage and it should be a period of cohabitation characterized by exclusivity
— meaning no third party was involved at anytime within the 5 years and continuity — that is unbroken.
Otherwise, if that continuous 5-year cohabitation is computed without any distinction as to whether the parties
were capacitated to marry each other during the entire five years, then the law would be sanctioning immorality
and encouraging parties to have common law relationships and placing them on the same footing with those who
lived faithfully with their spouse. Marriage being a special relationship must be respected as such and its
requirements must be strictly observed. The presumption that a man and a woman deporting themselves as
husband and wife is based on the approximation of the requirements of the law. The parties should not be
afforded any excuse to not comply with every single requirement and later use the same missing element as a
pre-conceived escape ground to nullify their marriage. There should be no exemption from securing a marriage
license unless the circumstances clearly fall within the ambit of the exception. It should be noted that a license is
required in order to notify the public that two persons are about to be united in matrimony and that anyone who is
aware or has knowledge of any impediment to the union of the two shall make it known to the local civil
registrar. 17 The Civil Code provides:

Art. 63: . . . This notice shall request all persons having knowledge of any impediment to the marriage to
advice the local civil registrar thereof. . . .

Art. 64: Upon being advised of any alleged impediment to the marriage, the local civil registrar shall
forthwith make an investigation, examining persons under oath. . . .

This is reiterated in the Family Code thus:

Art. 17 provides in part: . . . This notice shall request all persons having knowledge of any impediment to
the marriage to advise the local civil registrar thereof. . . .

Art. 18 reads in part: . . . In case of any impediment known to the local civil registrar or brought to his
attention, he shall note down the particulars thereof and his findings thereon in the application for a
marriage license. . . .

This is the same reason why our civil laws, past or present, absolutely prohibited the concurrence of multiple
marriages by the same person during the same period. Thus, any marriage subsequently contracted during the
lifetime of the first spouse shall be illegal and void, 18 subject only to the exception in cases of absence or where
the prior marriage was dissolved or annulled. The Revised Penal Code complements the civil law in that the
contracting of two or more marriages and the having of extramarital affairs are considered felonies, i.e., bigamy
and concubinage and adultery. 19 The law sanctions monogamy.

In this case, at the time of Pepito and respondent's marriage, it cannot be said that they have lived with each
other as husband and wife for at least five years prior to their wedding day. From the time Pepito's first marriage
was dissolved to the time of his marriage with respondent, only about twenty months had elapsed. Even
assuming that Pepito and his first wife had separated in fact, and thereafter both Pepito and respondent had
started living with each other that has already lasted for five years, the fact remains that their five-year period
cohabitation was not the cohabitation contemplated by law. It should be in the nature of a perfect union that is
valid under the law but rendered imperfect only by the absence of the marriage contract. Pepito had a subsisting
marriage at the time when he started cohabiting with respondent. It is immaterial that when they lived with each
other, Pepito had already been separated in fact from his lawful spouse. The subsistence of the marriage even
where there was actual severance of the filial companionship between the spouses cannot make any
cohabitation by either spouse with any third party as being one as "husband and wife".

Having determined that the second marriage involved in this case is not covered by the exception to the
requirement of a marriage license, it is void ab initio because of the absence of such element.

The next issue to be resolved is: do petitioners have the personality to file a petition to declare their father's
marriage void after his death?

Contrary to respondent judge's ruling, Article 47 of the Family Code 20 cannot be applied even by analogy to
petitions for declaration of nullity of marriage. The second ground for annulment of marriage relied upon by the
trial court, which allows "the sane spouse" to file an annulment suit "at anytime before the death of either party" is
inapplicable. Article 47 pertains to the grounds, periods and persons who can file an annulment suit, not a suit for
declaration of nullity of marriage. The Code is silent as to who can file a petition to declare the nullity of a
marriage. Voidable and void marriages are not identical. A marriage that is annulable is valid until otherwise
declared by the court; whereas a marriage that is void ab initio is considered as having never to have taken
place 21 and cannot be the source of rights. The first can be generally ratified or confirmed by free cohabitation or
prescription while the other can never be ratified. A voidable marriage cannot be assailed collaterally except in a
direct proceeding while a void marriage can be attacked collaterally. Consequently, void marriages can be
questioned even after the death of either party but voidable marriages can be assailed only during the lifetime of
the parties and not after death of either, in which case the parties and their offspring will be left as if the marriage
had been perfectly valid. 22 That is why the action or defense for nullity is imprescriptible, unlike voidable
marriages where the action prescribes. Only the parties to a voidable marriage can assail it but any proper
interested party may attack a void marriage. Void marriages have no legal effects except those declared by law
concerning the properties of the alleged spouses, regarding co-ownership or ownership through actual joint
contribution, 23 and its effect on the children born to such void marriages as provided in Article 50 in relation to
Article 43 and 44 as well as Article 51, 53 and 54 of the Family Code. On the contrary, the property regime
governing voidable marriages is generally conjugal partnership and the children conceived before its annulment
are legitimate.

Contrary to the trial court's ruling, the death of petitioner's father extinguished the alleged marital bond between
him and respondent. The conclusion is erroneous and proceeds from a wrong premise that there was a marriage
bond that was dissolved between the two. It should be noted that their marriage was void hence it is deemed as if
it never existed at all and the death of either extinguished nothing.

Jurisprudence under the Civil Code states that no judicial decree is necessary in order to establish the nullity of a
marriage. 24 "A void marriage does not require a judicial decree to restore the parties to their original rights or to
make the marriage void but though no sentence of avoidance be absolutely necessary, yet as well for the sake of
good order of society as for the peace of mind of all concerned, it is expedient that the nullity of the marriage
should be ascertained and declared by the decree of a court of competent jurisdiction." 25 "Under ordinary
circumstances, the effect of a void marriage, so far as concerns the conferring of legal rights upon the parties, is
as though no marriage had ever taken place. And therefore, being good for no legal purpose, its invalidity can be
maintained in any proceeding in which the fact of marriage may be material, either direct or collateral, in any civil
court between any parties at any time, whether before or after the death of either or both the husband and the
wife, and upon mere proof of the facts rendering such marriage void, it will be disregarded or treated as non-
existent by the courts." It is not like a voidable marriage which cannot be collaterally attacked except in direct
proceeding instituted during the lifetime of the parties so that on the death of either, the marriage cannot be
impeached, and is made good ab initio. 26 But Article 40 of the Family Code expressly provides that there must be
a judicial declaration of the nullity of a previous marriage, though void, before a party can enter into a second
marriage 27 and such absolute nullity can be based only on a final judgment to that effect. 28 For the same reason,
the law makes either the action or defense for the declaration of absolute nullity of marriage
imprescriptible. 29 Corollarily, if the death of either party would extinguish the cause of action or the ground for
defense, then the same cannot be considered imprescriptible.

However, other than for purposes of remarriage, no judicial action is necessary to declare a marriage an absolute
nullity. For other purposes, such as but not limited to determination of heirship, legitimacy or illegitimacy of a
1âwphi1

child, settlement of estate, dissolution of property regime, or a criminal case for that matter, the court may pass
upon the validity of marriage even in a suit not directly instituted to question the same so long as it is essential to
the determination of the case. This is without prejudice to any issue that may arise in the case. When such need
arises, a final judgment of declaration of nullity is necessary even if the purpose is other than to remarry. The
clause "on the basis of a final judgment declaring such previous marriage void" in Article 40 of the Family Code
connotes that such final judgment need not be obtained only for purpose of remarriage.
WHEREFORE, the petition is GRANTED. The assailed Order of the Regional Trial Court, Toledo City, Cebu,
Branch 59, dismissing Civil Case No. T-639, is REVERSED and SET ASIDE. The said case is ordered
REINSTATED. 1âwphi1.nêt

SO ORDERED.

Davide, Jr., C.J., Puno and Kapunan, JJ., concur.


Pardo, J., on official business abroad.

Footnotes

1
The dispositive portion of the Order dated March 27, 1998 issued by Judge Ferdinand J. Marcos of
Regional Trial Court (RTC) — Branch 59, Toledo City, reads: "WHEREFORE, premises considered,
defendant's motion to dismiss is hereby granted and this instant case is hereby ordered dismissed
without costs." (p. 6; Rollo, p. 21).

2
Order, p. 4; Rollo, p. 19.

3
Minute Resolution dated July 13, 1998; Rollo, p. 39.

4
Minute Resolution dated October 7, 1998; Rollo, p. 50.

5
Tamano v. Ortiz, 291 SCRA 584 (1998).

6
Now Article 3, Family Code. Art. 53. No marriage shall be solemnized unless all the requisites are
complied with:

(1) Legal capacity of the contracting parties; their consent, freely given;

(2) Authority of the person performing the marriage; and

(3) A marriage license, except in a marriage of exceptional character.

7
Now Article 4, Family Code. Art. 80. The following marriages shall be void from the beginning:

xxx xxx xxx

(3) Those solemnized without a marriage license, save marriages of exceptional character.

xxx xxx xxx

8
Art. 58. Save marriages of an exceptional character authorized in Chapter 2 of this Title, but not those
under article 75, no marriage shall be solemnized without a license first being issued by the local civil
registrar of the municipality where either contracting party habitually resides.

9
Perido v. Perido, 63 SCRA 97 (1975).

Sec. 12, Article II, 1987 Constitution; Hernandez v. CA, G.R. No. 126010, December 8, 1999; See
10

also Tuason v. CA, 256 SCRA 158 (1996).

11
Sec. 2, Article XV (The Family), 1987 Constitution.

Art. 1, Family Code provides: "Marriage is a special contract of permanent union between a man and a
12

woman entered into in accordance with law for the establishment of conjugal or family life. . . .

13
Santos v. CA, 58 SCAD 17 (1995); 310 Phil. 21, 41 (1995).

14
Now Article 34, Family Code. Art. 76. No marriage license shall be necessary when a man and a
woman who have attained the age of majority and who, being unmarried, have lived together as husband
and wife for at least five years, desire to marry each other. The contracting parties shall state the
foregoing facts in an affidavit before any person authorized by law to administer oaths. The official, priest
or minister who solemnized the marriage shall also state in an affidavit that he took steps to ascertain the
ages and other qualifications of the contracting parties and that he found no legal impediment to the
marriage.

15
Report of the Code Commission, p. 80.

16
Rollo, p. 29.

17
Art. 63 and 64, Civil Code; Article 17 and 18, Family Code.

18
Art. 83, Civil Code provides "Any marriage subsequently contracted by any person during the lifetime of
the first spouse of such person with any person other than such first spouse shall be illegal and void from
its performance, unless:

(1) the first marriage was annulled or dissolved; or

(2) the first spouse had been absent for seven consecutive years. . . .

Art. 41 of the Family Code reads: "A marriage contracted by any person during the subsistence of
a previous marriage shall be null and void, unless before the celebration of the subsequent
marriage, the prior spouse had been absent for four consecutive years. . ."

19
Arts. 333 and 334, Revised Penal Code.

Art. 47. The action for annulment of marriage must be filed by the following persons and within the
20

periods indicated herein:

(1) For causes mentioned in number 1 of Article 45 by the party whose parent or guardian did not
give his or her consent, within five years after attaining the age of twenty-one; or by the parent or
guardian or person having legal charge of the minor, at any time before such party has reached
the age of twenty-one;

(2) For causes mentioned in number 2 of Article 45, by the sane spouse, who had no knowledge
of the other's insanity; or by any relative or guardian or person having legal charge of the insane,
at anytime before the death of either party, or by the insane spouse during a lucid interval or after
regaining sanity;

(3) For causes mentioned in number 3 of Article 45, by the injured party, within five years after the
discovery of the fraud;

(4) For causes mentioned in number 4 of Article 45, by the injured party, within five years from the
time the force, intimidation or undue influence disappeared or ceased;

For causes mentioned in numbers 5 and 6 of Article 45, by the injured party, within five years
after the marriage.

21
Suntay v. Cojuanco-Suntay, 300 SCRA 760 (1998); People v. Retirement Board, 272 III. App. 59 cited
in I Tolentino, Civil Code, 1990 ed. p. 271.

In re Conza's Estate, 176 III. 192; Miller v. Miller, 175 Cal. 797, 167 Pac. 394 cited in I Tolentino, Civil
22

Code, 1990 ed., p. 271.

23
Art. 148-149, Family Code; Article 144, Civil Code.

Odayat v. Amante, 77 SCRA 338 (1977); Weigel v. Sempio-Dy, 143 SCRA 499 (1986); People v.
24

Mendoza, 95 Phil. 845 (1954); 50 O.G. (10) 4767 cited in People v. Aragon, 100 Phil. 1033 (1957); 53
O.G. 3749.

25
35 Am. Jur. 219-220.

26
18 RCL 446-7; 35 Am Jur. 221.

Apiag v. Cantero, 335 Phil. 511 (1997); 268 SCRA 47 (1997); Atienza v. Judge Brillantes, Jr., 60 SCAD
27

119; 312 Phil. 939 (1995).


28
Domingo v. CA, 226 SCRA 572 (1993).

Art. 39, Family Code as amended by E.O. 209 and 227 s. 1987 and further amended by R.A. No. 8533
29

dated February 23, 1998.


Republic of the Philippines
SUPREME COURT
Manila

EN BANC

G.R. No. L-9005 June 20, 1958

ARSENIO DE LORIA and RICARDA DE LORIA, petitioners,


vs.
FELIPE APELAN FELIX, respondent.

Guido Advincula and Nicanor Lapuz for petitioners.


Nicodemus L. Dasig for respondent.

BENGZON, J.:

Review of a decision of the Court of Appeals, involving the central issue of the validity of the marriage in articulo
mortis between Matea de la Cruz and Felipe Apelan Felix.

It appears that long before, and during the War of the Pacific, these two persons lived together as wife and
husband at Cabrera Street, Pasay City. They acquired properties but had no children. In the early part of the
liberation of Manila and surrounding territory, Matea be came seriously ill. Knowing her critical condition, two
young ladies of legal age dedicated to the service of God, named Carmen Ordiales and Judith Vizcarra 1 visited
and persuaded her to go to confession. They fetched Father Gerardo Bautista, Catholic parish priest of Pasay.
The latter, upon learning that the penitent had been living with Felipe Apelan Felix without benefit of marriage,
asked both parties to ratify their union according to the rites of his Church. Both agreed. Whereupon the priest
heard the confession of the bed-ridden old woman, gave her Holy Communion, administered the Sacrament of
Extreme Unction and then solemnized her marriage with Felipe Apelan Felix in articulo mortis,2 Carmen Ordiales
and Judith Vizcarra acting as sponsors or witnesses. It was then January 29 or 30, 1945.

After a few months, Matea recovered from her sickness; but death was not to be denied, and in January 1946,
she was interred in Pasay, the same Fr. Bautista performing the burial ceremonies.

On May 12, 1952, Arsenio de Loria and Ricarda de Loria filed this complaint to compel defendant to an
accounting and to deliver the properties left by the deceased. They are grandchildren of Adriana de la Cruz,
sister of Matea, and claim to be the only surviving forced heirs of the latter. Felipe Apelan Felix resisted the
action, setting up his rights as widower. They obtained favorable judgment in the court of first instance, but on
appeal the Court of Appeals reversed and dismissed the complaint.

Their request for review here was given due course principally to consider the legal question-which they amply
discussed in their petition and printed brief — whether the events which took place in January 1945 constituted,
in the eyes of the law, a valid and binding marriage.

According to the Court of Appeals:

There is no doubt at all in the mind of this Court, that Fr. Gerardo Bautista, solemnized the marriage in
articulo mortis of Defendant Apelan Felix and Matea de la Cruz, on January 29 and 30, 1945, under the
circumstances set forth in the reverend's testimony in court. Fr. Bautista, a respectable old priest of
Pasay City then, had no reason to side one or the other. . . . Notwithstanding this positive evidence on the
celebration or performance of the marriage in question, Plaintiffs-Appellees contend that the same was
not in articulo mortis, because Matea de la Cruz was not then on the point of death. Fr. Bautista clearly
testified, however, that her condition at the time was bad; she was bed-ridden; and according to his
observation, she might die at any moment (Exhibit 1), so apprehensive was he about her condition that
he decided in administering to her the sacrament of extreme unction, after hearing her confession. . . .
.The greatest objection of the Appellees and the trial court against the validity of the marriage under
consideration, is the admitted fact that it was not registered.

The applicable legal provisions are contained in the Marriage Law of 1929 (Act No. 3613) as amended by
Commonwealth Act No. 114 (Nov. 1936) specially sections 1, 3, 20 and 21.

There is no question about the officiating priest's authority to solemnize marriage. There is also no question that
the parties had legal capacity to contract marriage, and that both declared before Fr. Bautista and Carmen
Ordiales and Judith Vizcarra that "they took each other as husband and wife."

The appellants' contention of invalidity rests on these propositions:


(a) There was no "marriage contract" signed by the wedded couple the witnesses and the priest, as required by
section 3 of the Marriage Law; and

(b) The priest filed no affidavit, nor recorded the marriage with the local civil registry.

The factual basis of the first proposition — no signing — may seriously be doubted. The Court of Appeals made
no finding thereon. Indeed if anything, its decision impliedly held such marriage contract to have been executed,
since it said "the marriage in articulo mortis was a fact", and the only question at issue was whether "the failure of
Fr. Bautista to send copies of the certificate of marriage in question to the Local Civil Registrar and to register the
said marriage in the Record of Marriages of the Pasay Catholic Church . . . renders the said marriage invalid."
And such was the only issue tendered in the court of first instance. (See p. 14, 34, Record on Appeal.)

However, we may as well face this second issue: Does the failure to sign the "marriage certificate or contract"
constitute a cause for nullity?

Marriage contract is the "instrument in triplicate" mentioned in sec. 3 of the Marriage Law which provides:

Sec. 3. Mutual Consent. — No particular form for the ceremony of marriage is required, but the parties
with legal capacity to contract marriage must declare, in the presence of the person solemnizing the
marriage and of two witnesses of legal age, that they take each other as husband and wife. This
declaration shall be set forth in an instrument in triplicate, signed by signature or mark by the contracting
parties and said two witnesses and attested by the person solemnizing the marriage. . . . (Emphasis
ours).

In the first place, the Marriage Law itself, in sections 28, 29 and 30 enumerates the causes for annulment of
marriage. Failure to sign the marriage contract is not one of them.

In the second place, bearing in mind that the "essential requisites for marriage are the legal capacity of the
contracting parties and their consent" (section 1), the latter being manifested by the declaration of "the parties" "in
the presence of the person solemnizing the marriage and of two witnesses of legal age that they take each other
as husband and wife" — which in this case actually occurred. 3 We think the signing of the marriage contract or
certificate was required by the statute simply for the purpose of evidencing the act. 4 No statutory provision or
court ruling has been cited making it an essential requisite — not the formal requirement of evidentiary value,
which we believe it is. The fact of marriage is one thing; the proof by which it may be established is quite another.

Certificate and Record. — Statutes relating to the solemnization of marriage usually provide for the
issuance of a certificate of marriage and for the registration or recording of marriage . . . Generally
speaking, the registration or recording of a marriage is not essential to its validity, the statute being
addressed to the officials issuing the license, certifying the marriage, and making the proper return and
registration or recording. (Sec. 27 American Jurisprudence "Marriage" p. 197-198.)

Formal Requisites. — . . . The general rule, however, is that statutes which direct that a license must be
issued and procured, that only certain persons shall perform the ceremony, that a certain number of
witnesses shall be present, that a certificate of the marriage shall be signed, returned, and recorded, and
that persons violating the conditions shall be guilty of a criminal offense, are addressed to persons in
authority to secure publicity and to require a record to be made of the marriage contract. Such statutes do
not void common-law marriages unless they do so expressly, even where such marriage are entered into
without obtaining a license and are not recorded. It is the purpose of these statutes to discourage
deception and seduction, prevent illicit intercourse under the guise of matrimony, and relieve from doubt
the status of parties who live together as man and wife, by providing competent evidence of the marriage.
. . . (Section 15 American Jurisprudence "Marriage" pp. 188-189.) Emphasis Ours. (See also Corpus
Juris Secundum "Marriage" Sec. 33.)

And our law says, "no marriage shall be declared invalid because of the absence of one or several formal
requirements of this Act . . . ." (Section 27.)

In the third place, the law, imposing on the priest the duty to furnish to the parties copies of such marriage
certificate (section 16) and punishing him for its omission (section 41) implies his obligation to see that such
"certificate" is executed accordingly. Hence, it would not be fair to visit upon the wedded couple in the form of
annulment, Father Bautista's omission, if any, which apparently had been caused by the prevailing disorder
during the liberation of Manila and its environs.

Identical remarks apply to the priest's failure to make and file the affidavit required by sections 20 and 21. It was
the priest's obligation; non-compliance with it, should bring no serious consequences to the married pair,
specially where as in this case, it was caused by the emergency.
The mere fact that the parish priest who married the plaintiff's natural father and mother, while the latter
was in articulo mortis, failed to send a copy of the marriage certificate to the municipal secretary, does not
invalidate said marriage, since it does not appear that in the celebration thereof all requisites for its
validity were not present, the forwarding of a copy of the marriage certificate not being one of the
requisites. (Jones vs. Hortiguela, 64 Phil. 179.) See also Madridejo vs. De Leon, 55 Phil. 1.

The law permits in articulo mortis marriages, without marriage license; but it requires the priest to make the
affidavit and file it. Such affidavit contains the data usually required for the issuance of a marriage license. The
first practically substitutes the latter. Now then, if a marriage celebrated without the license is not voidable (under
Act 3613),5 this marriage should not also be voidable for lack of such affidavit.

In line with the policy to encourage the legalization of the union of men and women who have lived publicly in a
state of concubinage6, (section 22), we must hold this marriage to be valid.

The widower, needless to add, has better rights to the estate of the deceased than the plaintiffs who are the
grandchildren of her sister Adriana. "In the absence of brothers or sisters and of nephews, children of the former,
. . . the surviving spouse . . . shall succeed to the entire estate of the deceased. (Art 952, Civil Code.)

Wherefore, the Court of Appeals' decision is affirmed, with costs. So ordered.

Paras, C. J., Montemayor, Reyes, A., Bautista Angelo, Concepcion, Reyes, J.B.L., Endencia, and Felix,
JJ., concur.

Footnotes

1 Now a nun at Sta. Escolastica College.

2 In his presence, Matea and Felipe expressed mutual consent to be thenceforward husband and wife.

3 p. 49 Record on Appeal.

4 And to prevent fraud, as petitioners contend, p. 30 brief. See Corpuz Juris Secundum, Vol. 55 p. 899.

5 Because it is a formal requisite" (Section 7 as amended. See American Jurisprudence, supra. However,
the New Civil Code seemingly rules otherwise. (Art. 80 (3) ).

6 Section 22 Act 3613; Article 76 New Civil Code.

You might also like